Você está na página 1de 152

TEMPLE UNIVERSITY

MATHEMATICS DEPARTMENT
A Study Guide For Real Analysis
By: Ziad Adwan

REAL
ANALYSIS
(451 Completely Solved problems)

Copyright c 2005 Ziad Adwan. All rights reserved

TEMPLE UNIVERSITY
MATHEMATICS DEPARTMENT
Study Guide for REAL ANALYSIS
The Problems
By: Ziad Adwan

1. If A and B are sets, show that A B if and only if A \ B = A:

2. Prove the Second De Morgan Law : If A, B, and C are sets, then An(B \ C) = (AnB) [ (AnC) :

3. For each n 2 N; let An = f(n + 1)k : k 2 Ng:


(a) What is A1 \ A2 ? [
\
(b) Determine the sets fAn : n 2 Ng and fAn : n 2 Ng:

4. Draw diagrams in the plane of the Cartesian products A B for the given sets A and B.
(a) A = fx 2 R : 1 x 2 or 3 x 4g; B = fx 2 R : x = 1 or x = 2g:
(b) A = f1; 2; 3g; B = fx 2 R : 1 x 3g

5. Let f (x) = 1=x2 ; x 6= 0; x 2 R:


(a) Determine the direct image f (E) where E = fx 2 R : 1 x 2g:
(b) Determine the inverse image f 1 (G) where G = fx 2 R : 1 x 4g:

6. Let g(x) = x2 and f (x) = x + 2 for x 2 R; and let h be the composite function h = g f:
(a) Find the direct image h(E) of E = fx 2 R : 0 x 1g:
(b) Determine the inverse image h 1 (G) where G = fx 2 R : 0 x 4g:

7. Let f (x) = x2 for x 2 R; and let E = fx 2 R : 1 x 0g and F = fx 2 R : 0 x 1g:


Show that E \ F = f0g and f (E \ F) = f0g; while f (E) = f (F) = fy 2 R : 0 y 1g:
Hence, f (E \ F) is a proper subset of f (E) \ f (F): What happens if 0 is deleted from the sets E and F?

8. Show that if f : A ! B and E, F are subsets of A, then f (E[F) = f (E)[f (F) and f (E\F) f (E)\f (F):

9. Show that if f : A ! B and G, H are subsets of B, then f 1 (G [ H) = f 1 (G) [ f 1 (H) and


f 1 (G \ H) = f 1 (G) \ f 1 (H):

p
10. Show that the function f dened by f (x) = x= x2 + 1; x 2 R; is a bijection of R onto fy : 1 < y < 1g:

11. (a) Show that if f : A ! B is injective and E A; then f 1 (f (E)) = E: Give an example to show
that equality need not hold if f is not injective.
(b) Show that if f : A ! B is surjective and H B; then f (f 1 (H)) = H: Give an example to show that
equality need not hold if f is not surjective.

12. (a) Suppose that f is an injection. Show that f 1 f (x) = x for all x 2 D(f ) and that
f f 1 (y) = y for all y 2 R(f ):
(b) If f is a bijection of A onto B, show that f 1 is a bijection of B onto A.

13. Let f : A ! B and g : B ! C be functions.


(a) Show that if g f is injective, then f is injective.
(b) Show that if g f is surjective, then g is surjective.

14. Prove that 1=1 2 + 1=2 3 +


+ 1=n(n + 1) = n= (n + 1) for all n 2 N:

15. Prove that 3 + 11 +


+ (8n 5) = 4n2 n for all n 2 N:
2


16. Prove that 12 22 + 32 42 +
+ ( 1)n+1 n2 = ( 1)n+1 n(n + 1)=2 for all n 2 N:

17. Prove that 52n 1 is divisible by 8 for all n 2 N:

18. Conjecture a formula for the sum of the rst n odd natural numbers 1 + 3 + 5 +
+ (2n 1); and prove
your formula using mathematical induction.

19. Prove the following Second Version of the Principle of Mathematical Induction: Let n0 2 N and
let P (n) be a statement for each natural number n n0 : Suppose that
(1) The statement P (n0 ) is true.
(2) For all k n0 ; the truth of P (k) implies the truth of P (k + 1):
Then P (n) is true for all n n0 :

20. Prove that 2n < n! for all n 4; n 2 N:

21. Prove that a nonempty set T1 is nite if and only if there is a bijection from T1 onto a nite set T2 :

22. (a) Prove that if A is a set with m 2 N elements and C A is a set with 1 element, then AnC is
a set with m 1 elements.
(b) Prove that if C is an innite set and B is a nite set, then CnB is an innite set.

23. Exhibit a bijection between N and a proper subset of itself.

24. Prove that a set T1 is denumerable if and only if there is a bijection from T1 onto a denumerable set T2 .

25. Give an example of a countable collection of nite sets whose union is not nite.

26. Prove in detail that if S and T are denumerable, then S [ T is denumerable.

27. Use mathematical induction to prove that if a set S has n elements, then P(S) has 2n elements.

28. Prove that if a; b 2 R; then


(a) (a + b) = ( a) + ( b) ;
(b) ( a) ( b) = a b;
(c) 1=( a) = (1=a);
(d) (a=b) = ( a)=b if b 6= 0:

29. Solve the following equations, justifying each step by referring to an appropriate property or theorem.
(a) 2x + 5 = 8;
(b) x2 = 2x;
(c) x2 1 = 3;
(d) (x 1)(x + 2) = 0:

30. If a 2 R satises a a = a; prove that either a = 0 or a = 1:

31. Show that there does not exist a rational number t such that t2 = 3:

32. (a) Show that if x; y are rational numbers, then so are x + y and xy:
(b) Prove that if x is a rational number and y is an irrational number, then x + y is an irrational number.
If, in addition, x 6= 0; then show that xy is an irrational number.
p
33. Let K = fs + t 2 : s; t 2 Qg: Show that K satises the following:
(a) If x1 ; x2 2 K; then x1 + x2 2 K and x1 x2 2 K:
(b) If x 6= 0 and x 2 K; then 1=x 2 K:
(Thus, the set K is a subeld of R. With the order inherited from R, the set K is an ordered eld that lies
between Q and R).

34. (a) If a < b and c d; prove that a + c < b + d:


(b) If 0 < a < b and 0 c d; prove that 0 ac bd:

35. If a; b 2 R; show that a2 + b2 = 0 if and only if a = b = 0:

36. If 0 a < b; show that a2 ab < b2 : Show by example that it does not follow that a2 < ab < b2 :

37. Find all real numbers x that satisfy the following inequalities.
(a) x2 > 3x + 4;
(b) 1 < x2 < 4;
(c) 1=x < x;
(d) 1=x < x2 :

38. Let a; b 2 R; and suppose that for every " > 0 we have a b + ": Show that a b:

39. (a) If 0 < c < 1; show that 0 < c2 < c < 1:


(b) If 1 < c; show that 1 < c < c2 :

n
40. Use mathematical induction to show that if a 2 R and m; n 2 N; then am+n = am an and (am ) = amn :

41. If a; b 2 R; show that ja + bj = jaj + jbj if and only if ab 0:

42. If x; y; z 2 R and x z; show that x y z if and only if jx yj + jy zj = jx zj :


Interpret this geometrically.

43. Show that jx aj < if and only if a


<x<a+ :

44. If a < x < b and a < y < b; show that jx yj < b a: Interpret this geometrically.

45. Find all x 2 R that satisfy the following inequalities:


(a) j4x 5j 13;
(b) x2 1
3:

46. Find all x 2 R that satisfy both j2x 3j < 5 and jx + 1j > 2 simultaneously.

47. Determine and sketch the set of pairs (x; y) 2 R R that satisfy:
(a) jxj = jyj ;
(b) jxj + jyj = 1;
(c) jxyj = 2;
(d) jxj jyj = 2:

48. Let > 0 and > 0; and a 2 R: Show that V (a) \ V (a) and V (a) [ V (a) are
neighborhoods
of a for appropriate values of :

49. Show that if a; b 2 R; and a 6= b; then there exists


neighborhoods U of a and V of b such that
U \ V = ?:

50. Let S1 = fx 2 R : x 0g: Show that the set S1 has lower bounds, but no upper bounds.
Show that inf S1 = 0:

51. Let S2 = fx 2 R : x > 0g: Does S2 have lower bounds? Does S2 have upper bounds?
Does inf S2 exist? Does sup S2 exist? Prove your statements.

52. Let S3 = f1=n : n 2 Ng: Show that sup S3 = 1 and inf S3 0:

53. Let S be a nonempty subset of R that is bounded below. Prove that inf S = supf s : s 2 Sg:

54. If a set S R contains one of its upper bounds, show that this upper bound is the supremum of S:

55. Let S R be nonempty. Show that u 2 R is an upper bound of S if and only if the conditions t 2 R
and t > u imply that t 2
= S:

56. Let S R be nonempty. Show that if u = sup S; then for every number n 2 N the number u 1=n
is not an upper bound of S; but the number u + 1=n is an upper bound of S:

57. Let S be a bounded set in R and let S0 be a nonempty subset of S:


Show that inf S inf S0 sup S0 sup S:

58. Let S R and suppose that s = sup S 2 S: If u 2


= S; show that sup(S [ fug) = supfs ; ug:

59. Use the previous exercise and mathematical induction to show that a nonempty nite set S R
contains its supremum.

60. Show that a number u is the supremum of a nonempty set S R if and only if u satises the conditions:
(1) s u for all s 2 S;
(2) if v < u; then there exists s0 2 S such that v < s0 :

61. If S = f1=n 1=m : n; m 2 Ng; nd inf S and sup S:

62. Let S R be nonempty. Suppose that a number u in R has the properties:


(i) For every n 2 N the number u 1=n is not an upper bound of S; and
(ii) For every n 2 N the number u + 1=n is an upper bound of S:
Prove that u = sup S:

63. Let X be a nonempty set and let f : X ! R have bounded range in R: Let a 2 R: Assuming the fact that
if S is a nonempty subset of R that is bounded above, then sup(a + S) = a + sup S; Show that
(i) supfa + f (x) : x 2 Xg = a + supff (x) : x 2 Xg; and
(ii) inffa + f (x) : x 2 Xg = a + infff (x) : x 2 Xg:

64. Let A and B be bounded nonempty subsets of R; and let A + B = fa + b : a 2 A; b 2 Bg: Prove that:
(i) sup(A + B) = sup(A) + sup(B); and
(ii) inf(A + B) = inf(A) + inf(B):

65. Let X be a nonempty set, and let f and g be dened on X and have bounded ranges in R: Prove that:
(i) supff (x) + g(x) : x 2 Xg supff (x) : x 2 Xg + supfg(x) : x 2 Xg; and
(ii) infff (x) : x 2 Xg + inffg(x) : x 2 Xg infff (x) + g(x) : x 2 Xg:
Give examples to show that each of these inequalities can be either equalities or strict inequalities.

66. Let X = Y = fx 2 R : 0 < x < 1g: Dene h1 ; h2 : X Y ! R by


h1 (x; y) = 2x + y; and h2 (x; y) =

0
1

if x < y
if x y

(a) For each x 2 X; nd f1 (x) = supfh1 (x; y) : y 2 Y g and f2 (x) = supfh2 (x; y) : y 2 Y g; then nd
infff1 (x) : x 2 Xg and infff2 (x) : x 2 Xg:
(b) For each y 2 Y; nd g1 (y) = inffh1 (x; y) : x 2 Xg and g2 (y) = inffh2 (x; y) : x 2 Xg; then nd
supfg1 (y) : y 2 Y g and supfg2 (y) : y 2 Y g:

67. Let X and Y be nonempty sets and let h : X Y ! R have bounded range in R: Let f : X ! R and
g : Y ! R be dened by f (x) = supfh(x; y) : y 2 Y g; and g(y) = inffh(x; y) : x 2 Xg: Prove that
supfg(y) : y 2 Y g infff (x) : x 2 Xg:

68. Show that if a > 0; then there exists a positive real number z such that z 2 = a:

69. If u > 0 is any real number and x < y; show that there exists a rational number r such that x < ru < y:

70. If S R is nonempty, show that S is bounded if and only if there exists a bounded closed interval I
such that S I:

71. If S R is a nonempty bounded set, and IS = [inf S; sup S]; show that S IS : Moreover, if J is any
closed bounded interval containing S; show that IS J:

1
\
72. Let In = [0; 1=n] for n 2 N: Prove that
In = f0g:
n=1

1
\
73. Let Kn = (n; 1) for n 2 N: Prove that
Kn = ?:
n=1

7
74. Give the two binary representations of 83 and 16
:

75. Show that if ak ; bk 2 f0; 1; :::; 9g and if


a1
10

a2
102

an
10n

b1
10

b2
102

bm
10m

6= 0;

then n = m; and ak = bk for k = 1; 2; :::; n:

76. For any b 2 R, prove that lim nb = 0:


n!1

77. Use the denition of a limit of a sequence to establish the following limits:
2n
n2 1
(b) lim n+1
= 2;
(c) lim 3n+1
= 32 ;
(d) lim 2n
= 12 :
(a) lim n2n+1 = 0;
2 +3
2n+5
n!1
n!1
n!1
n!1

78. Show that:


p
n
n
1
2n
(a) lim pn+7
= 0;
(b) lim n+2
= 2;
(c) lim n+1
= 0;
(d) lim ( n21)+1n = 0:
n!1
n!1
n!1
n!1

1
79. Let xn = ln(n+1)
for n 2 N:
(a) Use the denition of limit to show that lim (xn ) = 0:
n!1

(b) Find a specic value of K( ) as required in the denition of limit for each of (i) = 1=2; and (ii) = 1=10:

80. Prove that lim (xn ) = 0 if and only if lim (jxn j) = 0: Give an example to show that the convergence
n!1

n!1

of (jxn j) need not imply the convergence of (xn ) :

81. Prove that if lim (xn ) = x and if x > 0; then there exists M 2 N such that xn > 0 for all n M:
n!1

82. Let b 2 R satisfy 0 < b < 1: Show that lim (nbn ) = 0: (Hint: Use the binomial theorem).
n!1

83. For xn given by the following formulas, establish either the convergence or the divergence of
the sequence X = (xn ):
2
1)n n
n
n2
+3
(a) xn = n+1
;
(b) xn = ( n+1
;
(c) xn = n+1
;
(d) xn = 2n
n2 +1 :

84. Give an example of two divergent sequences X and Y such that:


(a) Their sum converges,
(b) Their product converges.

85. Show that the following sequences are not convergent:


(a) (2n ) ;
(b) ( 1)n n2 :
6


86. Find the limits of the following sequences:
p
2
1)n
p
(d) lim nn+1
:
(a) lim (2 + 1=n) ;
(b) lim (n+2
;
(c) lim pnn+11 ;
n
n!1
n!1
n!1
n!1

87. If (bn ) is a bounded sequence and lim (an ) = 0; show that lim (an bn ) = 0:
n!1
n!1

an+1 +bn+1
88. If 0 < a < b; determine lim
:
an +bn
n!1

89. Use the squeeze Theorem to determine the limits of the following:
2
1=n2
(a) n1=n ;
(b) (n!)
:

90. Recall the following Theorem: "Let (xn ) be a sequence of positive real numbers such that
L = lim (xn+1 =xn ) exists. If L < 1; then (xn ) converges and lim (xn ) = 0":
n!1
n!1
Apply this theorem to the following sequences, where a; b satisfy: 0 < a < 1; b > 1:
(a) (an ) ;
(b) (bn =2n ) ;
(c) (n=bn ) ;
(d) 23n =32n :

91. Let X = (xn ) be a sequence of positive real numbers such that lim (xn+1 =xn ) = L > 1:
n!1
Show that X is not a bounded sequence and hence is not convergent.

1=n
92. Let (xn ) be a sequence of positive real numbers such that lim xn
= L < 1:
n!1
n
Show that there exists a number r with 0 < r < 1 such that 0 < xn < r for all su ciently large n 2 N:
Use this to show that lim (xn ) = 0:
n!1

93. Suppose that (xn ) is a convergent sequence and (yn ) is such that for any > 0 there exists M such
that jxn yn j < for all n M: Does it follow that (yn ) is convergent?

94. Let x1 = 8 and xn+1 = 12 xn + 2 for n 2 N. Show that (xn ) is bounded and monotone. Find lim (xn ) :
n!1

95. Let x1 > 1 and xn+1 = 2 1=xn for n 2 N. Show that (xn ) is bounded and monotone. Find lim (xn ) :
n!1
p

96. Let x1 2 and xn+1 = 1 + xn 1 for n 2 N. Show that (xn ) is decreasing and bounded below by 2.
Find lim (xn ) :
n!1

p
p
97. Let y1 = p; where p > 0; and yn+1 = p + yn for n 2 N. Show that (yn ) converges and nd lim (yn ) :
n!1
p
(Hint: One upper bound is 1 + 2 p):

98. Let (an ) be an increasing sequence and (bn ) be a decreasing sequence, and assume that an bn 8n 2 N:
Show that lim (an )
lim (bn ) ; and thereby deduce the Nested Intervals Property from the Monotone
n!1
n!1
Convergence Theorem.

99. Let A be an innite subset of R that is bounded above and let u = sup A: Show that there exists
an increasing sequence (xn ) with xn 2 A for all n 2 N such that u = lim (xn ) :
n!1

100. Let (xn ) be a bounded sequence, and for each n 2 N let sn = supfxk : k ng and tn = inffxk : k ng:
Prove that (sn ) and (tn ) are monotone and convergent. Also, prove that if lim (sn ) = lim (tn ) ; then
n!1

n!1

(xn ) is convergent.

101. Establish the convergence or divergence of the sequence (yn ) ; where


1
1
1
yn = n+1
+ n+2
+
+ 2n
for n 2 N:

102. Let xn = 1=12 + 1=22 +


+ 1=n2 for each n 2 N: Prove that (xn ) is increasing and bounded, and hence

converges. [Hint: Note that if k 2; then 1=k 2 1=k(k 1) = 1=(k 1) 1=k:]:

103. Establish the convergence and nd the limits of the following sequences:
1
(a) (1 + 1=n)n+1 ;
(b) (1 + 1=n)2n ;
(c) (1 + n+1
)n ;
(d) ((1 1=n)n ) :
p

104. Calculate 2; correct to within 4 decimals.

105. Give an example of an unbounded sequence that has a convergent subsequence.

106. Show that the following sequences are divergent.


(a) (1 ( 1)n + 1=n) ;
(b) (sin(n =4)) :

107. Let X = (xn ) and Y = (yn ) be given sequences, and let the "shu- ed" sequence Z = (zn ) be dened
by: z1 = x1 ; z2 = y1 ;
; z2n 1 = xn ; z2n = yn ;
: Show that Z is convergent if and only if both X and Y
are convergent and lim X = lim Y:

108. Establish the convergence and nd the limits of the following sequences:
2
2
(d) ((1 + 2=n)n ) :
(a) (1 + 1=n2 )n ;
(b) ((1 + 1=2n)n ) ;
(c) (1 + 1=n2 )2n ;

109. Determine the limits of the following:


1=2n
(a) (3n)
;
(b) (1 + 1=2n)3n :

110. Suppose that every subsequence of X = (xn ) has a subsequence that converges to 0. Show that
lim X = 0:

111. Show that if (xn ) is unbounded, then there exists a subsequence (xnk ) such that lim (1=xnk ) = 0:
k!1

112. Let (xn ) be a bounded sequence and let s = supfxn : n 2 Ng: Show that if s 2
= fxn : n 2 Ng; then
there is a subsequence of (xn ) that converges to s:

113. Let (In ) be a nested sequence of closed bounded intervals. For each n 2 N; let xn 2 In : Use the BolzanoWeierstrass Theorem to show that there exists a number 2 R such that 2 In 8n 2 N:
[That is, Use the Bolzano-Weierstrass Theorem to prove the Nested Intervals Property].

114. Show directly from the denition that the following are Cauchy sequences.
1
1
(a) n+1
;
(b) 1 + 2!
+
+ n!
:
n

115. Show directly from the denition that the following are not Cauchy sequences.
n
n
(a) (( 1) ) ;
(b) n + ( n1) ;
(c) (ln n)

116. Show directly from the denition that if (xn ) and (yn ) are Cauchy sequences, then (xn + yn ) and (xn yn )
are Cauchy sequences.
p

117. If xn = n; show that (xn ) satises lim jxn+1 xn j = 0; but that it is not a Cauchy sequence.
n!1

118. Let (xn ) be a Cauchy sequence such that xn is an integer for every n 2 N: Show that (xn ) is ultimately
constant.

119. Show directly that a bounded, monotone increasing sequence is a Cauchy sequence.

120. If 0 < r < 1 and jxn+1 xn j < rn for all n 2 N; show that (xn ) is a Cauchy sequence.

121. Show that if (xn ) is an unbounded sequence, then there exists a properly divergent subsequence.
8


122. Give examples of properly divergent sequences (xn ) and (yn ) with yn 6= 0 for all n 2 N such that
(a) (xn =yn ) is convergent;
(b) (xn =yn ) is properly divergent:

123. Show that if xn > 0 for all n 2 N; then lim (xn ) = 0 if and only if lim (1=xn ) = +1:
n!1
n!1

124. p
Establish the proper
sequences:
p divergence of the following
p
p
(a) ( n) ;
(b)
n+1 ;
(c)
n 1 ;
(d) n= n + 1 :

125. Is the sequence (n sin n) properly divergent?

126. Let (xn ) be properly divergent and let (yn ) be such that lim (xn yn ) belongs to R: Show that (yn )
n!1
converges to 0.

127. Let (xn ) and (yn ) be sequences of positive numbers such that lim (xn =yn ) = 0:
n!1

(a) Show that if lim (xn ) = +1; then lim (yn ) = +1:
n!1

n!1

(b) Show that if (yn ) is bounded, then lim (xn ) = 0:


n!1

128. p
Investigate the convergence
or divergence of the following
sequences:
p
p
p
p
n2 + 2 ;
(b)
n= n2 + 1 ;
(c)
n2 + 1= n ;
(d) (sin n) :
(a)

129. Let (xn ) and (yn ) be sequences of positive numbers such that lim (xn =yn ) = +1:
n!1

(a) Show that if lim (yn ) = +1; then lim (xn ) = +1:
n!1

n!1

(b) Show that if (xn ) is bounded, then lim (yn ) = 0:


n!1

130. Show that if lim (an =n) = L; where L > 0; then lim (an ) = +1:
n!1
n!1

131. Show that the convergence of a series is not aected by changing a nite number of its terms.
(of course, the value of the sum may be changed).

1
1
1
X
X
X
132. If
xn and
yn are convergent, show that
(xn + yn ) is convergent.
n=1

n=1

n=1

1
X
133. (a) Show that the series
cos n is divergent.
(b) Show that the series

1
X

n=1

cos n
n2

is convergent.

n=1

1
X
( 1)n
p
134. Show that the series
is convergent.
n
n=1

1
1
X
X
135. If
an with an > 0 is convergent, then is
a2n always convergent?
n=1

n=1

Either prove or give a counterexample.

1
1
X
X
p
136. If
an with an > 0 is convergent, then is
an always convergent?
n=1

n=1

Either prove or give a counterexample.

1
1
X
X
137. If
an with an > 0 is convergent, and if bn = (a1 + n +an ) for n 2 N; then show that
bn is always
n=1

n=1

divergent.

138. Determine a condition on jx 1j that will assure that:


(a) x2 1 < 12 ;
(b) x2 1 < 1=103 ;
1
(c) x2 1 < n for a given n 2 N;
(d) x3 1 < n1 for a given n 2 N:

139. Let c be a cluster point of A R and let f : A ! R: Prove that lim f (x) = L if and only if
x!c

lim jf (x) Lj = 0:
x!c

140. Let f : R ! R and let c 2 R: Show that lim f (x) = L if and only if lim f (x + c) = L:
x!c
x!0

2
141. Let I = (0; a) where a > 0; and let g(x) = x for x 2 I: For any points x; c 2 I; show that
g(x) c2
2a jx cj : Use this inequality to prove that lim x2 = c2 for any c 2 I:
x!c

142. Let I be an interval in R; let f : I ! R, and let c 2 I: Suppose there exists constants K and L such that
jf (x) Lj < K jx cj for x 2 I: Show that lim f (x) = L:
x!c

143. Show that lim x3 = c3 for any c 2 R:


x!c
p
p
144. Show that lim x = c for any c > 0:
x!c

145. Use the denition of limit to show that


x+5
(a) lim x2 + 4x = 12;
(b) lim 2x+3
= 4:
x!2
x! 1

146. Show that the following limits do not exist:


(a) lim x12 (x > 0);
(b) lim p1x (x > 0);
x!0

x!0

(c) lim (x+sgn(x));


(d) lim sin 1=x2
x!0
x!0

147. Suppose the function f : R ! R has limit L at 0; and let a > 0: If g : R ! R is dened by
g(x) = f (ax) for x 2 R; show that lim g(x) = L:
x!0

x
if x 2 Q
148. Let f : R ! R be dened by: f (x) =
:
0
if x 2
=Q
(a) Show that f has a limit at x = 0:
(b) Use a sequential argument to show that if c 6= 0; then f does not have a limit at c:

149. Determine the following limits:


(a) lim (x + 1) (2x + 3)
x!1

(c) lim

x!2

1
x+1

1
2x

(x 2 R);

(b) lim xx2 +22

(x > 0);

(d) lim xx+1


2 +2

(x 2 R):

x!1

(x > 0);

x!0

150. Determine the following limits:


q
2
(x > 0);
(b) lim xx 24 (x > 0);
(a) lim 2x+1
x+3
x!2 p

x!2

(c)

2
1
lim (x+1)
x
x!0

x 1
x!1 x 1

(x > 0);

(d) lim

(x > 0):

p p
1+3x
151. Find lim 1+2x
where x > 0:
x+2x2
x!0

152. Prove that lim cos (1=x) does not exist but that lim x cos (1=x) = 0:
x!0
x!0

10

153. Let f; g be dened on A R to R; and let c be a cluster point of A: Suppose that f is bounded
on a neighborhood of c and that lim g(x) = 0: Prove that lim f (x) g (x) = 0:
x!c
x!c

154. Use the denition of the limit to prove that if f; g are dened on A R to R; c is a cluster point of A;
lim f (x) = L; and lim g(x) = M; then lim [f (x) + g(x)] = L + M:
x!c
x!c
x!c

155. Let n 2 N be such that n 3: Derive the inequality x2 xn x2 for 1 < x < 1: Then use the fact
that lim x2 = 0 to show that lim xn = 0:
x!0
x!0

156. Give examples of functions f and g such that f and g do not have limits at a point c; but such that both
f + g and f g have limits at c:

157. Determine whether the following limits exist in R :


(a) lim sin 1=x2

(b) lim x sin 1=x2


x!0 p
(d) lim x sin 1=x2

(x 6= 0);

x!0

(c) lim sgn(sin (1=x))

(x 6= 0);

x!0

x!0

(x 6= 0);

(x > 0):

158. Let A R; let fp: A ! R; and let c 2 R be a cluster pointpof A: In addition,


suppose that f (x) 0
p
for all x 2 A; and let f be the function dened for x 2 A by
f (x) = f (x):
q
p
If lim f (x) exists, prove that lim f (x) = lim f (x):
x!c
x!c
x!c

1=2
159. Let f (x) = jxj
for x 6= 0: Show that lim f (x) = lim f (x) = +1:
x!0+
x!0

160. Let c 2 R and let f be dened for x 2 (c; 1) and f (x) > 0 for all x 2 (c; 1):
Show that lim f (x) = 1 if and only if lim 1=f (x) = 0:
x!c
x!c

161. Evaluate the following limits, or show that they do not exist.
x
x!1+ x 1
p
lim x+2
x!0+ x
p
lim x+1
x!0 px
lim px 5
x!1 x+3

(a) lim

(x 6= 1);

(c)

(x > 0);

(e)

(x >

(b) lim x x 1
x!1

x+2
p
x!1 x
p
(f ) lim x+1
x!1 px
(h) lim pxx+xx
x!1

(d) lim

1);

(x 6= 1);
(x > 0);
(x > 0);

(x > 0):
(x > 0);
(g)

162. Suppose that f and g have limits in R as x ! 1 and that f (x) g(x) for all x 2 (a; 1):
Prove that lim f (x)
lim g(x):
x!1
x!1

163. Let f be dened on (0; 1) to R: Prove that lim f (x) = L if and only if lim f (1=x) = L:
x!1
x!0+

164. Show that if f : (a; 1) ! R is such that lim xf (x) = L; where L 2 R; then lim f (x) = 0:
x!1
x!1

165. suppose that lim f (x) = L; where L > 0; and that lim g(x) = 1: Show that lim f (x)g(x) = 1:
x!c
x!c
x!c
If L = 0; show by example that this conclusion may fail.

166. Find functions f and g dened on (0; 1) such that lim f = 1 and lim g = 1; and lim (f g) = 0:
x!1

x!1

x!1

Can you nd such functions, with g(x) > 0 for all x 2 (0; 1); such that lim f =g = 0?
x!1

167. Prove the following Sequential Criterion for Continuity: A function f : A ! R is continuous at
a point c 2 A if and only if for every sequence (xn ) in A that converges to c; the sequence (f (xn ))
converges to f (c):

11

168. Let f be dened for all x 2 R; x 6= 2; by f (x) = (x2 + x 6)=(x 2): Can f be dened at x = 2
in such a way that f is continuous at this point?

169. Let f : R ! R be continuous at c and let f (c) > 0: Show that there exists a neighborhood V (c)
of c such that if x 2 V (c); then f (x) > 0:

170. Let f : R ! R be continuous on R and let S = fx 2 R : f (x) = 0g be the zero set of f: If (xn )
is in S and x = lim (xn ) ; show that x 2 S:
n!1

171. Let K > 0 and let f : R ! R satisfy the condition jf (x) f (y)j K jx yj for all x; y 2 R: Show
that f is continuous at every point c 2 R:

172. Suppose that f : R ! R is continuous on R and that f (r) = 0 for every rational number r: Prove
that f (x) = 0 for all x 2 R:

173. Determine the points of discontinuity of the following functions and state which theorems are used
in each case.
p
p
2
(x 2 R)
(b) g(x) = x + x (x 0)
(a) f (x) = x x+2x+1
2 +1
p
p
1+jsin xj
(c) h(x) =
(x 6= 0)
(d) k(x) = cos 1 + x2 (x 2 R)
x

174. Show that if f : A ! R is continuous on A R and if n 2 N; then the function f n dened by:
n
f n (x) = (f (x)) for x 2 A; is continuous on A:

175. Let g be dened on R by g(1) = 0; and g(x) = 2 if x 6= 1; and let f (x) = x + 1 for all x 2 R:
Show that lim g f 6= g f (0): Why doesnt this contradict Theorem 5.2.6?
x!0

176. Give an example of a function f : [0; 1] ! R that is discontinuous at every point of [0; 1] but
such that jf j is continuous on [0; 1]:

177. Let f; g be continuous from R to R; and suppose that f (r) = g(r) for all rational numbers r: Is it
true that f (x) = g(x) for all x 2 R?

178. Let f : R ! R is continuous on R; and let P = fx 2 R : f (x) > 0g: If c 2 P; show that there
exists a neighborhood V (c) P:

179. Let I = [a; b] and let f : I ! R be a continuous function such that f (x) > 0 for each x 2 I:
Prove that there exists a number > 0 such that f (x)
for each x 2 I:

180. Let f be continuous on the interval [0; 1] to R and such that f (0) = f (1): Prove that there exists
a point c 2 [0; 12 ] such that f (c) = f (c + 12 ): Conclude that there are, at any time, antipodal points
on the earths equator that have the same temperature. (Hint: consider g(x) = f (x) f (x + 12 )):

181. Show that the equation x = cos x has a solution on the interval [0; =2]: Use the Bisection method
and a calculator to nd an approximate solution of this equation, with error less than 10 3 :

182. Let I = [a; b], let f : I ! R be continuous on I; and assume that f (a) < 0; f (b) > 0:
Let W = fx 2 I : f (x) < 0g; and let w = sup W: Prove that f (w) = 0:

183. Suppose that f : R ! R is continuous on R and that lim f = 0 and lim f = 0:


x! 1

x!1

Prove that f is bounded on R and attains either a maximum or a minimum on R:


Give an example to show that both a maximum and a minimum need not be attained.

184. Show that the function f (x) = x12 is uniformly continuous on A = [1; 1); but that it is not uniformly
12

continuous on B = (0; 1):

185. Show that if f and g are uniformly continuous on A R and if they are both bounded on A; then
their product f g is uniformly continuous on A:

186. Prove that if f and g are each uniformly continuous on R; then their composite function f g is
uniformly continuous on R:
p
187. If g(x) = x for x 2 [0; 1]; show that there does not exist a constant K such that jg(x)j K jxj for all
x 2 [0; 1]: Conclude that the uniformly continuous function g is not a Lipschitz function on [0; 1]:

188. Show that if a function f : R ! R is Lipschitz on R, then f is uniformly continuous on R:

189. Show that the function f (x) = 1=x is uniformly continuous on the set A = [a; 1); where a is a positive
constant.

190. Show that the function f (x) = x2 is not uniformly continuous on the set A = [0; 1):

191. Show that the function f (x) = sin(1=x) is not uniformly continuous on the set B = (0; 1):

1
192. Show that the function f (x) = 1+x
2 for x 2 R is uniformly continuous on R:

193. Show that if f and g are uniformly continuous on A R; then f + g is uniformly continuous on A:

194. If f is uniformly continuous on A R; and jf (x)j k > 0 for all x 2 A; show that 1=f is uniformly
continuous on A:

195. Prove that if f is uniformly continuous on a bounded set A R; then f is bounded on A:

196. Show that if f is continuous on [0; 1) and uniformly continuous on [a; 1) for some positive constant a;
then f is uniformly continuous on [0; 1):

197. Let A R and suppose that f : A ! R has the following property:


8 > 0 9g : A ! R such that g is uniformly continuous on A and jf (x) g (x)j < for all x 2 A:
Show that f is uniformly continuous on A:

198. (a) Show that if f is uniformly continuous on (a; b] and on [b; c); then f is also uniformly
continuous on (a; c):
(b) Prove that any function f : R ! R that is continuous and periodic must be uniformly continuous.

199. Show that if f : R ! R is continuous and such that lim f (x) and lim f (x) are nite, then f is
x! 1

x!1

uniformly continuous on R.

200. A function f : R ! R is continuous at 0 and satises the following conditions:


f (0) = 0
and
f (x + y) f (x) + f (y) for any x; y 2 R:
Prove that f is uniformly continuous on R:

201. A point x 2 R is said to be an interior point of A R in case there is a neighborhood V of x such


that V
A: Show that a set A R is open if and only if every point of A is an interior point of A:

202. A point x 2 R is said to be a boundary point of A R in case every neighborhood V of x contains


points in A and points in Ac : Show that a set A and its complement Ac have exactly the same boundary
points.

13


203. Show that a set G R is open if and only if it does not contain any of its boundary points.

204. Show that a set F R is closed if and only if it contains all of its boundary points.

205. If A R; let A be the union of all open sets that are contained in A; the set A is called the interior
of A: Show that:
(a) A is an open set,
(b) A is the largest open set contained in A; and
(c) A point z belongs to A if and only if z is an interior point of A:

206. If A R; let A be the intersection of all closed sets containing A; the set A is called the closure of A:
Show that:
(a) A is a closed set,
(b) A is the smallest closed set containing A; and
(c) A point w belongs to A if and only if w is either an interior point or a boundary point of A:

207. Exhibit an open cover of N that has no nite subcover.

208. Exhibit an open cover of the set A = f1=n : n 2 Ng that has no nite subcover.

209. Prove that if K1 and K2 are compact sets in R; then K = K1 [ K2 is compact.

210. Prove that the intersection of an arbitrary collection of compact sets in R is compact.

211. Let K 6= ? be compact in R and let c 2 R:


(a) Prove that there exists a point a 2 K such that jc aj = inffjc xj : x 2 Kg:
(b) Prove that there exists a point b 2 K such that jc bj = supfjc xj : x 2 Kg:

1 if 0 x 1
212. Let h : R ! R be dened by h(x) =
: Find an open set G such that h 1 (G) is
0
otherwise
not open, and a closed set F such that h 1 (F ) is not closed.

213. Prove that f : R ! R is continuous if and only if for each closed set F in R; the inverse image f 1 (F )
is closed in R:

214. Show that if f : R ! R is continuous, then the set fx 2 R : f (x) < g is open in R for each 2 R:

215. Show that if f : R ! R is continuous, then the set fx 2 R : f (x)


g is closed in R for each 2 R:

216. Show that if f : R ! R is continuous, then the set fx 2 R : f (x) = kg is closed in R for each k 2 R:

217. Give an example of a function f : R ! R such that the set fx 2 R : f (x) = 1g is neither open
nor closed in R:

218. Let I = [a; b] and let f : I ! R and g : I ! R be continuous functions on I: Show that the set
fx 2 R : f (x) = g(x)g is closed in R:

219. Show that each of the following functions is a metric on the given space:
(a) S = R2 ; d1 (P1 ; P2 ) = jx1 x2 j + jy1 y2 j ; where P1 = (x1 ; y1 ) and P2 = (x2 ; y2 ):
(b) S = R2 ; d1 (P1 ; P2 ) = sup fjx1 x2 j ; jy1 y2 jg ; where P1 = (x1 ; y1 ) and P2 = (x2 ; y2 ):
Z1
(c) S = C[0; 1]; d1 (f; g) = jf gj :
0

14

(d) S = C[0; 1]; d1 (f; g) = supfjf (x)

g(x)j : x 2 [0; 1]g:


0 if s = t
(e) S = Any nonempty set; d(s; t) =
: This is called the discrete metric on S.
1 if s 6= t

220. If Pn = (xn ; yn ) 2 R2 and d1 is the metric in the previous problem part (b), show that (Pn ) converges
to P = (x; y) with respect to this metric if and only if (xn ) and (yn ) converge to x and y, respectively.

221. Let S be a nonempty set and let d be the discrete metric dened above. Show that in the metric space
(S; d); a sequence (xn ) in S converges to x if and only if there is a K 2 N such that xn = x for all n K:

222. Show that if d is the discrete metric on a set S; then every subset of S is both open and closed in (S; d):

223. Prove that in any metric space (S; d), an


neighborhood of a point is an open set.

x2 if x 2 Q
224. Let f : R ! R be dened by f (x) =
: Show that f is dierentiable at x = 0; and
0 if x 2
=Q
nd f 0 (0):

x2 sin(1=x2 ) if x 6= 0
225. Let g : R ! R be dened by g(x) =
: Show that g is dierentiable for
0
if x = 0
all x 2 R; Also, show that the derivative g 0 is not bounded on the interval [ 1; 1]:

226. Assume that there exists a function L : (0; 1) ! R such that L0 (x) = 1=x for x > 0: Calculate the
derivatives of the following functions:
(a) f (x) = L(2x + 3) for x > 0;
3
(b) g(x) = L(x2 ) for x > 0;
(c) h(x) = L(ax) for a > 0; x > 0;
(d) k(x) = L(L(x)) when L(x) > 0; x > 0:

0
227. Given that the function h(x) = x3 + 2x + 1 for x 2 R has an inverse h 1 on R; nd the value of h 1 (y)
at the points corresponding to x = 0; 1; 1:

228. Given that the restriction of the cosine function cos to I = [0; ] is strictly decreasing and that cos 0 = 1;
cos = 1; let J = [ 1; 1]; and let arccos : J ! R be the function inverse to the restriction of cos to I:
Show that arccos is dierentiable on ( 1; 1) and D(arccos(y)) = p 1 2 for y 2 ( 1; 1):
1 y

Show that arccos is not dierentiable at 1 and 1:

229. Let f : I ! R be dierentiable at c 2 I: Establish the Straddle Lemma:


Given > 0; there exists > 0 such that if u; v 2 I satisfy c
<u<c<v <c+ ;
then we have that jf (v) f (u) (v u)f 0 (c)j
(v u) :
(Hint: add and subtract the term f (c) cf 0 (c)):

230. Prove that if f : R ! R is an even function [that is, f ( x) = f (x) for all x 2 R] and has a derivative
at every point, then the derivative f 0 is an odd function [that is, f 0 ( x) = f 0 (x) for all x 2 R]: Also,
prove that if g : R ! R is a dierentiable odd function, then g 0 is an even function.

231. If f : R ! R is dierentiable at c 2 R; show that f 0 (c) = lim (n ff (c + 1=n) f (c)g) : However, show
n!1

by an example that the existence of the limit of this sequence does not imply the existence of f 0 (c):

232. Use the Mean Value Theorem to prove that jsin x sin yj jx yj for all x; y 2 R:

0 if x < 0
233. If h(x) =
; prove there does not exist a function f : R ! R such that f 0 (x) = h(x) for
1 if x 0
all x 2 R: Give examples of two functions, not diering by a constant, whose derivatives equal h(x) for
15

all x 6= 0:

234. Let I be an interval. Prove that if f is dierentiable on I and if the derivative f 0 is bounded on I; then
f satises a Lipschitz condition on I:

235. Let f; g be dierentiable functions on R and suppose that f (0) = g(0) and f 0 (x) g 0 (x) for all x 0:
Show that f (x) g(x) for all x 0:

236. Suppose that f : [0; 2] ! R is continuous on [0; 2] and dierentiable on (0; 2); and that f (0) = 0;
f (1) = 1; f (2) = 1:
(a) Show that there exists c1 2 (0; 1) : f 0 (c1 ) = 1:
(b) Show that there exists c2 2 (1; 2) : f 0 (c2 ) = 0:
(c) Show that there exists c 2 (0; 2) : f 0 (c) = 1=3:

237. Use the Mean Value Theorem to prove that x x 1 < ln x < x 1 for x > 1:
[Hint: Use the fact that D ln x = 1=x for x > 0]:

238. Let f : [a; b] ! R be continuous on [a; b]; and dierentiable in (a; b): Show that if lim f 0 (x) = A;
x!a

then f 0 (a) exists and is equal to A:

2x4 + x4 sin(1=x) if x 6= 0
239. Let f : R ! R be dened by f (x) =
: Show that f has an absolute
0
if x = 0
minimum at x = 0; but that its derivative has both positive and negative values in every neighborhood of 0.

240. Give an example of a uniformly continuous function on [0; 1] that is dierentiable on (0; 1) but whose
derivative is not bounded on (0; 1):

241. Suppose that f and g are continuous on [a; b], dierentiable on (a; b), that c 2 [a; b]; and that g(x) 6= 0
(x)
exists in R; show that we
for x 2 [a; b]; x 6= c: Let A = lim f (x) and B = lim g(x): If B = 0; and if lim fg(x)
x!c

x!c

x!c

(x)
must have A = 0: [Hint: f (x) = fg(x)
g(x)]:

242. In addition to the suppositions of the preceding exercise, let g(x) > 0 for x 2 [a; b]; x 6= c: If A > 0 and
(x)
(x)
B = 0; prove that we must have lim fg(x)
= 1: If A < 0 and B = 0; prove that we must have lim fg(x)
= 1:
x!c
x!c

x2 for x 2 Q
243. Let f (x) =
; and let g(x) = sin x for x 2 R: Use Theorem 6.3.1 to show that
0 for x 2
=Q
lim f (x) = 0: Explain why LHospitals Rule I cannot be used.
x!0 g(x)

244. Evaluate the following limits:


px
(a) lim lnx2x (0; 1);
(b) lim ln
(0; 1);
x
x!1

x!1

x
(0; 1):
(c) lim x ln sin x (0; );
(d) lim x+ln
x!1 x ln x
x!0

245. Evaluate the following limits:


(a) lim x2x (0; 1);
(b) lim (1 + x3 )x (0; 1);
x!0+

x!0

1
(c) lim (1 + x3 )x (0; 1);
(d) lim x1 arctan
(0; 1):
x
x!1
x!0+

246. Let g(x) = x3 for x 2 R: Find g 0 (x) and g 00 (x) for x 2 R; and g 000 (x) for x 6= 0: Show that g 000 (0) does
not exist.

247. Use induction to prove Leibnizs rule for the n-th derivative of a product:
n
X
(n)
n
(n k)
(f g) (x) =
(x)g (k) (x):
k f
k=0

16

3
5 3
x
:
Use
this
inequality
to
approximate
1:2
248. If x > 0; show that (1 + x)1=3
1 + 13 x 19 x2
81
p
3
and 2:

249. If f (x) = ex ; show that the remainder term in Taylors Theorem converges to 0 as n ! 1; for each
xed x0 and x:

2
e 1=x for x 6= 0
: Show that h(n) (0) = 0 for all n 2 N: Conclude that the remainder
250. Let h(x) =
0
for x = 0
term in Taylors Theorem for x0 = 0 does not converge to 0 as n ! 1; for x 6= 0: [Hint: By LHospitals
Rule, lim h(x)
= 0 for any k 2 N: Use Leibnizs Rule to calculate h(n) (x) for x 6= 0]:
k
x!0 x

251. We wish to approximate sin by a polynomial on [ 1; 1] so that the error is less than 0:001. Show that
3
x5
1
we have sin x
x x6 + 120
< 5040
for jxj 1:
R
b
252. If f 2 R[a; b] and jf (x)j M for all x 2 [a; b]; show that a f
M (b a):

253. Suppose f is bounded on [a; b] and that there exists two sequences of tagged partitions of [a; b] such that
:
:
:
:
= R[a; b]:
P n ! 0 and Qn ! 0; but such that lim S(f ; P n ) 6= lim S(f ; Qn ): Show that f 2
n!1
n!1

1 if x 2 Q \ [0; 1]
254. Let f be the Dirichlet function dened by f (x) =
: Show that f 2
= R[0; 1]:
0 if x 2 [0; 1] Q

255. Suppose that f : [a; b] ! R and that f (x) = 0 except for a nite number of points c1 ; :::; cn in [a; b]:
Rb
Prove that f 2 R[a; b] and that a f = 0:

256. If g 2 R[a; b] and if f (x) = g(x) except for a nite number of points in [a; b]; prove that f 2 R[a; b]
Rb
Rb
and that a f = a g:

257. Letq0 a < b; let f (x) = x2 for x 2 [a; b]; and let P = f[xi 1 ; xi ]gni=1 be a partition of [a; b]: For each i;
let qi = 13 x2i + xi 1 xi + x2i 1 :
(a) Show that qi satises 0 xi 1 qi
(b) Show that f (qi )(xi xi 1 ) = 13 x3i
:

xi :
x3i 1 :

(c) If Q is the tagged partition with the same subintervals as P and the tags qi ; show that
:
S(f ; Q) = 13 b3 a3 :
Rb
Rb
(d) Show that f 2 R[a; b] and a f = a x2 dx = 13 b3 a3 :

x + 1 if x 2 Q \ [0; 1]
258. Consider the function h(x) =
: Show that h 2
= R[0; 1]:
0
if x 2 [0; 1] Q

k
if x = k1 ; k 2 N
259. Let H(x) =
: Show that H 2
= R[0; 1]:
0 if x 2 [0; 1] f k1 : k 2 Ng
:
260. If S(f ; P) is any Riemann sum of f : [a; b] ! R, show that there exists a step function ' : [a; b] ! R
:
Rb
such that a ' = S(f ; P):
R
b
261. Suppose that f is continuous on [a; b]; that f (x) 0 for all x 2 [a; b] and that a f = 0: Prove that
f (x) = 0 for all x 2 [a; b]:

262. Show that the continuity hypothesis in the preceeding exercise cannot be dropped.
R R
b
b
263. If f and g are continuous on [a; b] and if a f = a g; prove that there exists c 2 [a; b] such that
f (c) = g(c):
17


264. Give an example of a function f : [a; b] ! R such that f 2 R[c; b] for each c 2 (a; b) but f 2
= R[a; b]:

265. (Mean Value Theorem for Integrals) If f is continuous on [a; b]; a < b; show that there exists c 2 [a; b]
Rb
such that a f = f (c)(b a):

266. If : [a; b] ! R takes on only a nite number of distinct values. is a step function?

267. If (x) = x and !(x) = x and if (x) f (x) !(x) for all x 2 [0; 1]; does it follow from the squeeze
theorem that f 2 R[0; 1]?

268. If f is bounded by M on [a; b] and if the restriction of f to every interval [c; b] where c 2 (a; b) is
Rb
Rb
Riemann integrable, show that f 2 R[a; b] and that lim c f = a f:
c!a+

M for x 2 [a; c)
M
for x 2 [a; c)
and ! c (x) =
:
f (x) for x 2 [c; b]
f (x) for x 2 [c; b]
Apply the squeeze theorem for c su ciently near a].

sin (1=x) for x 2 (0; 1]


269. Show that g(x) =
belongs to R[0; 1]:
0
for x = 0

270. If f is bounded and there is a nite set E such that f is continuous at every point of [a; b]nE; show that
f 2 R[a; b]:

271. Let f be continuous on [a; b]; let f (x) 0 for x 2 [a; b]; and let
[Hint: Let

c (x)

Mn =
Show that lim Mn = sup ff (x)g :
n!1

Rb
a

fn

1=n

x2[a;b]

n+1
272. If n 2 N and Hn (x) = xn+1 for x 2 [a; b]; show that the rst form of the fundamental theorem of
Rb
n+1
n+1
calculus implies that a xn dx = b n+1a : What is the set E for which Hn0 (x) 6= xn ?

R3
x for jxj 1
273. If g(x) =
and if G(x) = 12 x2 1 ; show that 2 g(x)dx = G(3) G( 2) = 5=2:
x for jxj < 1

1 2
Rb
for x < 0
2x
274. Let B(x) =
: Show that a jxj dx = B(b) B(a):
1 2
for x 0
2x
R
z
275. If f 2 R[a; b] and if c 2 [a; b]; the function dened by Fc (z) = c f for z 2 [a; b] is called the indenite
integral of f with basepoint c: Find a relation between Fa and Fc :
8

9
< x for 0 x < 1 =
Rx
1 for 1 x < 2
276. Let f : [0; 3] ! R be dened by f (x) =
: Obtain formulas for F (x) = 0 f and
:
;
x for 2 x 3
sketch the graphs of f and F: Where is F dierentiable? Evaluate F 0 (x) at all such points.

R x+c
277. If f : R ! R is continuous and c > 0; dene g : R ! R by g(x) = x c f (t)dt. Show that g is
dierentiable on R and nd g 0 (x):
R R
x
1
278. If f : [0; 1] ! R is continuous and 0 f = x f for all x 2 [0; 1]; show that f (x) = 0 for all x 2 [0; 1]:

279. (a) If Z1 and Z2 are null sets, show that Z1 [ Z2 is a null set.
1
S
(b) More generally, if Zn is a null set for each n 2 N; show that
Zn is a null set.
n=1

18


280. Let f; g 2 R[a; b]:
Rb
2
(a) If t 2 R; show that a (tf g)
0:
Rb
Rb 2 1 Rb 2
(b) Use (a) to show that 2 a f g
t a f + t a g for t > 0:
Rb 2
Rb
Rb
Rb
(c) If a f = 0; show that a f g = 0; and if a g 2 = 0; show that a f g = 0:
(d) Prove the following Schwartz Inequality:
Rb
a

fg

Rb
a

jf gj

Rb
a

f2

Rb
a

g2

x
= 0 for all x 2 R; x 0:
281. Show that lim x+n
n!1

nx
282. Show that lim 1+n
2 x2 = 0 for all x 2 R:
n!1

nx
283. Evaluate lim 1+nx
for x 2 R; x 0:
n!1

xn
284. Evaluate lim 1+x
0:
n for x 2 R; x
n!1

285. Evaluate lim sin(nx)


for x 2 R; x 0:
n!1 1+nx

286. Evaluate lim e nx for x 2 R; x 0:


n!1

287. Show that lim xe nx = 0 for x 2 R; x 0:


n!1

288. Show that lim x2 e nx = 0 and lim n2 x2 e nx = 0 for x 2 R; x 0:


n!1
n!1

2n
289. Show that lim (cos x) exists for all x 2 R. What is its limit?
n!1
n

o1
x
290. Show that if a > 0; then the convergence of the sequence x+n
is uniform on [0; a]; but not
n=1

uniform on [0; 1):


n o1

nx
291. Show that if a > 0; then the convergence of the sequence 1+n2 x2
is uniform on [a; 1); but not
n=1

uniform on [0; 1):


n o1

xn
292. Show that if 0 < b < 1; then the convergence of the sequence 1+xn
is uniform on [0; b]; but not
n=1

uniform on [0; 1]:


n

o1
sin(nx)
293. Show that if a > 0; then the convergence of the sequence 1+nx
is uniform on [a; 1); but not
n=1

uniform on [0; 1):

294. Show that the sequence x2 e nx converges uniformly on [0; 1):

1
295. Show that if a > 0; then the sequence n2 x2 e nx n=1 converges uniformly on [a; 1); but it does not
converge uniformly on [0; 1):

296. Show that if (fn ) and (gn ) converge uniformly on the set A to f and g; respectively, then (fn + gn )
converges uniformly on A to f + g:

297. Show that if fn (x) = x + n1 and f (x) = x for x 2 R; then (fn ) converges uniformly on R to f; but the
sequence fn2 does not converge uniformly on R. (Thus, the product of uniformly convergent sequences
of functions may not converge uniformly).

19


298. Let (fn ) and (gn ) be sequences of bounded functions on A that converge uniformly on A to f and g;
respectively. Show that (fn gn ) converges uniformly on A to f g:

299. Let (fn ) be a sequence of functions that converges uniformly to f on A and that satises jfn (x)j M
for all n 2 N and all x 2 A: If g is continuous on the interval [ M; M ]; show that the sequence (g fn )
converges uniformly to g f on A:

xn
does not converge uniformly on [0; 2] by showing that the limit
300. Show that the sequence 1+x
n
function is not continuous on [0; 2]:

301. Construct a sequence of functions on [0; 1] each of which is discontinuous at each point of [0; 1]
and which converges uniformly to a function that is continuous at every point.

302. Suppose (fn ) is a sequence of continuous functions on an interval I that converges uniformly on I to
a function f: If (xn ) I converges to x0 2 I; show that lim (fn (xn )) = f (x0 ) :
n!1

303. Let f : R ! R be uniformly continuous on R and let fn (x) = f (x + n1 ) for x 2 R: Show that (fn )
converges uniformly on R to f:

1
304. Let fn (x) = (1+x)
n for x 2 [0; 1]: Find the pointwise limit f of the sequence (fn ) on [0; 1]:
Does (fn ) converge uniformly to f on [0; 1]?

305. Suppose that the sequence (fn ) converges uniformly to f on the set A; and suppose that each fn
is bounded on A: (That is, for each n there is a constant Mn such that jfn (x)j Mn for all x 2 A):
Show that the function f is bounded on A:

nx
306. Let fn (x) = 1+nx
2 for x 2 A = [0; 1): Show that each fn is bounded on A; but the pointwise limit f
of the sequence is not bounded on A: Does (fn ) converge uniformly to f on A?

n
307. Let fn (x) = xn for x 2 [0; 1]: Show that the sequence (fn ) of dierentiable functions converges uniformly
to a dierentiable function f on [0; 1]; and that the sequence (fn0 ) converges on [0; 1] to a function g; but
that g(1) 6= f 0 (1):

308. Let gn (x) = e nx =n for x 0; n 2 N: Examine the relation between lim (gn ) and lim (gn0 ) :
n!1
n!1
R
2
2
309. Show that lim 1 e nx dx = 0:
n!1

nx
310. Let fn (x) = 1+nx
for x 2 [0; 1]: Show that (fn ) converges nonuniformly to an integrable function
R1
R1
f and that 0 f (x)dx = lim 0 fn (x)dx:
n!1
R
1
311. Let gn (x) = nx(1 x)n for x 2 [0; 1]; n 2 N: Discuss the convergence of (gn ) and 0 gn (x)dx :

312. Let fr1 ; r2 ;


; rn ; g be an enumeration of the rational numbers in [0; 1]; and let
fn (x) =

1 if x 2 fr1 ; r2 ;
; rn g
0
otherwise

Show that fn 2 R[0; 1] for each n 2 N; that f1 (x)

f2 (x)

fn (x)

:
; and that f (x) = lim (fn (x))
n!1

is the Dirichlet function which is not Riemann integrable on [0; 1]:

313. Show that if a convergent series contains only a nite number of negative terms, then it is absolutely
convergent.

20


314. Show that if a series is conditionally convergent, then the series obtained from its positive terms is
divergent and the series obtained from its negative terms is divergent.

1
P
315. If
xn is conditionally convergent, give an argument to show that there exists a rearrangement
n=1

whose partial sums diverge to 1:

1
P
316. Find an explicit expression for the n-th partial sum of
ln 1 n12 and show that this series
n=2

converges to ln 2: Is this convergence absolute?

1
1
P
P
317. (a) If
an is absolutely convergent and (bn ) is a bounded sequence, show that
an bn is absolutely
convergent.

n=1

n=1

(b) Give an example to show that if the convergence of

1
P

an is conditional and (bn ) is a bounded

n=1

1
P
sequence, then
an bn may diverge.
n=1

1
1
P
P
318. Give an example of a convergent series
an such that
a2n is not convergent.
n=1
n=1

1
P
319. Give an example of a divergent series
an with (an ) decreasing and such that lim (nan ) = 0:
n!1
n=1

1
P
an is absolutely convergent.
320. If (an ) is a sequence and if lim n2 an exists in R; show that
n!1
n=1

1
1
P
P
321. If (ank ) is a subsequence of (an ), then the series
ank is called a subseries of
an : Show that
n=1

1
P

n=1

an is absolutely convergent if and only if every subseries of it is convergent.

1
P
1
322. Let a > 0: Show that the series
1 and is convergent if a > 1:
1+an is divergent if 0 < a
n=1

323. Establish the convergence or divergence of the series whose nth term is:
1=2
(a) (n (n + 1))
;
1=2
(b) n2 (n + 1)
;
(c) n!=nn ;
n
(d) ( 1) n= (n + 1) :

1
P
p
324. If a; b > 0; show that
(an + b) converges if p > 1 and diverges if p 1:
n=1

325. Discuss the series whose nth term is:


(a) 3 5 7 n!(2n+1) ;
n=1

(b)
(c)

(n!)2
(2n)! ;
2 4 (2n)
3 5 (2n+1) ;
2 4 (2n)
5 7 (2n+3) :

(d)

326. Show that the series 1 + 21 13 + 14 + 15 16 + +


is divergent.

+ n1 ln n: Show that (cn ) is a decreasing sequence of positive


327. For n 2 N; let cn = 11 + 12 +
numbers. The limit C of this sequence is called Eulers Constant and is approximately equal to 0:577.
Show that if we put
21

bn =

1
1

1
2

1
3

1
2n

then the sequence (bn ) converges to ln 2: [Hint: bn = c2n cn + ln 2:]

328. Test the following series for convergence and for absolute convergence:
1
1
1
1
P
P
P
P
n+1 ln n
( 1)n+1
( 1)n+1
( 1)n+1 n
(a)
(b)
(c)
;
(d)
( 1)
n2 +1 ;
n+1 ;
n+2
n :
n=1
n=1
n=1
n=1

329. Give an example to show that the Alternating Series Test may fail if (an ) is not a decreasing sequence.

330. Consider the series


1

1
2

1
3

1
4

1
5

1
6

1
7

++

where the signs come in pair. Does it converge?

1
1
P
P
an
an
331. Let an 2 R for n 2 N and let p < q: If the series
np is convergent, show that the series
nq is
n=1

n=1

also convergent.

1
p
P
n
332. If p and q are positive numbers, show that
( 1) (lnnn)
is a convergent series.
q
n=2

333. Discuss the series whose nth term is:


n
n
n
n
nn
nn
(a) ( 1) (n+1)
(b) (n+1)
(c) ( 1) (n+1)
(d) (n+1)
n+1 ;
n+1 ;
nn ;
nn+1 :

1
1
P
P
334. If the partial sums of
an are bounded, show that the series
an e nt converges for t > 0:
n=1
n=1

1
1
1
P
P
P
an
sn
335. If the partial sums sn of
an are bounded, show that the series
n converges to
n(n+1) :
n=1
n=1
n=1

336. Show that the hypothesis that the sequence (xn ) is decreasing in the Dirichlets Test can be replaced
1
P
by the hypothesis that
jxn xn+1 j is convergent.
n=1

1
P
337. Show that if the partial sums sn of the series
ak satisfy jsn j M nr for some r < 1; then the
series

1
P

k=1

k=1

ak
k

converges.

1
1
P
P
338. Suppose that
an is a convergent series of real numbers. Either prove that
bn converges or
n=1

n=1

give a counter-example, where


we dene bn by:
p
p
an
an
(b) n (an 0)
(c) ann (an 0);
(a) n ;
(d) n1=n an :

1
P
339. Discuss the convergence and uniform convergence of the series
fn where fn is given by:
n=1

(a) x + n
(b) (nx)
(x 6= 0)
(c) sin(x=n )
(d) (xn + 1)
(x 0)

1
1
P
P
340. If
an is an absolutely convergent series, show that the series
an sin nx is absolutely and
n=1

n=1

uniformly convergent.

1
P
341. Determine the radius of convergence of the power series
an xn , where an is given by:
n=1

(a) 1=npn ;
(f ) n n :

(b) n =n!;

(c) nn =n!;

(d) (ln n)

22

(n

2);

(e) (n!) =(2n)!;


1 if n = k 2
1 if n = k!
342. If an =
and bn =
for k 2 N, nd the radius od convergence
0 otherwise
0 otherwise
1
1
P
P
of the power series
an xn and of
bn xn :
n=1
n=1

1
P
343. If 0 < p jan j q for all n 2 N; nd the radius of convergence of
an xn :
n=1

1
P
344. Let f (x) =
an xn for jxj < R: If f (x) = f ( x) for all jxj < R; show that an = 0 for all n odd.
n=1

345. Prove that if f is dened for jxj < r and if there exists a constant B such that f (n) (x)
B for all
jxj < r and n 2 N; then the Taylor series expansion
1
P

n=1

f (n) (0) n
n! x

converges to f (x) for jxj < r:


R
2
x
346. Find a series expansion for 0 e t dt for x 2 R:
R
347. Let f : R ! R be a Lebesgue integrable function, i.e., R f dm < 1.
(a) Prove that m(fx 2 R : f (x) = 1g) = 0:
(b) Prove that 8 > 0; m(fx 2 R : jf (x)j
g) < 1:
(c) Prove that the Lebesgue integral is absolutely continuous
with respect to the Lebesgue measure, i.e.,
R
if f is Lebesgue integrable on A, then 8 > 0 9 >
0
:
jf
j
dm
< whenever B A with m(B) < :
B
R
(d) Prove that 8 > 0; 9 a compact set K R : RnK jf j dm < :
R
(e) Prove that 8 > 0; 9 M > 0 and a measurable set A R : jf (x)j M on A; and RnA jf j dm < :

348. Show that if f is a Lebesgue integrable function on A and An = fx 2 A : jf (x)j ng; then
lim n m(An ) = 0:
n!1

R
349. Show that if f 0 on a set A, m(A) > 0; and A f dm = 0; then f = 0 a.e. on A.
R
350. Show that if B f dm = 0 for every measurable subset B of A; m(A) > 0; then f = 0 a.e. on A.

R
R
351. Show that if f is Lebesgue integrable on A and A f dm = A jf j dm; then either f 0 a.e. on A,
or f 0 a.e. on A.
R
352. Let ffn g be a sequence of nonnegative and measurable functions on A such that lim A fn dm = 0:
m

n!1

Show that fn ! 0: ( fn ! 0 means fn converges to 0 in measure).

353. Show that in problem 6, convergence in measure cannot be replaced by a.e. convergence.

R
jfn j
354. Let ffn g be a sequence of measurable functions on a set A, m(A) < 1: Show that lim A 1+jfn j dm = 0
n!1

if and only if fn ! 0:

355. Show that the assumption m(A) < 1 is essential in problem 8.

356. Suppose that f is nonnegative and measurable on A, m(A) < 1: Let Ak = fx 2 A : k f (x) < k + 1g:
1
X
Prove that f is Lebesgue integrable on A if and only if
km(Ak ) < 1:
k=0

357. Suppose that f is nonnegative and measurable on A, m(A) < 1: Let Bk = fx 2 A : f (x) kg:
23

Prove that f is Lebesgue integrable on A if and only if

1
X

k=0

m(Bk ) < 1:

1
X
358. Suppose that f is nonnegative and integrable on A, m(A) < 1: For > 0; dene S( ) =
k m(Ak );
k=0
R
where Ak = fx 2 A : k
f (x) < (k + 1) g: Prove that lim S( ) = A f dm:
!0

359. RLet ffn g be


R a sequence of nonnegative functions converging to a functionRf on R; and
R suppose that
lim
fn dm = R f dm < 1: Show that for each measurable set A R; lim A fn dm = A f dm:
n!1 R
n!1

360. Suppose that


R ffn g is a sequence
R of measurable
R functions Ron A such that jfn j g; where g is integrable
on A. Show that A limfn dm lim A fn dm lim A fn dm
limfn dm:
A

361. Suppose that ffRn g is a sequence


of real-valued functions integrable on A, m(A) < 1: Show that
R
if fn
f , then lim A fn dm = A f dm: ( fn
f means fn converges uniformly to f on A).
n!1
A
A

362. Let fn (x) = nxn 1 (n + 1)xn ; x 2 (0; 1):


1
1
X
X
R
R
(a) Show that (0;1)
fn dm 6=
f dm:
(0;1) n
(b) Show that

1
X
R

n=1

(0;1)

n=1

n=1

jfn j dm = 1:

1
X
R
363. Let ffn g be a sequence of measurable functions on A such that
jf j dm < 1:
A n
(a) Show that the function f =
(b) Show that

1
R X
A

n=1

fn dm =

1
X

n=1
1
X

n=1

n=1

fn is Lebesgue integrable on A.

fn dm:

x
R1
sin( )
364. Compute lim 0 (1+ xn)n dx and justify your answer.
n
n!1

R n xnix
365. Compute lim 0 (1 n ) e dx and justify your answer.
n!1

n
for 0 < x < n1
366. Let fn (x) =
:
x<1
0 for n1
(a) Find lim fn (x):
n!1
R1
(b) Find lim 0 fn (x)dx:
n!1

(c) Is there a function g(x) 2 L1 (0; 1) : g(x) fn (x) for all n?

R b
367. (a) Let (a; b) (0; 2 ) be an open interval. Show that lim a cos(nx)dx = 0:
R n!1
(b) Let A (0; 2 ) be a measurable set. Show that lim A cos(nx)dx = 0:
n!1

368. Let fn (x) = 1+nn2 x2 :


(a) Does fn ! 0 a.e. on [0; 1]?
(b) Does fn ! 0 in L1 [0; 1]?
R1
(c) Find lim 0 sinx x fn (x)dx and justify your answer.
n!1

R1
sin x
369. Compute lim
dx and justify your answer.
1 1+nx2
n!1
R
n
370. Let f be a nonnegative Lebesgue integrable function on (0; 1): Prove that lim n1 0 xf (x)dx = 0:
n!1

24


371. Let f be nonnegative and measurable on [0; 1]: Let A = fx 2 [0; 1] : f (x) > 1g:
R1
n
Show that lim 0 (f (x)) dx exists if and only if m(A) = 0:
n!1
R
b
372. Show that if f is Riemann integrable on [a; b] and f (x) = 0 for x 2 [a; b] \ Q; then a f (x)dx = 0:

373. Prove the following Schwartz inequality:


2
Rb
Rb
Rb
2
2
If f and g are Lebesgue integrable on (a; b); then a f (x)g(x)dx
(f (x)) dx
(g(x)) dx :
a
a

374. Let f : [a; b] ! R be nonnegative and Lebesgue integrable.


2
2
2
Rb
Rb
Rb
Prove that a f (x) cos xdx + a f (x) sin xdx
f
(x)dx
:
a

R
x2
for x 2 [0; 1]nQ
375. Find [0;1] f dm; where f (x) =
: Is f Riemann integrable on [0; 1]?
1
for x 2 [0; 1] \ Q

376. Let C be the Cantor set.


8
9
for x 2 [0; 1=2]nC =
< sin( x)
R
cos( x)
for x 2 [1=2; 1]nC
(a) Find [0;1] f dm; where f (x) =
:
:
;
x2
for x 2 C
8
9
0
for x 2 C
>
>
<
=
n 1
2[
(b) Let f be dend on [0; 1] as follows: f (x) =
;where Cn;k is a removed
>
Cn;k >
for x 2
: n
;
k=1
R
interval of length 31n : Find [0;1] f dm:

377. Dene the Cantor-Lebesgue function ' : [0; 1] ! [0; 1] as follows:


1
X
an
If x is an element of the Cantor set C and x =
3n with an = 0 or 2; then we put
n=1

'(x) = '(

1
X

n=1

an
3n )

1
X

an 1
2 2n ;

n=1

that is, if an is the n-th ternary digit for x; then the n-th binary digit for '(x) is
We extend ' to [0; 1] by setting:

an
2 :

'(x) = supf'(y) : y 2 C; y < xg


(a) Show that the Cantor function maps the Cantor set C onto [0; 1].
(b) Show that ' is increasing and continuous on [0; 1]:
(c) Show thatR'0 (x) = 0 a.e. on [0; 1]:
(d) Compute [0;1] 'dm and justify your answer.

1 for x 2 A
378. Let A [0; 1] be a perfect nowhere dense set with m(A) > 0: Show that A (x) =
0 for x 2
=A
is not Riemann integrable on [0; 1] but it is Lebesgue integrable on [0; 1].
X

1
379. Let frn g be the sequence of rational numbers and dene f (x) =
2n :
fn:rn <xg

(a) Show that f is continuous on the irrationals.


(b) Show thatR f is discontinuous on the rationals.
(c) Compute [0;1] f dm and justify your answer.

1
X
1 p 1
380. Let frn g be the sequence of rational numbers in [a; b] and dene f (x) =
:
2n
n=1

25

jx rn j

Rb
(a) Show that a f (x)dx < 1:
Rb
2
(b) Show that a (f (x)) dx = 1:

381. Let fxn g be a sequence in [0; 1] and fan g be a sequence of nonnegative real numbers such that
1
1
X
X
p an
an < 1: Show that
converges for almost all x 2 [0; 1]:

n=1

n=1

jx xn j

382. Suppose f : [0; 1] ! [0; 1) is measurable and satises m(fx 2 [0; 1] : f (x) yg) y(log1 y)2 8 y 2:
Show that f is Lebesgue integrable on [0; 1]:
R
p
383. For p > 0; let B = ff : R ! R such that f is measurable and R jf j dm 1g:
m
Suppose that fn 2 B; and fn ! f on R: Prove that f 2 B:

384. TrueRor False: If f is Lebesgue integrable on R; then there exists a continuous function g : R ! R
such that R jf gj dm = 0:

385. We say that functions fn ; n = 1;


R 2; 3; :::; integrable on A are equi-integrable if 8 > 0 9 > 0 such that
for every measurable subset B of A, B jfn j dm < for n = 1; 2; 3; :::; whenever m(B) < :
Show that ifR ffn g is a convergent
sequence, say fn ! f; of equi-integrable functions on a set A, m(A) < 1;
R
then lim A fn dm = A f dm:
n!1
R R
m
386. (Fatou in measure) Show that if fn 0 and fn ! f on A, then A f dm lim A fn dm:

m
387. (Monotone
Convergence
in measure) Show that if fn 0; fn fn+1 ; and fn ! f on A,
R
R
then lim A fn dm = A f dm:
n!1

m
388. (Lebesgue DCT in measure) Show that if fn ! f on A, and there
g integrable
R exists a function
R
on A such that jfn (x)j g(x) for all n 2 N and for all x 2 A; then lim A fn dm = A f dm:
n!1

389. (Equi-integrability in measure) Show that if ffn g is a sequence of equi-integrable functions on


R
R
m
a set A, m(A) < 1; and fn ! f on A, then lim A fn dm = A f dm:
n!1

a.e.
390. Let f; fn : RR ! R be integrable functions such thatR fn ! f onRR:
Show that lim R jfn f j dm = 0 if and only if lim R jfn j dm = R jf j dm:
n!1
n!1

R jxj
2
391. Use Fubinis theorem to prove that Rn e
dx = n=2 :

R 1
x sin x
1
392. Let a > 1: Prove that 0 1+(nx)
a dx = o( n ):

R1
393. Compute lim 0 (1+ x dx
n x1=n and justify your answer.
n!1
n)

R1
394. Compute lim 0 log(x+n)
e x cos x dx and justify your answer.
n
n!1
R
1
nx
395. (a) Compute lim 0 1+n
2 x2 dx and justify your answer.
n!1
R 1 n3=2 x
(b) Compute lim 0 1+n2 x2 dx and justify your answer.
n!1

1
X
n
p
R =2
396. Compute
1
sin
x
cos xdx and justify your answer.
0
n=0

R 1
397. Compute lim 0 (1 + nx ) n sin( nx )dx and justify your answer.
n!1

R 1 1+nx
398. Compute lim 0 (1+x)n dx and justify your answer.
n!1

26


R1
1 for x = n1
is Riemann integrable on [0; 1] and compute 0 f (x)dx:
399. Show that f (x) =
0 otherwise

400. (Variant of Fatous Theorem) Let fgn g be a sequence of integrable functions on A such that
a.e.
gn ! g in L1 (A): Suppose that ffn g is a sequence
R functions on A
R of measurable
R such that fn ! f on A,
limfn dm = A f dm:
and fn gn a.e. for each n 2 N: Prove that lim A fn dm
A

401. (Variant of Fatous Theorem in measure) Let fgn g be a sequence of integrable functions on A
such that gn ! g in L1 (A): Suppose that ffn g is a sequence of measurable functions on A such that
R
R
m
limfn dm:
fn ! f on A, and fn gn a.e. for each n 2 N: Prove that lim A fn dm
A

402. (Variant of Lebesgues DCT) Let fgn g be a sequence of integrable functions on A such that
a.e.
gn ! g in L1 (A): Suppose that ffn g is a sequence ofR measurableRfunctions on A such that fn ! f on A,
and jfn j gn a.e. for each n 2 N: Prove that lim A fn dm = A f dm:
n!1

403. (Variant of Lebesgues DCT in measure) Let fgn g be a sequence of integrable functions on A
such that gn ! g in L1 (A): Suppose that ffn g is a sequence of measurable functions on A such that
R
R
m
fn ! f on A, and jfn j gn a.e. for each n 2 N: Prove that lim A fn dm = A f dm:
n!1

a.e.
404. Let 1 p < 1; fn 2 Lp (R) such thatR fn ! f on R: Suppose that
p
(i) 9n1 > 0 and A R with m(A) < 1 : RnA jfn j dm 1 8n n1 ; and
R
p
(ii) 9n0 > 0 and 0 < < 1 : whenever m(F) < ; F jfn j dm 1 8n n0 :
Show that f 2 Lp (R):

405. Prove the following Tchebyshevs Inequality:


Let f be nonnegative and measurable on A:
R
1
f dm:
If > 0; then m(fx 2 A : f (x) > g)
A
p

406. Let fn (x) = ne nx on [0; 1]: Prove that:


(a) fn (x) ! 0 pointwise in (0; 1]:
R1
(b) 0 (fn (x))2 dx C for all n 2 N:
(c) fn does not converge in L2 (0; 1):
R1
(d) 0 fn (x)g(x)dx ! 0 for each g 2 L2 (0; 1):

1
407. Let fn (x) =
1=2 on (0; 1): Prove that:
jx n1 j
(a) fn (x) converges pointwise on (0; 1):
(b) fn (x) converges in measure on (0; 1):
(c) fn converges in L1 (0; 1):
(d) @ g 2 L1 (0; 1) : fn (x) g(x) for a.e. x 2 (0; 1) and for all n 2 N:

a.e.
408. Let p 1: Suppose fn 2 Lp (R), supn jfn j 2 Lp (R); and fn ! f in R. Prove that f 2 Lp (R) and
that fn ! f in Lp (R):

409. Let A R be measurable with m(A) < 1: Let ffn g be a sequence of functions in L1 (A) such that
R
m
1=2
fn ! 0 on A, and kfn kL1 (A) M for all n 2 N. Prove that lim A jfn gj dm = 0 for all g 2 L1 (A):
n!1

410. Let f be a measurable function on [0; 1] and let A = fx 2 [0; 1] : f (x) 2 Zg:
R1
2n
Prove that A is measurable and that lim 0 [cos( f (x))] dx = m(A):
n!1

411. Show that if m(A) < 1; and 0 < p1 < p2 < 1; then Lp2 (A) Lp1 (A); and the inequality
kf kp1

kf kp2 (m(A)) p1

1
p2

27

holds for f 2 Lp2 (A):


R
x
1
412. Given f 2 L2 ([0; 1]) dene K(x) = x4=3
f (t)dt:
0
Show that kKk1 C kf k2 where C is a constant independent of f:

413. Recall that the convolution of two integrable functions f and g is dened by:
(f
Let f

0 such that

g) (x) =

f (x

y)g(y)dy:

f (x)dx = A < 1: Dene the sequence fn = f


|

{z

n tim es

f:
}

Prove that fn 2 L1 (R) for each n 2 N, and fn ! 0 in L1 (R):

m
414. Let ffn g be a sequence of measurable functions on A, m(A) < 1; such that fn ! f on A,
and jfnR (x)j C for xR2 A, and n = 1; 2; 3; :::: Show that if g is continuous on [ C; C]; then
lim A g(fn )dm = A g(f )dm:
n!1

m
415. Let ffnRg be a sequence Rof measurable functions on A, m(A) < 1; such that fn ! f on A.
Then lim A sin(fn )dm = A sin(f )dm:
n!1

416. Suppose that g satises a Lipschitz condition on R; i.e., 9C > 0 : 8x; y 2 R; jg(x) g(y)j C jx yj :
Show that if f is Lebesgue integrable on [a; b]; then g(f ) is Lebesgue integrable on [a; b]:

417. Suppose that g satises a Lipschitz condition on R; i.e., 9C > 0 : 8x; y 2 R; jg(x) g(y)j C jx yj :
m
Show that if ffn g is a sequence of measurable functions on [a; b] such that fn ! f and there is a Lebesgue
R
R
integrable function G such that jfn (x)j G(x); then lim
g(fn )dm = [a;b] g(f )dm:
[a;b]
n!1

R
R
p
p
418. Let ffn g be a sequence ofR functions converging to f on A such that RA jfn j dm < 1R and A jf j dm < 1;
p
p
p
1 p < 1: Show that lim A jfn f j dm = 0 if and only if lim A jfn j dm = A jf j dm:
n!1
n!1

a.e.
419. Let ffn g be a sequence of functions in Lp [a; b]; 1 < p < 1; such that fn ! f on [a; b]:
Suppose that 9C > 0 : kfn kp C for n = 1; 2; 3; :::: Prove that 8g 2 Lq [a; b]; p1 + 1q = 1;
R
R
lim
fn gdm = [a;b] f gdm:
[a;b]
n!1

420. Let ffn g be a sequence of functions in L2 (R): Suppose that kfn kL2 (R) M for all n 2 N; and
R
R
a.e.
fn ! f on R: Prove that lim R fn gdm = R f gdm for all g 2 L2 (R):
n!1

421. Let ffn g be a sequence of functions in Lp [a; b]; 1 p < 1; which converges in norm to f 2 Lp [a; b];
a.e.
and let fgn g be a sequence of measurable functions such that jgn j C for n = 1; 2; 3; :::; and gn ! g:
Show that fn gn ! f g in Lp [a; b]:

a.e.
422. Let f; fn : R ! R be measurable functions such that fn ! f on R and there exists a function g
integrable on R such that jfn (x)j g(x) a.e. for all n 2 N. Show that fn ! f almost uniformly on R:
(That is, 8 > 0; 9 a measurable set A R : m(RnA) < and fn
f ):
A

423. Suppose that f is nonnegative and measurable


on a set A, where
R
R 20 < m(A) < 1: Suppose also that
1
1
there are positive and such that m(A)
f
dm
and
:
m(A) A f dm
A
2

Show that if 0 < < 1 and A = fx 2 A : f (x)


g; then m(A ) m(A)(1
)2 :

424. We say that a measurable function f on a set A is essentially bounded if m(fx 2 A : jf (x)j > rg) = 0
for some real number r: In this case, we dene the essential supremum of f by:
kf k1 = inffr : m(fx 2 A : jf (x)j > rg) = 0g:
28

1=p
R
p
Show that if f is essentially bounded on [a; b]; then lim
jf j dm
= kf k1 :
[a;b]
p!1

425. Prove the following Hlders Inequality: If 1 p 1 and p1 + p10 = 1; then kf gk1 kf kp kgkp0 ;
1=p0
R
R
R
R
1=p R
p
p0
that is, A jf gj dm
jf
j
dm
jgj
dm
; and A jf gj dm kf k1 A jgj dm:
A
A

426. (a) Prove the following Minkowskis Inequality: If 1 p 1; then kf + gkp kf kp + kgkp ;
R
R
R
1=p
1=p
1=p
p
p
p
jf j dm
+ A jgj dm
; and kf + gk1 kf k1 + kgk1 :
that is, A jf + gj dm
A
(b) Show that Minkowskis Inequality fails for 0 < p < 1:

1=p
R1
427. Let f : [0; 1] ! R be positive and continuous. For p 6= 0; let N (p) = 0 f p (x)dx
:
(a) Compute lim N (p):
p!1

(b) Compute lim N (p):


p! 1

(c) Compute lim N (p):


p!0

R b
428. Suppose f 2 C 1 ([a; b]); f (a) = f (b) = 0; and a f 2 (x)dx = 1:
Rb
(a) Show that a xf (x)f 0 (x)dx = 12 :
Rb 0
Rb 2 2
2
(b) Show that 41
(f
(x))
dx
x f (x)dx :
a
a
R
R 1
1
429. (a) Find min 0 (1 + x2 )f 2 (x)dx; where A = ff 2 C([0; 1]) : 0 f (x)dx = 1g; and nd a function for
f 2A

which the minimum is attained.


R1
R1
R1
(b) Let A = ff 2 C([0; 1]) : 0 f (x)dx = 3; and 0 xf (x)dx = 2g: Find min 0 f 2 (x)dx; and a function for
f 2A

which the minimum is attained.

430. Show that if f is continuous on [0; 1]; dierentiable on (0; 1); f (0) = 0; and 0 < f 0 (x) 1 on (0; 1);
2
R1
R1
3
then 0 f (x)dx
(f (x)) dx: Show also that equality occurs if and only if f (x) = x:
0

R b f(a)+f
(b)
431. Show that if f is a convex function on [a; b]; then f a+b
(b
a)
f
(x)dx
(b a):
2
2
a
R
b
432. Show that if f 2 C([a; b]) is positive and strictly concave on [a; b]; then a f (x)dx > 12 (b a) max f (x):
x2[a;b]

433. Prove the following Opials inequality: If f is continuously dierentiable on [0; a] and f (0) = 0; then
Ra
Ra
2
jf (x)f 0 (x)j dx a2 0 (f 0 (x)) dx: Show also that the constant a2 is the best possible.
0

434. Let f 0 be measurable on [0; 1] and f n g be a sequence of positive numbers such that n # 0 as n ! 1:
R1
R1
Prove that 0 f (x)dx < 1 if and only if lim n f (x)dx < 1:
n!1

435. Assume that f 2 C 1 ([0; 1)); f 0 is monotonic on [0; 1); and lim f (x) = l < 1:
x!1
R1
Show that for 0 < a < b; 0 f (bx) x f (ax) dx = (l f (0)) ln ab :
R
bx
ax
1
436. (a) Find 0 e x e dx; 0 < a < b:
R1
(b) Find 0 cos(bx)x2cos(ax) dx; 0 < a < b:
r

2
R1p
R1
437. (a) Show that if f 2 C[0; 1] and jf (x)j 1 8x 2 [0; 1]; then 0 1 f 2 (x)dx
1
f (x)dx :
0

1=2
R1
R1
1=2 R 1
2
(b) Show that if f 2 C 1 [0; 1) and lim f (x) = 0; then 0 f 2 (x)dx 2 0 x2 f 2 (x)dx
(f 0 (x)) dx
:
0
x!1

438. Prove the following Hlder inequality: If f1 ; f2 ; :::; fn are nonegative and Riemann integrable on [a; b];

29

and if

1;

2 ; :::;

> 0 are such that

n
X

= 1; then

i=1

Rb
a

n
Y

fi (x) dx

i=1

n
Y
Rb
a

fi (x)dx

i=1

439. Suppose that f and g are nonnegaive and Riemann integrable on [a; b]: For p 6= 0; Let q be its conjugate,
i.e., p1 + 1q = 1: Prove that:
1=p R b
1=q
Rb
Rb p
(a) If p > 1; then a f (x)g(x)dx
f (x)dx
g q (x)dx
:
a
a
1=p R b
1=q
Rb
Rb p
(b) If 0 < p < 1; and f and g are positive, then a f (x)g(x)dx
f (x)dx
g q (x)dx
:
a
a

440. Suppose that f is continuous on [0; 1] and there is a > 0 such that 0 f (x) a2=3 8x 2 [0; 1]:
R1
R1p
Show that if 0 f (x)dx = a; then 0 f (x)dx a2=3 :

441. Prove the following Minkowski inequality for integrals: If f1 ; f2 ; :::; fn are nonnegative and Riemann
integrable on [a; b]; then
1=p
1=p
1=p
Rb
Rb p
Rb
p
(a) If p > 1; then a (f1 (x) +
+ fn (x)) dx
f (x)dx
+
+ a fnp (x)dx
:
a 1
1=p
1=p
1=p
Rb p
Rb
Rb
p
(b) If 0 < p < 1; and fi 0, then a (f1 (x) +
+ fn (x)) dx
f (x)dx
+ + a fnp (x)dx
:
a 1

442. Assume that f1 ; f2 ; :::; fn are nonnegative and Riemann integrable on [a; b]:
Rb
Rb p
Rb
p
(a) If p > 1; then a (f1 (x) +
+ fn (x)) dx
f1 (x)dx +
+ a fnp (x)dx:
a
Rb
Rb p
Rb
p
(b) If 0 < p < 1; then a (f1 (x) +
+ fn (x)) dx
f (x)dx +
+ a fnp (x)dx:
a 1

443. Suppose that f 2 C([0; c]); c > 0; is strictly increasing on [0; c] and f (0) = 0: Let f 1 (x) denote
R f (x) 1
Rx
f (t)dt = xf (x):
the inverse of f: Show that for x 2 [0; c]; 0 f (t)dt + 0

444. Prove the following Young inequality: Suppose that f 2 C([0; c]); c > 0; is strictly increasing on [0; c]
and f (0) = 0: Let f 1 (x) denote the inverse of f:
Rb
Ra
(a) Show that for any a 2 [0; c] and for any b 2 [0; f (c)]; we have 0 f (t)dt + 0 f 1 (t)dt ab:
(b) Show that equality holds in part (a) if and only if b = f (a):

445. Show that for a; b 0; (1 + a) ln(1 + a) (1 + a) + (eb b) ab:

446. Suppose that f is positive and Riemann integrable on [a; b]:


Rb
Rb 1
(a) Show that (b a)2
f (x)dx
dx :
a
a f (x)
Rb
Rb 1
(m+M )2
(b) Show that if 0 < m f (x) M; then a f (x)dx
dx
a)2 :
4mM (b
a f (x)

1
X
447. Given an innite sequence of complex numbers (cn ) ; let f (x) =
cn einx : Show:
(a) If
(b) If

1
X

n=1
1
X

n=1

n=1

jcn j < 1; then f is continuous on R:


n jcn j < 1; then f is continuously dierentiable on R:

Z1
2
448. Show that the function f (t) =
e tx dx is nite and continuous for t 1; dierentiable for t > 1;
1

and satises the dierential equation 2tf 0 (t) + f (t) = 0:

449. Let f be continuous on [0; 1]; and suppose that g is a nonnegative continuous function on R with

30

g(x + 1) = g(x): Show that lim

n!1

Z1
0

01
10 1
1
Z
Z
f (x)g(nx)dx = @ f (x)dxA @ g(x)dxA :
0

450. Let f 2 L1 (R). Show that there exists a sequence xn ! 1 such that xn f (xn ) ! 0:
R
451. Let g : Rd ! R be
= 1; let g (x) = d g(x= ) for > 0; and dene
R nonnegative with Rd g(y)dy
d
f (x) = (g f ) (x) = Rd g (y)f (x y)dy: If f : R ! R is continuous and bounded, then show that
for any compact subset K Rd , f
f as ! 0:
K

31

TEMPLE UNIVERSITY
MATHEMATICS DEPARTMENT
Study Guide for REAL ANALYSIS
The Solutions
By: Ziad Adwan

1. If A and B are sets, show that A B if and only if A \ B = A:

Proof. (=)) Suppose that A B. We want to show that A \ B = A:


Let x 2 A \ B: Then x 2 A and x 2 B: Hence, A \ B A (1) :
On the other hand, if we let x 2 A; then since we are assuming that A B; we deduce that x 2 B:
Hence, x 2 A \ B and thus, A A \ B (2) :
Now, (1) and (2) imply that A \ B = A:
((=) Suppose that A \ B = A: We want to show that A B:
Let x 2 A: Since we are assuming that A \ B = A; we get x 2 A \ B and consequently, x 2 B:

2. Prove the Second De Morgan Law : If A, B, and C are sets, then An(B \ C) = (AnB) [ (AnC) :

Proof. ( ) Let x 2 An(B \ C) and suppose that x 2


= AnB: Since A = (AnB) [ (A \ B) ; we get that
x 2 A \ B: Now, x 2 An(B \ C) and x 2 A \ B =) x 2
= A \ C: Since A = (AnC) [ (A \ C) ; we
get that x 2 AnC:
( ) Note that AnB An(B \ C) and AnC An(B \ C): Thus, (AnB) [ (AnC) An(B \ C):
Hence, An(B \ C) = (AnB) [ (AnC) :

3. For each n 2 N; let An = f(n + 1)k : k 2 Ng:


(a) What is A1 \ A2 ? [
\
(b) Determine the sets fAn : n 2 Ng and fAn : n 2 Ng:

Solution. (a) A1 = f(1 + 1)k : k 2 Ng = f2k : k 2 Ng = f2; 4; 6; 8; 10; :::g; and


A2 = f(2 + 1)k : k 2 Ng = f3k : k 2 Ng = f3; 6; 9; 12; 15; :::g:
Thus,
= f6k : k 2 Ng = A5 :
[A1 \ A2 = f6; 12; 18; 24; :::g \
(b) fAn : n 2 Ng = Nnf1g and fAn : n 2 Ng = ?:

4. Draw diagrams in the plane of the Cartesian products A B for the given sets A and B.
(a) A = fx 2 R : 1 x 2 or 3 x 4g; B = fx 2 R : x = 1 or x = 2g:
(b) A = f1; 2; 3g; B = fx 2 R : 1 x 3g

Solution. (a)

32

(b)

5. Let f (x) = 1=x2 ; x 6= 0; x 2 R:


(a) Determine the direct image f (E) where E = fx 2 R : 1 x 2g:
(b) Determine the inverse image f 1 (G) where G = fx 2 R : 1 x 4g:

Solution. The graph of f (x) = 1=x2 is:

Note that f is even, strictly increasing on ( 1; 0) and strictly decreasing on (0; +1):
(a) Since f is strictly decreasing on [1; 2]; f ([1; 2]) = [f (2); f (1)] = [1=4; 1]:
(b) f 1 (G) = [ 1; 1=2] [ [1=2; 1]:

6. Let g(x) = x2 and f (x) = x + 2 for x 2 R; and let h be the composite function h = g f:
(a) Find the direct image h(E) of E = fx 2 R : 0 x 1g:
(b) Determine the inverse image h 1 (G) where G = fx 2 R : 0 x 4g:

2
Solution. h(x) = g f (x) = g(f (x)) = (x + 2) : The graph of h(x) is:

33

Note that h is strictly decreasing on ( 1; 2) and strictly increasing on ( 2; +1):


(a) Since h is increasing on [0; 1]; h([0; 1]) = [h(0); h(1)] = [4; 9]:
(b) h 1 ([0; 4]) = [ 4; 0]:

7. Let f (x) = x2 for x 2 R; and let E = fx 2 R : 1 x 0g and F = fx 2 R : 0 x 1g:


Show that E \ F = f0g and f (E \ F) = f0g; while f (E) = f (F) = fy 2 R : 0 y 1g:
Hence, f (E \ F) is a proper subset of f (E) \ f (F): What happens if 0 is deleted from the sets E and F?

Proof. E \ F = fx 2 R : 1 x 0 and 0 x 1g = f0g; f (E \ F) = f (0) = 02 = 0:


f (E) = fy = f (x) :

0g = fy = x2 :

0g

f is decreasing on E

fy 2 R : 0

1g:

f is increasing on F

Also, f (F) = fy = f (x) : 0 x 1g = fy = x : 0 x 1g


=
fy 2 R : 0 y 1g:
If 0 is deleted from the sets E and F; we get that E \ F = ?; f (E \ F) = ?; and
f (E) = f (F) = fy 2 R : 0 < y 1g:

8. Show that if f : A ! B and E, F are subsets of A, then f (E[F) = f (E)[f (F) and f (E\F) f (E)\f (F):

Proof. Let us show that f (E [ F) = f (E) [ f (F):


( ) Let y 2 f (E [ F): Then there is an x 2 E [ F such that y = f (x): Since x 2 E [ F; x 2 E or x 2 F:
Hence, f (x) 2 f (E) or f (x) 2 f (F): Thus, y 2 f (E) or y 2 f (F) and so y 2 f (E) [ f (F):
( ) Since E E [ F and F E [ F; we get that f (E) f (E [ F) and f (F) f (E [ F):
Hence, f (E) [ f (F) f (E [ F):
Next, let us show that f (E \ F) f (E) \ f (F):
Let y 2 f (E \ F): Then there is an x 2 E \ F such that y = f (x): Since x 2 E \ F; x 2 E and x 2 F:
Hence, f (x) 2 f (E) and f (x) 2 f (F): Thus, y 2 f (E) and y 2 f (F) and so y 2 f (E) \ f (F):

9. Show that if f : A ! B and G, H are subsets of B, then f 1 (G [ H) = f 1 (G) [ f 1 (H) and


f 1 (G \ H) = f 1 (G) \ f 1 (H):

Proof. Let us show that f 1 (G [ H) = f 1 (G) [ f 1 (H):


( ) Let x 2 f 1 (G [ H): Then there is a y 2 G [ H such that x 2 f 1 (y): Since y 2 G [ H; y 2 G or y 2 H:
Hence, x 2 f 1 (G) or x 2 f 1 (H) and so x 2 f 1 (G) [ f 1 (H):
( ) Since G G [ H and H G [ H; we get that f 1 (G) f 1 (G [ H) and f 1 (H) f 1 (G [ H):
Hence, f 1 (G) [ f 1 (H) f 1 (G [ H):
Next, let us show that f 1 (G \ H) = f 1 (G) \ f 1 (H):
( ) Since G \ H G and G \ H H; we get that f 1 (G \ H)
34

(G) and f

(G \ H)

(H):

Hence, f 1 (G \ H) f 1 (G) \ f 1 (H):


( ) Let x 2 f 1 (G) \ f 1 (H): Then x 2 f 1 (G) and x 2 f 1 (H): Since f is a function, f assigns a unique
element to each x in its domain. Consequently, there is a y 2 G \ H such that x 2 f 1 (y):
But f 1 (y) f 1 (G \ H).

p
10. Show that the function f dened by f (x) = x= x2 + 1; x 2 R; is a bijection of R onto fy : 1 < y < 1g:

Proof. Here is a graph of f (x):

p
p
p
Note that for all x 2 R; jf (x)j = x= x2 + 1 = x2 = x2 + 1 < 1:
Hence, 1 < f (x) < 1 for all x 2 R:
To show that f is one to one, pick 0 < x1 < x2 : Then x21 < x22 =)
x21 +1
x21

x22 +1
x22

x21
2
x1 +1

x22
2
x2 +1

1
x21

>

1
x22

=) 1 +

1
x21

>1+

1
x22

=)
>
=)
<
=) f (x1 ) < f (x2 ): Thus, f is strictly increasing on (0; +1):
Since f is odd, f is also strictly increasing on ( 1; 0) and so f is strictly increasing on R:
Hence, f is one to one on R:
To show that f is onto, pick y 2 ( 1; 1): Then x = p y 2 is a well-dened element in R:
1 y

Moreover, f (x) = f ( p y

1 y

) = y:
2

Therefore, f is a bijection of R onto fy : 1 < y < 1g:

11. (a) Show that if f : A ! B is injective and E A; then f 1 (f (E)) = E: Give an example to show
that equality need not hold if f is not injective.
(b) Show that if f : A ! B is surjective and H B; then f (f 1 (H)) = H: Give an example to show that
equality need not hold if f is not surjective.

Proof. (a) ( ) Let x 2 f 1 (f (E)): Then f (x) 2 f (E) and since f is injective, x 2 E:
( ) Let x 2 E: Then f (x) 2 f (E) and so x 2 f 1 (f (x)) 2 f 1 (f (E)):
Thus, f 1 (f (E)) = E:
Example for (a). Let f (x) = x2 on R and let E = [0; 1]: Then f (E) = [0; 1] but f

(f (E)) = [ 1; 1] 6= E:

(b) ( ) This is clear.


( ) Let x 2 H: Since f is surjective, there exists y 2 A such that f (y) = x:
Now, f (y) 2 H =) y 2 f 1 (H) =) x 2 f (f 1 (H)):
Thus, f (f 1 (H)) = H:
Example for (b). Let A = B = f0; 1g and dene f : A ! B by f (0) = f (1) = 0: Note that f is not surjective.
Then if H = B; we get that f (f 1 (H)) = f (A) = f0g $ H:

35

12. (a) Suppose that f is an injection. Show that f 1 f (x) = x for all x 2 D(f ) and that
f f 1 (y) = y for all y 2 R(f ):
(b) If f is a bijection of A onto B, show that f 1 is a bijection of B onto A.

Proof. (a) This follows immediately from the previous problem with E = fxg and F = fyg:
(b) We have to prove that f

is both injective and surjective.


(a)

f
is injective: Let x; y 2 B: Then f 1 (x) = f 1 (y) =) f (f 1 (x)) = f (f 1 (y)) =) x = y:
f 1 is surjective: Let x 2 A: Then y = f (x) 2 B and so, by (a), f 1 (y) = f 1 (f (x)) = x:
Hence, f 1 is a bijection of B onto A.

13. Let f : A ! B and g : B ! C be functions.


(a) Show that if g f is injective, then f is injective.
(b) Show that if g f is surjective, then g is surjective.

g f is injective
Proof. (a) Let x; y 2 A: Then f (x) = f (y) =) g(f (x)) = g(f (y))
=)
x = y:
Thus, f is injective.
1

(b) Let z 2 C: Since g f is surjective, we can nd x 2 A such that g(f (x)) = z:


Let y = f (x): Then g(y) = z and so g is surjective.

14. Prove that 1=1 2 + 1=2 3 +


+ 1=n(n + 1) = n= (n + 1) for all n 2 N:

Proof. We use induction. The statement is true for n = 1 since 1=1 2 = 1=(1 + 1) = 1=2:
Suppose the statement is true for n 1; i.e., 1=1 2 + 1=2 3 +
+ 1=(n 1)n = (n 1) =n:
Then 1=1 2 + 1=2 3 +
+ 1=n(n + 1) = [1=1 2 + 1=2 3 +
+ 1=(n 1)n] + 1=n(n + 1)
2
1)+n
n3
= [(n 1) =n] + 1=n(n + 1) = n(n
n2 (n+1) = n2 (n+1) = n= (n + 1) :
Thus, by the induction theorem, 1=1 2 + 1=2 3 +
+ 1=n(n + 1) = n= (n + 1) for all n 2 N:

15. Prove that 3 + 11 +


+ (8n 5) = 4n2 n for all n 2 N:

Proof. We use induction. The statement is true for n = 1 since 8(1) 5 = 4(1)2 1 = 3:
Suppose the statement is true for n 1; i.e., 3 + 11 +
+ (8(n 1) 5) = 4(n 1)2 (n 1):
Then 3 + 11 +
+ (8n 5) = [3 + 11 +
+ (8(n 1) 5)] + (8n 5)
2
2
= 4(n 1)
(n 1) + (8n 5) = 4n
n:
Thus, by the induction theorem, 3 + 11 +
+ (8n 5) = 4n2 n for all n 2 N:

16. Prove that 12 22 + 32 42 +


+ ( 1)n+1 n2 = ( 1)n+1 n(n + 1)=2 for all n 2 N:

Proof. We use induction. The statement is true for n = 1 since ( 1)1+1 (1)2 = ( 1)1+1 1(1 + 1)=2 = 1:
Suppose the statement is true for n 1; i.e., 12 22 + 32 42 +
+ ( 1)n (n 1)2 = ( 1)n (n 1)n=2:
Then 12 22 + 32 42 +
+ ( 1)n+1 n2 = 12 22 + 32 42 +
+ ( 1)n (n 1)2 + ( 1)n+1 n2
n
n+1 2
n+1
2
= [( 1) (n 1)n=2]+( 1)
n = ( 1)
n
(n 1)n=2 = ( 1)n+1 (n(n + 1)=2) = ( 1)n+1 n(n+1)=2:
2
2
2
2
Thus, by the induction theorem, 1
2 +3
4 +
+ ( 1)n+1 n2 = ( 1)n+1 n(n + 1)=2 for all n 2 N:

17. Prove that 52n 1 is divisible by 8 for all n 2 N:

Proof. We use induction. The statement is true for n = 1 since 52 1 = 24 = 8 3 is divisible by 8.


Suppose the statement is true for n 1; i.e., 52(n 1) 1 is divisible by 8, say 52(n 1) 1 = 8k for some k 2 N:
Then 52n 1 = 52n 2+2 1 = 52(n 1)+2 1 = 25(52(n 1) ) 1 = 25(52(n 1) 1 + 1) 1 = 25(8k + 1) 1
= 25 (8k) + 24 = 8(25k + 3) is divisible by 8.
Thus, by the induction theorem, 52n 1 is divisible by 8 for all n 2 N:

36

18. Conjecture a formula for the sum of the rst n odd natural numbers 1 + 3 + 5 +
+ (2n 1); and prove
your formula using mathematical induction.

Proof. Since 1 + 2 + 3 + + 2n = (2n)(2n + 1)=2 = n(2n + 1), and 2 + 4 + 6 + + 2n = 2(1 + 2 + 3 + + n)


= 2 [n(n + 1)=2] = n(n + 1); we conjecture that 1 + 3 + 5 +
+ (2n 1) = n(2n + 1) n(n + 1) = n2 for all
n 2 N: To prove this, we use induction. The statement is true for n = 1 since 2(1) 1 = 12 = 1:
Suppose the statement is true for n 1; i.e., 1 + 3 + 5 +
+ (2(n 1) 1) = (n 1)2
Then 1 + 3 + 5 +
+ (2n 1) = [1 + 3 + 5 +
+ (2(n 1) 1)] + (2n 1) = (n 1)2 + (2n 1)
2
2
2n + 1 + (2n 1) = n :
= n
Thus, by the induction theorem, 1 + 3 + 5 +
+ (2n 1) = n2 for all n 2 N:

19. Prove the following Second Version of the Principle of Mathematical Induction: Let n0 2 N and
let P (n) be a statement for each natural number n n0 : Suppose that
(1) The statement P (n0 ) is true.
(2) For all k n0 ; the truth of P (k) implies the truth of P (k + 1):
Then P (n) is true for all n n0 :

Proof. Let S = fn 2 N : P (n0 1 + n) is trueg: Then S is a subset of N with the following properties:
(1) The number 1 2 S: This is because P (n0 1 + 1) = P (n0 ) is true.
(2) For all k 2 N; if k 2 S; then P (n0 1 + k) is true and so P ((n0 1 + k) + 1) = P (n0 1 + (k + 1)) is true.
Hence, k + 1 2 N:
Therefore, the principle of mathematical induction implies that S = N:
That is, P (n0 1 + n) is true for all n 2 N and so, P (n) is true for all n n0 :

20. Prove that 2n < n! for all n 4; n 2 N:

Proof. We use induction. The statement is true for n = 4 since 24 = 16 < 24 = 4!:
Suppose the statement is true for n 1; i.e., 2n 1 < (n 1)!: (Here, n 5):
Then 2n = 2(2n 1 ) < 2 ((n 1)!) < n ((n 1)!) = n!:
Thus, by the induction theorem, 2n < n! for all n 4; n 2 N:

21. Prove that a nonempty set T1 is nite if and only if there is a bijection from T1 onto a nite set T2 :

Proof. (=)) Suppose that T1 is nite. We need to show that there is a nite set T2 and a bijection
' : T1 ! T2 : Take T2 = T1 and ' = identity. Then T2 is nite and ' is a bijection from T1 onto T2 :
((=) Suppose that there is a bijection ' from T1 onto a nite set T2 : Write T2 = fy1 ; y2 ; :::; yn g:
Consider the set A = f' 1 (y1 ); ' 1 (y2 ); :::; ' 1 (yn )g T1 : We will show that A = T1 :
For this, let x 2 T1 : Then '(x) 2 T2 and so '(x) = yj for some 1 j n:
Since ' is bijective, we get that x = ' 1 ('(x)) = ' 1 (yj ): Thus, T1 A:
Hence, T1 = A and so T1 is nite.

22. (a) Prove that if A is a set with m 2 N elements and C A is a set with 1 element, then AnC is
a set with m 1 elements.
(b) Prove that if C is an innite set and B is a nite set, then CnB is an innite set.

Proof. (a) Write A = fa1 ; a2 ; :::; am g and let C A be a set with 1 element.
Then C = faj g for some 1 j m:
Now, AnC = fx 2 A : x 2
= Cg
= fx 2 A : x 6= aj g
= fa1 ; a2 ; :::; aj 1 ; aj+1 ; :::; am g is a set with m 1 elements.
(b) Let C be an innite set and B be a nite set. Write B = fb1 ; b2 ; :::; bm g:
We proceed by contradiction. Suppose that CnB is a nite set, say CnB = fc1 ; c2 ; :::; cn g:
Then C = CnB [ B = fc1 ; c2 ; :::; cn ; b1 ; b2 ; :::; bm g is a set with n + m elements
37

This means that C is a nite set, a contradiction!


Therefore, C is an innite set.

23. Exhibit a bijection between N and a proper subset of itself.

Solution. Let A = f2; 4; 6; 8; :::g = f2n : n 2 Ng N be the set of all even natural numbers.
Since 1 2
= A; A is a proper subset of N.
Now, dene ' : A ! N as follows:
'(2n) = n;

for n 2 N:

We claim that ' is bijective, and here is the proof:


(i) ' is injective. Let x 6= y 2 A; say x = 2n1 and y = 2n2 where n1 ; n2 2 N:
Then 2n1 6= 2n2 =) '(x) = n1 6= n2 = '(y): Thus, ' is injective.
(ii) ' is surjective. Let n 2 N, and let x = 2n: Then '(x) = '(2n) = n: Thus, ' is surjective.
Therefore, ' is bijective and we are done.

24. Prove that a set T1 is denumerable if and only if there is a bijection from T1 onto a denumerable set T2 .

Proof. (=)) Suppose that T1 is denumerable. We need to show that there is a denumerable set T2 and
a bijection ' : T1 ! T2 : Take T2 = T1 and ' = identity. Then T2 is denumerable and ' is a bijection
from T1 onto T2 :
((=) Suppose that there is a bijection ' from T1 onto a denumerable set T2 : Write T2 = fy1 ; y2 ; :::; yn ; :::g:
Consider the set A = f' 1 (y1 ); ' 1 (y2 ); :::; ' 1 (yn ); :::g T1 : Then A is a denumerable subset of T1 .
We will show that A = T1 : For this, let x 2 T1 : Then '(x) 2 T2 and so '(x) = yj for some j 2 N.
Since ' is bijective, we get that x = ' 1 ('(x)) = ' 1 (yj ): Thus, T1 A:
Hence, T1 = A and so T1 is denumerable.

Alternatively for ((=), since T2 is denumerable, there exists a bijection : N ! T2 and since we have
a bijection ' : T2 ! T1 ; we get that the composition '
: N ! T1 is a bijection.
Thus, T1 is denumerable.

25. Give an example of a countable collection of nite sets whose union is not nite.

Solution. Let A1 = f1g; A2 = f2g; A3 = f3g; :::; An = fng; ::::


1
1
[
[
Then each An is nite. Yet,
An =
fng = f1; 2; 3; :::g = N is innite.
n=1

n=1

26. Prove in detail that if S and T are denumerable, then S [ T is denumerable.

Proof. Write S = fs1 ; s2 ; :::; sn ; :::g and T = fr1 ; r2 ; :::; rn ; :::g:


Let A = S \ T: We have three cases.
Case 1 T nA is innite. Write T nA = ft1 ; t2 ; :::; tn ; :::g; and note that S \ (T nA) = ?:
Then S [ T = S [ (T nA) = fs1 ; t1 ; s2 ; t2 ; :::; sn ; tn ; :::g:
Dene ' : N ! S [ T as follows:
'(n) =

s(n+1)=2
tn=2

if n is odd
if n is even

38

Since S \ (T nA) = ?; ' is injective, and it is pretty clear that ' is surjective. Hence, ' is bijective.
Case 2 T nA is nite. Write T nA = ft1 ; t2 ; :::; tm g; and note that S \ (T nA) = ?:
Then S [ T = S [ (T nA) = ft1 ; t2 ; :::; tm ; s1 ; s2 ; :::; sn ; :::g:
Dene ' : N ! S [ T as follows:
'(n) =

tn
sn

if n = 1; 2; :::; m
if n m + 1

Since S \ (T nA) = ?; ' is injective, and it is pretty clear that ' is surjective. Hence, ' is bijective.
Case 3 T nA = ?. In this case, T = S and so S [ T = S is denumerable.

27. Use mathematical induction to prove that if a set S has n elements, then P(S) has 2n elements.

Proof. If S has 1 element, say S = fsg; then P(S) = f?; fsg = Sg has 21 elements.
Suppose that the result is true for any set with n 1 elements, that is, if a set has n 1 elements, then
its power set has 2n 1 elements.
Let S be a set with n elements, say S = fs1 ; s2 ; :::; sn g; and let A = fs1 ; s2 ; :::; sn 1 g: Then S = A [ fsn g:
By the induction hypothesis, P(A) has 2n 1 elements, say P(A) = fA1 = ?; A2 ; A3 ; :::; A2n 1 1 ; A2n 1 = Ag:
Now, P(S) = P(A) [ fA1 [ fsn g; A2 [ fsn g; :::; A2n 1 1 [ fsn g; A2n 1 [ fsn gg
= P(A) [ ffsn g; A2 [ fsn g; :::; A2n 1 1 [ fsn g; Sg has 2n 1 + 2n 1 = 2n elements.
Thus, by the induction theorem, we conclude that if a set S has n elements, then P(S) has 2n elements.
Another elementary proof. Let S = fs1 ; s2 ; :::; sn g: Then an element in P(S) is a subset of S:
We adopt the following convention: For each A 2 P(S); we assign the indicator ( 1 ; 2 ; :::; n );
0
if sj 2
=A
where j =
: For example, (0; 0; :::; 0) is assigned to ?; (1; 1; :::; 1) is assigned to S;
1
if sj 2 A
(1; 0; 1; 0; 0; :::; 0) is assigned to fs1 ; s3 g; and so on. Let I be the set of all possible indicators.
It is easy to check that the function ' : P(S) ! I is a bijection, and so P(S) is nite and has the same number
of elements as the set I: But I has |2 2 2{z
2} = 2n elements.
n tim es

28. Prove that if a; b 2 R; then


(a) (a + b) = ( a) + ( b) ;
(b) ( a) ( b) = a b;
(c) 1=( a) = (1=a);
(d) (a=b) = ( a)=b if b 6= 0:

Proof. (a) We have (a + b) + (( a) + ( b)) = a + b + ( a) + ( b) = a + ( a) + b + ( b)


= (a + ( a)) + (b + ( b)) = 0 + 0 = 0: Thus, (a + b) = ( a) + ( b) :

(b) We have ( a) ( b) = ( 1 a) ( 1 b) = ( 1) a ( 1) b = (( 1) ( 1)) (a b)


= 1 (a b) = a b: (Note that 1 = 1 ( 1) = ( 1) ( 1) and so ( 1) ( 1) = ( 1) = 1):
(c) Since ( 1) ( 1) = 1; we get that
= (1 a1 ) = ( a1 ):

1=

1
( 1) :

Thus,

1
( a)

1
( 1) a

1
( 1)

1
a

1
a

a
(d)
1 ab = 1 a 1b = ( 1 a) 1b = ( a) 1b = ( ba) :
b =

29. Solve the following equations, justifying each step by referring to an appropriate property or theorem.
(a) 2x + 5 = 8;
(b) x2 = 2x;
(c) x2 1 = 3;
(d) (x 1)(x + 2) = 0:

Solution. (a) 2x + 5 = 8 =) 2x + 5 + ( 5) = 8 + ( 5) =) 2x + 0 = 8 5 =) 2x = 3 =) ( 12 ) 2x = ( 12 ) 3

39

=) 1 x =

3
2

=) x = 32 :

(b) x2 = 2x , x2 + ( 2x) = 2x + ( 2x) =) x2

2x = 0 =) x (x + ( 2)) = 0 =) x = 0 or x =

(c) x2 1 = 3 =) x2 1 + ( 3) = 3 + ( 3) =) x2
=) x 2 = 0 or x + 2 = 0 =) x = 2 or x = 2:

4 = 0 =) (x

( 2) = 2:

2) (x + 2) = 0

(d) (x 1)(x + 2) = 0 =) x 1 = 0 or x + 2 = 0 =) x = 1 or x = 2:

30. If a 2 R satises a a = a; prove that either a = 0 or a = 1:

Proof. a a = a =) (a a) + ( a) = a + ( a) =) (a a) + (a ( 1)) = 0 =) a (a + ( 1)) = 0


=) a (a 1) = 0 =) a = 0 or a 1 = 0 =) a = 0 or a = 1:

31. Show that there does not exist a rational number t such that t2 = 3:

Proof. Suppose, to the contrary, that there is a rational number t such that t2 = 3:
Write t = pq ; where p and q 6= 0 are relatively prime.

Then t2 = 3 =) ( pq )2 = 3 =) p2 = 3q 2 =) p2 is divisible by 3
2

p =3q

see rem ark b elow

=)

p is divisible by 3

( )

=) p = 3k for some k 2 Z =) 9k 2 = 3q 2 =) q 2 = 3k 2 =) q 2 is divisible by 3 =) q is divisible by 3


Thus, ( ) and ( ) =) p and q are not relatively prime, a contradiction. Hence, t cannot be rational.

Remark. Let us show that if p is not divisible by 3, then so is p2 .


Proof. p is not divisible by 3 =) p = 3k + 1 or p = 3k + 2 for some k 2 Z
=) p2 = 9k 2 + 6k + 1 or p2 = 9k 2 + 12k + 4 for some k 2 Z
=) p2 = 3(3k 2 + 2k) + 1 or p2 = 3(k 2 + 4k + 1) + 1 for some k 2 Z
=) p2 is not divisible by 3.
Thus, if p2 is divisible by 3, then so is p.

32. (a) Show that if x; y are rational numbers, then so are x + y and xy:
(b) Prove that if x is a rational number and y is an irrational number, then x + y is an irrational number.
If, in addition, x 6= 0; then show that xy is an irrational number.

Proof. (a) Suppose that x = pq and y = rs are rational numbers. Of course, q; s 2 Znf0g:
Then qs 2 Znf0g; pr 2 Z; and ps + qr 2 Z:
2 Q; and xy = pq rs = pr
Thus, x + y = pq + rs = ps+qr
qs
qs 2 Q:
(b) Suppose to the contrary that x + y is a rational number. Then, since x is rational, so is x.
Thus, by (a), y = (x + y) + ( x) is a rational number, a contradiction. Thus, x + y is an irrational number.
For the second statement, suppose to the contrary that xy is a rational number.
Then, since x is a nonzero rational number, so is x1 .
Thus, by (a), y = ( x1 )(xy) is a rational number, a contradiction. Thus, xy is an irrational number.
p
33. Let K = fs + t 2 : s; t 2 Qg: Show that K satises the following:
(a) If x1 ; x2 2 K; then x1 + x2 2 K and x1 x2 2 K:
(b) If x 6= 0 and x 2 K; then 1=x 2 K:
(Thus, the set K is a subeld of R. With the order inherited from R, the set K is an ordered eld that lies
between Q and R).

Proof. Before we start, keep the results of the previous problem in mind.

40

p
p
(a) Let x1 ; x2 2 K; say x
p1 = s1 + t1 2;pand x2 = s2 + t2 2 for s1p; t1 ; s2 ; t2 2 Q:
Then x1 + x2 = s1 + t1 2 + s2 + t2 2 = (s1 + s2 ) + (t1 + t2 ) 2 2 K since s1 + s2 ; t1 + t2 2 Q:
p
p
p
p
p 2
Also, x1 x2 = s1 + t1 2 s2 + t2 2 = s1 s2 + s1 t2 2 + s2 t1 2 + t1 t2
2
p
= (s1 s2 + 2t1 t2 ) + (s1 t2 + s2 t1 ) 2 2 K since s1 s2 + 2t1 t2 ; s1 t2 + s2 t1 2 Q:

p
(b) Let 0 6= x 2 K: Then x = s + t 2 for
p s; t 2 Q and 1s and t are not simultaneously 0.
1
2 2 K since 2t 2 Q:
If s = 0 and t 6= 0; then x1 = tp1 2 = 2t
1
1
If s 6= 0 and t = 0; then x = s 2 K since 1s 2 Q:
p
p
p
t
If s 6= 0 and t 6= 0; then x1 = s+t1p2 = s+t1p2 ss ttp22 = ss2 t2t22 = s2 s2t2 + s2 2t
2:
2
p
s
t
2
2
2
2
Note that s
2t 6= 0 since 2 2
= Q: Also, s
2t 2 Q and so s2 2t2 and s2 2t2 2 Q:
p
t
2 2 K:
Thus, x1 = s2 s2t2 + s2 2t
2

34. (a) If a < b and c d; prove that a + c < b + d:


(b) If 0 < a < b and 0 c d; prove that 0 ac bd:

Proof. (a) Since a < b, a + c < b + c: Also, since c d; b + c b + d: Thus, a + c < b + d:


(b) We rst show that if 0 < a < b; then a1 > 1b :
1
1
Since ab > 0, we get that ab
> 0 (because ab ( ab
) = 1 > 0 and ab > 0)
1
1
1
Thus, 0 < a < b =) 0 < ab a < ab b =) b < a1 :
Now, since c 0; c ( 1b ) c ( a1 ) d ( a1 ) =) (a b) c ( 1b ) (a b) d ( a1 )
=) b ( 1b ) a c (a ( a1 )) b d =) 1 a c 1 b d =) ac bd:
Since c 0 and a > 0; we have that ac 0:
Thus, 0 ac bd:

35. If a; b 2 R; show that a2 + b2 = 0 if and only if a = b = 0:

Proof. (=)) Suppose that a 6= 0: Then a2 > 0 and since b2 0; we get that a2 + b2 > 0:
Similarly, if b 6= 0; then a2 + b2 > 0: Thus, if a2 + b2 = 0; then a = b = 0:
((=) If a = b = 0; then a2 + b2 = 02 + 02 = 0 + 0 = 0:
Thus, a2 + b2 = 0 if and only if a = b = 0:

36. If 0 a < b; show that a2 ab < b2 : Show by example that it does not follow that a2 < ab < b2 :

b a>0
Proof. a2 ab. This follows because ab a2 = a(b a) > a 0; and so ab a2 :
ab

b2 . This follows because b2

ab = b(b

a)

b a>0

>

b > 0; and so ab < b2 :

Example. Let a = 0 and b = 1: Then a2 = 02 = 0; ab = 0 1 = 0; and b2 = 12 = 1:


Thus, in this case, a2 = ab:

37. Find all real numbers x that satisfy the following inequalities.
(a) x2 > 3x + 4;
(b) 1 < x2 < 4;
(c) 1=x < x;
(d) 1=x < x2 :

Proof. (a) x2 > 3x + 4 () x2 3x 4 > 0 () (x 4)(x + 1) > 0


() Either (x 4) > 0 and (x + 1) > 0;or (x 4) < 0 and (x + 1) < 0
() Either x > 4 and x > 1; or x < 4 and x < 1
() Either x > 4, or x < 1
() x 2 Rn[ 1; 4]:
41

(b) 1 < x2 < 4 () x2 > 1 and x2 < 4 () x2 1 > 0 and 4


() (x 1) (x + 1) > 0 and (2 x) (2 + x) > 0:

x2 > 0

Now, (x 1) (x + 1) > 0 () Either (x 1) > 0 and (x + 1) > 0;or (x


() Either x > 1 and x > 1; or x < 1 and x < 1
() Either x > 1, or x < 1
() x 2 Rn[ 1; 1]:

1) < 0 and (x + 1) < 0

Also, (2 x) (2 + x) > 0 () Either (2 x) > 0 and (2 + x) > 0;or (2


() Either x < 2 and x > 2;or x > 2 and x < 2
() x 2 ( 2; 2) [ ? = ( 2; 2) :

x) < 0 and (2 + x) < 0

Note that (Rn[ 1; 1]) \ ( 2; 2) = ( 2; 1) [ (1; 2): Thus, 1 < x2 < 4 () x 2 ( 2; 1) [ (1; 2):
(c) If x > 0; then

1
x

< x () x2 > 1 () x2

If x < 0; then x1 < x () 1 > x2 () 1


Thus, x1 < x () x 2 (1:1) [ ( 1; 0):

(b)

1 > 0 () x 2 (1:1):
as in (b)

x2 > 0 () x 2 ( 1; 0):

(d) Note that this is true for all x < 0 since x2 > 0:
x2 +x+1>0, for x>0

If x > 0; then x1 < x2 () x3 1 > 0 () (x 1)(x2 + x + 1) > 0


()
x 1>0
() x > 1 () x 2 (1; 1):
Thus, x1 < x2 () x < 0 or x > 1 () x 2 Rn[0; 1]:

38. Let a; b 2 R; and suppose that for every " > 0 we have a b + ": Show that a b:

Proof. Suppose to the contrary that a > b: Then if we take "0 = 12 (a b) > 0; we have b < b + "0 < a:
Thus, there is an "0 > 0 such that a > b + "0 ; a contradiction. Hence, a > b is false and so a b:

39. (a) If 0 < c < 1; show that 0 < c2 < c < 1:


(b) If 1 < c; show that 1 < c < c2 :

Proof. (a) Since c > 0 and c < 1; c c < 1 c and so c2 < c: Also, since c > 0; c2 > 0 c = 0:
Thus, 0 < c2 < c < 1:
(b) Since c > 1; c c > 1 c and so c2 > c: Thus, 1 < c < c2 :

n
40. Use mathematical induction to show that if a 2 R and m; n 2 N; then am+n = am an and (am ) = amn :

n
Proof. Fix m 2 N:We will prove that if a 2 R and n 2 N; then am+n = am an and (am ) = amn :
We use mathematical induction for both arguments simultaneously.
For n = 1; am+1 = am a1 and (am )1 = am 1 :
n 1
Suppose the argument is true for n 1; that is, am+(n 1) = am an 1 and (am )
= am(n 1) :
n
n
1
Then am+n = am+(n 1) a = am an 1 a = am an ; and (am ) = (am )
(am ) = am(n 1) am = amn :
m+n
m n
m n
mn
Thus, by the induction theorem a
= a a and (a ) = a
for all n 2 N:
n
Similarly, if we x n 2 N, then we get that am+n = am an and (am ) = amn for all m 2 N:
n
Thus, for any m; n 2 N; am+n = am an and (am ) = amn :

41. If a; b 2 R; show that ja + bj = jaj + jbj if and only if ab 0:

Proof. (=)) If ab < 0; then either a < 0 and b > 0; or a > 0 and b < 0:
If a < 0 and b > 0; we get that jaj = a and jbj = b: Thus, jaj + jbj = a + b > b > ja + bj :
If a > 0 and b < 0; we get that jaj = a and jbj = b: Thus, jaj + jbj = a b > a > ja + bj :
42

((=) ab 0 =) a 0 and b 0; or a 0 and b 0:


If a 0 and b 0; then a + b 0 and so jaj = a; jbj = b; and ja + bj = a + b:
Thus, ja + bj = a + b = jaj + jbj :
If a 0 and b 0; then a + b 0 and so jaj = a; jbj = b; and ja + bj = (a + b) = a b:
Thus, ja + bj = a b = jaj + jbj :

42. If x; y; z 2 R and x z; show that x y z if and only if jx yj + jy zj = jx zj :


Interpret this geometrically.

Proof. (=)) x y z =) y x 0; z y 0; z x 0 =) jx yj = y x; jy zj = z y; jz xj = z x
=) jx yj + jy zj = y x + z y = z x = jx zj :
((=) Suppose jx yj + jy zj = jx zj : Since x z; we have that jx zj = z x:
Let y < x: Then y < z; and so jx yj + jy zj = x y + z y 6= z x = jx zj (or else, y = x):
Let y > z: Then y > x; and so jx yj + jy zj = y x + y z 6= z x = jx zj (or else, y = z):
Thus, x y z:

43. Show that jx aj < if and only if a


<x<a+ :

Proof. jx aj < if and only if


< x a < if and only if a
<x<a+ :

44. If a < x < b and a < y < b; show that jx yj < b a: Interpret this geometrically.

Proof. Suppose without loss of generality that a < x < y < b:


y<b

Since a < x; we get that x < a: Thus, jx yj = y x < b x < b a:


Geometrically, this means that if two points lie between two xed points, then the distance between
the two points in the middle is less than the distance between the two xed points.

45. Find all x 2 R that satisfy the following inequalities:


(a) j4x 5j 13;
(b) x2 1
3:

Solution. (a) j4x 5j 13 =) 13 4x 5 13 =) 8 4x 18 =) 2 x 4:5


=) x 2 [ 2; 4:5]:
x2 0

(b) x2 1
3 =) 3 x2 1 3 =) 2 x2 4 =) x2 4 =) jxj 2 =) 2 x 2
=) x 2 [ 2; 2]:

46. Find all x 2 R that satisfy both j2x 3j < 5 and jx + 1j > 2 simultaneously.

Proof. j2x 3j < 5 and jx + 1j > 2 =) 5 < 2x 3 < 5 and fx + 1 > 2 or x + 1 < 2g
=) 2 < 2x < 8 and fx > 1 or x < 3g =) 1 < x < 4 and fx > 1 or x < 3g
=) x 2 ( 1; 4) \ f(1; 1) [ ( 1; 3)g =) x 2 f( 1; 4) \ (1; 1)g [ f( 1; 4) \ ( 1; 3)g
=) x 2 f(1; 4)g [ f?g = (1; 4):

47. Determine and sketch the set of pairs (x; y) 2 R R that satisfy:
(a) jxj = jyj ;
(b) jxj + jyj = 1;
(c) jxyj = 2;
(d) jxj jyj = 2:

Solution. (a) jxj = jyj =) x = y or x = y =) f(x; y) 2 R R : jxj = jyjg


= f(x; x) 2 R Rg [ f(x; x) 2 R Rg: This is sketched below:

(b) jxj + jyj = 1: This is sketched below:

43

(c) jxyj = 2 =) xy = 2 or xy = 2 =) y = x2 or y = x2
=) f(x; y) 2 R R : jxyj = 2g = f(x; x2 ) 2 R Rg [ f(x;
This is sketched below:
x
jyj = 2: jxj =
x
8
x y
>
>
<
x+y
Hence, jxj jyj =
x y
>
>
:
x+y
(d) jxj

x 0
x<0
if
if
if
if

x 0
x 0
x<0
x 0

and jyj =
and
and
and
and

9
y 0 >
>
=
y<0
:
y 0 >
>
;
y<0

y
y

2
x)

2R

y 0
y<0

Rg:

: Thus,

jyj =

Thus, f(x; y) 2 R R : jxj jyj = 2g = f(x; y) 2 R+ R+ : x y = 2g [ f(x; y) 2 R+


[f(x; y) 2 R
R+ : x y = 2g [ f(x; y) 2 R
R : x + y = 2g:
This is sketched below:

y
y

y 0
y<0

R : x + y = 2g

48. Let > 0 and > 0; and a 2 R: Show that V (a) \ V (a) and V (a) [ V (a) are
neighborhoods
of a for appropriate values of :

Proof. V (a) \ V (a) = fx 2 R : a


< x < a + g \ fx 2 R : a
<x<a+ g
= (a
; a + ) \ (a
; a + ) = (a
; a + ) where = minf ; g:
Thus, V (a) \ V (a) is a
neighborhood of a:
V (a) [ V (a) = fx 2 R : a
< x < a + g [ fx 2 R : a
<x<a+ g
= (a
; a + ) [ (a
; a + ) = (a
; a + ) where = maxf ; g:
Thus, V (a) [ V (a) is a
neighborhood of a:

49. Show that if a; b 2 R; and a 6= b; then there exists


neighborhoods U of a and V of b such that
U \ V = ?:

Proof. Since a 6= b; either a < b or b < a: Say a < b:Then b a > 0 and so we can nd an > 0
such that 0 < < b a: Let = 2 ; and consider the neighborhoods U = V (a) of a and V = V (b)
of b: Now, 0 < < b a =) 0 < + < b a =) a + < b
:
Since V (a) = (a
; a + ); V (b) = (b
; b + ); and a + < b
; we get that
(a
; a + ) \ (b
; b + ) = ?:

50. Let S1 = fx 2 R : x 0g: Show that the set S1 has lower bounds, but no upper bounds.
Show that inf S1 = 0:

Proof. Any number y 0 is a lower bound for S1 : An upper bound for S1 , if it exists, must be positive.
But if we add 1 to this supposedly upper bound, then we are still in S1 : Thus, S1 has no upper bounds.
Now, 0 is a lower bound for S1 since s 0 for all s 2 S1 : Moreover, if y is a lower bound for S1 ; then
y 0: (Note that y cannot be positive because then y0 = y y2 is positive, y0 2 S1 ; and y0 < y):
Thus, inf S1 = 0:

51. Let S2 = fx 2 R : x > 0g: Does S2 have lower bounds? Does S2 have upper bounds?
Does inf S2 exist? Does sup S2 exist? Prove your statements.

Solution. Any number y 0 is a lower bound for S2 : Also, S2 cannot have upper bounds by the same
reasoning as in the previous problem. As in the previous problem, inf S2 = 0: Since S2 does not have any
upper bounds, sup S2 does not exist.

44

52. Let S3 = f1=n : n 2 Ng: Show that sup S3 = 1 and inf S3 0:

Proof. 1 2 S and if we choose any v < 1; we get that 1 is an element of S for which v < 1:
Thus, sup S3 = 1:
Now, 0 < 1=n for all n 2 N: hence, 0 is a lower bound for S3 : Moreover, if t is any lower bound for S3 ;
then t 0 ( If t > 0; then we can nd n large enough so that 1=n < t and so t is not a lower bound).
Hence, inf S3 = 0:

53. Let S be a nonempty subset of R that is bounded below. Prove that inf S = supf s : s 2 Sg:

Proof. Since S is bounded below, u = inf S exists and v = sup( S) exists.


Thus, s v for all s 2 S and so s
v for all s 2 S: Since u = inf S; we get that v u ( ) :
Also, s u for all s 2 S and so s
u for all s 2 S. Since v = sup( S); we get that v
u and
( )
so v u
:
Now, ( ) and ( ) =) u = v:

54. If a set S R contains one of its upper bounds, show that this upper bound is the supremum of S:

Proof. Let u 2 S be an upper bound for S: Then if v is any upper bound for S; we get that s v for all
s 2 S: In particular, u v: Hence, u is an upper bound of S and u v for any upper bound v of S.
Therefore, sup S = u:

55. Let S R be nonempty. Show that u 2 R is an upper bound of S if and only if the conditions t 2 R
and t > u imply that t 2
= S:

Proof. (=)) Suppose that u 2 R is an upper bound of S: Then s u for all s 2 S:


Consequently, for any t 2 R such that t > u; we have s u < t for all s 2 S: Hence, t 2
= S:
((=) Suppose that the conditions t 2 R and t > u imply that t 2
= S: Let s 2 S: Then s u or else s 2
= S:
That is, if s 2 S; then s u: Thus, u is an upper bound of S.

56. Let S R be nonempty. Show that if u = sup S; then for every number n 2 N the number u 1=n
is not an upper bound of S; but the number u + 1=n is an upper bound of S:

Proof. u 1=n < u = sup S =) there is v 2 S such that v > u 1=n: Hence, for every number n 2 N
the number u 1=n is not an upper bound of S:
Now, since u = sup S; u is an upper bound for S and so s u for all s 2 S: Consequently, since u < u + 1=n
for all n 2 N, we get that s u < u + 1=n for all s 2 S and for all n 2 N:

57. Let S be a bounded set in R and let S0 be a nonempty subset of S:


Show that inf S inf S0 sup S0 sup S:

Proof. Since S is bounded and S0 is a nonempty subset of S; we get that S0 is also bounded.
Thus, u = inf S; v = sup S; u0 = inf S0 ; and v0 = sup S0 all exist.
Now, s u for all s 2 S and so, in particular, s u for all s 2 S0 :
Hence, by denition of inmum, we get that u0 u:
Since u0 s v0 for all s 2 S0 ; we get that u0 v0 :
Now, s v for all s 2 S and so, in particular, s v for all s 2 S0 :
Hence, by denition of supremum, we get that v0 v:
Thus, u u0 v0 v:

58. Let S R and suppose that s = sup S 2 S: If u 2


= S; show that sup(S [ fug) = supfs ; ug:

Proof. Let S # = S [ fug: We consider two cases:


45

Case 1 . s

u: In this case, s

Case 2 . s > u: In this case, s

u for all s 2 S # and since u 2 S # ; we get that u = sup S # :


s for all s 2 S # and since s 2 S # ; we get that s = sup S # :

Thus, cases (1) and (2) =) sup(S # ) = supfs ; ug:

59. Use the previous exercise and mathematical induction to show that a nonempty nite set S R
contains its supremum.

Proof. If S = fsg contains only one element, then sup S = s 2 S:


Suppose the result is true for n 1; and let S be a set which contains n elements, say S = fs1 ; :::; sn 1 ; sn g:
Then S = S 0 [ fsn g where S 0 = fs1 ; :::; sn 1 g: By the induction hypothesis, sup S 0 2 S 0 S:
Since sn 2
= S 0 ; the previous problem implies that sup(S 0 [ fsn g) = supfsup S 0 ; sn g 2 S:

60. Show that a number u is the supremum of a nonempty set S R if and only if u satises the conditions:
(1) s u for all s 2 S;
(2) if v < u; then there exists s0 2 S such that v < s0 :

Proof. (=)) Suppose that u is the supremum of a nonempty set S R: By denition, this means that
(1) u is an upper bound of S, and
(2) If v is any upper bound of S; then u v:
Note that condition (1) implies that s u for all s 2 S which is exactly condition (1).
Now, Let v < u and suppose that s v for all s 2 S: Then v is another upper bound for S:
Hence, by condition (2), we get that u v; a contradiction. Hence, there must be an s0 2 S such that v < s0 :
Thus, condition (2) holds, too.
((=) Condition (1) implies that u is an upper bound of S:
Now, let v be any upper bound of S and suppose that v < u: Then condition (2) implies that there exists
s0 2 S such that v < s0 ; contradicting the assumption that v is an upper bound of S:
Hence, if v is any upper bound of S; then u v: This shows that condition (2) is satised.

61. If S = f1=n 1=m : n; m 2 Ng; nd inf S and sup S:

1
1
Solution. n 2 N =) 0 < n1
1; and m 2 N =) 0 < m
1 =) 1
m < 0:
Thus, 0 1 < 1=n 1=m < 1 + 0 =) 1 < 1=n 1=m < 1 for any n; m 2 N:
Hence, 1 is a lower bound for S and 1 is an upper bound for S:
We claim that inf S = 1 and sup S = 1; and here is the proof:
Let x > 1: Then x + 1 > 0 and so, by the Archimedean property, 9 n 2 N : 0 < n1 < x + 1:
Hence, n1 1 < x: But n1 1 2 S (for m = 1): Therefore, 1 = inf S:
1
Let x < 1: Then 1 x > 0 and so, by the Archimedean property, 9 m 2 N : 0 < m
< 1 x:
1
1
Hence, x < 1 m : But 1 m 2 S (for n = 1): Therefore, 1 = sup S:

62. Let S R be nonempty. Suppose that a number u in R has the properties:


(i) For every n 2 N the number u 1=n is not an upper bound of S; and
(ii) For every n 2 N the number u + 1=n is an upper bound of S:
Prove that u = sup S:

Proof. Let s 2 S: Then s u + 1=n for all n 2 N; and so s u: Thus, u is an upper bound for S:
Now, let v < u: We have to show that there exists s 2 S : v < s:
For this, note that u v > 0; and so by the Archimedean property, 9 n 2 N : 0 < n1 < u v:
Hence, v < u n1 : But u 1=n is not an upper bound of S for every n 2 N =) v is not an upper bound of S:
Thus, there exists s0 2 S : v < s0 ; and consequently, u = sup S:

63. Let X be a nonempty set and let f : X ! R have bounded range in R: Let a 2 R: Assuming the fact that
46

if S is a nonempty subset of R that is bounded above, then sup(a + S) = a + sup S; Show that
(i) supfa + f (x) : x 2 Xg = a + supff (x) : x 2 Xg; and
(ii) inffa + f (x) : x 2 Xg = a + infff (x) : x 2 Xg:

Proof. Let S = f (X) = ff (x) : x 2 Xg: Then S is a nonempty subset of R that is bounded above. Hence,
(i) sup(a + f (X)) = a + sup f (X) =) supfa + f (x) : x 2 Xg = a + supff (x) : x 2 Xg:
(ii) inffa + f (x) : x 2 Xg = supf (a + f (x)) : x 2 Xg = [supf a + ( f (x)) : x 2 Xg]
by (i)

=
[ a + supf f (x) : x 2 Xg] = a supf f (x) : x 2 Xg = a + infff (x) : x 2 Xg:

64. Let A and B be bounded nonempty subsets of R; and let A + B = fa + b : a 2 A; b 2 Bg: Prove that:
(i) sup(A + B) = sup(A) + sup(B); and
(ii) inf(A + B) = inf(A) + inf(B):

Proof. We use the fact that if S is a nonempty subset of R that is bounded, and a 2 R; then
sup(a + S) = a + sup S and inf(a + S) = a + inf S: This was proved in the previous problem.
(i) Let u = sup A and v = sup B and x a 2 A: Then sup(a + B) = a + sup B = a + v:
Now, sup(A + B) = supfa + v : a 2 Ag = supfA + v : a 2 Ag = (sup A) + v = u + v:
(ii) Let u = inf A and v = inf B and x a 2 A: Then inf(a + B) = a + inf B = a + v:
Now, inf(A + B) = inffa + v : a 2 Ag = inffA + v : a 2 Ag = (inf A) + v = u + v:

65. Let X be a nonempty set, and let f and g be dened on X and have bounded ranges in R: Prove that:
(i) supff (x) + g(x) : x 2 Xg supff (x) : x 2 Xg + supfg(x) : x 2 Xg; and
(ii) infff (x) : x 2 Xg + inffg(x) : x 2 Xg infff (x) + g(x) : x 2 Xg:
Give examples to show that each of these inequalities can be either equalities or strict inequalities.

Proof. Since f and g have bounded ranges in R; u = sup f (X); v = sup g(X); u0 = inf f (X); and
v 0 = inf g(X) all exist in R.
(i) For all x 2 X; f (x) u and g(x) v =) f (x) + g(x) u + v and so, since this happens 8x 2 X;
we deduce that sup(f (X) + g(X)) u + v = sup f (X) + sup g(X):
(ii) For all x 2 X; f (x) u0 and g(x) v 0 =) f (x) + g(x) u0 + v 0 and so, since this happens 8x 2 X;
we deduce that inf(f (X) + g(X)) u0 + v 0 = inf f (X) + inf g(X):
Examples. (a) Let X = R; and let f1 (x) = g1 (x) = 0 8x 2 X: Then f1 (x) + g1 (x) = 0 8x 2 X:
Hence, supff1 (x) + g1 (x) : x 2 Xg = 0 and supff1 (x) : x 2 Xg + supfg1 (x) : x 2 Xg = 0 + 0 = 0:
Also, infff1 (x) + g1 (x) : x 2 Xg = 0 and infff1 (x) : x 2 Xg + inffg1 (x) : x 2 Xg = 0 + 0 = 0:
(b) Let X = R; and let f2 (x) =

1 x 0
1 x>0

; and g2 (x) =

1 x 0
1 x>0

Then f2 (x) + g2 (x) = 0 8x 2 X:


Hence, supff1 (x) + g1 (x) : x 2 Xg = 0 and supff1 (x) : x 2 Xg + supfg1 (x) : x 2 Xg = 1 + 1 = 2:
Also, infff1 (x) + g1 (x) : x 2 Xg = 0 and infff1 (x) : x 2 Xg + inffg1 (x) : x 2 Xg = 1 + 1 = 2:

66. Let X = Y = fx 2 R : 0 < x < 1g: Dene h1 ; h2 : X Y ! R by


h1 (x; y) = 2x + y; and h2 (x; y) =

0
1

if x < y
if x y

(a) For each x 2 X; nd f1 (x) = supfh1 (x; y) : y 2 Y g and f2 (x) = supfh2 (x; y) : y 2 Y g; then nd
infff1 (x) : x 2 Xg and infff2 (x) : x 2 Xg:
(b) For each y 2 Y; nd g1 (y) = inffh1 (x; y) : x 2 Xg and g2 (y) = inffh2 (x; y) : x 2 Xg; then nd
47

supfg1 (y) : y 2 Y g and supfg2 (y) : y 2 Y g:

Solution. (a) Let x 2 X: Then


f1 (x) = supfh1 (x; y) : y 2 Y g = supf2x + y : y 2 Y g = supf2x + Y g = 2x + sup Y = 2x + 1:
f2 (x) = supfh2 (x; y) : y 2 Y g = supf0; 1g = 1:
Hence, infff1 (x) : x 2 Xg = inff2x + 1 : x 2 (0; 1)g = 1; and infff2 (x) : x 2 Xg = inff1g = 1:
(b) Let y 2 Y: Then
g1 (y) = inffh1 (x; y) : x 2 Xg = inff2x + y : x 2 Xg = inff2X + yg = y + inf(2X) = y + 0 = y:
g2 (y) = inffh2 (x; y) : x 2 Xg = inff0; 1g = 0:
Hence, supfg1 (y) : y 2 Y g = supfy : y 2 (0; 1)g = 1; and supfg2 (y) : y 2 Y g = supf0g = 0:

67. Let X and Y be nonempty sets and let h : X Y ! R have bounded range in R: Let f : X ! R and
g : Y ! R be dened by f (x) = supfh(x; y) : y 2 Y g; and g(y) = inffh(x; y) : x 2 Xg: Prove that
supfg(y) : y 2 Y g infff (x) : x 2 Xg:

Proof. Let x0 2 X and y0 2 Y: Then g(y0 ) = inffh(x; y0 ) : x 2 Xg h(x; y0 ) for all x 2 X:


So, in particular, g(y0 ) h(x0 ; y0 ) supfh(x0 ; y) : y 2 Y g = f (x0 ):
Thus, g(y) f (x) for any x 2 X and any y 2 Y:
Hence, supfg(y) : y 2 Y g f (x) 8x 2 X:
Therefore, supfg(y) : y 2 Y g infff (x) : x 2 Xg:

68. Show that if a > 0; then there exists a positive real number z such that z 2 = a:

Proof. Let S = fs 2 R : 0 s; s2 < ag: Since 0 2 S; S 6= ?: Also, S is bounded above by a + 1;


because if t > a + 1; then t2 > (a + 1)2 = a2 + 2a + 1 > a so that t 2
= S:
Therefore, z = sup S exists in R: Note that z > 0:
We will prove that z 2 = a by ruling out the other two possibilities: z 2 < a and z 2 > a:
Suppose that z 2 < a: We will show that this assumption contradicts the fact that z = sup S by nding
an n 2 N such that z + 1=n 2 S; thus implying that z is not an upper bound for S:
1
Note that (z + 1=n)2 = z 2 + 2z
z 2 + n1 (2z + 1):
n + n2
1
Hence, if we can choose n so that n (2z + 1) < a z 2 ; then we get (z + 1=n)2 < a:
z2
z2
By assumption, we have a z 2 > 0 so that a2z+1
> 0 and by the Archimedean property 9n 2 N : n1 < a2z+1
:
2
Thus, (z + 1=n) < a and so z + 1=n 2 S:
Hence, z is not an upper bound for S; a contradiction. Therefore, we cannot have z 2 < a:
Suppose that z 2 > a: We will show that it is impossible to nd m 2 N : z 1=m is also an upper bound
for S; Contradicting the fact that z = sup S:
1
2z
2
Note that (z 1=m)2 = z 2 2z
m + m2 > z
m:
2z
2
Hence, if we can choose m so that m < z
a; then we get (z 1=m)2 > a:
2
2
1
< z 2z a :
By assumption, we have z 2 a > 0 so that z 2z a > 0 and by the Archimedean property 9m 2 N : m
Thus, (z 1=m)2 > a:
Now, if s 2 S; then s2 < a < (z 1=m)2 and so s < z 1=m =) z 1=m is an upper bound for S:
This contradicts the fact that z = sup S. Therefore, we cannot have z 2 > a:
Thus, we ruled out the two possibilities z 2 < a and z 2 > a; and so z 2 = a:

69. If u > 0 is any real number and x < y; show that there exists a rational number r such that x < ru < y:

Proof. x < y and u > 0 =) ux < uy =) 9 a rational number r : ux < r < uy :


Hence, x < ru < y:

48


70. If S R is nonempty, show that S is bounded if and only if there exists a bounded closed interval I
such that S I:

Proof. (=)) Suppose that S is bounded. Then u = inf S and v = sup S both exist in R:
Hence, u s v for all s 2 S; and so S [u; v] = I; which is a closed interval.
((=) Suppose there exists a bounded closed interval I = [a; b] such that S I:
Then a s b for all s 2 S and so S is bounded.

71. If S R is a nonempty bounded set, and IS = [inf S; sup S]; show that S IS : Moreover, if J is any
closed bounded interval containing S; show that IS J:

Proof. The rst part of this problem was done in the previous exercise. Now, let J = [a; b] be any closed
bounded interval such that S J: Let x 2 IS : Then inf S x sup S:
Since S J; we get that a s b for all s 2 S: Thus, by denition of inf and sup, we get that
a inf S and sup S b: That is, a inf S x sup S b and so x 2 [a; b] = J:
Hence, IS J:

1
\
72. Let In = [0; 1=n] for n 2 N: Prove that
In = f0g:
n=1

1
\
1
< x:
Proof. Clearly, 0 2
In : Let 0 < x 1. By the Archimedean property, 9m 2 N : m
n=1

Thus, x 2
= In for all n

m: Hence, x 2
=

1
\

In ; and so

n=1

1
\

n=1

In = f0g:

1
\
73. Let Kn = (n; 1) for n 2 N: Prove that
Kn = ?:
n=1

Proof. Let x > 0: By the Archimedean property, 9m 2 N : x < m: Thus, x 2


= Kn for all n m:
1
\
Therefore, x 2
=
Kn : It is also clear that Kn only contains positive numbers.
Hence,

n=1

1
\

Kn = ?:

n=1

7
74. Give the two binary representations of 38 and 16
:

1
X
1
Solution. (i) Since 38 14 = 18 =
2n ; we get that
3
8

1
4

1
8

1
23 ;

and

= (0:011000

1
22

3
8

1
4

n=4
1
X

1
2n

1
22

1
X

n=4

Hence,

3
8

(ii) Since

7
16

1
4

3
16

and

n=4

)2 = (0:010111
3
16

1
8

1
16

)2 :

1
X

1
2n ;

1
4

1
8

we get that

n=5
7
16

1
4

1
8

1
16

1
22

1
23

1
24 ;

and

7
16

1
2n :

1
X

n=5

1
2n

1
22

1
23

1
X

1
2n :

n=5

7
Hence, 16
= (0:0111000 0 )2 = (0:0110111 1 )2 :

75. Show that if ak ; bk 2 f0; 1; :::; 9g and if

49

a1
10

a2
102

an
10n

b1
10

b2
102

bm
10m

6= 0;

then n = m; and ak = bk for k = 1; 2; :::; n:

Proof. Suppose that m < n:


bm+1
b2
bm
b1
b2
bm
bn
b1
Then we can write 10
+ 10
+ 10
+ 10
+ 10
2 +
m = 10 + 102 +
m + 10m+1 +
n;
where bm+j = 0 for all j = 1; :::; n m:
a2
an
b1
b2
bn
+ 10
+ 10
Now, we are given that a101 + 10
2 +
n = 10 + 102 +
n:
Assume that there is aj 6= bj : Let j be the rst index in f1; :::; ng so that aj 6= bj .
Then a1 = b1 ; a2 = b2 ; :::; aj 1 = bj 1 ; but aj 6= bj . Say, aj < bj :
a
aj
b
b
a
an
bn
9
+ 10
+ 109n < 10jj + 10j+1
+ 10
Then 10jj + 10j+1
j+1 +
n
j+1 +
n for any bj+k 2 f0; :::; 9g:
10j + 10j+1 +
an
b1
a1
a2
b2
bn
This contradicts our assumption that 10 + 102 +
+ 10n = 10 + 102 +
+ 10
n:
Hence, aj = bj for all j = 1; 2; :::; n:
Thus, am+j = bm+j = 0 for all j = 1; :::; n m; and so m = n:

76. For any b 2 R, prove that lim nb = 0:


n!1

Proof. If b = 0; then nb = 0 for all n 2 N and so lim nb = 0: Hence, assume that b 6= 0:


n!1
Let > 0 be given. Then jbj > 0 and so, by the Archimedean property, we can nd N 2 N large enough
b
so that 0 < N1 < jbj : Thus, n1 < jbj for all n N; and so jbj
N:
n = n < for all n
b
Since our choice of > 0 was arbitrary, we deduce that lim n = 0:
n!1

77. Use the denition of a limit of a sequence to establish the following limits:
2n
n2 1
(b) lim n+1
= 2;
(c) lim 3n+1
= 32 ;
(d) lim 2n
= 12 :
(a) lim n2n+1 = 0;
2 +3
2n+5
n!1
n!1
n!1
n!1

Proof. (a) Let > 0 be given. Then, by the Archimedean property, we can nd N 2 N large enough
so that 0 < N1 < : Thus, n1 < for all n N:
Note that since n2 < n2 + 1 we have n2n+1 < n1 for all n 2 N:
But then n2n+1 < for all n N:

Since our choice of

> 0 was arbitrary, we deduce that lim

n!1

n
n2 +1

= 0:

(b) Let > 0 be given. Then 2 > 0 and so, by the Archimedean property, we can nd N 2 N large enough
1
2
so that 0 < N1 < 2 : Thus, n+1
< n1 < 2 for all n N and so n+1
< for all n N:
Thus,

2n
n+1

2
n+1

2 =

Since our choice of

2
n+1

<

for all n

N:

> 0 was arbitrary, we deduce that lim

n!1

2n
n+1

= 2:

(c) Let > 0 be given. Then 13 > 0 and so, by the Archimedean property, we can nd N 2 N large enough
1
13
< for all n N:
so that 0 < N1 < 13 : Thus, 4n+10
< n1 < 13 for all n N and so 4n+10
Thus,

3n+1
2n+5

3
2

Since our choice of

13
4n+10

13
4n+10

<

for all n

N:

> 0 was arbitrary, we deduce that lim

n!1

3n+1
2n+5

= 32 :

(d) Let > 0 be given. Then 5 > 0 and so, by the Archimedean property, we can nd N 2 N large enough
so that 0 < N1 < 5 : Thus, 4n21+6 < n1 < 5 for all n N and so 4n25+6 < for all n N:
Thus,

n2 1
2n2 +3

1
2

5
4n2 +6

5
4n2 +6

<

for all n

N:
2

n 1
Since our choice of > 0 was arbitrary, we deduce that lim 2n
= 12 :
2 +3
n!1

78. Show that:


p
n
n
2n
1
(a) lim pn+7
= 0;
(b) lim n+2
= 2;
(c) lim n+1
= 0;
(d) lim ( n21)+1n = 0:
n!1

n!1

n!1

50

n!1


Proof. (a) Let > 0 be given. Then 2 > 0 and so, by the Archimedean property, we can nd N 2 N
1
1
large enough so that 0 < N1 < 2 : Thus, n+7
< n1 < 2 for all n N and so pn+7
< for all n N:
Since our choice of

> 0 was arbitrary, we deduce that lim

n!1

p1
n+7

= 0:

(b) Let > 0 be given. Then 4 > 0 and so, by the Archimedean property, we can nd N 2 N large enough
1
4
so that 0 < N1 < 4 : Thus, n+2
< n1 < 4 for all n N and so n+2
< for all n N:
Thus,

2n
n+2

2 =

4
n+2

Since our choice of

4
n+2

<

for all n

N:

> 0 was arbitrary, we deduce that lim

n!1

2n
n+2

= 2:

(c) Let > 0 be given. Then 2 > 0 and so, by the Archimedean p
property, we can nd N 2 N large enough
n
1
1
n
2
2
so that 0 < N < : Thus, (n+1)2 < n < for all n N and so n+1
< for all n N:
Since our choice of

> 0 was arbitrary, we deduce that lim

n!1

p
n
n+1

= 0:

(d) Let > 0 be given. Then, by the Archimedean property, we can nd N 2 N large enough so that
n
1
n
for all n N .
0 < N1 < : Thus, ( n21)+1n = (n+1)
2 < n <
n

Since our choice of > 0 was arbitrary, we deduce that lim ( n21)+1n = 0:
n!1

1
79. Let xn = ln(n+1)
for n 2 N:
(a) Use the denition of limit to show that lim (xn ) = 0:
n!1

(b) Find a specic value of K( ) as required in the denition of limit for each of (i) = 1=2; and (ii) = 1=10:

Proof. (a) Let > 0 be given. Then, by the Archimedean property, we can nd N 2 N large enough
so that e1=
1 < N: This implies that e1=
1 < n for all n N:
1
Hence, for all n N; we get n > e1=
1 =) n + 1 > e1= =) ln(n + 1) > 1= =) ln(n+1)
< :
Since our choice of

> 0 was arbitrary, we deduce that lim

n!1

1
ln(n+1)

= 0:

(b) From what we did in (a), K(1=2) > e2 1 = 6: 39 =) K(1=2) = 7:


Also, K(1=10) > e10 1 = 22025: 47 =) K(1=10) = 22026:

80. Prove that lim (xn ) = 0 if and only if lim (jxn j) = 0: Give an example to show that the convergence
n!1

n!1

of (jxn j) need not imply the convergence of (xn ) :

Proof. (=)) Suppose lim (xn ) = 0 and let > 0 be given. Then there exists N 2 N large enough so that
n!1

jxn 0j = jxn j < for all n N:


Thus, since jjxn j 0j = jxn j ; and since our choice of
((=) Suppose lim (jxn j) = 0 and let
n!1

> 0 was arbitrary, we deduce that lim (jxn j) = 0:


n!1

> 0 be given. Then there exists N 2 N large enough so that

jjxn j 0j = jxn j < for all n N:


Since our choice of > 0 was arbitrary, we deduce that lim (xn ) = 0:
n!1

Let xn = ( 1)n : Then (jxn j) = f1; 1; 1; :::g and so lim (jxn j) = 1; yet (xn ) does not converge.
n!1

81. Prove that if lim (xn ) = x and if x > 0; then there exists M 2 N such that xn > 0 for all n M:
n!1

Proof. Since x > 0; we can nd > 0 such that 0 < < x: Since lim (xn ) = x; we can nd N 2 N
n!1

large enough so that jxn xj <


Hence,
< xn x < =) x
But x
> 0:

for all n N:
< xn < x + for all n

51

N:


82. Let b 2 R satisfy 0 < b < 1: Show that lim (nbn ) = 0: (Hint: Use the binomial theorem).
n!1

Proof. Since 0 < b < 1; we get that 1=b > 1 and so 1=b = 1 + k for some k > 0:
n
n
Thus, nbn = (1=b)
n =
(1+k)n :
n
X
n
n i
n 2
+ kn :
Now, (1 + k) =
+ k n = 1 + nk + n(n2 1) k 2 +
i k = 1 + nk + 2 k +
i=0

n
= n(n n1) 2 = k2 (n2 1) :
k
(n2 )k2
2
k2
Let > 0 be given. Then 2 > 0 and so, by the Archimedean property, we can nd N 2 N large enough
2
2
so that N1 < k2 : Hence, for all n N + 1; we have n 1 1 < k2 ; and so k2 (n2 1) < :
n
2
Thus, jnbn j = nbn = (1+k)
N + 1:
n <
k2 (n 1) < for all n
Since our choice of > 0 was arbitrary, we deduce that lim (nbn ) = 0:
n!1

83. For xn given by the following formulas, establish either the convergence or the divergence of
the sequence X = (xn ):
2
1)n n
n
n2
+3
(a) xn = n+1
;
(b) xn = ( n+1
;
(c) xn = n+1
;
(d) xn = 2n
n2 +1 :

n
Proof. (a) We claim that lim n+1
= 1: Let > 0 be given. Then by the Archimedean property,

Note that for n

n
(1+k)n

2; we have

<

n!1

we can nd N 2 N large enough so that


Thus,

n
n+1

1 =

1
n+1

Since our choice of

1
n+1

<

1
N +1

for all n

< : Hence,

1
n+1

<

for all n

N:

N:

> 0 was arbitrary, we deduce that lim

n!1

n
n+1

= 1:

1) n
n
1
(b) First note that jxn j = ( n+1
= n+1
2 for all n 2 N:
Thus, at least we know that the limit is not 0.
Since (xn ) has an innite number of positive terms and an innite number of negative terms and the limit
of (xn ) is not 0, we conclude that the limit does not exist and so the sequence diverges.
2

n
(c) We will show that the sequence (xn ) = n+1
is not bounded and so is not convergent.
Let R > 0 be any positive real number. Then, by the Archimedean property, we can nd n 2 N large
enough so that R < n 1: Then R(n + 1) < (n 1)(n + 1) = n2 1 < n2 :
n2
n2
Thus, R < n+1
; and since our choice of R > 0 was arbitrary, we conclude that the sequence (xn ) = n+1
is not bounded and so is not convergent.
2

+3
(d) We will show that the sequence (xn ) = 2n
converges to 2.
n2 +1
2
2
2
(n +1)+(n +1)+1
+3
n2 +1+n2 +1+1
Note that 2n
=
= 1 + 1 + n21+1 = 2 + n21+1 :
n2 +1 =
n2 +1
n2 +1
Let > 0 be given. Then by the Archimedean property, we can nd N 2 N large enough so that
Hence, n1 < for all n N:

Thus,

2n2 +3
n2 +1

2 =

2+

1
n2 +1

2 =

1
n2 +1

<

1
n

<

for all n

1
N

< :

N:
2

+3
Since our choice of > 0 was arbitrary, we deduce that lim 2n
= 2:
n2 +1
n!1

84. Give an example of two divergent sequences X and Y such that:


(a) Their sum converges,
(b) Their product converges.

n
n+1
Solution. (a) Let X = (( 1) ) and Y = ( 1)
: Then X and Y are both divergent sequences.
n

n+1

Yet, X + Y = ( 1) + ( 1)

= (0) converges to 0.

52

n+1

(b) Let X = (( 1) ) and Y = ( 1)


n

: Then X and Y are both divergent sequences.

n+1

Yet, XY = ( 1) ( 1)
= ( 1) converges to 1.

85. Show that the following sequences are not convergent:


(a) (2n ) ;
(b) ( 1)n n2 :

Proof. (a) Let R > 1 be any positive real number greater than 1. We will show that we can nd n 2 N
R
so that 2n > R: Choose, by the Archimedean property, n 2 N large enough so that n > ln
ln 2 :
Then n ln 2 > ln R =) ln 2n > ln R =) 2n > R:
Since our choice of R > 1 was arbitrary and we were able to nd a member of the sequence that is larger
than R; we conclude that (2n ) is not bounded and so is not convergent.
(b) Let R > 1 be any positive real number greater than 1. We will show that we can nd n 2 N
so that ( 1)n n2 > R: Choose, by the Archimedean property, an even integer n 2 N large enough so that
n2 > R: Then ( 1)n n2 = n2 > R: (Here, since n is even, ( 1)n = 1):
Since our choice of R > 1 was arbitrary and we were able to nd a member of the sequence that is larger
than R; we conclude that ( 1)n n2 is not bounded and so is not convergent.

86. Find the limits of the following sequences:


p
2
1)n
p
(a) lim (2 + 1=n) ;
(b) lim (n+2
;
(c) lim pnn+11 ;
(d) lim nn+1
:
n
n!1
n!1
n!1
n!1

b ecause they converge


2
Solution. (a) lim (2 + 1=n) = lim 4 + 2=n + 1=n2
=
lim (4)+ lim (2=n)+ lim 1=n2
n!1
n!1
n!1
n!1
n!1
= 4 + 0 + 0 = 4:
(b) Since lim
(c) Note

1
n+2

n!1
p
that pnn+11

If we show that

= 0; and

1
n+2

p
n+1 2
p
=1
n+1
lim p 2 = 0; then
n!1 n+1

( 1)n
n+2

; we conclude, by problem 80, that lim

n!1

p2 :
n+1

p
pn 1
n+1

we will have lim

n!1

( 1)n
n+2

= 0:

= 1:

Let > 0 be given. Then 4 > 0 and so, by the Archimedean property, we can nd N 2 N large enough
2
1
so that N1+1 < 4 ; and so pn+1
< 2 for all n N:
1
1
1
p
Since pn+1
for all n 2 N; we get that pn+1
< 2 for all n N:
n+1
2
Hence, pn+1 < for all n N .
2
Since our choice of > 0 was arbitrary, we deduce that lim pn+1
= 0:
Thus, lim

n!1

p
pn 1
n+1

n!1

= 1:
b ecause they converge

1
1
p
(d) lim nn+1
= lim p1n + np
=
lim p1n + lim np
= 0 + 0 = 0:
n
n
n
n!1
n!1
n!1
n!1

87. If (bn ) is a bounded sequence and lim (an ) = 0; show that lim (an bn ) = 0:
n!1
n!1

Proof. Since (bn ) is bounded, we can nd M > 0 : jbn j < M for all n 2 N:
Let > 0 be given and proceed as follows:
Since lim (an ) = 0; we can nd N 2 N large enough so that jan j < M for all n N:
n!1

Hence, for all n N; we have jan bn j = jan j jbn j < M M = :


Since our choice of > 0 was arbitrary, we deduce that lim (an bn ) = 0:
n!1

n+1
n+1
88. If 0 < a < b; determine lim a an +b
:
+bn
n!1

n+1
n+1
Solution. One claims that lim a an +b
= b: To show this, one proceeds as follows:
+bn

Let r =

a
b

< 1: Then

n!1
an+1 +bn+1
an +bn

b =

bn+1
bn

r n+1 +1
r n +1

53

b = b

r n+1 +1
r n +1

b =b

r n+1 +1
r n +1

n+1

1
= b r rn +1r = brn rrn +1
< brn :
Let > 0 be given. Since lim rn = 0; we can nd N 2 N large enough so that rn <

Thus,

n+1

n+1

+b
an +bn

n!1
n

b < br <

for all n

for all n

N:

N:
n+1

n+1

= b:
Since our choice of > 0 was arbitrary, we deduce that lim a an +b
+bn
n!1

89. Use the squeeze Theorem to determine the limits of the following:
2
1=n2
(a) n1=n ;
(b) (n!)
:
2
2
1=n
1=n
Proof. (a) We have that 8n 2 N; 1 n nn =) 1 n1=n
(nn )
= (n)
! 1 as n ! 1:
2
1=n
Thus, by the squeeze Theorem, lim n
= 1:
n!1

(b) Note that n! = n(n

1)

3 2 1
1=n2

n
| n

n
{z n n n} = n for all n 2 N:

n tim es
2
n 1=n

1=n

Thus, 8n 2 N; 1 n! n =) 1 (n!)
(n )
= (n)
! 1 as n ! 1:
1=n2
Thus, by the squeeze Theorem, lim (n!)
= 1:
n!1

90. Recall the following Theorem: "Let (xn ) be a sequence of positive real numbers such that
L = lim (xn+1 =xn ) exists. If L < 1; then (xn ) converges and lim (xn ) = 0":
n!1
n!1
Apply this theorem to the following sequences, where a; b satisfy: 0 < a < 1; b > 1:
(a) (an ) ;
(b) (bn =2n ) ;
(c) (n=bn ) ;
(d) 23n =32n :

Proof. (a) Here xn = an ; L = lim (xn+1 =xn ) = lim an+1 =an = lim (a) = a < 1:
n!1

n!1

n!1

Thus, by the above theorem, (an ) converges and lim (an ) = 0:


n!1

n+1

n+1

= lim 2b = 2b :
(b) Here xn = bn =2n ; L = lim (xn+1 =xn ) = lim b bn =2
=2n
n!1
n!1
n!1
Hence, L < 1 if and only if b < 2:
Thus, by the above theorem, (bn =2n ) converges for b < 2 and lim (bn =2n ) = 0:
n!1

(c) Here xn = n=bn ; L = lim (xn+1 =xn ) = lim


n!1

n!1

n+1

n+1=b
n=bn

= lim
n

n!1

n+1
bn

1
b

< 1:

Thus, by the above theorem, (n=b ) converges and lim (n=b ) = 0:


n!1

(8=9)n+1
(8=9)n
n!1
3n
lim 2 =32n =
n!1

(d) Here xn = 23n =32n = (8=9)n ; L = lim (xn+1 =xn ) = lim


n!1

3n

2n

= lim (8=9) = 8=9 < 1:


n!1

Thus, by the above theorem, 2 =3


converges and
0:

91. Let X = (xn ) be a sequence of positive real numbers such that lim (xn+1 =xn ) = L > 1:
n!1
Show that X is not a bounded sequence and hence is not convergent.

Proof. Since L > 1; we can nd a real number R with 1 < R < L.


Since lim (xn+1 =xn ) = L; we can nd N 2 N large enough so that R < xxn+1
< L for all n N:
n
n!1

Thus, in particular, xxNN+1 > R; and so xN +1 > RxN :


+2
Also, xxNN+1
> R; and so, xN +2 > RxN +1 > R2 xN :
xN +3
Also, xN +2 > R; and so, xN +3 > RxN +2 > R3 xN :
..
.
Continuing in this fashion, we get that xN +n > Rn xN for all n 2 N; and so, xn > Rn N xN for all n N + 1:
But R > 1 =) Rn ! 1 as n ! 1:
Hence, X = (xn ) is not bounded and hence is not convergent.

1=n
92. Let (xn ) be a sequence of positive real numbers such that lim xn
= L < 1:
n!1

54

Show that there exists a number r with 0 < r < 1 such that 0 < xn < rn for all su ciently large n 2 N:
Use this to show that lim (xn ) = 0:
n!1

Proof. Since L < 1; we can nd a real number r such that L < r < 1:
1=n
1=n
Since lim xn
= L < 1; we can nd N 2 N large enough so that xn < r 8n N: (Here, you can
n!1

take = r L and apply the denition of a limit):


1=n
Hence, 8n N; 0 < xn < r =) 0 < xn < rn :
n
Since r < 1; lim r = 0; and hence, by the squeeze theorem, lim (xn ) = 0:
n!1
n!1

93. Suppose that (xn ) is a convergent sequence and (yn ) is such that for any > 0 there exists M such
that jxn yn j < for all n M: Does it follow that (yn ) is convergent?

Proof. Yes, and we will actually show that if lim (xn ) = x; then lim (yn ) = x:
n!1
n!1
Let > 0 be given. Then:
There exists M such that jxn yn j < 2 for all n M (by hypothesis)
There exists N such that jxn xj < 2 for all n N (by denition of limit).
Let K = maxfM; N g:
Then, jyn xj = jyn xn + xn xj jyn xn j + jxn xj < 2 + 2 = for all n K:
Since our choice of > 0 was arbitrary, we deduce that lim (yn ) = x:
n!1

1
94. Let x1 = 8 and xn+1 = 2 xn + 2 for n 2 N. Show that (xn ) is bounded and monotone. Find lim (xn ) :
n!1

Proof. Direct calculation shows that x2 = 6: Hence, we have 4 < x2 < x1 :


We show, by induction, that 4 < xn for all n 2 N: It is true for n = 1; 2:
If xk > 4 holds for some k 2 N; then xk+1 = 21 xk + 2 > 12 (4) + 2 = 4:
Therefore, by the induction theorem, xn > 4 for all n 2 N:
We now show, by induction, that xn+1 < xn for all n 2 N:
The truth of this assertion has been veried for n = 1:
Now suppose that xk+1 < xk for some k; then xk+2 = 21 xk+1 + 2 < 12 xk + 2 = xk+1 :
Thus, xk+1 < xk implies xk+2 < xk+1 : Therefore, xn+1 < xn for all n 2 N:
We have thus shown that the sequence (xn ) is decreasing and bounded below by 4.
It follows from the Monotone Convergence Theorem that (xn ) converges to a limit that is at least 4:
Let L = lim xn : Then L = lim xn+1 :
n!1

n!1

1
2 xn

Now, xn+1 =
+ 2 =) lim xn+1 = 12 lim xn + 2 =) L = 12 L + 2 =) L = 4:
n!1
n!1
Therefore, lim xn = 4:
n!1

95. Let x1 > 1 and xn+1 = 2 1=xn for n 2 N. Show that (xn ) is bounded and monotone. Find lim (xn ) :
n!1

Proof. Since x1 > 1; 1=x1 < 1 and so x2 = 2 1=x1 > 1:


We show, by induction, that xn > 1 for all n 2 N: It is true for n = 1; 2:
If xk > 1 holds for some k 2 N; then xk+1 = 2 1=xk > 2 1 = 1:
Therefore, by the induction theorem, xn > 1 for all n 2 N:
We now show, by induction, that xn+1 < xn for all n 2 N:
For n = 1; we have 0 < (x1 1)2 = x21 2x1 + 1 =) 2x1 < x21 + 1 =) 2 < x1 + 1=x1
=) 2 1=x1 < x1 : But x2 = 2 1=x1 : Hence, x2 < x1 :
Now suppose that xk+1 < xk for some k: Then 1=xk+1 > 1=xk =) 1=xk+1 < 1=xk
=) 2 1=xk+1 < 2 1=xk : Therefore, xk+2 = 2 1=xk+1 < 2 1=xk = xk+1 :
Thus, xk+1 < xk implies xk+2 < xk+1 : Therefore, xn+1 < xn for all n 2 N:
We have thus shown that the sequence (xn ) is decreasing and bounded below by 1.
It follows from the Monotone Convergence Theorem that (xn ) converges to a limit that is at least 1:
Let L = lim xn : Then L = lim xn+1 :
n!1

n!1

55

Now, xn+1 = 2

1=xn =) lim xn+1 = 2


n!1

=) L = 2

lim xn

1
L

=) (L

1) = 0 =) L = 1:

n!1

Therefore, lim xn = 1:
n!1
p

96. Let x1 2 and xn+1 = 1 + xn 1 for n 2 N. Show that (xn ) is decreasing and bounded below by 2.
Find lim (xn ) :
n!1
p

Proof. Since x1 2; x1 1 1 and so x2 = 1 + x1 1 2:


We show, by induction, that xn 2 for all n 2 N:pIt is true for n = 1; 2:
If xk 2 holds for some k 2 N; then xk+1 = 1 + xk 1 > 1 + 1 = 2:
Therefore, by the induction theorem, xn 2 for all n 2 N:
We now show, by induction, that
n 2 N:
p xn+1 xn for all p
For n = 1; we have x1 1
x1 1 =) x1 1 + xn 1 = x2 :
p
p
Now suppose
that
x
x
for
some
k:
Then
x
1
x
1
=)
x
+
1
xk + 1
k+1
k
k+1
k
k+1
p
p
p
p
=) 1 + xk+1 + 1 1 + xk + 1: But 1 + xk+1 + 1 = xk+2 and 1 + xk + 1 = xk+1 :
Thus, xk+1 xk implies xk+2 xk+1 : Therefore, xn+1 xn for all n 2 N:
We have thus shown that the sequence (xn ) is decreasing and bounded below by 2.
It follows from the Monotone Convergence Theorem that (xn ) converges to a limit that is at least 2:
Let L = lim xn : Then L = lim xn+1 :
n!1
n!1
q
p
p
2
Now, xn+1 = 1 + xn 1 =) lim xn+1 = 1 +
lim xn 1 =) L = 1 + L 1 =) (L 1) = L 1
n!1

n!1

=) L = 2: Therefore, lim xn = 2:
n!1

p
p
97. Let y1 = p; where p > 0; and yn+1 = p + yn for n 2 N. Show that (yn ) converges and nd lim (yn ) :
n!1
p
(Hint: One upper bound is 1 + 2 p):

p
Proof. Let us show that 1 + 2 p is an upper bound for (yn ) : We use induction.
p
p
For n = 1; y1 = p < 1 + 2 p:
p
Suppose that the result is true for n; that is, yn 1 + 2 p:
q
p
p
p
p
p
p
2
Then yn+1 = p + yn
p+1+2 p=
p + 1 = p + 1 < 2 p + 1:
p
Thus, by the induction theorem, we deduce that yn 2 p + 1 for all n 2 N:
Now, we prove by induction
the sequence (yn ) is increasing.
pthatp
p
p
Note that y2 = p + y1 = p + p > p = y1 :
p
p
Also, yn > yn 1 =) yn+1 = p + yn > p + yn 1 = yn :
Thus, by the induction theorem, yn < yn+1 for all n 2 N:
Hence, by the Monotone Convergence Theorem, the sequence (yn ) converges. Say, lim (yn ) = L:
n!1
p
p
p
1
1+4p Since L>0
1+ 1+4p
2
2
=)
L
=
:
Then L = p + L =) L = p + L =) L
L p = 0 =) L =
2
2

98. Let (an ) be an increasing sequence and (bn ) be a decreasing sequence, and assume that an bn 8n 2 N:
Show that lim (an )
lim (bn ) ; and thereby deduce the Nested Intervals Property from the Monotone
n!1
n!1
Convergence Theorem.

Proof. Since (an ) is increasing, we have a1 a2 a3


an
:
Since (bn ) is decreasing, we have b1 b2 b3
bn
:
Now, by above and since an bn 8n 2 N; we have that a1 an bn b1 8n 2 N:
Thus, both sequences are bounded and since they are monotone, they both converge.
Let L1 = lim (an ) ; and L2 = lim (bn ) : Suppose that L2 < L1 : Then 9 > 0 : L2 + < L1 :
n!1

n!1

Since L1 = lim (an ) ; 9N1 2 N large enough so that L2 + < an


n!1

Since L2 = lim (bn ) ; 9N2 2 N large enough so that bn


n!1

Combining ( ) and ( ), one gets: bn < an for all n


Hence, L1 L2 and one is done.

L2 +

L1 for all n
for all n

N2

N1
(

maxfN1 ; N2 g; a contradiction.

56

( )


99. Let A be an innite subset of R that is bounded above and let u = sup A: Show that there exists
an increasing sequence (xn ) with xn 2 A for all n 2 N such that u = lim (xn ) :
n!1

Proof. If u 2 A; then we can take the sequence (xn ) = fu; u; u; :::g: Thus, we may assume that u 2
= A:
Since u = sup A; we can nd an element x1 2 A : (u 1; u):
Since u = sup A; we can nd an element x2 2 A : x2 > x1 and x2 2 (u 1=2; u):
Since u = sup A; we can nd an element x3 2 A : x3 > x2 and x3 2 (u 1=3; u):
..
.
Since u = sup A; we can nd an element xn 2 A : xn > xn
..
.

and xn 2 (u

1=n; u):

Note that xn is an increasing sequence of points in A by our construction, and that lim (xn ) = u:
n!1

100. Let (xn ) be a bounded sequence, and for each n 2 N let sn = supfxk : k ng and tn = inffxk : k ng:
Prove that (sn ) and (tn ) are monotone and convergent. Also, prove that if lim (sn ) = lim (tn ) ; then
n!1

n!1

(xn ) is convergent.

Proof. First note that (sn ) is decreasing since the sup is taken over a "smaller" set as n increases.
Similarly, (tn ) is increasing. Since (xn ) is bounded, we can nd a; b 2 R : a xn b for all n 2 N:
Thus, by denition of sup and inf; we get that a tn xn sn b for all n 2 N:
Hence, the sequences (sn ) and (tn ) are monotone and bounded and so are convergent.
Now, if lim (sn ) = lim (tn ) = L; then by the squeeze theorem, lim (xn ) = L:
n!1
n!1
n!1

101. Establish the convergence or divergence of the sequence (yn ) ; where


1
1
1
+ n+2
+
+ 2n
for n 2 N:
yn = n+1

1
1
1
1
1
1
n
Proof. We rst note that yn = n+1
+ n+2
+
+ 2n
< n+1
+ n+1
+
+ n+1
= n+1
< 1 for all n 2 N:
Thus, the sequence (yn ) is bounded above. We will show that (yn ) is also increasing, thus convergent.
To do so, we have to prove that yn yn+1 for all n 2 N:
7
y1 = 12 ; and y2 = 31 + 14 = 12
> 12 = y1 : h
i
h
i
1
1
1
+ 2n+2
=
n+2 + n+3 +
1
1
1
1
2n+1 + 2n+2
n+1 = 2n+1

Note that yn+1 =

1
1
1
+ 2n
n+2 + n+3 +
1
1
2n+2 = (2n+1)(2n+2)

1
1
+ 2n+1
+ 2n+2
= yn

1
n+1

1
1
+ 2n+1
+ 2n+2
:

But note that


> 0:
Hence yn+1 > yn for all n 2 N, and one is done.

102. Let xn = 1=12 + 1=22 +


+ 1=n2 for each n 2 N: Prove that (xn ) is increasing and bounded, and hence
converges. [Hint: Note that if k 2; then 1=k 2 1=k(k 1) = 1=(k 1) 1=k:]:

1
Proof. Since xn+1 = xn + (n+1)
2 ; It is clear that (xn ) is increasing.
Now we show that (xn ) is bounded above.
Indeed, xn = 1=12 + 1=22 + 1=32 +
+ 1=(n 1)2 + 1=n2 = 1 + 1=22 + 1=32 +
+ 1=(n 1)2 + 1=n2
h
i h
i
by the Hint
1 + 11 12 + 12 13 +
+ n 1 2 n 1 1 + n 1 1 n1 = 1 + 1 n1 = 2 n1
2 for all n 2 N:
Thus, the sequence (xn ) is increasing and bounded, and hence converges.

103. Establish the convergence and nd the limits of the following sequences:
1
(a) (1 + 1=n)n+1 ;
(b) (1 + 1=n)2n ;
(c) (1 + n+1
)n ;
(d) ((1 1=n)n ) :

Proof. (a) (1 + 1=n)n+1 = ((1 + 1=n)n ) ((1 + 1=n)) = X Y:


Note that lim X = e and lim Y = 1: Thus, lim X Y = (lim X) (lim Y ) = e 1 = e:

57

(b) (1 + 1=n)2n = ((1 + 1=n)n ) ((1 + 1=n)n ) = X X:


Note that lim X = e. Thus, lim X X = (lim X) (lim X) = e e = e2 :
1
1
1
(c) (1 + n+1
)n = (1 + n+1
)n+1 (1 + n+1
) 1 = X Y:
Note that lim X = e and lim Y = 1: Thus, lim X Y = (lim X) (lim Y ) = e 1 = e:

(d) Note that (1


Hence, ((1

1=n)n = ( nn 1 )n = (1=

1=n)n ) = 1= 1 +

n
n 1

)n = 1= 1 +

1
n 1

1= 1 +

1
n 1

n 1

n 1

1= 1 +

1
n 1

= X Y:

Note that lim X = 1=e and lim Y = 1: Thus, lim X Y = (lim X) (lim Y ) = (1=e) 1 = 1=e:
p

104. Calculate 2; correct to within 4 decimals.

p
Solution. Recall that the sequence (sn ) that converges to 2 was constructed as follows:
Let s1 > 0 and set sn+1 = 12 sn + s2n for n 2 N:
p
s2 2
Recall the inequality 0 sn
2 sn s2n = nsn for all n 2:
Now, sn+1 =

1
2

sn +

2
sn

s2n +2
2sn

1 for all n

1: (since (sn

1) + 1

1)

1
sn

Hence, s2

1 for all n 2 and so


1 for all n 2:
p
s2n 2
Thus, 0 sn
2
s2n 2 for all n 2:
sn
p
To make sn
2 < 0:00001; it then su ces, by the above inequality, to make s2n
That is, we have to nd n 2 N : s2n < 2:00001:
1 17
2
Let s1 = 1: Then s2 = 21 1 + 21 = 32 ; s3 = 12 32 + 23 = 17
=
12 ; s4 = 2 12 + 17
2

12

2 < 0:00001:
577
408 :

Note that s24 = 577


2: 000 006 007 < 2:00001:
408
p
577
Thus, 2 s4 = 408 = 1: 414 215 686 is correct to within 4 decimals. (Actually, 5 decimals).
p
On your calculator, 2 = 1: 414 213 562 :

105. Give an example of an unbounded sequence that has a convergent subsequence.

0 if n is even
Solution. Let xn =
:
n if n is odd
Then (xn ) is clearly unbounded, and the subsequence (x2n ) converges to 0.

106. Show that the following sequences are divergent.


(a) (1 ( 1)n + 1=n) ;
(b) (sin(n =4)) :

Proof. (a) Let xn = 1 ( 1)n + 1=n:


Then for nk = 2k; we have xnk = 1 ( 1)2k + 1=2k = 1 1 + 1=2k = 1=2k for all k 1:
Thus, lim (xnk ) = lim (1=2k) = 0:
k!1

k!1

Now, for nk = 2k + 1; we have xnk = 1 ( 1)2k+1 + 1=2k = 1 + 1 + 1=2k = 2 + 1=2k for all k
Thus, lim (xnk ) = lim (2 + 1=2k) = 2:
k!1

1:

k!1

Hence, we found two subsequences with dierent limits, and so (xn ) is divergent.
(b) Let yn = sin(n =4):
Then for nk = 4k; we have ynk = sin(4k =4) = sin(k ) = 0 for all k
Thus, lim (ynk ) = lim (0) = 0:
k!1

k!1

Now, for nk = 8k + 6; we have ynk = sin((8k + 6) =4) = sin(2k +


Thus, lim (ynk ) = lim ( 1) = 1:
k!1

1:

3
2

)=

1 for all k

1:

k!1

Hence, we found two subsequences with dierent limits, and so (yn ) is divergent.

58

107. Let X = (xn ) and Y = (yn ) be given sequences, and let the "shu- ed" sequence Z = (zn ) be dened
by: z1 = x1 ; z2 = y1 ;
; z2n 1 = xn ; z2n = yn ;
: Show that Z is convergent if and only if both X and Y
are convergent and lim X = lim Y:

Proof. (=)) Suppose that Z is convergent. Say lim Z = L; and let > 0 be given.
Then 9N 2 N large enough so that jzn Lj < for all n N:
In particular, jxn Lj < for all n N and jyn Lj < for all n N:
Thus, both X and Y are convergent and lim X = lim Y = L = lim Z:
((=) Suppose that both X and Y are convergent and lim X = lim Y = L: Let > 0 be given.
Then 9N1 ; N2 2 N : jxn Lj < for all n N1 and jyn Lj < for all n N2 :
Let N = maxfN1 ; N2 g: Then jzn Lj < for all n 2N + 2:
Thus, Z is convergent and lim Z = L:

108. Establish the convergence and nd the limits of the following sequences:
2
2
(a) (1 + 1=n2 )n ;
(b) ((1 + 1=2n)n ) ;
(c) (1 + 1=n2 )2n ;
(d) ((1 + 2=n)n ) :

2
Proof. (a) (1 + 1=n2 )n is a subsequence of the sequence ((1 + 1=n)n ) :
Since lim ((1 + 1=n)n ) = e; we conclude that lim

(1 + 1=n2 )n

n!1

n!1

= e:

(b) [(1 + 1=2n)n ] = (1 + 1=2n)2n e for all n 2 N:


p
Thus, ((1 + 1=2n)n ) is increasing and bounded above by e: So it converges. Say, L = lim ((1 + 1=2n)n ) :
n!1

Now, as in (a), lim (1 + 1=2n)2n = e since (1 + 1=2n)2n is a subsequence of the sequence ((1 + 1=n)n ) :
n!1
2n
p
e
2
Thus, (1 + 1=2n)n = (1+1=2n)
e:
(1+1=2n)n =) L = L =) L = e =) L =
p
n
But (1 + 1=2n)
1 > 0 for all n 2 N =) L = e:
(c) Let X = (1 + 1=n2 )2n

: Then (1 + 1=n2 )2n

By (a), lim X = e; and so lim


n!1

n!1

(1 + 1=n2 )2n

= (1 + 1=n2 )n

(1 + 1=n2 )n

= X X:

= e e = e2 :

(d) ((1 + 2=n)n ) is increasing and bounded above, so it converges.


Moreover, (1 + 2=2n)2n = (1 + 1=n)2n is a subsequence of ((1 + 2=n)n ) that converges to e2 :
Hence, lim ((1 + 2=n)n ) = e2 :
n!1

109. Determine the limits of the following:


1=2n
(a) (3n)
;
(b) (1 + 1=2n)3n :

p1=n
1=2n
Proof. (a) (3n)
=
3
n1=2n = X Y:
p 1=n
Since 1
3
n1=n for all n 2; the squeeze theorem implies that lim X = 1:
1=2n
Since 1 n
n1=n for all n 1; the squeeze theorem implies that lim Y = 1:
1=2n
Hence, lim (3n)
= (lim X) (lim Y ) = 1 1 = 1:
n!1

(b) (1 + 1=2n)3n = (1 + 1=2n)2n ((1 + 1=2n)n )p


= X Y:
As in the previous exercise, lim X = e; and lim Y =
p e: 3=2
Thus, lim (1 + 1=2n)3n = (lim X) (lim Y ) = e
e=e :
n!1

110. Suppose that every subsequence of X = (xn ) has a subsequence that converges to 0. Show that
lim X = 0:

Proof. We proceed by contradiction. Suppose that lim X 6= 0: Then we can nd a subsequence (xnk ) of X
such that, for some r > 0 , jxnk j r > 0 for all k 1: Thus, any subsequence xnkm of (xnk ) also satises
59

xnkm
r > 0 for all m 1: Hence, lim xnkm 6= 0:
m!1
Therefore, we have constructed a subsequence of X with no subsequence that converges to 0.
This contradicts our assumption that every subsequence of X = (xn ) has a subsequence that converges to 0.
Hence, lim X = 0:

111. Show that if (xn ) is unbounded, then there exists a subsequence (xnk ) such that lim (1=xnk ) = 0:
k!1

Proof. Since (xn ) is unbounded, we can nd xn1 2 (xn ) : jxn1 j > 1; xn2 2 (xn ) : jxn2 j > maxfjxn1 j ; 2g;
:
xn3 2 (xn ) : jxn3 j > maxfjxn2 j ; 3g;
; xnk 2 (xn ) : jxnk j > maxf xnk 1 ; kg;

Thus, 0 < xn1 < k1 for all k = 1; 2; 3; :::; and so by the squeeze theorem, we get that lim (1=xnk ) = 0:
k
k!1

112. Let (xn ) be a bounded sequence and let s = supfxn : n 2 Ng: Show that if s 2
= fxn : n 2 Ng; then
there is a subsequence of (xn ) that converges to s:

Proof. Since s = supfxn : n 2 Ng; we can nd an element xn1 : xn1 2 (s 1; s):


Since s = supfxn : n 2 Ng; we can nd an element xn2 : n2 > n1 ; xn2 > xn1 and xn2 2 (s 1=2; s):
Since s = supfxn : n 2 Ng; we can nd an element xn3 : n3 > n2 ; xn3 > xn2 and xn3 2 (s 1=3; s):
..
.

Since s = supfxn : n 2 Ng; we can nd an element xnk : nk > nk 1 ; xnk > xnk 1 and xnk 2 (s 1=k; s):
..
.
Note that xnk is an increasing sequence of points by our construction, and that lim (xnk ) = s:
k!1

113. Let (In ) be a nested sequence of closed bounded intervals. For each n 2 N; let xn 2 In : Use the BolzanoWeierstrass Theorem to show that there exists a number 2 R such that 2 In 8n 2 N:
[That is, Use the Bolzano-Weierstrass Theorem to prove the Nested Intervals Property].

Proof. Since I1 I2
In
; we get that xn 2 I1 for all n 2 N:
Hence, the sequence (xn ) is a bounded sequence, and so, by the Bolzano-Weierstrass Theorem, there is
a subsequence (xnk ) of (xn ) such that (xnk ) converges. Say, lim (xnk ) = :
k!1

We claim that 2 In for all n 2 N, and here is the proof:


Suppose, by contradiction, that there is N 2 N : 2
= IN :
Then, since the sequence of intervals is nested, we get that 2
= In for all n N:
Say, IN = [aN ; bN ]: Since 2
= IN ; we can nd > 0 small enough so that (
; + ) \ IN = ?:
Again, since the sequence of intervals is nested, (
; + ) \ In = ? for all n N ( ) :
Since lim (xnk ) = ; we can nd K 2 N large enough so that jxnk
j < for all k K:
k!1

That is, xnk 2 (


; + ) for all k K: But xnk 2 Ink for all k 2 N; a contradiction to ( ).
Hence, our claim is proved, and so we are done.

114. Show directly from the denition that the following are Cauchy sequences.
1
1
;
(b) 1 + 2!
+
+ n!
:
(a) n+1
n

Proof. (a) Let > 0 be given, and let N 2 N be such that N1 < 2 : Then for all n > m N; we have:
m+1
1
1
1
1
1
2
jxn xm j = n+1
= n1
n
m
m
n + m
N + N = N < :
n+1
Thus, n is a Cauchy sequence.

(b) Let > 0 be given, and let N 2 N be such that N1 < 2 : Then for all n > m N; we have:
1
1
1
1
1
1
1
jxn xm j = 1 + 2!
+
+ n!
1 + 2!
+
+ m!
= (m+1)!
+ (m+2)!
+
+ n!
1
1
m(m+1) + (m+1)(m+2)
1
1
1
1
2
m + n < N + N = N
1
1
Thus, 1 + 2! +
+ n!

1
(n 1)n

1
m

1
m+1

< :
is a Cauchy sequence.

60

1
m+1

1
m+2

1
n 1

1
n

1
m

1
n


115. Show directly from the denition that the following are not Cauchy sequences.
n
n
(a) (( 1) ) ;
(b) n + ( n1) ;
(c) (ln n)

Proof. (a) Let 0 = 1; and N 2 N be any positive integer. We will show that we can always nd n; m N
n
m
such that j( 1)
( 1) j > 1: Indeed, let n N be any even number, and let m = n + 1:
n
m
Then j( 1)
( 1) j = j1 ( 1)j = 2 > 1 = 0 :
n
Thus, (( 1) ) is not a Cauchy sequence.
(b) Let 0 = 1=2; and N 2 N be any positive integer. We will show that we can always nd n; m N
n
1)m
such that n + ( n1)
m+ ( m
> 1=2: Indeed, let n N be any even number, with n1 < 1=4;
and let m = n + 1:
n
n
1)m
1)n+1
1
= n + n1
Then n + ( n1)
m+ ( m
= n + ( n1)
n + 1 + ( n+1
n + 1 n+1
=

1
n

1+

Thus, n +

1
n+1
( 1)
n

=
n

1+

1
n(n+1)

1
n(n+1)

1
n

>1

>1

1=4 = 3=4 > 1=2 =

0:

is not a Cauchy sequence.

(b) Let 0 = 1; and N 2 N be any positive integer. We will show that we can always nd n; m N
such that jln n ln mj > 1: Indeed, let m = N , and choose n = 4N:
Hence, jln n ln mj = jln 4N ln N j = ln 4N
N = ln 4 > 1 = 0 :
Thus, (ln n) is not a Cauchy sequence.

116. Show directly from the denition that if (xn ) and (yn ) are Cauchy sequences, then (xn + yn ) and (xn yn )
are Cauchy sequences.

Proof. First, we show that (xn + yn ) is a Cauchy sequence. Let > 0 be given.
Then since (xn ) and (yn ) are Cauchy sequences, we can nd N 2 N large enough so that whenever n; m N;
we have jxn xm j < =2 and jyn ym j < =2:
Thus, for all n; m N; we have:
j(xn + yn ) (xm + ym )j = j(xn xm ) + (yn ym )j jxn xm j + jyn ym j < =2 + =2 = :
Thus, (xn + yn ) is a Cauchy sequence.
Next, we show that (xn yn ) is a Cauchy sequence.
Before we do this, let us show that the sequences (xn ) and (yn ) are bounded.
Let = 1. Then since (xn ) and (yn ) are Cauchy sequences, we can nd N 2 N large enough so that whenever
n; m N; we have jxn xm j < 1 and jyn ym j < 1:
In particular, jxn xN j < 1 and jyn yN j < 1 for all n N:
Thus, jxn j < 1 + jxN j and jyn j < 1 + jyN j for all n N:
Let M1 = maxfjx1 j ; jx2 j ;
; jxN 1 j ; 1 + jxN jg and M2 = maxfjy1 j ; jy2 j ;
; jyN 1 j ; 1 + jyN jg:
Then jxn j M1 and jyn j M2 for all n 2 N:
Thus, the sequences (xn ) and (yn ) are bounded.
Now, let > 0 be given. Then since (xn ) and (yn ) are Cauchy sequences, we can nd N 2 N large enough
so that whenever n; m N; we have jxn xm j < 2M2 and jyn ym j < 2M1 :
Hence, for all n; m N; jxn yn xm ym j = jxn yn xn ym + xn ym xm ym j = jxn (yn ym ) + ym (xn xm )j
jxn j jyn ym j + jym j jxn xm j M1 jyn ym j + M2 jxn xm j < M1 2M1 + M2 2M2 = 2 + 2 = :
Thus, (xn yn ) is a Cauchy sequence.
p

117. If xn = n; show that (xn ) satises lim jxn+1 xn j = 0; but that it is not a Cauchy sequence.
n!1
p

p p
p p p
n
p
Proof. lim jxn+1 xn j = lim
n+1
n = lim
n+1
n pn+1+
n+1+ n
=

n!1
1 p
lim pn+1+
n
n!1

Now, let

n!1

n!1

= 0:

= 1=2; and N 2 N be any positive integer. We will show that we can always nd n; m
61

xm j > 1=2: Indeed, for n = N 2 and m = (N + 1) ; we have:


q
p
2
xm j = N 2
(N + 1) = jN (N + 1)j = 1 > 1=2 = 0 :

such that jxn


jxn

Thus, (xn ) is not a Cauchy sequence.

118. Let (xn ) be a Cauchy sequence such that xn is an integer for every n 2 N: Show that (xn ) is ultimately
constant.

Proof. We are asked to show that after nitely many terms, the sequence (xn ) becomes constant.
Suppose not. This means that for any N 2 N; we can nd n > N : xn 6= xN :
But since xn and xN are integers, and xn 6= xN ; we get that jxn xN j 1:
Thus, if we let 0 = 1=2; we get that (xn ) is not a Cauchy sequence.
(choose, e.g., m = N; and n > N as above).
Hence, if (xn ) is a Cauchy sequence such that xn is an integer 8n 2 N; then (xn ) is ultimately constant.

119. Show directly that a bounded, monotone increasing sequence is a Cauchy sequence.

Proof. A bounded monotone increasing sequence is convergent and hence is Cauchy.

120. If 0 < r < 1 and jxn+1 xn j < rn for all n 2 N; show that (xn ) is a Cauchy sequence.

Proof. Recall from an earlier exercise that the sequence (nrn ) converges to 0.
Let > 0 be given. Then 9N 2 N : nrn < r for all n N; and so nrn 1 < for all n N:
Now, for any n > m N; we have:
n
n
n
X1
X1
X1
jxn xm j =
(xk+1 xk )
jxk+1 xk j <
rk < (n m)rn 1 < nrn 1 < :
k=m

k=m

k=m

Hence, (xn ) is a Cauchy sequence.

121. Show that if (xn ) is an unbounded sequence, then there exists a properly divergent subsequence.

Proof. Since (xn ) is an unbounded sequence, we can nd a positive integer n1 : jxn1 j > 1:
Since (xn ) is an unbounded sequence, we can nd a positive integer n2 : n2 > n1 ; and jxn2 j > maxf2; jxn1 jg:
Since (xn ) is an unbounded sequence, we can nd a positive integer n3 : n3 > n2 ; and jxn3 j > maxf3; jxn2 jg:
..
.
Since (xn ) is an unbounded sequence, we can nd a positive integer nk : nk > nk 1 ; and jxnk j > maxfk; xnk
..
.
Continuing in this fashion, we get a subsequence (xnk ) of (xn ) such that lim jxnk j = +1:

g:

k!1

Now, lim jxnk j = +1 =) (xnk ) has a subsequence xnkm : Either lim xnkm = +1; or lim xnkm =
m!1

k!1

m!1

1:

Say, lim xnkm = +1: Then xnkm is a properly divergent subsequence of (xn ) :
m!1
Thus, every unbounded sequence has a properly divergent subsequence.

122. Give examples of properly divergent sequences (xn ) and (yn ) with yn 6= 0 for all n 2 N such that
(a) (xn =yn ) is convergent;
(b) (xn =yn ) is properly divergent:

Proof. (a) Let (xn ) = (n) and (yn ) = (n) : Then both (xn ) and (yn ) are properly divergent.
Yet, (xn =yn ) = (1) which is convergent.
(a) Let (xn ) = n2 and (yn ) = (n) : Then both (xn ) and (yn ) are properly divergent.
Also, (xn =yn ) = (n) which is properly divergent:

123. Show that if xn > 0 for all n 2 N; then lim (xn ) = 0 if and only if lim (1=xn ) = +1:
n!1

n!1

62


Proof. Let R > 0 be a large real number. (Note that since xn > 0 for all n 2 N; we have jxn j = xn ):
Then, we have:
R arbitrary
lim (xn ) = 0 , 9N 2 N : xn < R1 for all n N , x1n > R for all n N
,
lim (1=xn ) = +1:
n!1
n!1

124. p
Establish the proper
sequences:
p divergence of the following
p
p
(a) ( n) ;
(b)
n+1 ;
(c)
n 1 ;
(d) n= n + 1 :

Proof. Recall that a sequence (xn ) of positive real numbers is properly divergent if and only if 8R > 0
9N = N (R) 2 N : xn > R for all n N:
This is the idea that we will use to show proper divergence in what follows.
(a) Let R > 0 be a large real number. Then there exists p
N 2 N : N > R2 :
p
Hence, for allpn N; we have n N > R2 and so n > R2 = R:
Thus, lim ( n) = +1:
n!1

2
(b) Let R > 0 be a large real number. Then there exists N 2 N : N +
p1 > R :
p
2
2
Hence, for allpn N; we have n + 1 N + 1 > R and so n + 1 > R = R:
Thus, lim
n + 1 = +1:
n!1

(c) Let R > 0 be a large real number. Then there exists N 2 N : N p1 > R2 :
p
Hence, for allpn N; we have n 1 N 1 > R2 and so n 1 > R2 = R:
Thus, lim
n 1 = +1:
n!1

p
p
(d) Note rst that
for any positive integer n; n2 > n2 1 = (n 1)(n + 1) and so n > n 1 n + 1:
p
n
Hence, pn+1
> n 1 for all n 2 N:
Let R > 0 be a large real number. Then there exists N 2 Np: N 1 >pR2 :
Hence, for all np N; we have n 1 N 1 > R2 and so n 1 > R2 = R:
n
Thus, pn+1
> n 1 > R for all n N:
p
Thus, lim
n 1 = +1:
n!1

125. Is the sequence (n sin n) properly divergent?

> 0 for x 2 ((2k 2) ; (2k 1) ) , k 2 N


Proof. No. Recall that the function sin x is
:
<0
for x 2 ((2k 1) ; 2k ) , k 2 N
Here is a graph to refresh your memory:

Since the length of each of the above intervals is ; we can nd an integer n within each interval.
Hence, the sequence (n sin n) has innitely many positive terms and innitely many negative terms.
This means that the sequence (n sin n) cannot be properly divergent.

126. Let (xn ) be properly divergent and let (yn ) be such that lim (xn yn ) belongs to R: Show that (yn )
n!1
converges to 0.

Proof. Say, lim (xn yn ) = a 2 R: Suppose to the contrary that (yn ) does not converge to 0.
n!1

Then 9 a subsequence (ynk ) of (yn ) such that jynk j r > 0 for some r and for all k 2 N:
Since (xn ) is properly divergent, lim (jxn j) = +1; and so lim (jxnk j) = +1:
n!1

k!1

jaj+1
r

Thus, we can nd K 2 N large enough so that jxnk j >


Now, for all k K; we have jxnk ynk j r jxnk j > jaj + 1:
63

for all k

K:

Hence, jxnk ynk j jaj > 1 for all k K; and so jxnk ynk aj > 1 for all k K: (Recall: jx yj jxj jyj):
Thus, we found a subsequence (xnk ynk ) of (xn yn ) that does not converge to a; a contradiction.
Therefore, (yn ) converges to 0.

127. Let (xn ) and (yn ) be sequences of positive numbers such that lim (xn =yn ) = 0:
n!1

(a) Show that if lim (xn ) = +1; then lim (yn ) = +1:
n!1

n!1

(b) Show that if (yn ) is bounded, then lim (xn ) = 0:


n!1

Proof. (a) Let zn = y1n : Then we have that (xn ) is properly divergent, and lim (xn zn ) = 0 belongs to R:
n!1

Hence, by the previous exercise, lim (zn ) = 0: Thus, by another previous exercise, we know that if we have
n!1

a sequence (zn ) of positive numbers, then lim (zn ) = 0 , lim (1=zn ) = +1:
n!1

n!1

But 1=zn = yn : Hence, lim (yn ) = +1:


n!1

(b) Since (yn ) is bounded, there is a positive real number M : yn M for all n 2 N:
Let > 0 be given. Since lim (xn =yn ) = 0; 9N 2 N large enough so that xn =yn < M for all n N:
n!1
Thus, xn < M yn
M
=
for all n N:
M
This is exactly what it means for lim (xn ) to be 0.
n!1

128. p
Investigate the convergence
or divergence of the following
sequences:
p
p
p
p
n2 + 2 ;
(b)
n= n2 + 1 ;
(c)
n2 + 1= n ;
(d) (sin n) :
(a)
p

Proof. (a) We have n2 + 2 > np2 for all n 2 N and so n2 + 2 > n for all n 2 N:
But lim n = +1: Hence, lim n2 + 2 = +1:
n!1

n!1

n
n
n
1
(b) We have 0 < n2 +1
n2 +1 < n2 =p
n for all n 2 N:
n= n2 + 1 = 0:
Hence, by the squeeze theorem, lim
n!1
q
q
p
2
2
(c) Note that for all n 2 N; we have npn+1 = n n+1 = n +
p
p
2
But lim n = +1: Hence, lim npn+1 = +1:

1
n

>

n:

n!1

n!1

(d) This sequence is bounded and divergent.

129. Let (xn ) and (yn ) be sequences of positive numbers such that lim (xn =yn ) = +1:
n!1

(a) Show that if lim (yn ) = +1; then lim (xn ) = +1:
n!1

n!1

(b) Show that if (xn ) is bounded, then lim (yn ) = 0:


n!1

Proof. Note that since lim (xn =yn ) = +1; and xn =yn > 0; we have that lim (yn =xn ) = 0:
n!1
n!1
Thus, this problem is a reformulation of the previous to the previous problem.

130. Show that if lim (an =n) = L; where L > 0; then lim (an ) = +1:
n!1
n!1

Proof. Since lim (an =n) = L > 0; we can assume, without loss of generality, that an > 0 for all n 2 N:
n!1

Also, since lim (an =n) = L; we can nd N 2 N large enough so that

n!1
Hence, L ann < ann L < 12 L for all
Thus, an > 12 Ln for all n N:
But lim 12 Ln = +1: Hence, lim (an )
n!1
n!1

N; and so,

an
n

>

1
2L

an
n

for all n

L < 12 L for all n

N:

N:

= +1:

131. Show that the convergence of a series is not aected by changing a nite number of its terms.
(of course, the value of the sum may be changed).

64

1
X

Proof. Let

xn be a convergent series, say

1
X

xn = L:
n=1
xn1 ; xn2 ; :::; xnk ; where

n=1

Pick any arbitrary nite terms, say


n1 < n2 <
< nk : Change them as you wish.
1
X
Let
yn be the resulting series. (of course, yn = xn for all n nk + 1):
n=1
!
!
nk
nk
nk
nk
nk
1
1
1
X
X
X
X
X
X
X
X
Then
yn =
yn +
yn =
yn +
xn =
yn + L
xn =
(yn xn ) + L:
n=1

Now,
Thus,

nk
X

n=1

(yn

n=1
1
X

n=1

n=nk +1

n=1

n=nk +1

n=1
nk
X

xn ) is a nite sum, (it might not be 0), say

n=1

(yn

n=1

xn ) = a:

n=1

yn = a + L < 1:

1
1
1
X
X
X
132. If
xn and
yn are convergent, show that
(xn + yn ) is convergent.
n=1

n=1

n=1

Proof. Let sn = x1 +
+ xn and tn = y1 +
+ yn .
1
1
X
X
That is, (sn ) and (tn ) are the sequences of partial sums of
xn and
yn ; respectively.
Note that the sequence of partial sums, (un ) ; of

1
X

n=1

n=1

(xn + yn ) is given by:

n=1

un = (x1 + y1 ) +
+ (xn + yn ) = (x1 +
+ xn ) + (y1 +
+ yn ) = sn + tn :
1
1
X
X
Thus, since
xn and
yn are convergent, the sequences (sn ) and (tn ) are convergent.
n=1

n=1

Hence, (un ) = (sn + tn ) = (sn ) + (tn ) is convergent, and so


Note that we have also proved that

1
X

(xn + yn ) =

n=1

1
X

1
X

(xn + yn ) is convergent.

n=1
1
X

xn +

n=1

yn :

n=1

X
1

133. (a) Show that the series


cos n is divergent.
n=1
X1
cos n
(b) Show that the series
is convergent.
2
n=1 n

Proof. (a) The sequence (cos n) is divergent as we saw in an earlier exercise (might have been (sin n)):
Hence, lim cos n 6= 0; and so the series cannot converge.
n!1

(b) We have

1
X

n=1

cos n
n2

1
X

n=1

1
n2

< 1: Thus, the series

1
X

cos n
n2

converges absolutely, and hence,

n=1

is convergent.

1
X
( 1)n
p
134. Show that the series
is convergent.
n
n=1

Proof. Instead of following the book, let us be more general and prove a much more general result:

1
1
X
X
Theorem. Suppose that
an and
bn satisfy the following properties:
n=1

n=1
n
X

(i) The sequence of partial sums, sn =

ak ; is a bounded sequence; and

k=1

(ii) The sequence (bn ) is decreasing and lim (bn ) = 0:


n!1

65

Then

1
X

an bn converges.

n=1

Proof. Since (sn ) is bounded, we can nd M : jsn j M for all n 2 N:


Let > 0 be given. Then 9N 2 N : bn < 2M for all n N:
Now, for any k > m > N; we have:
k
m
k
k
X
X
X
X1
see Rem ark (1) b elow
an bn
an bn =
an bn
=
sn (bn
n=1

n=1

k
X1

n=m+1

(bn

bn+1 )

n=m+1

+ sk bk

sm bm+1

bn+1 ) + bk + bm+1 = M j2bm+1 j = 2M bm+1 < 2M 2M = :

n=m+1

1
X

Thus, by Cauchys Criterion, the sequence of partial sums of


1
X

an bn is convergent and so

n=1

an bn is convergent.

n=1

n
Now, in our situation, (an ) = (( 1) ) ; (bn ) = p1n :

1
if n is odd
: Thus, jsn j 1:
0
if n is even
Also, (bn ) is decreasing and lim (bn ) = 0: Thus, the theorem applies to our case.
Note that sn = a1 +

+ an =

n!1

Remark (1).

k
X

an bn =

n=m+1

=
=

k
X

n=m+1
k
X1

sn bn

sn bn

k
X

sn

1 )bn

k
X1

sn bn+1

n=m
k
X1

n=m+1

k
X

n=m+1

sn bn+1

n=m+1

(sn

(sn

sn

1 )bn

k
X

n=m+1

sk bk +

k
X1

sn bn

n=m+1

+ sk bk

sm bm+1 =

sm bm+1 +

sn bn

n=m+1
k
X1

k
X1

sn (bn

bn+1 )

n=m+1

+ sk bk

sn

1 bn

n=m+1

sn bn+1

n=m+1

k
X

sm bm+1 :

(This result is known as the Abel partial summation formula).


Remark (2). Note that the above theorem implies, in particular, that

1
X

n=1

( 1)n
na

is convergent 8a > 0:

1
1
X
X
135. If
an with an > 0 is convergent, then is
a2n always convergent?
n=1

n=1

Either prove or give a counterexample.

1
X
Proof. Yes. Since an > 0 and
an is convergent, we get that lim an = 0:
n!1

n=1

Let N 2 N be large enough so that 0 < an < 1 for all n N:


Then 0 < a2n < an < 1 for all n N:
1
1
1
N
1
X
X
X
X1
X
Thus,
a2n <
an < 1; and so
a2n =
a2n +
a2n < 1:
n=N

n=1

n=N

n=1

n=N

a2n
n!1 an

This can also be done using the limit comparison test; since lim
we conclude that

1
X

n=1

= lim an = 0; and
n!1

1
X

n=1

an < 1;

a2n < 1:

66

136. If

1
X

an with an > 0 is convergent, then is

n=1

1
X
p

an always convergent?

n=1

Either prove or give a counterexample.

1
1
1
1
X
X
X
X
p
1
1
1
an =
an =
Solution. No. Take an = n2 : Then
n is divergent.
n2 is convergent, but
n=1

n=1

n=1

n=1

1
1
X
X
137. If
an with an > 0 is convergent, and if bn = (a1 + n +an ) for n 2 N; then show that
bn is always
n=1

n=1

divergent.

Proof. bn = (a1 + n +an ) =) nbn = a1 +


+ an and since an > 0 for all n; we get that a1 < nbn 8n 2 N:
a1
Thus, bn > n 8n 2 N; and so, b1 + b2 +
+ bn > a11 + a21 +
+ an1 = a1 (1 + 12 +
+ n1 ) 8n 2 N:
1
1
1
X
X
X
1
Thus,
bn > a1
bn = +1:
n and so, by the limit comparison test,
n=1

n=1

n=1

138. Determine a condition on jx 1j that will assure that:


(a) x2 1 < 21 ;
(b) x2 1 < 1=103 ;
1
2
(c) x
1 < n for a given n 2 N;
(d) x3 1 < n1 for a given n 2 N:

Solution. Note that if jx 1j < 1; then x < 2 and so jx + 1j < 3:


Since x2 1 = jx 1j jx + 1j ; we get that x2 1 < 3 jx 1j for all x 2 (0; 2):
Also, if jx 1j < 1; then x < 2 and so x2 + x + 1
x2 + jxj + 1 < 4 + 2 + 1 = 7:
3
2
3
Since x
1 = jx 1j x + x + 1 ; we get that x
1 < 7 jx 1j for all x 2 (0; 2):

(a) From what we did above, let jx 1j < 61 :


1
(b) From what we did above, let jx 1j < 3 10
3:
1
(c) From what we did above, let jx 1j < 3n :
1
(d) From what we did above, let jx 1j < 7n
:

139. Let c be a cluster point of A R and let f : A ! R: Prove that lim f (x) = L if and only if
x!c

lim jf (x) Lj = 0:
x!c

Proof. (=)) Suppose that lim f (x) = L and let > 0 be given.
x!c

Then there exists > 0 : 0 < jx cj < =) jf (x) Lj < :


Since jf (x) Lj = jjf (x) Lj 0j ; we get that 0 < jx cj <
This is exactly what it means for lim jf (x) Lj to be 0.

=) jjf (x)

Lj

0j < :

x!c

((=) Suppose that lim jf (x)


x!c

Lj = 0 and let

> 0 be given.

Then there exists > 0 : 0 < jx cj < =) jjf (x) Lj 0j < :


Since jf (x) Lj = jjf (x) Lj 0j ; we get that 0 < jx cj < =) jf (x) Lj < :
This is exactly what it means for lim f (x) to be L.
x!c

140. Let f : R ! R and let c 2 R: Show that lim f (x) = L if and only if lim f (x + c) = L:
x!c
x!0

Proof. (=)) Suppose that lim f (x) = L, let > 0 be given, and set y = x c.
x!c

( )

:
Then there exists > 0 : 0 < jx cj < =) jf (x) Lj <
In terms of y; ( ) can be rewritten as: 0 < jyj = jy 0j < =) jf (y + c) Lj < :
Thus, lim f (y + c) = L: Note that we can replace y by x since they are dummy variables.
y!0

((=) Suppose that lim f (x + c) = L, let


x!0

> 0 be given, and set y = x + c.

67

Then there exists > 0 : 0 < jx 0j < =) jf (x + c) Lj <


:
In terms of y; ( ) can be rewritten as: 0 < jy cj < =) jf (y) Lj < :
Thus, lim f (y) = L: Note that we can replace y by x since they are dummy variables.
y!c

141. Let I = (0; a) where a > 0; and let g(x) = x2 for x 2 I: For any points x; c 2 I; show that
g(x) c2
2a jx cj : Use this inequality to prove that lim x2 = c2 for any c 2 I:
x!c

Proof. For any x; c 2 I = (0; a); we have that jx + cj jxj + jcj = x + c < a + a = 2a:
Hence, for any x; c 2 I = (0; a); g(x) c2 = x2 c2 = jx + cj jx cj < 2a jx cj :
Now, x c 2 I; let > 0 be given and set = 2a > 0:
Then 0 < jx cj < = 2a =) x2 c2 < 2a jx cj < 2a 2a = :
This is exactly what it means for lim x2 to be c2 :
x!c

142. Let I be an interval in R; let f : I ! R, and let c 2 I: Suppose there exists constants K and L such that
jf (x) Lj < K jx cj for x 2 I: Show that lim f (x) = L:
x!c

Proof. Let > 0 be given and set = K .


Than for all x 2 I with 0 < jx cj < ; we have that jf (x) Lj < K jx cj < K = :
This is exactly what it means for lim f (x) to be L:
x!c

143. Show that lim x3 = c3 for any c 2 R:


x!c

Proof. Let c 2 R: Then c is a cluster point of the intervel I = (c 1; c + 1):


2
Also, for all x 2 I; we have that x2 + cx + c2
x2 + jcj jxj + c2 < (c + 1) + jcj jc + 1j + c2 < 3 c2 + jcj + 1 :
Note that x3 c3 = x2 + cx + c2 jx cj :
Now, if we let L = c3 and K = 3 c2 + jcj + 1 ; then we have: x3 L < K jx cj for all x 2 I:
Thus, by the previous exercise, we get that lim x3 = c3 :
x!c
p
p
144. Show that lim x = c for any c > 0:
x!c

Proof. Let c > 0: Then c is a cluster


point
of the
p
p
p intervel
p I = (0; 2c):1
Also, for all x 2 I; we have that x + c > 0 + c = c; and so px+pc < p1c .
p
p
1p
Note that j x
cj = px+
jx cj :
c
p
p
Now, if we let L = c and K = p1c ; then we have: j x Lj < K jx cj for all x 2 I:
p
p
Thus, by the previous to the previous exercise, we get that lim x = c:
x!c

145. Use the denition of limit to show that


x+5
(a) lim x2 + 4x = 12;
(b) lim 2x+3
= 4:
x!2
x! 1

Proof. (a) Note that 2 is a cluster point of the intervel I = (1; 3); and that x2 + 4x 12 = jx + 6j jx 2j :
For all x 2 I; we have jx + 6j < 9:
Let > 0 be given and set = 9 : Then whenever x 2 I with 0 < jx 2j < ; we have that
x2 + 4x 12 = jx + 6j jx 2j < 9 = : Thus, lim x2 + 4x = 12:
x!2

(b) Note that

1 is a cluster point of the intervel I = ( 2; 0); and that


1
j2x+3j

x+5
2x+3

4 =

7
j2x+3j

For all x 2 I; we have j2x + 3j > j2( 2) + 3j = 1; and so


< 1:
Let > 0 be given and set = 7 : Then whenever x 2 I with 0 < jx + 1j < ; we have that
x+5
2x+3

jx + 1j :

7
x+5
jx + 1j < 7 = : Thus, lim 2x+3
= 4:
4 = j2x+3j
x! 1

146. Show that the following limits do not exist:


(a) lim x12 (x > 0);
(b) lim p1x (x > 0);
x!0

x!0

68

(c) lim (x+sgn(x));


(d) lim sin 1=x2
x!0
x!0

Proof. (a) Let f (x) = x12 for x > 0: Let xn = n1 ; n 2 N: Then lim (xn ) = 0:
n!1

However, the sequence (f (xn )) =


Hence,

lim 12
x!0 x

1
1=n2

= n

does not converge in R:

does not exist.

(b) Let f (x) =

p1
x

for x > 0: Let xn =

1
n2 ;

p1 2
1=n

However, the sequence (f (xn )) =

n 2 N: Then lim (xn ) = 0:


n!1

= (n) does not converge in R:

Hence, lim p1x does not exist.


x!0

(c) Let f (x) = x+sgn(x): Let xn =

( 1)n
n ;

n 2 N: Then lim (xn ) = 0:


n!1
( 1)n
+(
n

However, the sequence (f (xn )) = (xn + sgn(xn )) =


1) has a subsequence that converges
to 1 and a subsequence that converges to 1, for n even and n odd, respectively.
Hence, (f (xn )) does not converge in R:
Thus, lim (x+sgn(x)) does not exist.
x!0

p 1 ; and yn = p 1
; n 2 N: Then lim (xn ) = lim (yn ) = 0:
2 n
2 n+ 2
n!1
n!1
sin 1=x2n = (sin 2 n) = (0), and the sequence (f (yn )) = sin 1=yn2

(d) Let f (x) = sin 1=x2 : Let xn =

However, the sequence (f (xn )) =


= sin 2 n + 2 = (1) :
Hence, lim sin 1=x2 does not exist.
x!0

147. Suppose the function f : R ! R has limit L at 0; and let a > 0: If g : R ! R is dened by
g(x) = f (ax) for x 2 R; show that lim g(x) = L:
x!0

Proof. Let (xn ) be any sequence of points such that lim xn = 0: Then (axn ) also converges to 0.
n!1

Since lim f (x) = L; we get, by the sequential criterion for limits, that lim f (ax) = L:
x!0

x!0

but lim f (ax) = lim g(x):


x!0
x!0

x
if x 2 Q
148. Let f : R ! R be dened by: f (x) =
:
0
if x 2
=Q
(a) Show that f has a limit at x = 0:
(b) Use a sequential argument to show that if c 6= 0; then f does not have a limit at c:

Proof. (a) We claim that lim f (x) = 0: Let > 0 be given and set = :
x!0

Then for all x 2 R with 0 < jxj = jx


Hence, lim f (x) = 0:

0j < ; we have that jf (x)

f (0)j = jf (x)

0j = jf (x)j

jxj <

= :

x!0

(b) Recall that both Q and RnQ are dense in R. Thus, if c 6= 0; then we may choose two sequences (xn ) and
(yn ) of points converging to c so that xn 2 Q and yn 2
= Q for all n 2 N:
Since yn 2Q
=

Since x 2Q

Then lim f (xn )


=n
lim xn = c: Also, lim f (yn )
=
lim 0 = 0:
n!1
n!1
n!1
n!1
Since c 6= 0; we get that f does not have a limit at c:

149. Determine the following limits:


2
(a) lim (x + 1) (2x + 3)
(x 2 R);
(b) lim xx2 +22 (x > 0);
x!1

x!1

1
1
(c) lim x+1
(x > 0);
(d) lim xx+1
(x 2 R):
2
2x
x!2
x!0 +2

Solution. (a) Since lim x = 1; lim 1 = 1; lim 2x = 2 lim x = 2; and lim 3 = 3; we get that
x!1

x!1

x!1

x!1

69

x!1

lim (x + 1) =

lim x + lim 1 = 1 + 1 = 2; and lim (2x + 3) = 2 lim x + lim 3 = 2 + 3 = 5:

x!1

x!1

x!1

Thus, lim (x + 1) (2x + 3) =

x!1

lim (x + 1)

x!1

x!1

lim (2x + 3) = 2 5 = 10:

x!1

x!1

p
(b) Note that 1 is a cluster point of the interval (0; 2):
Also, lim x2 + 2 = lim x2 + lim 2 = 1 + 2 = 3; and lim x2
x!1

=1

2=

x!1

1 6= 0:
2

Hence, lim xx2 +22 =


x!1

lim (x2 +2)

x!1

lim (x2 2)

x!1

x!1

3
1

lim x2

2 =

x!1

x!1

x!1

lim 2

x!1

3:

(c) Note that 2 is a cluster point of the interval (0; 1):


lim 1

1
Also, lim x+1
=
x!2

x!2

x!2

x!2

1
x+1

Hence, lim

1
= 31 ; and lim 2x
=

x!2

lim (x+1)
1
2x

lim 1
x!2 x+1

lim 1

x!2

lim (2x)

x!2

lim 1
x!2 2x

= 14 :
1
3

1
4

1
12 :

(d) Note that x2 + 2 > 0 for all x 2 R; lim (x + 1) = 1; and lim x2 + 2 = 2:


x!0

lim (x+1)

lim x+1
2
x!0 x +2

x!0

1
2:

x!0

Hence,
= lim (x2 +2) =
x!0

150. Determine
the following limits:
q
2
2x+1
(a) lim x+3
(x > 0);
(b) lim xx 24 (x > 0);
x!2

x!2 p

1
(c) lim (x+1)
(x > 0);
(d) lim xx 11 (x > 0):
x
x!0
x!1
r
q
q
q
lim (2x+1)
2x+1
2x+1
5
Solution. (a) Using an exercise below, lim x+3 = lim x+3 = x!2
lim (x+3) =
5 = 1:
x!2

(b) lim xx
x!2

4
2

(c) lim (x+1)


x

lim (x x2)(x+2)
2
x!2
2

= lim x

x!0

x 1
x!1 x 1

(d) lim

x!0

= lim

x!1 (

x!2

= lim (x + 2) = 4:
x!2

= lim x(x+2)
= lim (x + 2) = 2:
x

+2x+1 1
x

x!2

x!0

x 1
p
x 1)( x+1)

x!0

1
= lim px+1
=
x!1

lim 1

x!1

lim x +

lim 1

x!1

1
1+1

= 12 :

x!1

p p
1+3x
151. Find lim 1+2x
where x > 0:
x+2x2
x!0

Solution. lim

x!0

= lim

x!0

p
1+2x
1+3x
x+2x2

= lim

p x
p
x(1+2x)( 1+2x+ 1+3x)

x!0

= lim

x!0

p
1+2x
1+3x
x+2x2

p
p
p1+2x+p1+3x
1+2x+ 1+3x

p 1 p
(1+2x)( 1+2x+ 1+3x)

1
2:

(1+2x) (1+3x)
p
p
(x+2x2 )( 1+2x+ 1+3x)

= lim

=@

x!0

lim 1

lim (1+2x)

x!0

x!0

lim (1+2x)+

x!0

lim (1+3x)

x!0

1
A

152. Prove that lim cos (1=x) does not exist but that lim x cos (1=x) = 0:
x!0
x!0

1
Proof. Let f (x) = cos (1=x) : Let xn = 2 1n ; and yn = 2 n+
; n 2 N: Then lim (xn ) = lim (yn ) = 0:
n!1

n!1

However, the sequence (f (xn )) = (cos (1=xn )) = (cos 2 n) = (1), and the sequence (f (yn )) = (cos (1=yn ))
= cos 2 n + 2 = (0) : Hence, lim cos (1=x) does not exist.
x!0

Now, note that for all x 2 R; we have 1 cos (1=x) 1 and so x x cos (1=x) x:
Since lim x = 0; and lim x = 0; the squeeze theorem implies that lim x cos (1=x) = 0:
x!0

x!0

x!0

70


153. Let f; g be dened on A R to R; and let c be a cluster point of A: Suppose that f is bounded
on a neighborhood of c and that lim g(x) = 0: Prove that lim f (x) g (x) = 0:
x!c
x!c

Proof. Let V be a neighborhood of c on which f is bounded, i.e., jf (x)j M for all x 2 V:


Let > 0 be given. Since lim g(x) = 0; 9 > 0 : whenever 0 < jx cj < ; we have that jg(x)j < M :
x!c

We may choose small enough so that (c


; c + ) V:
Hence, whenever 0 < jx cj < ; we have jf (x)g(x)j M jg(x)j < M M = :
Therefore, lim f (x) g (x) = 0:
x!c

154. Use the denition of the limit to prove that if f; g are dened on A R to R; c is a cluster point of A;
lim f (x) = L; and lim g(x) = M; then lim [f (x) + g(x)] = L + M:
x!c
x!c
x!c

Proof. Let > 0 be given.


Since lim f (x) = L; 9 1 > 0 : whenever 0 < jx cj < 1 ; we have that jf (x) Lj < 2 :
x!c

Since lim g(x) = M; 9


x!c

> 0 : whenever 0 < jx

cj <

2;

we have that jf (x)

Mj < 2:

Let = minf 1 ; 2 g:
Then whenever 0 < jx cj < ; we have that j(f (x) + g(x)) (L + M )j = j(f (x) L) + (g(x) M )j
jf (x) Lj + jg(x) M j < 2 + 2 = :
Therefore, lim [f (x) + g(x)] = L + M:
x!c

155. Let n 2 N be such that n 3: Derive the inequality x2 xn x2 for 1 < x < 1: Then use the fact
that lim x2 = 0 to show that lim xn = 0:
x!0
x!0

k
Proof. Note that if jxj 1; and k 2 N; then xk = jxj
1 and so 1 xk 1:
2
k+2
2
Thus, x
x
x for all k 2 N and all x 2 ( 1; 1):
Now, lim x2 = 0; and lim x2 = 0:
x!0

x!0

Therefore, by the squeeze theorem, lim xn = 0 for all n 3:


x!0

156. Give examples of functions f and g such that f and g do not have limits at a point c; but such that both
f + g and f g have limits at c:

Solution. (i) Let f (x) = x1 and g(x) = x1 :


Then lim f (x) and lim g(x) do not exist, but lim (f + g)(x) = lim x1 x1 = lim 0 = 0:
x!0

x!0

x!0

x!0

x!0

1 for x < 0
:
1
for x 0
Then lim f (x) = lim g(x) = 1; and lim f (x) = lim g(x) = 1: Thus, lim f (x) and lim g(x) do not exist.

(ii) Let f (x) = g(x) =


x!0

x!0

x!0+

x!0+

x!0

x!0

( 1) ( 1) = 1 for x < 0
Yet, f g(x) =
1; and so lim f g = lim 1 = 1 exists.
(1) (1) = 1
for x 0
x!0
x!0

157. Determine whether the following limits exist in R :


(a) lim sin 1=x2
(x 6= 0);
(b) lim x sin 1=x2
(x 6= 0);
x!0
x!0 p
(c) lim sgn(sin (1=x))
(x 6= 0);
(d) lim x sin 1=x2
(x > 0):
x!0
x!0

Solution. (a) Let f (x) = sin 1=x2 for x 6= 0: Let xn = p21 n ; and yn = p 1
; n 2 N:
2 n+ 2

Then lim (xn ) = lim (yn ) = 0:However, the sequence (f (xn )) = sin 1=x2n
n!1

n!1

and the sequence (f (yn )) = sin 1=yn2


2

Hence, lim sin 1=x


x!0

= sin 2 n +

does not exist .

Here is a graph of f (x) = sin 1=x2 :

71

= (1) :

= (sin 2 n) = (0),

(b) Let f (x) = x sin 1=x2 for x 6= 0: Since 1 sin z


jxj f (x) = x sin 1=x2
jxj for all x 2 R; x 6= 0:

1 for all z 2 R; we have the inequality

Since lim jxj = 0; the squeeze theorem implies that lim x sin 1=x2 = 0 :
x!0

x!0

Here is a graph of f (x) = x sin 1=x2 :

(c) Let f (x) =sgn(sin (1=x)) for x 6= 0: Let xn =


Then lim (xn ) = lim (yn ) = 0:
n!1

1
2 n+ 2

; and yn =

1
2 n+ 32

; n 2 N:

n!1

= (sgn (1)) = (1),


However, the sequence (f (xn )) = (sgn (sin (1=xn ))) = sgn sin 2 n + 2
3
and the sequence (f (yn )) = (sgn (sin (1=yn ))) = sgn sin 2 n + 2
= (sgn ( 1)) = ( 1) :
Hence, lim sgn(sin (1=x)) does not exist .
x!0

p
p
(d) Let f (x) = x sin 1=x2 for x > 0: Since 1 sin z 1 for all z 2 R; and x > 0 for x > 0;
p
p
p
we have the inequality
x f (x) = x sin 1=x2
x for all x 2 R; x > 0:
q
p
p
Since lim x = lim x = 0; the squeeze theorem implies that lim x sin 1=x2 = 0 :
x!0

x!0

Here is a graph of f (x) =

x!0

x sin 1=x2 :

72


158. Let A R; let fp: A ! R; and let c 2 R be a cluster pointpof A: In addition,
suppose that f (x) 0
p
for all x 2 A; and let f be the function dened for x 2 A by
f (x) = f (x):
q
p
If lim f (x) exists, prove that lim f (x) = lim f (x):
x!c
x!c
x!c
p
Proof. Say, lim f (x) = L: Since f (x) 0 for all x 2 A; L 0: Thus, L is well-dened.
x!c

Let > 0 be given.


p
Since lim f (x) = L; we can nd > 0 : whenever 0 < jx cj < ; we have jf (x) Lj < L :
x!c
p
p
p
p
Now, jf (x) Lj =
f (x)
L
f (x) + L :
p
p
p
p
p
f (x) + L =
f (x) + L
L since f (x) 0 for all x 2 A:
Also,
p
p
p1 jf (x)
Hence,
f (x)
L = pjf (x) Lj
Lj for all x 2 A:
p
L
f (x)+ L
p
p
p
p1 jf (x)
Therefore, whenever 0 < jx cj < ; we have
f (x)
L
Lj < p1L L = :
L
p
p
This is exactly what it means for lim f (x) to be L:
x!c

1=2
159. Let f (x) = jxj
for x 6= 0: Show that lim f (x) = lim f (x) = +1:
x!0+
x!0

Proof. Let > 0 be given and let = 12 > 0: Then for all x with 0 < jxj < = 12 ; we have that
p
0 < jxj < 1 =) f (x) = p1 > : Thus, lim f (x) = lim f (x) = +1:
jxj

Here is a graph of f (x) = jxj

x!0+

x!0

1=2

73


160. Let c 2 R and let f be dened for x 2 (c; 1) and f (x) > 0 for all x 2 (c; 1):
Show that lim f (x) = 1 if and only if lim 1=f (x) = 0:
x!c
x!c

Proof. (=)) Suppose that lim f (x) = 1 and let > 0 be given.
x!c

Then 9 > 0 : whenever 0 < jx


Thus, lim 1=f (x) = 0:

cj < ; we have f (x) >

=)

1
f (x)

< :

x!c

((=) Suppose that lim 1=f (x) = 0 and let


x!c

> 0 be given.

1
Then 9 > 0 : whenever 0 < jx cj < ; we have f (x)
< 1 =) f (x) > :
Thus, lim f (x) = 1:
x!c

161. Evaluate the following limits, or show that they do not exist.
(b) lim x x 1 (x 6= 1);
(a) lim x x 1 (x 6= 1);
x!1+

x!1

x+2
p
x!0+ x
p
lim x+1
x!0 px
lim px 5
x!1 x+3

(c) lim
(e)

x+2
p
x!1 x
p
(f ) lim x+1
x!1 px
(h) lim pxx+xx
x!1

(x > 0);
(x >

(d) lim

1);

(x > 0);
(x > 0);

(g)
(x > 0);
(x > 0):

Solution. (a) We claim that lim x x 1 = 1 and so it does not exist in R:


x!1+

To prove this, let > 1 be given. Set = 1 1 > 0:


Then if 0 < x 1 < = 1 1 ; we have x 1 1 >
1 =)
x
Thus, lim x 1 = 1:

1
x 1

+1>

=)

x
x 1

> :

> 0:

x!1+

(b) By what we did in part (a), we deduce that lim x x 1 does not exist.
x!1

(c) We claim that lim

x!0+

x+2
p
x

= 1 and so it does not exist in R:

p > p2 : Let
To prove this, note that for x > 0; x+2
> 0 be given. Set
x
x
p
4
2
2
p > :
Then if 0 < x < = 2 ; we have x < =) px > =) x+2
x
x+2
p
Thus, lim x = 1:

x!0+

(d) We claim that lim

x+2
p
x

= 1 and so it does not exist in R:


p
p
p =
To prove this, note that for x > 0; x+2
x + p2x > x: Let > 0 be given.
x
p
p >
Then for all x > 2 ; we have that x > =) x+2
x
p = 1:
Thus, lim x+2
x
x!1

x!1

(e) We claim that lim

x!0

x+1
x

does not exist in R:

To prove this, note that for x > 0; we have that

x+1
x

>

p1 :
x

(This is because

By a similar argument to the one given in part (c), we conclude that lim

x!0+

(f ) We claim that lim

x!1

x+1
x

= 0:

To prove this, note that for x > 0; we have that 0 <


Since lim

x!1

p1
x

x+1
x

= 0; the squeeze theorem implies that lim

(g) We claim that


Note that

p
px 5
x+3

x!1

p
lim pxx+35
x!1

1 =

x
p1 :
x =
x
x+1
= 0:
x

<

= 1:

p 8
x+3

p8 :
x+3

To prove our claim, let 0 < < 1 be given and let

=
74

x+1
x

x2 + 1 >
= 1:

x2 = x):

Then for x >

p
Thus, for x > ; pxx+35
p
Therefore, lim pxx+35 =
x!1

; we have that

1 =

p8
x+3

x>

3 =)

x+3
8

>

=)

p8
x+3

< :

>

=)

p2
x+1

< =)

< :

1:

p
px x =
1:
x!1 x+x
p
( 1) = pxx+xx +

(h) We claim that lim


Note that

p
px x
x+x

1 =

p
p2 x
x+x

To prove our claim, let 0 < < 1 be given and let


p
2
Then for x > = 2 1 ; we have that x > 2
=)

p2 x
x+x

< : Thus, for x > ;


p
lim pxx+xx
x!1

p
px x
x+x

( 1) =

p
p2 x :
x+x
2

1 =)
p

p2 x
x+x

x+1
2

p2
x+1

p
px
x

<

< :

Therefore,
= 1:

162. Suppose that f and g have limits in R as x ! 1 and that f (x) g(x) for all x 2 (a; 1):
Prove that lim f (x)
lim g(x):
x!1
x!1

Proof. Let L = lim f (x); and M = lim g(x): Suppose to the contrary that L > M:
x!1

Then 9 > 0 : M + < L

x!1

: Since L = lim f (x) and M = lim g(x); we can nd K1 > a and K2 > a :
x!1

x!1

whenever x > K1 we have jf (x) Lj < and whenever x > K2 we have jg(x) M j < :
Thus, for all x > K = maxfK1 ; K2 g; we have that jf (x) Lj < and jg(x) M j < :
Hence, for all x > K; we have that f (x) > L
> M + > g(x).
This is a cotradiction to the fact that f (x) g(x) for all x 2 (a; 1):
Therefore, L M:

163. Let f be dened on (0; 1) to R: Prove that lim f (x) = L if and only if lim f (1=x) = L:
x!1
x!0+

Proof. (=)) Suppose that lim f (x) = L and let > 0 be given.
x!1

Then 9K > 0 : whenever x > K; we have that jf (x) Lj < : Let =


Thus, whenever 0 < x < ; we have x1 > K and so f ( x1 ) L < :
This is exactly what it means for lim f (1=x) to be L:

1
K:

x!0+

((=) Suppose that lim f (1=x) = L and let


x!0+

> 0 be given.

Then 9 > 0 : whenever 0 < x < ; we have that jf (1=x) Lj < : Let K = 1 :
Thus, whenever x > K; we have x1 < and so jf (x) Lj < :
This is exactly what it means for lim f (x) to be L:
x!1

164. Show that if f : (a; 1) ! R is such that lim xf (x) = L; where L 2 R; then lim f (x) = 0:
x!1
x!1

Proof. Since lim xf (x) = L; we can nd > 0 : whenever x > ; jxf (x) Lj < 1:
x!1

Since jxf (x)j jLj < jxf (x)


=) jf (x)j < 1+jLj
jxj :

Lj ; we deduce that whenever x > ; jxf (x)j

Summing up, we have shown that whenever x > ; we have 0 < jf (x)j <

jLj < 1 =) jxf (x)j < 1 + jLj

1+jLj
jxj :

Now, since lim 1+jLj


= 0; the squeeze theorem implies that lim f (x) = 0:
x!1 jxj
x!1

165. suppose that lim f (x) = L; where L > 0; and that lim g(x) = 1: Show that lim f (x)g(x) = 1:
x!c
x!c
x!c
If L = 0; show by example that this conclusion may fail.

Proof. Let > 0 be given.


Since lim f (x) = L and L > 0; 9 1 > 0 : 0 < jx cj < 1 =) f (x)
> 0:
x!c

75

Since lim g(x) = 1; 9

x!c

> 0 : 0 < jx

Let = minf 1 ; 2 g: Then 0 < jx


Thus, lim f (x)g(x) = 1:

cj <

cj <

=) g(x) >

:
= :

=) f (x)g(x) >

x!c

For an example, let f (x) = 0 and g(x) =

1
jxj

for x 6= 0: Then lim f (x) = 0; lim g(x) = 1:


x!0

x!0

Yet, for all x 6= 0; f (x)g(x) = 0; and so lim f (x)g(x) = 0:


x!0

166. Find functions f and g dened on (0; 1) such that lim f = 1 and lim g = 1; and lim (f g) = 0:
x!1

x!1

x!1

Can you nd such functions, with g(x) > 0 for all x 2 (0; 1); such that lim f =g = 0?
x!1

Solution. Let f (x) = g(x) = x: Then lim f = 1, lim g = 1; and lim (f g) = lim 0 = 0:
x!1

x!1

x!1

x!1

The answer to the question is no. If g(x) > 0 for all x 2 (0; 1); lim f = 1, lim g = 1; and lim (f
x!1

f
g

then we can write


Now, Since lim (f
x!1

x!1

x!1

g) = 0;

+ 1:
f g
x!1 g

g) = 0 and lim g = 1; we get that lim


x!1

= 0:

Hence, lim f =g = 0 + 1 = 1 6= 0:
x!1

167. Prove the following Sequential Criterion for Continuity: A function f : A ! R is continuous at
a point c 2 A if and only if for every sequence (xn ) in A that converges to c; the sequence (f (xn ))
converges to f (c):

Proof. A function f : A ! R is continuous at a point c 2 A if and only if lim f (x) = f (c):


x!c

Now, by the sequential criterion for limits, we get that lim f (x) = f (c) if and only if for every sequence (xn )
x!c

in A that converges to c; the sequence (f (xn )) converges to f (c):

168. Let f be dened for all x 2 R; x 6= 2; by f (x) = (x2 + x 6)=(x 2): Can f be dened at x = 2
in such a way that f is continuous at this point?

Proof. Note that f (x) = (x2 + x 6)=(x 2) = (x 2)(x + 3)=(x 2) = x + 3: This suggests that we dene
f (2) = 5 and f will be continuous at x = 2:

169. Let f : R ! R be continuous at c and let f (c) > 0: Show that there exists a neighborhood V (c)
of c such that if x 2 V (c); then f (x) > 0:

Proof. Let = f (c)


> 0 : whenever jx cj < ; we
2 > 0: Since f : R ! R be continuous at c; there exists
f (c)
have that jf (x) f (c)j < = 2 : In other words, for all x 2 V (c); we have f (c)
f (c) < f (c)
2 < f (x)
2 or
f (c)
equivalently, f (c)
<
f
(x)
<
3
and
so
f
(x)
>
0:
2
2

170. Let f : R ! R be continuous on R and let S = fx 2 R : f (x) = 0g be the zero set of f: If (xn )
is in S and x = lim (xn ) ; show that x 2 S:
n!1

Proof. Since x = lim (xn ) and f is continuous at x; the sequential criterion for continuity implies that
n!1

f (x) = lim (f (xn )) : But xn 2 S =) f (xn ) = 0 for all n 2 N: Hence, f (x) = lim (f (xn )) = lim (0) = 0:
n!1
n!1
n!1
This is exactly what it means for x to be in S:

171. Let K > 0 and let f : R ! R satisfy the condition jf (x) f (y)j K jx yj for all x; y 2 R: Show
that f is continuous at every point c 2 R:

Proof. Let c 2 R and let > 0 be given. Then for = K ; the following is true:
8 x 2 R with jx cj < ; we have that jf (x) f (c)j K jx cj < K K = : Thus, f is continuous at c:
But since our choice of c 2 R was arbitrary, we conclude that f is continuous at every point c 2 R:
76


172. Suppose that f : R ! R is continuous on R and that f (r) = 0 for every rational number r: Prove
that f (x) = 0 for all x 2 R:

Proof. Suppose, to the contrary, that there is a point x0 2 R : f (x0 ) 6= 0: Since f is continuous on R; we
get that, in particular, f is continuous at x0 : Let (rn ) be a sequence of rational numbers that converges to x0 :
Note that f (rn ) = 0 for all n 2 N:
Now, by the sequential criterion for continuity, f (x0 ) = lim (f (rn )) = lim (0) = 0; a contradiction.
n!1

n!1

Hence, f (x) = 0 for all x 2 R:

173. Determine the points of discontinuity of the following functions and state which theorems are used
in each case.
p
p
2
(x 2 R)
(b) g(x) = x + x (x 0)
(a) f (x) = x x+2x+1
2 +1
p
p
1+jsin xj
(c) h(x) =
(x 6= 0)
(d) k(x) = cos 1 + x2 (x 2 R)
x

Proof. (a) Since both f1 (x) = x2 + 2x + 1 and f2 (x) = x2 + 1 are continuous everywhere on R; and
x2 + 1 1; we get that f (x) is continuous everywhere on R:
p
p
p
(b) Since both g1 (x) = x + x and g2 (x) = x are continuous on [0; 1) and x + x
we get that g2 g1 = g is continuous on [0; 1):
(c) Since h1 (x) = 1 + jsin xj is continuous on R with h1 (x)
we get that h2 h1 is continuous on R:

1; and h2 (x) =

Now, since h3 (x) = x is continuous on R; we get that h(x) =

h2 h1 (x)
h3 (x)

0 for all x

0;

x is continuous on [0; 1);

1+jsin xj
x

is continuous for x 6= 0:

p
(d) Since k1 (x) = 1 + x2 is continuous on R with k1 (x) 1; and k2 (x) = x is continuous on [0; 1);
we get that k2 k1 is continuous on R:
p
Now, since k3 (x) = cos x is continuous on R; we get that k(x) = k3 k2 k1 (x) = cos 1 + x2 is
continuous for x 6= 0:

174. Show that if f : A ! R is continuous on A R and if n 2 N; then the function f n dened by:
n
f n (x) = (f (x)) for x 2 A; is continuous on A:

Proof. Since the function g(x) = xn is continuous on R for all n 2 N; and since f : A ! R is continuous
n
on A R and f (A) R; we get that g f (x) = f n (x) = (f (x)) is continuous on A:

175. Let g be dened on R by g(1) = 0; and g(x) = 2 if x 6= 1; and let f (x) = x + 1 for all x 2 R:
Show that lim g f 6= g f (0): Why doesnt this contradict Theorem 5.2.6?
x!0

0 if x = 0
Proof. g f (x) = g(x + 1) =
: Thus, lim g f (x) = 2:
2 if x 6= 0
x!0
Now, g f (0) = g(1) = 0: Thus, lim g f 6= g f (0):
x!0

This doesnt contradict Theorem 5.2.6 because g is not continuous on f (R) = R:

176. Give an example of a function f : [0; 1] ! R that is discontinuous at every point of [0; 1] but
such that jf j is continuous on [0; 1]:

1 if x 2 [0; 1] \ Q
Proof. Let f (x) =
: Then, by the sequential criterion for continuity, we see that
1 if x 2
= [0; 1]nQ
f is not continuous at any point x 2 [0; 1]: (If x is rational, approach it by a sequence of irrational points and
if x is irrational, approach it with a sequence of rational points).
Note that jf (x)j = 1 for all x 2 [0; 1] is a continuous function.

77

177. Let f; g be continuous from R to R; and suppose that f (r) = g(r) for all rational numbers r: Is it
true that f (x) = g(x) for all x 2 R?

Proof. Yes, it is true and here is the proof. Let h(x) = (f g) (x) = f (x) g(x) for x 2 R:
Then h is continuous on R since both f and g are. Moreover, h(x) = 0 for all x 2 Q:
Hence, by problem 172 above, we must have h(x) = 0 for all x 2 R:
That is, f (x) = g(x) for all x 2 R:

178. Let f : R ! R is continuous on R; and let P = fx 2 R : f (x) > 0g: If c 2 P; show that there
exists a neighborhood V (c) P:

Proof. Let c 2 P: Then f (c) > 0: We saw, in problem 169, that there exists a neighborhood V (c) of c
such that if x 2 V (c); then f (x) > 0: Thus, if x 2 V (c); then x 2 P: Hence, V (c) P:

179. Let I = [a; b] and let f : I ! R be a continuous function such that f (x) > 0 for each x 2 I:
Prove that there exists a number > 0 such that f (x)
for each x 2 I:

Proof. By the Maximum-Minimum Theorem, f attains its minimum on I = [a; b]; say at a point c 2 [a; b]:
(There might, of course, be more than one global minimum).
Thus, f (x) f (c) > 0 for all x 2 [a; b]: Let = f (c):

180. Let f be continuous on the interval [0; 1] to R and such that f (0) = f (1): Prove that there exists
a point c 2 [0; 21 ] such that f (c) = f (c + 12 ): Conclude that there are, at any time, antipodal points
on the earths equator that have the same temperature. (Hint: consider g(x) = f (x) f (x + 12 )):

Proof. Let g(x) = f (x) f (x + 12 ): Then g(0) = f (0) f ( 12 ) and g( 12 ) = f ( 12 ) f (1):


If f ( 12 ) = f (1); then we are done and our point c = 0 or c = 21 : (both will work).
If f ( 21 ) 6= f (1); then, since f (0) = f (1); either g(0) < 0 < g( 12 ) or g(0) > 0 > g( 12 ):
Thus, by the Location of Roots Theorem, there exists a number c 2 (0; 12 ) : g(c) = 0:
In other words, there exists a number c 2 (0; 21 ) : f (c) = f (c + 12 ):
Now, let R be the radius of earth. A standard parametrization of the circle x2 + y 2 = R2 ; "the equator",
is given by r(x) = (R cos (2 x) ; R sin (2 x)) for x 2 [0; 1]:
The Temperature T : Equator ! R is a continuous function and hence, f = T r : [0; 1] ! R is continuous.
Note that f (0) = f (1) and so, by above, there exists a point c 2 [0; 12 ] such that f (c) = f (c + 12 ):
That is, T r(c) = T r(c + 21 ) =) T ((R cos (2 c) ; R sin (2 c))) = T ((R cos (2 c + ) ; R sin (2 c + ))) :
But cos (2 c + ) = cos (2 c) ; and sin (2 c + ) = sin (2 c) :
Hence, T ((R cos (2 c) ; R sin (2 c))) = T (( R cos (2 c) ; R sin (2 c))):
That is, the Temperature at the point (R cos (2 c) ; R sin (2 c)) on the equator is the same as the Temperature
at the point ( R cos (2 c) ; R sin (2 c)) :
Note that the two points above are antipodal.

181. Show that the equation x = cos x has a solution on the interval [0; =2]: Use the Bisection method
and a calculator to nd an approximate solution of this equation, with error less than 10 3 :

Proof. Let f (x) = x cos x: Then 1 = f (0) < 0 < f ( 2 ) = 2 :


Thus, by the Location of Roots Theorem, there exists a number c 2 (0; =2) : f (c) = 0:
In other words, there exists a number c 2 (0; =2) : c = cos c:
Using the Bisection method, we get that the solution has to be in the interval [0:7390; 0:7391]:

182. Let I = [a; b], let f : I ! R be continuous on I; and assume that f (a) < 0; f (b) > 0:
Let W = fx 2 I : f (x) < 0g; and let w = sup W: Prove that f (w) = 0:
78


Proof. By continuity of f; we get that f (w) 0: To show that f (w) = 0; assume not, i.e., f (w) < 0:
Then, by continuity of f; we can nd a neighborhood V (w) such that f (x) < 0 for all x 2 V (w) ( ) :
Now, since w = sup W; we must have f (x) 0 for all x > w:
This contradicts ( ): Hence, f (w) = 0:

183. Suppose that f : R ! R is continuous on R and that lim f = 0 and lim f = 0:


x! 1

x!1

Prove that f is bounded on R and attains either a maximum or a minimum on R:


Give an example to show that both a maximum and a minimum need not be attained.

Proof. Since lim f = 0 and lim f = 0; there exists M > 0 large enough so that jf (x)j < 1 for all jxj > M:
x! 1

x!1

Now, let I = [ M; M ]: Since f is continuous on I, f is bounded on I; say jf (x)j L for all x 2 I:


Let M 0 = maxf1; Lg: Then jf (x)j < M 0 for all x 2 R: Hence, f is bounded on R:
Suppose that f does not attain a minimum on R: Then f (x) > 0 for all x 2 R: (why?)
Now, let c 2 I = [ M; M ] be such that f (c) = L: (This exists since I is closed, bounded and f is continuous).
What we will do now is technical. Let K > M be such that jf (x)j < L2 for all jxj > K:
Then consider the closed, bounded interval J = [ K; K]: Since f is continuous on J and we know for sure
that c 2 J (why?), we get that f attains its global maximum at a point (not necessarily c) in J:
By our choice of J; we are sure that this point is a global maximum for f on R:
A similar argument may be applied in the case where f does not attain a maximum on R:
Example. Let f (x) = e

x2

for x 2 R: We know that f (x) > 0 for x 2 R; that lim f = 0 and lim f = 0:

Also, as above, f can only have a global maximum but not a global minimum.
Here is a graph of f (x) = e

x2

x! 1

x!1

: (There are an uncountable number of examples, of course).

184. Show that the function f (x) = x12 is uniformly continuous on A = [1; 1); but that it is not uniformly
continuous on B = (0; 1):

y 2 x2
1
Proof. For all 1 x < y < 1; jf (x) f (y)j = x12
=
= (y xx)(y+x)
= jx yj (y+x)
2 y2
y2
x2 y 2
x2 y 2
2 jx yj : Thus, f (x) is Lipschitz on [1; 1) and hence is uniformly continuous on [1; 1):
Now, let = 1: We will show that for all 0 < < 1; we can always nd x; y 2 (0; 1) : jf (x)
So, x 0 < < 1 and let x 2 (0; ): Let y = x2 : Then y 2 (0; ) and so jx yj < :

f (y)j > 1:

1
1
4
3
Now, jf (x) f (y)j = x12
y 2 = x2
x2 = x2 > 3; since x < 1:

185. Show that if f and g are uniformly continuous on A R and if they are both bounded on A; then
their product f g is uniformly continuous on A:

Proof. Say, jf j M1 and jgj M2 on A: Let M = maxfM1 ; M2 g: Then jf j M and jgj M on A:


Let > 0 be given. Since f and g are uniformly continuous on A, 9 > 0 : whenever x; y 2 A with
jx yj < ; we have that jf (x) f (y)j < 2M and jg(x) g(y)j < 2M :
Now, whenever x; y 2 A with jx yj < ; we have that
jf (x)g(x) f (y)g(y)j = jf (x)g(x) f (x)g(y) + f (x)g(y) f (y)g(y)j jf (x)g(x) f (x)g(y)j+jf (x)g(y) f (y)g(y)j
= jf (x)j jg(x) g(y)j + jg(y)j jf (x) f (y)j M 2M + M 2M = :
This is exactly what it means for the product function f (x)g(x) to be uniformly continuous on A:

186. Prove that if f and g are each uniformly continuous on R; then their composite function f g is
uniformly continuous on R:

79

Proof. Let > 0 be given and proceed as follows:


Since f is uniformly continuous, 9 0 > 0 : whenever x; y 2 R with jx yj < 0 ; we have that jf (x) f (y)j < :
Since g is uniformly continuous, 9 > 0 : whenever x; y 2 R with jx yj < ; we have that jg(x) g(y)j < 0 :
Thus, whenever x; y 2 R with jx yj < ; we have that jg(x) g(y)j < 0 and so jf (g(x)) f (g(y))j < :
This is exactly what it means for the composite function f g to be uniformly continuous on R:
p
187. If g(x) = x for x 2 [0; 1]; show that there does not exist a constant K such that jg(x)j K jxj for all
x 2 [0; 1]: Conclude that the uniformly continuous function g is not a Lipschitz function on [0; 1]:
p
g(x)
x
p1 and so, lim
= +1:
Proof. For x 2 (0; 1]; we have that jg(x)j
x
jxj = x =
x
x!0+

g(x)
x

Hence,
is not bounded on (0; 1] and so that there does not exist a constant K such that jg(x)j
for all x 2 [0; 1]: Hence, g is not a Lipschitz function on [0; 1]:

K jxj

Now, since [0; 1] is a closed and bounded interval and g is continuous on [0; 1], we conclude that g is
uniformly continuous on [0; 1]:

188. Show that if a function f : R ! R is Lipschitz on R, then f is uniformly continuous on R:

Proof. Since f is Lipschitz on R; there exists K > 0 : jf (x) f (y)j K jx yj for all x; y 2 R:
Let > 0 be given and set = K : Then for all x; y 2 R with jx yj < ; we have that
jf (x) f (y)j K jx yj < K = :
This is exactly what it means for f to be uniformly continuous on R:

189. Show that the function f (x) = 1=x is uniformly continuous on the set A = [a; 1); where a is a positive
constant.

1 1
1
Proof. Note that for all x; y 2 [a; 1); one has x1
a; y
a ; and therefore,

1
jf (x) f (y)j = x1 y1 = xy
jx yj a12 jx yj :
Hence, f is Lipschitz on [a; 1) and thus uniformly continuous on [a; 1):

190. Show that the function f (x) = x2 is not uniformly continuous on the set A = [0; 1):

Proof. Note rst that for any x; y 2 [0; 1); jf (x) f (y)j = x2 y 2 = jx yj jx + yj
Let = 1: Then for all > 0; set x = + 1 and y = x + 2 = 32 + 1 :
Then jx yj = 2 < ; but jf (x) f (y)j = 2 52 + 2 = 54 2 + 1 > 1 = :
This is exactly what it means for f (x) = x2 not to be uniformly continuous on [0; 1):

191. Show that the function f (x) = sin(1=x) is not uniformly continuous on the set B = (0; 1):

1
Proof. Let = 21 ; (xn ) = 2 1n and (yn ) = 2 n+
: Then:
2

(i) (xn

yn ) =

=2
[2 n][2 n+ 2 ]

! 0 as n ! 1:

(ii) jf (xn ) f (yn )j = sin(2 n) sin(2 n + 2 ) = j0 1j = 1 12 = :


This is exactly what it means for f (x) = sin(1=x) not to be uniformly continuous on (0; 1):

1
192. Show that the function f (x) = 1+x
2 for x 2 R is uniformly continuous on R:

1
Proof. We will show that f (x) = 1+x
2 is Lipschitz on R and so uniformly continuous on R:
jx2 y2 j
jx+yjjx yj
1
1
For this, note that for any x; y 2 R; one has jf (x) f (y)j = 1+x
2
1+y 2 = (1+x2 )(1+y 2 ) = (1+x2 )(1+y 2 ) :

jxj
jyj
Now, (1+xjx+yj
2 )(1+y 2 )
(1+x2 )(1+y 2 ) + (1+x2 )(1+y 2 )
That is, for any x; y 2 R; one has jf (x) f (y)j

1 + 1 = 2:
2 jx yj :
80

1
Hence, f (x) = 1+x
2 is Lipschitz on R and so uniformly continuous on R:

193. Show that if f and g are uniformly continuous on A R; then f + g is uniformly continuous on A:

Proof. Let > 0 be given. Then there exists > 0 such that for all x; y 2 A with jx yj < ; we have
that jf (x) f (y)j < 2 and jg(x) g(y)j < 2 :
Thus, for all x; y 2 A with jx yj < ; we have that j(f + g) (x) (f + g) (y)j = jf (x) + g(x) f (y) g(y)j
= jf (x) f (y) + g(x) g(y)j jf (x) f (y)j + jg(x) g(y)j < 2 + 2 = :
This is exactly what it means for f + g to be uniformly continuous on A:

194. If f is uniformly continuous on A R; and jf (x)j k > 0 for all x 2 A; show that 1=f is uniformly
continuous on A:

Proof. Let > 0 be given. Since f is uniformly continuous on A; there exists > 0 : whenever x; y 2 A
with jx yj < ; we have jf (x) f (y)j < k 2 :
1
1
1
Thus, whenever x; y 2 A with jx yj < ; we have f (x)
f (y)j k12 k 2 = :
f (y) = jf (x)jjf (y)j jf (x)
This is exactly what it means for 1=f to be uniformly continuous on A:

195. Prove that if f is uniformly continuous on a bounded set A R; then f is bounded on A:

Proof. Since f is uniformly continuous on A; there exists > 0 : whenever x; y 2 A with jx yj < ;
we have jf (x) f (y)j < 1: Since A is bounded, we can nd a nite number of points x1 < x2 <
< xn
n
[
in A such that A
[xj
; xj + ] :
j=1

Let M = maxf1 + jf (x1 )j ; 1 + jf (x2 )j ; :::; 1 + jf (xn )jg and let x 2 A:


Then x 2 [xj
; xj + ] for some 1 j n; and so jx xj j < :
Hence, jf (x) f (xj )j < 1 =) jf (x)j < 1 + jf (xj )j M:
Since our choice of x 2 A was arbitrary, we conclude that jf (x)j < M for all x 2 A:
Thus, f is bounded on A:

196. Show that if f is continuous on [0; 1) and uniformly continuous on [a; 1) for some positive constant a;
then f is uniformly continuous on [0; 1):

Proof. f is continuous on [0; 1) =) f is continuous on [0; a + 1]:


Since [0; a + 1] is closed and bounded, f is uniformly continuous on [0; a + 1]:
Let > 0 be given. Then there exists 0 < 1 < 1 : whenever x; y 2 [0; a + 1] with jx yj < 1 ;
we have that jf (x) f (y)j < :
Also, since f is uniformly continuous on [a; 1); there exists 0 < 2 < 1 : whenever x; y 2 [a; 1)
with jx yj < 2 ; we have that jf (x) f (y)j < :
Let = minf 1 ; 2 g and note that for all x; y 2 R with jx yj < ; we have that either x; y 2 [0; a + 1];
or x; y 2 [a; 1):
Therefore, for all x; y 2 R with jx yj < ; we have that jf (x) f (y)j < :
This is exactly what it means for f to be uniformly continuous on [0; 1):

197 Let A R and suppose that f : A ! R has the following property:


8 > 0 9g : A ! R such that g is uniformly continuous on A and jf (x) g (x)j < for all x 2 A:
Show that f is uniformly continuous on A:

Proof. Let > 0 be given. Then there exists a function g =3 : A ! R such that g =3 is uniformly continuous
on A and f (x) g =3 (x) < =3 for all x 2 A:
Since g =3 is uniformly continuous on A; there exists > 0 such that whenever x; y 2 A with jx yj < ;
we have that g =3 (x) g =3 (y) < =3:

81

Thus, for all x; y 2 A with jx yj < ; we have that


jf (x) f (y)j = f (x) g =3 (x) + g =3 (x) g =3 (y) + g =3 (y) f (y)
f (x) g =3 (x) + g =3 (x) g =3 (y) + g =3 (y) f (y)
< =3 + =3 + =3 = :
Therefore, f is uniformly continuous on A:

198. (a) Show that if f is uniformly continuous on (a; b] and on [b; c); then f is also uniformly
continuous on (a; c):
(b) Prove that any function f : R ! R that is continuous and periodic must be uniformly continuous.

Proof. (a) Given > 0; there are 1 > 0 and 2 > 0 such that jf (x) f (b)j < 2 if 0 b x < 21
and jf (y) f (b)j < 2 if 0 y b < 22 :
Setting = minf 1 ; 2 g; we get that for any x 2 (b 2 ; b] and any y 2 [b; b + 2 ); jf (x)
For x; y 2 (a; b] or x; y 2 [b; c); ( ) is clearly satised with some positive > 0:

f (y)j <

( )

(b) Let p be the period of f: Then f is continuous on each interval [kp; (k + 1)p] for all k 2 Z:
Since [kp; (k + 1)p] is closed and bounded, f is uniformly continuous on each of those intervals.
So, by the solution of (a), one can show that f is uniformly continuous on R.

199. Show that if f : R ! R is continuous and such that lim f (x) and lim f (x) are nite, then f is
x! 1

x!1

uniformly continuous on R.

Proof. Set lim f (x) = L and lim f (x) = l:


x!1

x! 1

Then, given > 0; there is A > 0 : jf (x) Lj < 2 for x A and jf (x) lj < 2 for x
A:
This implies that if x; y 2 [A; 1) or x; y 2 ( 1; A]; then jf (x) f (y)j < :
Of course, since [ A; A] is closed and bounded, f is uniformly continuous on [ A; A]:
Finally, as in the solution of part (a) of the previous problem, one can show that f is uniformly
continuous on R.

200. A function f : R ! R is continuous at 0 and satises the following conditions:


f (0) = 0
and
f (x + y) f (x) + f (y) for any x; y 2 R:
Prove that f is uniformly continuous on R:

Proof. By the continuity of f at 0, given > 0 there is > 0 : jf (x)j < for jxj < :
Hence, the subadditivity of f implies that; for x 2 R and jtj < ;
f (x + t) f (x) f (t) <
and
f (x) f (x + t) f ( t) <
Thus, jf (x + t) f (x)j < ; which proves the uniform continuity of f on R.

201. A point x 2 R is said to be an interior point of A R in case there is a neighborhood V of x such


that V
A: Show that a set A R is open if and only if every point of A is an interior point of A:

Proof. (=)) Let A R be open and let x 2 A: By denition of open-ness, there is a neighborhood V
of x such that V
A: This means that x is an interior point of A:
((=) Suppose that every point of A is an interior point of A and let x 2 A: Being an interior point of A,
there is a neighborhood V of x such that V
A: Thus, for each x 2 A there is a neighborhood V of x
such that V
A: Thus, A R is open.

202. A point x 2 R is said to be a boundary point of A R in case every neighborhood V of x contains


points in A and points in Ac : Show that a set A and its complement Ac have exactly the same boundary
points.

Proof. x is a boundary point of A if and only if every neighborhood V of x contains points in A and points

82

in Ac if and only if every neighborhood V of x contains points in A = (Ac ) and points in Ac if and only if
x is a boundary point of Ac :

203. Show that a set G R is open if and only if it does not contain any of its boundary points.

Proof. (=)) Suppose that G R is open and suppose that x is a boundary point of G: Then every
neighborhood V of x contains points in G and points in Gc : Thus, no neighborhood of x is contained
entirely in G: Hence, x is not an interior point of G and thus x 2
= G:
((=) Suppose that G does not contain any of its boundary points and let x 2 G: Not being a boundary
point, x has a neighborhood V that does not intersect Gc : But V \ Gc = ? () V
G: Hence, x is an
interior point of G: Since this is true for any x 2 G; we conclude that G is open.

204. Show that a set F R is closed if and only if it contains all of its boundary points.

Proof. We can use the previous two exercises as follows:


By Denition
By previous Exercise
A set F R is closed
()
F c is open
()
F c does not contain any of its boundary
F and F c have the sam e b oundary p oints

points
()
F contains all of its boundary points.

205. If A R; let A be the union of all open sets that are contained in A; the set A is called the interior
of A: Show that:
(a) A is an open set,
(b) A is the largest open set contained in A; and
(c) A point z belongs to A if and only if z is an interior point of A:

Proof. (a) A is a union of open sets and so is an open set. Note that, by denition, A
A:
(b) Let G be an open set contained in A: Since A is the union of all open sets that are contained in A; we
conclude that G A . Thus, A is the largest open set contained in A:
(c) z 2 A () z 2 V
A for some open set V
A () z is an interior point of A:

206. If A R; let A be the intersection of all closed sets containing A; the set A is called the closure of A:
Show that:
(a) A is a closed set,
(b) A is the smallest closed set containing A; and
(c) A point w belongs to A if and only if w is either an interior point or a boundary point of A:

Proof. (a) A is the intersection of closed sets and so is closed. Note that, by denition, A A:
(b) Let F be a closed set containing A: Since A is the intersection of all closed sets containing A; we
conclude that A F: Thus, A is the smallest closed set containing A:
c
c
c
(c) A is closed () A is open () Every point x 2 A is an interior point of A () Every point
c
c
c
A
Ac () Every point x 2 A is neither an interior point nor
x 2 A has a neighborhood Vx
a boundary point of A () Every point x 2 A is either an interior point or a boundary point of A:

207. Exhibit an open cover of N that has no nite subcover.

Solution. For n = 1; 2; 3; ::: let Gn = ( n; n): Note that G1 G2


Gn
is an increasing
1
[
sequence of sets. We claim that G =
Gn = R.
n=1

Obviously, G R: Now, let x 2 R: By the Archimedian property, 9N 2 N large enough so that


x < N: Thus, x 2 ( N; N ) G: Since x was arbitrary, we conclude that R G: Hence, G = R.
In particular, N G and so G is an open cover of N:
Suppose that N can be covered by a nite number of the Gn s, say N Gn1 [ Gn2 [
[ G nk :
Let m = maxfn1 ; n2 ; :::; nk g: Then, since the Gn s form an increasing sequence, we conclude that
G n1 [ G n2 [
[ Gnk = Gm = ( m; m):
83

Thus, N

( m; m); a contradiction since m 2 N but m 2


= ( m; m): Thus, G has no nite subcover.

Remark. Another solution is obtained by taking Gn = (n

1; n + 1) for n 1:
1
[
Then note that the only Gn \ N = fng for every n and thus, G =
Gn N is an open cover.
n=1

By our construction, it cannot reduce to a nite subcover because each point n 2 N is contained in
exactly one Gn ; namely, Gn itself.

208. Exhibit an open cover of the set A = f1=n : n 2 Ng that has no nite subcover.

Solution. We will follow the same method we used in the remark of the previous exercise. This method
is not applicable all the time. Actually, it is using the fact that the sets are discrete with no limit points.
We want to nd a sequence of sets Gn such that Gn \ A = f1=ng and nothing else from A:
1
; n 1 1 ) for n 1: Now, proceed as in the remark of the previous exercise.
This is easy, let Gn = ( n+1
Remark. Try the same exercise but with A = f0g [ f1=n : n 2 Ng : Can you nd an open cover that has
no nite subcover?

209. Prove that if K1 and K2 are compact sets in R; then K = K1 [ K2 is compact.


[

Proof. Let fG g 2A be an open cover of K; i.e., K G =


G and each G is open in R:
Then K1

2A

G: By compactness of K1 and K2 ; we conclude that we can nd


k
k+1+m
[
[
0
00
two nite subcovers G =
Gn and G =
Gn for some k; m 2 N such that K1 G0
and K2

G and K2

G00 : Thus, K

n=1
k+1+m
[
n=1

n=k+1

Gn is a nite subcover of K: Hence, K = K1 [ K2 is compact.

210. Prove that the intersection of an arbitrary collection of compact sets in R is compact.

\
Proof. Suppose that K is compact in R For in some index set A: Let K =
K :
2A

We will show that K is closed and bounded and hence, by Heine-Borel Theorem, compact.
Since each K is compact, Heine-Borel Theorem =) each K is closed and bounded.
Since arbitrary intersection of closed sets is closed, K is closed .
Since K K for each 2 A; x an in A; call it 0 :
Now, K 0 is bounded and K K 0 =) K is bounded .
Thus, K is closed and bounded and hence compact.

211. Let K 6= ? be compact in R and let c 2 R:


(a) Prove that there exists a point a 2 K such that jc aj = inffjc xj : x 2 Kg:
(b) Prove that there exists a point b 2 K such that jc bj = supfjc xj : x 2 Kg:

Proof. (a) The inmum exists because, by boundedness of K; fjc xj : x 2 Kg is bounded in R.


Let = inffjc xj : x 2 Kg:
Let n 2 N: By denition of inf; there exists a point an 2 K : + n1 > jc an j :
By the Bolzano-Weierstrass Theorem, there is a subsequence (ank ) of (an ) : a = lim ank 2 K:
k!1

Since

> jc

ank j

1
nk

for every k;

lim

k!1

Of course, being the inf and because a 2 K;


Hence, = jc aj :

jc

jc

ank j
aj :

84

1
nk

= jc

aj :

(b) The supremum exists because, by boundedness of K; fjc xj : x 2 Kg is bounded in R.


Let = supfjc xj : x 2 Kg:
1
Let n 2 N: By denition of sup; there exists a point bn 2 K :
bn j :
n < jc
By the Bolzano-Weierstrass Theorem, there is a subsequence (bnk ) of (bn ) : b = lim bnk 2 K:
k!1

Since

< jc

bn k j

1
nk

for every k;

lim

k!1

jc

b nk j

1
nk

= jc

bj :

Of course, being the sup and because b 2 K;


jc bj :
Hence, = jc bj :

1 if 0 x 1
212. Let h : R ! R be dened by h(x) =
: Find an open set G such that h 1 (G) is
0
otherwise
not open, and a closed set F such that h 1 (F ) is not closed.

Solution. Let G = (1=2; 3=2): Then h 1 (G) = fx 2 R : 1=2 < f (x) < 3=2g = fx 2 R : 0 x 1g = [0; 1]:
Hence, G is open but h 1 (G) is closed.
Let F = [ 1=2; 1=2]: Then h 1 (F ) = fx 2 R : 1=2 f (x) 1=2g = fx 2 R : x 2
= [0; 1]g = Rn[0; 1]:
Hence, F is closed but h 1 (F ) is open.

213. Prove that f : R ! R is continuous if and only if for each closed set F in R; the inverse image f 1 (F )
is closed in R:

Proof. First note that for any subset G R; f 1 (RnG) = Rnf 1 (G): This is an elementary result.
In what follows, we will use the fact that f : R ! R is continuous if and only if for each open set G in R;
the inverse image f 1 (G) is open in R:
(=)) suppose that f : R ! R is continuous, and let F be closed in R: Then G = RnF is open and so
f 1 (G) = f 1 (RnF ) = Rnf 1 (F ) is open. Thus, f 1 (F ) is closed in R:
((=) Suppose that for each closed set F in R; the inverse image f 1 (F ) is closed in R: Let G be open in R:
Then RnG is closed and so f 1 (RnG) is closed in R: But f 1 (RnG) = Rnf 1 (G):
Thus, f 1 (G) is open and so f is continuous.

214. Show that if f : R ! R is continuous, then the set fx 2 R : f (x) < g is open in R for each 2 R:

Proof. Note that for any 2 R; the set fx 2 R : f (x) < g = f 1 ( 1; ) is open since f is continuous,
and ( 1; ) is open.

215. Show that if f : R ! R is continuous, then the set fx 2 R : f (x)


g is closed in R for each 2 R:

Proof. Note that for any 2 R; the set fx 2 R : f (x)


g = f 1 ( 1; ] is closed since f is continuous,
and ( 1; ] is closed.

216. Show that if f : R ! R is continuous, then the set fx 2 R : f (x) = kg is closed in R for each k 2 R:

Proof. Note that for any k 2 R; the set fx 2 R : f (x) = kg = f 1 fkg is closed since f is continuous,
and fkg is closed.

217. Give an example of a function f : R ! R such that the set fx 2 R : f (x) = 1g is neither open
nor closed in R:

1 for x 2 Q
Proof. Let f (x) =
: Then the set fx 2 R : f (x) = 1g = Q which is neither open
0 for x 2
=Q
nor closed. Note that Q is not open because of the density of the irrationals, and Q is not closed because
of its density in R.

85


218. Let I = [a; b] and let f : I ! R and g : I ! R be continuous functions on I: Show that the set
fx 2 R : f (x) = g(x)g is closed in R:

Proof. Let h(x) = f (x) g(x): Then h : I ! R is continuous.


Moreover, the set fx 2 R : f (x) = g(x)g = fx 2 R : h(x) = 0g = h 1 f0g which is closed in I since h
is continuous.

219. Show that each of the following functions is a metric on the given space:
(a) S = R2 ; d1 (P1 ; P2 ) = jx1 x2 j + jy1 y2 j ; where P1 = (x1 ; y1 ) and P2 = (x2 ; y2 ):
(b) S = R2 ; d1 (P1 ; P2 ) = sup fjx1 x2 j ; jy1 y2 jg ; where P1 = (x1 ; y1 ) and P2 = (x2 ; y2 ):
Z1
(c) S = C[0; 1]; d1 (f; g) = jf gj :
0

(d) S = C[0; 1]; d1 (f; g) = supfjf (x)

g(x)j : x 2 [0; 1]g:


0 if s = t
(e) S = Any nonempty set; d(s; t) =
: This is called the discrete metric on S.
1 if s 6= t

Proof. (a) (i) (Positivity) d1 (P1 ; P2 ) = jx1 x2 j + jy1 y2 j 0 for all P1 ; P2 2 R2 :


(ii) (Deniteness) d1 (P1 ; P2 ) = 0 =) jx1 x2 j + jy1 y2 j = 0 =) jx1 x2 j = 0 and jy1 y2 j = 0
=) x1 x2 = 0 and y1 y2 = 0 =) x1 = x2 and y1 = y2 =) P1 = P2 :
(iii) (Symmetry) d1 (P1 ; P2 ) = jx1 x2 j + jy1 y2 j = jx2 x1 j + jy2 y1 j = d1 (P2 ; P1 ) for all P1 ; P2 2 R2 :
(iv) (Triangle Inequality) Let P1 = (x1 ; y1 ), P2 = (x2 ; y2 ); and P3 = (x3 ; y3 ): Then d1 (P1 ; P2 ) =
jx1 x2 j + jy1 y2 j = jx1 x3 + x3 x2 j + jy1 y3 + y3 y2 j jx1 x3 j + jx3 x2 j + jy1 y3 j + jy3 y2 j
= jx1 x3 j + jy1 y3 j + jx3 x2 j + + jy3 y2 j = d1 (P1 ; P3 ) + d1 (P3 ; P2 ):
(b) (i) (Positivity) d1 (P1 ; P2 ) = sup fjx1 x2 j ; jy1 y2 jg 0 for all P1 ; P2 2 R2 :
(ii) (Deniteness) d1 (P1 ; P2 ) = 0 =) sup fjx1 x2 j ; jy1 y2 jg = 0 =) jx1 x2 j = 0 and jy1 y2 j = 0
=) x1 x2 = 0 and y1 y2 = 0 =) x1 = x2 and y1 = y2 =) P1 = P2 :
(iii) (Symmetry) d1 (P1 ; P2 ) = sup fjx1 x2 j ; jy1 y2 jg = sup fjx2 x1 j + jy2 y1 jg = d1 (P2 ; P1 )
for all P1 ; P2 2 R2 :
(iv) (Triangle Inequality) Let P1 = (x1 ; y1 ), P2 = (x2 ; y2 ); and P3 = (x3 ; y3 ):
Then jx1 x2 j jx1 x3 j + jx3 x2 j sup fjx1 x3 j ; jy1 y3 jg + sup fjx3 x2 j ; jy3 y2 jg ; and
jy1 y2 j jy1 y3 j + jy3 y2 j sup fjx1 x3 j ; jy1 y3 jg + sup fjx3 x2 j ; jy3 y2 jg :
Hence, sup fjx1 x2 j ; jy1 y2 jg sup fjx1 x3 j ; jy1 y3 jg + sup fjx3 x2 j ; jy3 y2 jg :
This is exactly d1 (P1 ; P2 ) d1 (P1 ; P3 ) + d1 (P3 ; P2 ):
(c) (i) (Positivity) d1 (f; g) =

Z1

jf

gj

0 for all f; g 2 C[0; 1]:

(ii) (Deniteness) d1 (f; g) = 0 =)

Z1

jf

gj = 0

since jf

gj is continuous and

=)

jf

gj

0 =) f

=) f

g:

(iii) (Symmetry) d1 (f; g) =

Z1
0

jf

gj =

Z1

jg

f j = d1 (g; f ) for all f; g 2 C[0; 1]:

(iv) (Triangle Inequality) Let f; g; h 2 C[0; 1]: Then d1 (f; g) =

Z1
0

Z1
0

jf

hj +

Z1

jh

gj = d1 (f; h) + d1 (h; g):

86

jf

gj =

Z1
0

jf

h+h

gj

(d) (i) (Positivity) d1 (f; g) = supfjf (x) g(x)j : x 2 [0; 1]g 0 for all f; g 2 C[0; 1]:
(ii) (Deniteness) d1 (f; g) = 0 =) supfjf (x) g(x)j : x 2 [0; 1]g = 0 =) jf (x) g(x)j = 0 for all x 2 [0; 1]
=) f g 0 =) f g:
(iii) (Symmetry) d1 (f; g) = supfjf (x) g(x)j : x 2 [0; 1]g = supfjg(x) f (x)j : x 2 [0; 1] = d1 (g; f )
for all f; g 2 C[0; 1]:
(iv) (Triangle Inequality) Let f; g; h 2 C[0; 1]: Then for all x 2 [0; 1]; jf (x) g(x)j jf (x) h(x)j
+ jh(x) g(x)j supfjf (x) h(x)j : x 2 [0; 1]g + supfjh(x) g(x)j : x 2 [0; 1]g:
Hence, supfjf (x) g(x)j : x 2 [0; 1]g supfjf (x) h(x)j : x 2 [0; 1]g + supfjh(x) g(x)j : x 2 [0; 1]g:
This can be written as d1 (f; g) d1 (f; h) + d1 (h; g):
0 if s = t
0 for all s; t 2 S:
1 if s 6= t
(ii) (Deniteness) d(s; t) = 0 =) s = t:
(iii) (Symmetry) Clearly, d(s; t) = d(t; s):
(iv) (Triangle Inequality) Let s; t; u 2 S:
If d(s; t) = 0 =) s = t =) d(s; u) = d(t; u) and each is either 0 or 1. Since 0 0 and 0 2; we are done.
If d(s; t) = 1 =) s 6= t =) For any u 2 S; either u = s; u = t; or u 2
= fs; tg:
In the rst case, we get that d(s; u) = 0; d(u; t) = 1 and so d(s; t) = 1 0 + 1 = d(s; u) + d(u; t):
In the second case, we get that d(s; u) = 1; d(u; t) = 0 and so d(s; t) = 1 1 + 0 = d(s; u) + d(u; t):
In the third case, we get that d(s; u) = 1; d(u; t) = 1 and so d(s; t) = 1 2 = 1 + 1 = d(s; u) + d(u; t):
Hence, the triangle inequality holds.

220. If Pn = (xn ; yn ) 2 R2 and d1 is the metric in the previous problem part (b), show that (Pn ) converges
to P = (x; y) with respect to this metric if and only if (xn ) and (yn ) converge to x and y, respectively.

Proof. (=)) Suppose that (Pn ) converges to P = (x; y) with respect to d1 ; and let > 0 be given.
Then there exists K 2 N : Pn 2 V (P ) = fz 2 R2 : d1 (z; P ) < g for all n K:
Now, fz 2 R2 : d1 (z; P ) < g = fz 2 R2 : sup fjz1 xj ; jz2 yjg < g:
Since Pn 2 V (P ) for all n K; we conclude that sup fjxn xj ; jyn yjg < for all n K:
This means that jxn xj < and jyn yj < for all n K:
Thus, (xn ) and (yn ) converge to x and y, respectively.
(e) (i) (Positivity) d(s; t) =

((=) Suppose that (xn ) and (yn ) converge to x and y, respectively. Let > 0 be given.
Then we can nd K 2 N : jxn xj < and jyn yj < for all n K:
Thus, sup fjxn xj ; jyn yjg < for all n K:
Hence, Pn 2 V (P ) for all n K and so (Pn ) converges to P = (x; y) with respect to d1 :

221. Let S be a nonempty set and let d be the discrete metric dened above. Show that in the metric space
(S; d); a sequence (xn ) in S converges to x if and only if there is a K 2 N such that xn = x for all n K:

Proof. (=)) Suppose that in the metric space (S; d); a sequence (xn ) in S converges to x:
Then for = 1=2; 9 K 2 N : xn 2 V1=2 (x) = fy 2 S : d(y; x) < 1=2g
Hence, xn = x for all n K:

By denition of d

fxg for all n

K:

((=) Suppose that there is a K 2 N such that xn = x for all n K and let > 0 be given.
Then d(xn ; x) = 0 for all n K and so xn 2 V (x) for all n K. Hence, (xn ) converges to x:

222. Show that if d is the discrete metric on a set S; then every subset of S is both open and closed in (S; d):

Proof. We will show that each singleton fx0 g is an open set when we have the discrete metric.
This is because for each x 2 fx0 g; x = x0 and V1=2 (x) = fxg = fx0 g fx0 g:
Thus, we have shown that 8x 2 fx0 g there is a neighborhood U = V1=2 (x) of x such that U fx0 g:
This is exactly what it means for the set fx0 g to be open in S:
87

Now, Let A

S be any subset. Then we can write A =

x2A

of open sets is open, we conclude that A is open.

fxg and since fxg is open and an arbitrary union

Now, let B S be any set. Then SnB S and so by what we did above, SnB is open.
Hence, B is closed, being the complement of an open set.
Thus, if d is the discrete metric on a set S; then every subset of S is both open and closed in (S; d):

223. Prove that in any metric space (S; d), an


neighborhood of a point is an open set.

Proof. Let x0 2 S and consider V (x0 ) = fx 2 S : d(x; x0 ) < g:


Let x 2 V (x0 ) and let =
d(x; x0 ) > 0:
We claim that V (x) V (x0 ). To prove this, let y 2 V (x):
Then d(y; x) < =) d(y; x0 ) d(y; x) + d(x; x0 ) < + d(x; x0 ) =
d(x; x0 ) + d(x; x0 ) = :
Hence, y 2 V (x0 ) and so V (x) V (x0 ).
Thus, we have shown that 8x 2 V (x0 ) there is a neighborhood U = V (x) of x such that U V (x0 ):
Therefore, V (x0 ) is open.

x2 if x 2 Q
224. Let f : R ! R be dened by f (x) =
: Show that f is dierentiable at x = 0; and
0 if x 2
=Q
nd f 0 (0):

jxj for all x 2 Rnf0g:


Proof. We note that jf (x)j x2 for all x 2 Rnf0g; and so f (x)
x
We want to show that f 0 (0) = 0; i.e., lim f (x)x
x!0

Let

> 0 be given. Then for

f (0)
0

= 0 or equivalently, since f (0) = 0; lim f (x)


x = 0:
x!0

= ; we have that 8 jxj < ;

f (x)
x

jxj < :

Hence, f 0 (0) = lim f (x)x 0f (0) = 0:


x!0

x2 sin(1=x2 ) if x 6= 0
225. Let g : R ! R be dened by g(x) =
: Show that g is dierentiable for
0
if x = 0
all x 2 R; Also, show that the derivative g 0 is not bounded on the interval [ 1; 1]:

Proof. For x 6= 0; we get, by the product rule and chain rule, g 0 (x) = 2x sin(1=x2 ) + x2 ( x32 cos(1=x2 ))
= 2x sin(1=x2 ) x2 cos(1=x2 ) which exists for all x 6= 0: Hence, g is dierentible for all x 6= 0:

For x = 0; we compute the limit, g 0 (0) = lim g(x)x


x!0

(Because x sin(1=x2 )

g(0)
0

jxj for all x 2 R):

= lim x
x!0

sin(1=x2 ) 0
x

= lim x sin(1=x2 ) = 0:
x!0

To show that g 0 is not bounded on the interval [ 1; 1]; we consider the sequence (xn ) = p21 n :
p
All the terms of this sequence are in [ 1; 1]; and g 0 (xn ) = p22 n sin(2 n) 2 2 n cos(2 n)
| {z }
| {z }
=0 for all n
=1 for all n
p
= 2 2 n which is not bounded since it goes to 1 as n ! 1:

226. Assume that there exists a function L : (0; 1) ! R such that L0 (x) = 1=x for x > 0: Calculate the
derivatives of the following functions:
(a) f (x) = L(2x + 3) for x > 0;
3
(b) g(x) = L(x2 ) for x > 0;
(c) h(x) = L(ax) for a > 0; x > 0;
(d) k(x) = L(L(x)) when L(x) > 0; x > 0:

2
Solution. (a) Using the chain rule, f 0 (x) = L0 (2x + 3) 2 = 2x+3
:
0
2 2
0 2
(b) Using the chain rule, g (x) = 3(L(x )) L (x ) 2x = 3(L(x2 ))2 x12 2x = x6 (L(x2 ))2 :
88

1
(c) Using the chain rule, h0 (x) = L0 (ax) a = ax
a = x1 :
1
1
1
0
0
0
(d) Using the chain rule, k (x) = L (L(x)) L (x) = L(x)
x = xL(x) :

0
227. Given that the function h(x) = x3 + 2x + 1 for x 2 R has an inverse h 1 on R; nd the value of h 1 (y)
at the points corresponding to x = 0; 1; 1:

Solution. h0 (x) = 3x2 + 2; h(0) = 1; h(1) = 4; and h( 1) = 2:


0
0
Hence, h 1 (1) = h 1 (h(0)) = h0 (h 11(h(0))) = h01(0) = 12 ;

1 0

(4) = h

1 0

(h(1)) =
0

1
h0 (h

1 (h(1)))

1
h0 (1)

= 15 ;

1
1
1
h 1 ( 2) = h 1 (h( 1)) = h0 (h 1 (h(
1))) = h0 ( 1) = 5 :

228. Given that the restriction of the cosine function cos to I = [0; ] is strictly decreasing and that cos 0 = 1;
cos = 1; let J = [ 1; 1]; and let arccos : J ! R be the function inverse to the restriction of cos to I:
Show that arccos is dierentiable on ( 1; 1) and D(arccos(y)) = p 1 2 for y 2 ( 1; 1):
1 y

Show that arccos is not dierentiable at 1 and 1:

Proof. Let y 2 ( 1; 1): Then y = cos x for a unique x 2 (0; ) and x = arccos y:

Thus, for y 2 ( 1; 1); D(arccos(y))


Since p

1
1 y2

1 as y !

x=arccos y

1
D(cos x)

(sin x)>0 on (0; )


y=cos x
1
1
p
=
=
sin x
1 cos2 x

1; we conclude that arccos is not dierentiable at

1
:
1 y2

1 and 1:

Here is a graph of g(x) = arccos x:

229. Let f : I ! R be dierentiable at c 2 I: Establish the Straddle Lemma:


Given > 0; there exists > 0 such that if u; v 2 I satisfy c
<u<c<v <c+ ;
then we have that jf (v) f (u) (v u)f 0 (c)j
(v u) :
(Hint: add and subtract the term f (c) cf 0 (c)):

Proof. Let > 0 be given. Since f : I ! R be dierentiable at c 2 I; there exists > 0 such that
if x 2 I satises 0 < jx cj < ; then f (x)x fc (c) f 0 (c) < :
Now, both u and v satisfy 0 < ju cj < ; 0 < jv cj < :
by the hint

Thus, jf (v) f (u) (v u)f 0 (c)j


=
jf (v) f (u) (v u)f 0 (c)
= jf (v) f (u) vf 0 (c) + uf 0 (c) (f (c) cf 0 (c)) + (f (c) cf 0 (c))j
jf (v) vf 0 (c) (f (c) cf 0 (c))j + j f (u) + uf 0 (c) + (f (c) cf 0 (c))j
= jf (v) f (c) + (c v) f 0 (c)j + jf (c) f (u) + (u c) f 0 (c)j
= vv cc jf (v) f (c) + (c v) f 0 (c)j + cc uu jf (c) f (u) + (u c) f 0 (c)j
= (v

c)

f (v) f (c)
v c

f 0 (c) + (c

u)

f (c) f (u)
c u

f 0 (c)

89

(f (c)

cf 0 (c)) + (f (c)

cf 0 (c))j

= (v c) f (v)v fc (c) f 0 (c) + (c u) f (u)u fc (c) f 0 (c)


< (v c) + (c u) = (v u) :

230. Prove that if f : R ! R is an even function [that is, f ( x) = f (x) for all x 2 R] and has a derivative
at every point, then the derivative f 0 is an odd function [that is, f 0 ( x) = f 0 (x) for all x 2 R]: Also,
prove that if g : R ! R is a dierentiable odd function, then g 0 is an even function.

Proof. Suppose that f : R ! R is a dierentiable even function. Let c 2 R:


f is even
=
lim f ( c h)h f ( c) = lim f ( c h)h f ( c) = lim f ( c+h)h f ( c)
Then f 0 (c) = lim f (c+h)h f (c)
h!0

h!0

h!0

= f 0 ( c):
Since our choice of c 2 R was arbitrary, we conclude that f 0 (x) =
Hence, f 0 is an odd function.

h!0

f 0 ( x) for all x 2 R:

Now, suppose that g : R ! R is a dierentiable odd function. Let c 2 R:


g is o dd
Then g 0 (c) = lim g(c+h)h g(c) = lim g( c hh)+g( c) = lim g( c h)h g(
h!0

h!0

h!0
0

c)

= lim g(
h!0

c+h) g( c)
h

= g 0 ( c):

Since our choice of c 2 R was arbitrary, we conclude that g (x) = g 0 ( x) for all x 2 R:
Hence, g 0 is an even function.

231. If f : R ! R is dierentiable at c 2 R; show that f 0 (c) = lim (n ff (c + 1=n) f (c)g) : However, show
n!1

by an example that the existence of the limit of this sequence does not imply the existence of f 0 (c):

Solution. Since f is dierentiable at c; f 0 (c) = lim f (c+h)h f (c) exists.


h!0

Hence, for any sequnce of points (xn ) such that xn ! 0; we must have that lim

n!1

In particular, let (xn ) = 1=n: Then lim

n!1

f (c+1=n) f (c)
1=n

For an example, let f (x) = jxj : Then lim

n!1

f (c+xn ) f (c)
xn

= f 0 (c):

= f 0 (c):

f (0+1=n) f (0)
1=n

= lim

n!1

j1=nj 0
1=n

= lim

1=n

n!1 1=n

= 1 exists.

Yet, f 0 (0) does not exist.

232. Use the Mean Value Theorem to prove that jsin x sin yj jx yj for all x; y 2 R:

Proof. Let f (x) = sin x: Then for any x < y; the Mean Value Theorem implies that there exists c 2 (x; y) :
sin y sin x = (cos c) (y x) : Thus, jsin y sin xj = jcos cj jy xj jy xj : Since our choice of x; y 2 R
was arbitrary, we conclude that jsin x sin yj jx yj for all x; y 2 R:

0 if x < 0
233. If h(x) =
; prove there does not exist a function f : R ! R such that f 0 (x) = h(x) for
1 if x 0
all x 2 R: Give examples of two functions, not diering by a constant, whose derivatives equal h(x) for
all x 6= 0:

Proof. If such an f exists, then it should be dierentiable everywhere with f 0 (x) = h(x): Thus, f is
dierentiable on [ 1; 1]: Also, f 0 ( 1) = h( 1) = 0; and f 0 (1) = h(1) = 1: Hence, by Darbouxs
theorem, there must exist a c 2 ( 1; 1) : f 0 (c) = h(c) = 1=2: But this is absurd by denition of h:
1 if x < 0
3
if x < 0
Let f (x) =
and g(x) =
:
x if x 0
x + 1 if x 0

234. Let I be an interval. Prove that if f is dierentiable on I and if the derivative f 0 is bounded on I; then
f satises a Lipschitz condition on I:

Proof. Say jf 0 (x)j M for all x 2 I: Let x < y 2 I: Then by the Mean Value Theorem, there exists
c 2 (x; y) : f 0 (c) = f (y)y xf (x) =) f (y)y fx(x) = jf 0 (c)j M =) jf (y) f (x)j M jy xj :
Since our choice of x and y was arbitrary, we conclude that f satises a Lipschitz condition on I with
90

Lipschitz constant M:

235. Let f; g be dierentiable functions on R and suppose that f (0) = g(0) and f 0 (x) g 0 (x) for all x 0:
Show that f (x) g(x) for all x 0:

Proof. Let h(x) = g(x) f (x): Then h(0) = 0; h0 (x) = g 0 (x) f 0 (x) 0 for all x 0:
For x = 0; it is given that f (0) = g(0); so let x > 0:
The by the Mean Value Theorem, 9c 2 (0; x) : h0 (c) = h(x)x 0h(0) = h(x)
0 =) h(x) 0 =) g(x) f (x):
x
Since our choice of x > 0 was arbitrary, we conclude that f (x) g(x) for all x 0:

236. Suppose that f : [0; 2] ! R is continuous on [0; 2] and dierentiable on (0; 2); and that f (0) = 0;
f (1) = 1; f (2) = 1:
(a) Show that there exists c1 2 (0; 1) : f 0 (c1 ) = 1:
(b) Show that there exists c2 2 (1; 2) : f 0 (c2 ) = 0:
(c) Show that there exists c 2 (0; 2) : f 0 (c) = 1=3:

Proof. (a) By the mean value theorem, 9c1 2 (0; 1) : f 0 (c1 ) = f (1)1 f0 (0) = 11 00 = 1:

(b) By the mean value theorem, 9c2 2 (1; 2) : f 0 (c2 ) = f (2)2 f1 (1) = 12 11 = 0:
(c) f is dierentiable on [c1 ; c2 ]; and f 0 (c2 ) = 0 < 1=3 < 1 = f 0 (c1 ): Therefore, by Darbouxs Theorem,
there exists c 2 (c1 ; c2 ) (0; 2) : f 0 (c) = 1=3:

237. Use the Mean Value Theorem to prove that x x 1 < ln x < x 1 for x > 1:
[Hint: Use the fact that D ln x = 1=x for x > 0]:

Proof. Let x > 1: Then f (x) = ln x is continuous on [1; x] and dierentiable on (1; x) and so by the Mean
Value Theorem, 9 c 2 (1; x) : 1c = f 0 (c) = f (x)x f1 (1) = lnxx 10 = xln x1 :
Since 1 < c < x; we get that x1 < 1c < 1:
Thus, x1 < xln x1 < 1 and so, x x 1 < ln x < x 1:
Since our choice of x > 1 was arbitrary, we conclude that x x 1 < ln x < x 1 for all x > 1:

238. Let f : [a; b] ! R be continuous on [a; b]; and dierentiable in (a; b): Show that if lim f 0 (x) = A;
x!a

then f 0 (a) exists and is equal to A:

Proof. Recall that f 0 (a) = lim f (x)x fa (a) : Let x 2 (a; b): Then, by the Mean Value Theorem, 9cx 2 (a; x) :
x!a

f 0 (cx ) = f (x)x fa (a) : Taking limits, A = lim f 0 (cx ) = lim f (x)x fa (a) = f 0 (a):
x!a
x!a

4
2x + x4 sin(1=x) if x 6= 0
239. Let f : R ! R be dened by f (x) =
: Show that f has an absolute
0
if x = 0
minimum at x = 0; but that its derivative has both positive and negative values in every neighborhood of 0.

proof. For any x 6= 0; f (x) = 2x4 + x4 sin(1=x) 2x4 x4 = x4 > 0: Since f (0) = 0; we conclude that
f (x) 0 for all x 2 R: Hence, f has an absolute minimum at x = 0:
Now, f 0 (x) = 8x3 + 4x3 sin(1=x) x2 cos(1=x):
1
Let (xn ) = 2 1n and yn = 2 n+
: Then both xn ! 0 and yn ! 0 as n ! 1:
2
Thus, every neighborhood of 0 contains an innite number of elements from both sequences.
3
2
1
Note that f 0 (xn ) = 8 2 1n
< 0 for all n large enough.
2 n
Also, f 0 (yn ) = 8
0

1
2 n+ 2

+4

1
2 n+ 2

> 0 for all n:

Thus, f has both positive and negative values in every neighborhood of 0.

240. Give an example of a uniformly continuous function on [0; 1] that is dierentiable on (0; 1) but whose
derivative is not bounded on (0; 1):

91

p
Solution. For x 0; let f (x) = x: Then f is continuous on [0; 1] and since [0; 1] is compact, f is uniformly
1
which is not bounded on (0; 1):
continuous on [0; 1]: Also, f 0 (x) = 2p
x

241. Suppose that f and g are continuous on [a; b], dierentiable on (a; b), that c 2 [a; b]; and that g(x) 6= 0
(x)
for x 2 [a; b]; x 6= c: Let A = lim f (x) and B = lim g(x): If B = 0; and if lim fg(x)
exists in R; show that we
x!c

x!c

x!c

(x)
must have A = 0: [Hint: f (x) = fg(x)
g(x)]:

(x)
Proof. Say, lim fg(x)
= L 2 R:
x!c
h
ih
i
(x)
(x)
By the hint, A = lim f (x) = lim fg(x)
g(x) = lim fg(x)
lim g(x) = LB = 0:
x!c
x!c
x!c
x!c

242. In addition to the suppositions of the preceding exercise, let g(x) > 0 for x 2 [a; b]; x 6= c: If A > 0 and
(x)
(x)
B = 0; prove that we must have lim fg(x)
= 1: If A < 0 and B = 0; prove that we must have lim fg(x)
= 1:
x!c
x!c

Proof. Let M > 0 be given. Since A = lim f (x) > 0; 9 1 > 0 : f (x) > A2 > 0 for all x 2 (c
1 ; c + 1 ) nfcg:
x!c

Since g(x) > 0 for x 2 [a; b]nfcg; and B = lim g(x) = 0; we conclude that 9
x!c

> 0 : 0 < g(x) <

A
2M

for all

1
2M
x 2 (c
2 ; c + 2 ) nfcg: Thus, g(x) > A for all x 2 (c
2 ; c + 2 ) nfcg:
Let = minf 1 ; 2 g:
(x)
1
Then for all x 2 (c
; c + ) fcg; we have fg(x)
= (f (x)) g(x)
> A2 2M
A = M:

(x)
Since our choice of M > 0 was arbitrary, we conclude that lim fg(x)
= 1:
x!c
The other case is very similar.

x2 for x 2 Q
243. Let f (x) =
; and let g(x) = sin x for x 2 R: Use Theorem 6.3.1 to show that
0 for x 2
=Q
lim f (x) = 0: Explain why LHospitals Rule I cannot be used.
x!0 g(x)

Proof. We have:
(1) f and g are dened on [0; 2 ]:
(2) f (0) = g(0) = 0:
(3) g(x) 6= 0 for all x 2 (0; 2 ):

(4) f is dierentiable at a = 0: This is because

f (x) f (0)
x 0

f (x)
x

jxj for all x 6= 0: Moreover, f 0 (0) = 0:

(5) g is dierentiable at a = 0 and g 0 (0) = cos(0) = 1:


f (x)
x!0+ g(x)

Hence, by Theorem 6.3.1, we conclude that lim


Similarly, lim
x!0

f (x)
g(x)

f (0)
g 0 (0)

0
1

f 0 (0)
g 0 (0)

0
1

= 0:

(x)
= 0: Therefore, lim fg(x)
= 0:
x!0

LHospitals Rule I cannot be used because f is not continuous at any point dierent from 0, so it cannot
be dierentiable in any neighborhood of 0.

244. Evaluate the following limits:


px
(a) lim lnx2x (0; 1);
(b) lim ln
(0; 1);
x
x!1

x!1

x
(c) lim x ln sin x (0; );
(d) lim x+ln
(0; 1):
x!1 x ln x
x!0

LH ospitals Rule
1
1
Solution. (a) lim lnx2x
=
lim 1=x
2x = lim 2x2 = 1 = 0:
x!1

(b)

x!1

LH ospitals Rule
px
p
lim ln
=
lim 1=x
x!1 x
x!1 1=2 x

sin x
(c) lim x ln sin x = lim ln1=x
x!0

x!0

LH ospitals Rule

x!1

lim p2
x!1 x
lim

x!0

2
1

= 0:

x2 sin x
cos x

92

0
1

= 0:

1+ 1

LH ospitals Rule

1
x
x
=1
= 0:
(d) lim x+ln
=
lim
x!1 x ln x
x!1 1+ln x

245. Evaluate the following limits:


(a) lim x2x (0; 1);
(b) lim (1 + x3 )x (0; 1);
x!0+

x!0

1
(c) lim (1 + x3 )x (0; 1);
(d) lim x1 arctan
(0; 1):
x
x!1
x!0+

2=x
lim (2x ln x)
lim ( 2 ln x ) LH ospitals Rule
lim
2
Solution. (a) lim x2x = lim e2x ln x = ex!0+
= ex!0+ 1=x
=
ex!0+ 1=x
x!0+

x!0+

= e0 = 1:
(b) lim (1 +
x!0
3x
lim ( x+3
)

= ex!0

3
x ln(1+ x
)

3 x
x)

=e

= lim e

3
))
lim (x ln(1+ x

x!0

x!0

lim

x!1

1
x

1
arctan x

= lim

x!0+

x
x+arctan x+x2 arctan x

x!0+

(arctan x) x
x arctan x

LH ospitals Rule

LH ospitals Rule

LH ospitals Rule

= lim

ln(1+ 3 )
x
1=x

x!1

x!0+

ln(1+ 3 )
x
1=x

LH ospitals Rule

lim

x!0

3x2
x2 +3x

= e0 = 1:

(c) lim (1 + x3 )x = lim ex ln(1+ x ) = ex!1


(d) lim

=e

lim

x!0

lim

x!0+

lim

ex!1

lim

x!0+

3x2
x2 +3x

= e3 :

1
1
1+x2
x
+arctan
x
2
1+x

2x
1
x2
1+ 1+x
2 +2x arctan x+ 1+x2

0
1+0+0+0

= 0:

246. Let g(x) = x3 for x 2 R: Find g 0 (x) and g 00 (x) for x 2 R; and g 000 (x) for x 6= 0: Show that g 000 (0) does
not exist.

x3
for x 0
Proof. g(x) =
: Let us nd g 0 (0):
x3 for x < 0

x!0+

lim

g(x) g(0)
x 0

lim

g(x) g(0)
x 0

x!0

= lim

x!0+

= lim

3x
3x2

Hence, g 0 (x) =

g 0 (x) g 0 (0)
x 0
x!0+

lim

lim

x!0

g (x) g (0)
x 0

Thus, g 00 (0) =

g 00 (x) g 00 (0)
x 0
x!0+

lim

lim

x!0

g (x) g (0)
x 0
000

6x
6x
= lim

x!0+

= lim
x!0

x!0

for x 0
for x < 0

3x2 0
x!0+ x

= lim

x2 = 0:

= lim

exists and is equal to 0.


:

= lim 3x = 0:
x!0+

3x2 0
x

x!0
g 0 (x) g 0 (0)
lim
x 0
x!0

00

x!0+

= lim

Hence, g 00 (x) =

00

= lim x2 = 0:

x3 0
x

x!0
lim g(x)x 0g(0)
x!0
2

Thus, g 0 (0) =

x3 0
x

= lim
x!0

3x = 0:

exists and is equal to 0.

for x 0
for x < 0
6x 0
x

= lim 6 = 6:

6x 0
x

x!0+

= lim
x!0

6=

6:

Thus, g (0) does not exist.


6
for x > 0
Hence, g 000 (x) =
and g 000 (0) does not exist.
6 for x < 0

247. Use induction to prove Leibnizs rule for the n-th derivative of a product:

93

(n)

(f g)

n
X

(x) =

n
k

f (n

k)

(x)g (k) (x):

k=0

0
X
(0)
0
(0 k)
(x)g (k) (x):
Proof. For n = 0; (f g) (x) = f g(x) =
k f
k=0

For n = 1; (f g) (x) = f (x)g(x) + g (x)f (x) =

1
X

1
k

f (1

k)

(x)g (k) (x):

k=0

Now, suppose the result is true for n: Then we have to prove


" n it for n + 1:
#
h
i induction hyp othesis
X
(n+1)
(n)
n
d
d
(n k)
Well, (f g)
(x) = dx
(f g) (x)
=
(x)g (k) (x)
dx
k f
=

k=0

n
X

n
k

f (n

k+1)

(x)g (k) (x) + f (n

k=0

= f

(n+1)

(x)g(x) + f

(n)

(x)g (x) +

k)

(x)g (k+1) (x)

n
X

n
k

f (n

k+1)

(x)g (k) (x) + f (n

k)

(x)g (k+1) (x)

k=1

Now, we use the fact that for 1

= f (n+1) (x)g(x) + f (n) (x)g 0 (x) +


= f (n+1) (x)g(x) + f (n) (x)g 0 (x) +

n; we have
n
Xh
n+1
k

k=1
n
X

n+1
k

n+1
k

n
k 1

f (n

k+1)

n
k

f (n

n
k 1

k+1)

( )

: We continue:

(x)g (k) (x) + f (n

(x)g (k) (x) + f (n

k)

k)

(x)g (k+1) (x)

(x)g (k+1) (x)

k=1

n
X

n
k 1

f (n

k+1)

(x)g (k) (x) + f (n

k)

(x)g (k+1) (x)

k=1

= f (n+1) (x)g(x) + f (n) (x)g 0 (x) +


n
k 1

f (n

k+1)

k=1

=f

(x)g (x) + f
n
k

f (n

k)

(x)g (x) +

"

f (n) (x)g 0 (x) +

"

n
X

(x)g (k) (x)

(n+1)

(x)g(x) +

k+1)

n
X

(x)g (k) (x) +

n
X

n+1
k

"nk=01
Xh

"k=1
n
( ) X
=
"nk=0
X1
k=1

n
k

f (n
+

n+1
k

k=0
n
X1

n+1
k

k+1)

n
k 1

f (n
f (n

f (n

n+1
k

n
k 1

f (n

k)

n+1
k

n
k

f (n

(n k+1)

k)

(x)g (k) (x) +

f (n

k+1)

k)

k)

(x)g

n
X

k=1

(k)

(x)g

n
X
(x) +
# " k=1

n+1
k

(x)g (k+1) (x)


#

"

(k)

(x) +

(x)g (k+1) (x)

n
X

n+1
k

f (n

k)

(x)g (k+1) (x)

k=1

n+1
k

f (n

k)

(x)g (k+1) (x)

nf (x)g (n+1) (x) +

f (n

k)

n
X1

n
k 1

f (n

k)

(x)g (k+1) (x)

k=1

(x)g (k+1) (x)

nf (x)g (n+1) (x)


n
X1

(x)g (k) (x) + (n + 1)f (x)g (n+1) (x) +


k=1
#

(x)g (k+1) (x)

k)

(x)g (k+1) (x)

(x)g (k+1) (x)

f (n

(x)g (k+1) (x)

(n k+1)

k)

k=1
n
X

k=1

n
X

n
k 1

n
X

n+1
k

k=1

k=1

(x)g (k+1) (x)

k=0

=f
"

f (n

k=1

"

n
X1
(n)

n+1
k

k=1

n
X

(n)

n
X

nf (x)g (n+1) (x)

94

n+1
k

f (n

k)

(x)g (k+1) (x)

=
=
=

"

n
X

"k=0
n
X

k=0
n+1
X

n+1
k

n+1
k

n+1
k

(n k+1)

f (n

f (n

k+1)

k+1)

(x)g

(k)

(x) + (n + 1)f (x)g (n+1) (x)

nf (x)g (n+1) (x)

(x)g (k) (x) + f (x)g (n+1) (x)

(x)g (k) (x):

k=0

3
5 3
248. If x > 0; show that (1 + x)1=3
1 + 31 x 19 x2
x
:
Use
this
inequality
to
approximate
1:2
81
p
3
and 2:

Proof. Let f (x) = (1 + x)1=3 =) f 0 (x) = 13 (1 + x) 2=3 =) f 00 (x) = 29 (1 + x) 5=3 :


Thus, f (0) = 1; f 0 (0) = 13 ; and f 00 (0) = 29 :
Let x > 0: Then f; f 0 ; and f 00 exist and are continuous on [0; x + 1]:
10
Note that f 000 (x) = 27
(1 + x) 8=3 exists in the interval (0; x + 1):
Hence, by Taylors theorem, f (x) = P2 (x) + R2 (x) = 1 + 13 x 19 x2 + R2 (x);
1 000
5
where R2 (x) = 3!
f (c)x3 = 81
(1 + c) 8=3 x3 for some point 0 < c < x:
1
Note that if c > 0; then 1+c
< 1 and so (1 + c) 8=3 < 1:
5
5 3
Thus, (1 + x)1=3
1 + 13 x 19 x2 = R2 (x) = 81
(1 + c) 8=3 x3 < 81
x :
p
3
1
1
5
2
3
Now, 1:2 = f (0:2) 1 + 3 (0:2) 9 (0:2) = 1: 062 222 with an error
81 (0:2) = 0:000 5 :
p
3
1
1
5
3
Also, 2 = f (1) 1 + 3 9 = 1: 222 2 with an error
81 (1) = 0:062:
p
p
3
3
Compare these to 1:2 = 1: 062 658 and 2 = 1: 259 921 ; which we get if we use a calculator.

249. If f (x) = ex ; show that the remainder term in Taylors Theorem converges to 0 as n ! 1; for each
xed x0 and x:
c n+1
c n+1

x0 )
x0 j
Proof. Fix x0 and x 2 R: Then, by Taylors theorem, jRn (x)j = e (x(n+1)!
= e jx(n+1)!
:
jRn+1 (x)j
:
n!1 jRn (x)j

Now, let L = lim

ec jx x0 jn+2 =(n+2)!
n+1
c
=(n+1)!
n!1 e jx x0 j

jx x0 j
n!1 n+2

Then L = lim

= lim

= 0 < 1:

Hence, by Theorem 3.2.11, we get that lim jRn (x)j = 0 and so, lim Rn (x) = 0:
n!1
n!1

2
e 1=x for x 6= 0
250. Let h(x) =
: Show that h(n) (0) = 0 for all n 2 N: Conclude that the remainder
0
for x = 0
term in Taylors Theorem for x0 = 0 does not converge to 0 as n ! 1; for x 6= 0: [Hint: By LHospitals
= 0 for any k 2 N: Use Leibnizs Rule to calculate h(n) (x) for x 6= 0]:
Rule, lim h(x)
k
x!0 x

2
2
Proof. For x 6= 0; h(x) = e 1=x =) h0 (x) = x23 e 1=x = P1 ( x1 )h(x); where P1 ( x1 ) is a polynomial in x1 :
Now, we use induction on n to prove that h(n) (x) = Pn ( x1 )h(x)

( )

; where Pn ( x1 ) is a polynomial in

Suppose that h(n 1) (x) = Pn 1 ( x1 )h(x); where Pn 1 ( x1 ) is a polynomial in x1 :


0
Note that the derivative Pn 1 ( x1 ) = Qn ( x1 ) is another polynomial in x1 :
0
Then h(n) (x) = Pn 1 ( x1 ) h(x) + Pn 1 ( x1 )h0 (x) = Qn ( x1 )h(x) + Pn 1 ( x1 )P1 ( x1 )h(x)
= Qn ( x1 ) + Pn 1 ( x1 )P1 ( x1 ) h(x) = Pn ( x1 )h(x); where Pn ( x1 ) = Qn ( x1 ) + Pn 1 ( x1 )P1 ( x1 ) is
a polynomial in x1 : Thus, ( ) follows.
Note that h0 (0) = lim h(x)x
x!0

h(0)
0

= lim e
x!0

Actually, for all k 2 N; lim h(x)


= lim e
xk
x!0

Hence,

x!0

1=x2

x
1=x2

xk

e y
y! 1 1=y

= lim

y LH ospital
1
=
lim
2
2
y! 1 ey
y! 1 2yey
k LH ospital k tim es
y

= lim

e y
k
y! 1 1=y

= lim

lim Pn ( x1 )h(x) = 0 for any polynomial Pn ( x1 )

x!0

= lim
)

y! 1 ey

Now, we use induction to prove that h(n) (0) = 0 for all n 2 N:


95

0:

= 0:

1
x:

For n = 1; we showed it above. So, suppose h(n


Then, h
= lim

(n)

h(n

x!0

x!0
(x) 0
x 0

(n

(x) h
x 0

1)

1)

1)

(0) = 0

(0)

By induction hypothesis

(x)

x!0

= lim

1)

1)

(n

= lim h

(0) = lim

(n

Pn

1
1 ( x )h(x)

1
1 ( x )h(x)

= lim Qn
x!0

1
1( x )

For some polynomial Pn

x!0

Where Qn

1
1( x )

Pn

1
1( x )

=0
as in ( ) above.

251. We wish to approximate sin by a polynomial on [ 1; 1] so that the error is less than 0:001. Show that
3
x5
1
we have sin x
x x6 + 120
< 5040
for jxj 1:

Proof. We have f (x) = sin x; f 0 (x) = cos x; f 00 (x) = sin x; f 000 (x) = cos x; f (4) (x) = sin x;
f (5) (x) = cos x; f (6) (x) = sin x; and f (7) (x) = cos x:
Thus, f (0) = 0; f 0 (0) = 1; f 00 (0) = 0; f 000 (0) = 1; f (4) (0) = 0; f (5) (0) = 1; and f (6) (0) = 0:
6
X
f (k) (0) k
1 (7)
Let 1 x 1:Then, by Taylors theorem, f (x) = P6 (x) + R6 (x) =
(c)x7 ; for
k! x + 7! f
k=0

some c between 0 and x:


Now, jR6 (x)j = jf (x) P6 (x)j = f (x)
1 (7)
(c)x7
7! f

x3
6

x
7

1
7!

x5
120

1
7!

1
5040

But, jR6 (x)j =


= j cos(c)j jxj
=
for all 1 x 1:
1
Thus, approximating sin by the Taylors 6th-polynomial on [ 1; 1] gives an error
0:001:
7! = 0:000 1
R
b
252. If f 2 R[a; b] and jf (x)j M for all x 2 [a; b]; show that a f
M (b a):
R

R b Rb
b
Proof. We have M f (x) M for all x 2 [a; b] and so a M
f (x)
M:
a
a
Rb
(Recall that constant functions are Riemann integrable and a M = M (b a)).
Rb
Rb
M (b a):
Hence, M (b a)
f M (b a) and so a f
a

253. Suppose f is bounded on [a; b] and that there exists two sequences of tagged partitions of [a; b] such that
:
:
:
:
P n ! 0 and Qn ! 0; but such that lim S(f ; P n ) 6= lim S(f ; Qn ): Show that f 2
= R[a; b]:
n!1
n!1
R
b
Proof. Suppose that f 2 R[a; b] with a f = L and let > 0 be given. Then, by denition, there exists > 0
:

such that for any tagged partition P with P <


:

; we have S(f ; P)

Since P n ! 0 and Qn ! 0; we can nd N 2 N : P n <


Hence,

:
S(f ; Pn )

and S(f ; Qn )

L <

for all n

and Qn <

L <

L < :

for all n

N:

N:
:

Since our choice of > 0 was arbitrary, we conclude that lim S(f ; P n ) = L = lim S(f ; Qn ):
n!1
n!1

1 if x 2 Q \ [0; 1]
254. Let f be the Dirichlet function dened by f (x) =
: Show that f 2
= R[0; 1]:
0 if x 2 [0; 1] Q

Proof. Let Pn = f[ j n 1 ; nj ]gnj=1 be the partition of [0; 1] into n closed subintervals of equal length (= n1 ):
Then kPn k = n1 ! 0 as n ! 1:
:

Now, let P n = Pn with tags qj 2 Q and let Qn = Pn with tags rj 2


= Q:
:

Then P n ! 0, Qn ! 0:

Moreover, for all n 2 N; we have S(f ; Pn ) =

n
P

j=1

f (qj )( nj

96

j 1
n )

n
P

j=1

1
n

= 1; and

n
P

S(f ; Qn ) =

j=1

j 1
n )

f (rj )( nj
:

n
P

j=1

0 n1 = 0:
:

Hence, lim S(f ; P n ) = 1 6= 0 = lim S(f ; Qn ):


n!1

n!1

Thus, by the previous exercise, we conclude that f 2


= R[0; 1]:

255. Suppose that f : [a; b] ! R and that f (x) = 0 except for a nite number of points c1 ; :::; cn in [a; b]:
Rb
Prove that f 2 R[a; b] and that a f = 0:

Proof. Let M = maxfjf (c1 )j ; :::; jf (cn )jg and let > 0 be given. Here, there are n points where f is not 0;
:
each of which can belong
to
2
subintervals
in
a
given
tagged
partition
P:
Only these terms will make a nonzero
:
contribution to S(f ; P): Therefore, we choose = 2nM :
:

If P <

; let P 0 be the subset of P with tags dierent from c1 ; :::; cn ; and let P 1 be the subset of P with
:

tags at these points. Since S(f ; P0 ) = 0; it is seen that S(f ; P) = S(f ; P1 ):


:
Since there are at most 2n terms in S(f ; P1 ) and each term is between M and M ; we conclude that
:
2n( M ) < S(f ; P1 ) < 2n(M ):
:
:
:
Choosing = 2nM ; we get that
< S(f ; P1 ) = S(f ; P) < and so S(f ; P) 0 < :
:

Since our choice of the tagged partition P with P <


:

was arbitrary, we conclude that S(f ; P) = 0

for all tagged partitions P of [a; b] with P < :


Rb
Hence, f 2 R[a; b] and that a f = 0:

256. If g 2 R[a; b] and if f (x) = g(x) except for a nite number of points in [a; b]; prove that f 2 R[a; b]
Rb
Rb
and that a f = a g:

Proof. Let h(x) = f (x) g(x): Then h : [a; b] ! R and h(x) = 0 except for a nite number of points in [a; b]:
Rb
Hence, by the previous exercise, h 2 R[a; b] and that a h = 0:
Rb
Rb
Rb
Rb
Now, f (x) = h(x) + g(x) and g; h 2 R[a; b] =) f 2 R[a; b] and a f = a g + a h = a g:

257. Letq0 a < b; let f (x) = x2 for x 2 [a; b]; and let P = f[xi 1 ; xi ]gni=1 be a partition of [a; b]: For each i;

let qi = 13 x2i + xi 1 xi + x2i 1 :


(a) Show that qi satises 0 xi 1 qi
(b) Show that f (qi )(xi xi 1 ) = 13 x3i
:

xi :
x3i 1 :

(c) If Q is the tagged partition with the same subintervals as P and the tags qi ; show that
:
S(f ; Q) = 13 b3 a3 :
Rb
Rb
(d) Show that f 2 R[a; b] and a f = a x2 dx = 13 b3 a3 :

Proof. (a) We will show that 0 x2i 1 qi2 x2i and since the function f (x) = x2 is increasing on [0; 1);
we will conclude that 0 xi 1 qi xi :
1
2
Now, since 0 a xi 1 < xi b; we have that 2x2i 1 x2i + xi xi 1 =) 32 x2i 1
3 xi + xi xi 1
1
1
2
2
2
2
2
=) x2i 1 31 x2i 1
3 xi + xi xi 1 =) xi 1
3 xi + xi xi 1 + xi 1 = qi :
1
2
1
1 2
2
2
Also, x2i + xi xi 1 2x2i =) 3 x2i + xi xi 1
x2i
3 xi =) 3 xi + xi xi 1
3 xi
=) qi2 = 13 x2i + xi xi 1 + x2i
x2i :
Hence, 0 x2i 1 qi2 x2i and so 0 xi 1 qi xi :
(b) Recall that a3 b3 = (a b)(a2 + ab + b2 ):
We have f (qi )(xi xi 1 ) = qi2 (xi xi 1 ) = 13 x2i + xi xi
:

(c) We have S(f ; Q) =

n
P

i=1

f (qi )(xi

xi

1)

(b)

n
P

i=1

1
3

x3i

97

+ x2i (xi

x3i

1
3

xi
n
P

i=1

1)

x3i

1
3

x3i

x3i
1

x3i

= 13 (x3n

:
x30 )

1
3

b3

a3 :
:

(d) Let > 0 be given. Let P = f(Ii ; ti )gni=1 be an arbitrary tagged partition of [a; b] with P < ;
(where > 0 is to be determined later) so that xi xi 1 < for i = 1; :::; n:
:
:
:
Let Q be the tagged partition with the same subintervals as P but with the tags qi ; i.e., Q = f(Ii ; qi )gni=1
with qi is in the statement of the exercise.
:
:
Since both ti and qi 2 Ii and P = Q < ; we conclude that jti qi j < for all 1 i n:
Using the triangle inequality, one gets that
n
n
n
:
:
P
P
P
[f (ti ) f (qi )] (xi xi 1 )
f (qi )(xi xi 1 ) =
S(f ; P) S(f ; Q) =
f (ti )(xi xi 1 )
=

n
P

i=1
n
P

i=1

t2i

qi2

(xi

xi

n
P

1)

i=1

(a + b) (xi

xi

1)

i=1

t2i

qi2

(a + b)

i=1

(xi

n
P

<

b2 a2 ;

1
3

Since, by (c), S(f ; Q) =

1)

(xi

xi

1)

i=1

jti

qi j (ti + qi ) (xi

(a + b) (b

a) =

xi

(b2

1)

a2 ):

one gets that S(f ; P)


b3

i=1

i=1

Thus, choosing

xi

n
P

S(f ; Q) < :
:

1
3

a3 ; we conclude that S(f ; P)


:

b3

a3

< :

Hence, since our choice of the tagged partition P with P < was arbitrary, we conclude that
Rb
Rb
f 2 R[a; b] and a f = a x2 dx = 13 b3 a3 :

x + 1 if x 2 Q \ [0; 1]
258. Consider the function h(x) =
: Show that h 2
= R[0; 1]:
0
if x 2 [0; 1] Q

n
Proof. Let 0 = 12 and let n 2 N: Let Pn = j n 1 ; nj j=1 and consider the two tagged partitions
:

Pn =
Then,

:
n
j 1 j
;
;
t
and
Q
j
n =
n
n
j=1
:
:
1
1
Pn
n and Qn
n:
:

Moreover, S(h; P n ) =

n
P

h(tj )( nj

j=1
:

On the other hand, S(h; Qn ) =


:

n
P

j=1

Hence, S(h; P n )

j 1 j
n ; n

S(h; Qn ) =

"

j 1
n )

h(rj )( nj
1
n

n
P

tj

j=1

; rj

1
n

; where each tj 2 Q and each rj is irrational.

j=1

n
P

(tj + 1) =

j=1
j 1
n )

= 0:

+1

1>

1
n

n
P

tj

j=1

+1

1:

0:

This implies that h 2


= R[0; 1]:

k
if x = k1 ; k 2 N
: Show that H 2
= R[0; 1]:
259. Let H(x) =
0 if x 2 [0; 1] f k1 : k 2 Ng

n
Proof. Let 0 = 12 and let n 2 N: Let Pn = j n 1 ; nj j=1 and consider the two tagged partitions
:

P n = j n 1 ; nj ; tj j=1 and Qn =
and each rj is irrational.
:

1
n

j=1
:

On the other hand, S(H; Qn ) =


:

; rj

; where t1 =

j=1

1
n;

and all other tj are irrationals,

1
and Qn
n:
n
:
P
Moreover, S(H; P n ) =
H(tj )( nj

Then, P n

j 1 j
n ; n

n
P

j=1

j 1
n )

H(rj )( nj

1
n H(t1 )
j 1
n )

= 1:

= 0:

Hence, S(H; P n ) S(H; Qn ) = j1 0j = 1 > 0 :


This implies that H 2
= R[0; 1]:

98

260. If S(f ; P) is any Riemann sum of f : [a; b] ! R, show that there exists a step function ' : [a; b] ! R
:
Rb
such that a ' = S(f ; P):

n
:
P
Proof. Say, S(f ; P) =
f (tj )(xj xj 1 ): Dene ' : [a; b] ! R by
j=1

'(x) = f (tj )

for x 2 [xj

1 ; xj ]

Then ' is a step function and so ' 2 R[a; b]:


n
n
n R
n R
Rx
Rb
P
P
P
P
xj
xj
f (tj )(xj
f (tj ) xjj 1 dx =
f (tj )dx =
'=
Thus, a ' =
xj 1
xj 1
j=1

j=1

xj

j=1

j=1

1)

= S(f ; P):

R
b
261. Suppose that f is continuous on [a; b]; that f (x) 0 for all x 2 [a; b] and that a f = 0: Prove that
f (x) = 0 for all x 2 [a; b]:

Proof. Suppose that there is an x0 2 [a; b] such that f (x0 ) > 0: Then, by continuity of f; we can nd
a closed interval [c; d] [a; b] such that x0 2 [c; d] and f (x) > 12 f (x0 ) for all x 2 [c; d]:
Rb
Rc
Rd
Rb
Rd
Rd
Now, 0 = a f = a f + c f + d f 0 + c f + 0 > c 12 f (x0 )dx = 12 f (x0 ) (d c) > 0; a contradiction.
Hence, f (x) = 0 for all x 2 [a; b]:

262. Show that the continuity hypothesis in the preceeding exercise cannot be dropped.

Rb
1
if x = a
Proof. Let f (x) =
: Then, f (x) 0 for all x 2 [a; b], and a f = 0:
0 if x 2 (a; b]
Nevertheless, f is not identically 0 on [a; b]:
R R
b
b
263. If f and g are continuous on [a; b] and if a f = a g; prove that there exists c 2 [a; b] such that
f (c) = g(c):

R b
Proof. Let h(x) = f (x) g(x) for x 2 [a; b]: Then h is continuous on [a; b] and a h = 0:
If h(x) 0 for all x 2 [a; b]; then, by the above exercise, h(x) = 0 for all x 2 [a; b].
Hence, f (x) = g(x) for all x 2 [a; b]:

If h(x) 0 for all x 2 [a; b]; then, similarly, h(x) = 0 for all x 2 [a; b]:
Hence, f (x) = g(x) for all x 2 [a; b]:
If h(x1 ) < 0 and h(x2 ) > 0 for some x1 ; x2 2 [a; b]; then, by Bolzanos intermediate value property for
continuous functions, 9x1 < c < x2 such that h(c) = 0: Hence, f (c) = g(c).

264. Give an example of a function f : [a; b] ! R such that f 2 R[c; b] for each c 2 (a; b) but f 2
= R[a; b]:

0
if x = 0
Solution. Let f (x) =
: Then f is continuous on each closed interval [c; 1] where
1
if
x 2 (0; 1]
x
c 2 (0; 1) and so f 2 R[c; 1] for each c 2 (0; 1):
Now, f is not bounded on [0; 1] and hence f 2
= R[0; 1]:

265. (Mean Value Theorem for Integrals) If f is continuous on [a; b]; a < b; show that there exists c 2 [a; b]
Rb
such that a f = f (c)(b a):

Proof. Let m = inf f (x) and M = sup f (x): (They exist because f is continuous and [a; b] is compact).
x2[a;b]
x2[a;b]
Rb
Rb
Rb
Rb
Then a f
M = M (b a) and a f
m = m(b a):
a
a
( )
Rb
Rb
1
Hence, m(b a)
f M (b a) =) m b a a f M
:
a
99

But since f is continuos on [a; b] and m f (x) M , we get by Bolzanos intermediate value theorem that
f assumes all the values between m and M: In particular, from ( ), we see that there exists c 2 [a; b] such
Rb
Rb
that f (c) = b 1 a a f =) a f = f (c)(b a):

266. If : [a; b] ! R takes on only a nite number of distinct values. is a step function?

1 for x 2 [0; 1] \ Q
Proof. Not necessarily. Consider the Dirichlet function (x) =
:
0
for x 2 [0; 1]nQ
Then takes on the values 0 and 1 on [0; 1], but cannot be a step function or else it has to be Riemann
integrable and we showed in a previous exercise that the Dirichlet function is not Riemann integrable.

267. If (x) = x and !(x) = x and if (x) f (x) !(x) for all x 2 [0; 1]; does it follow from the squeeze
theorem that f 2 R[0; 1]?

x for x 2 [0; 1] \ Q
Solution. No. Let f (x) =
: Then x f (x) x for all x 2 [0; 1]:
0
for x 2 [0; 1]nQ
Now, we show that f 2
= R[0; 1]:
n
Let 0 = 21 and let n 2 N: Let Pn = j n 1 ; nj j=1 and consider the two tagged partitions
:

:
n
j 1 j
n ; n ; tj j=1 and Qn =
:
:
1
1
Then, P n
and
Q
n
n
n:
n
:
P
Moreover, S(f ; P n ) =
f (tj )( nj
j=1
n
:
P

Pn =

On the other hand, S(f ; Qn ) =


:

j=1

j 1 j
n ; n

j 1
n )

f (rj )( nj

; rj

1
n

n
P

j=1

j=1

j
n

j 1
n )

; where each tj =

1
n2

n
P

j=1

= 0:

j=

n(n+1)
2n2

j
n

2 Q and each rj is irrational.

1
2

1
2n

1
2:

1
1
Hence, S(f ; P n ) S(f ; Qn ) = 12 + 2n
0
2 = 0:
This implies that f 2
= R[0; 1]:

268. If f is bounded by M on [a; b] and if the restriction of f to every interval [c; b] where c 2 (a; b) is
Rb
Rb
Riemann integrable, show that f 2 R[a; b] and that lim c f = a f:
c!a+

M for x 2 [a; c)
M
for x 2 [a; c)
and ! c (x) =
:
f (x) for x 2 [c; b]
f (x) for x 2 [c; b]
Apply the squeeze theorem for c su ciently near a].

Proof. Let > 0 be given. Then there exists c 2 (a; b) such that c a < 2M :
[Hint: Let

Let

c (x)

c (x)

M for x 2 [a; c)
f (x) for x 2 [c; b]

and ! c (x) =

M
for x 2 [a; c)
f (x) for x 2 [c; b]

Then both c and ! c are Riemann integrable on [a; b] by the given.


Also, since M f M on [a; b]; we have that c (x) f (x) ! c (x) for all x 2 [a; b]:
Rb
Rc
Rb
Rc
Rb
Rc
Moreover, a (! c
( M )) + c (f f ) = a 2M
c ) = a (! c
c ) + c (! c
c ) = a (M
= 2M (c a) < 2M 2M = :
Hence, by the squeeze theorem, f 2 R[a; b]:

Rc
Rc
f
=
f
M = M (c a) < M 2M = 2 < :
a
c
a
a
Rb
Rb
Thus, lim c f = a f:
c!a+

sin (1=x) for x 2 (0; 1]


269. Show that g(x) =
belongs to R[0; 1]:
0
for x = 0

Proof. Note that jg(x)j 1 for all x 2 [0; 1] and that for any c 2 (0; 1); g is continuous on [c; 1] and hence
Now,

Rb

Rb

100

g 2 R[c; 1]: Thus, by the previous exercise, we conclude that g 2 R[0; 1]:

270. If f is bounded and there is a nite set E such that f is continuous at every point of [a; b]nE; show that
f 2 R[a; b]:

Proof. Say, E = fc1 ; :::; cn g where a < c1 < c2 <


< cn < b: (If E \ fa; bg =
6 ?; then the proof is the same).
We will be done, by the additivity theorem, if we show that f 2 R[a; c1 ]; f 2 R[cn ; b]; and f 2 R[cj 1 ; cj ]
for 2 j n. Since the idea of the following proof will be the same for proving all of the above statements,
we will be done if we show that f 2 R[c1 ; c2 ]: So, let us do this.
Fix 2 (c1 ; c2 ): Then f is continuous on [ ; ] for every 2 (c1 ; ) and so f 2 R[ ; ] for all 2 (c1 ; ):
Also, f is bounded on [c1 ; ]: Hence, by a previous exercise, f 2 R[c1 ; ] for every 2 (c1 ; c2 ):
Also, f is bounded on [c1 ; c2 ]: Therefore, by the same previous exercise, f 2 R[c1 ; c2 ]:

271. Let f be continuous on [a; b]; let f (x) 0 for x 2 [a; b]; and let
Mn =
Show that lim Mn = sup ff (x)g :
n!1

Rb
a

fn

1=n

x2[a;b]

Proof. Since f is continuous on [a; b] and [a; b] is compact, M = sup ff (x)g exists at some point c 2 [a; b]:
x2[a;b]

If M = 0; then Mn = 0 and so we are done. Thus, suppose that M > 0: Assume that c 2 [a; b):
(A similar argument to what follows takes care of the case c = b):
Let 0 < < M be given. Since f is continuous on [a; b]; we can nd a > 0 such that 8x 2 [c; c + ]
n
we have that f (x) M
> 0 =) f n (x) (M
) for all x 2 [c; c + ]:
Rb
R c+ n R c+
n
n
Thus, a f n
) 1=n for all n 2 N (1) :
f
(M
) = (M
) =) Mn (M
c
c
Now, f (x)
=) Mn

M for all x 2 [a; b] =) f n (x)

M (b

1=n

a)

for all n 2 N

(2)

M n for all x 2 [a; b] =)

Rb
a

fn

Rb
a

M n = M n (b

[a; b];

a)

Therefore, (M
) 1=n Mn M (b a)1=n for all n 2 N:
Hence, by the squeeze theorem for sequences, M
lim Mn M:
n!1
Since our choice of 0 < < M was arbitrary, we conclude that lim Mn = M:
n!1

n+1
272. If n 2 N and Hn (x) = xn+1 for x 2 [a; b]; show that the rst form of the fundamental theorem of
Rb
n+1
n+1
calculus implies that a xn dx = b n+1a : What is the set E for which Hn0 (x) 6= xn ?

Proof. We have that:


n+1

(a) Hn (x) = xn+1 is a polynomial and so is continuous on R: In particular, Hn (x) is continuous on [a; b]:
(b) Hn0 (x) = xn for all x 2 R: In particular, Hn0 (x) = xn for all x 2 [a; b]:
(c) h(x) = xn is continuous on [a; b] and so h(x) 2 R[a; b]:
Thus, by the rst form of the fundamental theorem of calculus, we deduce that
Rb n
Rb
n+1
n+1
x dx = a h(x)dx = Hn (b) Hn (a) = b n+1a :
a

R3
x for jxj 1
273. If g(x) =
and if G(x) = 12 x2 1 ; show that 2 g(x)dx = G(3) G( 2) = 5=2:
x for jxj < 1

Proof. We have that:

101

(a) G(x) = 12 x2 1 is continuous on [ 2; 3] since it is a composition of continuous functions on R.


(b) G0 (x) = g(x) for x 2 [ 2; 3]nf 1; 1g:
(c) g(x) is continuous on [ 2; 3]nf 1; 1g and so g(x) 2 R[ 2; 3]:
Thus,
by the rst form of the fundamental theorem of calculus, we deduce that
R3
g(x)dx = G(3) G( 2) = 4 (3=2) = 5=2:
2

1 2
Rb
for x < 0
2x
: Show that a jxj dx = B(b) B(a):
274. Let B(x) =
1 2
for x 0
2x

Proof. We have that:


(a) B(x) is continuous at x = 0; and so B(x) is continuous on [a; b]:
x for x < 0
(b) B 0 (x) =
= jxj for x 2 [a; b]nf0g:
x for x > 0
(c) jxj is continuous on [a; b] and so jxj 2 R[a; b]:
Thus, by the rst form of the fundamental theorem of calculus, we deduce that
Rb
jxj dx = B(b) B(a):
a
R
z
275. If f 2 R[a; b] and if c 2 [a; b]; the function dened by Fc (z) = c f for z 2 [a; b] is called the indenite
integral of f with basepoint c: Find a relation between Fa and Fc :
R R R R R
c
c
z
z
c
f:
Proof. We have Fa (z) = a f = a f + c f = a f + Fc (z): Hence, Fc (z) = Fa (z)
a
8

9
< x for 0 x < 1 =
Rx
1 for 1 x < 2
276. Let f : [0; 3] ! R be dened by f (x) =
: Obtain formulas for F (x) = 0 f and
:
;
x for 2 x 3
sketch the graphs of f and F: Where is F dierentiable? Evaluate F 0 (x) at all such points.

R x R
x
Proof. If 0 x < 1; then F (x) = 0 f = 0 tdt = 21 x2 :
Rx
R1
Rx
R1
Rx
If 1 x < 2; then F (x) = 0 f = 0 f + 1 f = 0 tdt + 1 dt = 12 + (x 1) = x 12 :
Rx
R1
R2
R3
R1
R2
Rx
2
If 2 x 3; then F (x) = 0 f = 0 f + 1 f + 2 f = 0 tdt + 1 dt + 2 tdt = 12 + 1 + x2
2 = 12 x2 1 :
8
9
1 2
x
for
0
x
<
1
=
<
2
x 12
for 1 x < 2
Hence, F (x) =
and it is easy to see that it is continuous.
: 1 2
;
x
1
for
2
x
3
2
Here are the graphs of f and F :

f (x)

F (x)

F is dierentiable everywhere on [0; 3] except when x = 2:


102

F (2+h) F (2)
h
h!0+

This is because lim


and lim
h!0

F (2+h) F (2)
h

= lim
1
2

h!0+

((2+h)2

3
2

1)

3
2

h!0+

(2+h

= lim

1
2

h
h!0+ h

= lim

2h+ 12 h2
h
h!0+

= lim

= lim

h!0+

2 + 12 h = 2;

= lim 1 = 1:
h!0+

8
9
< x for 0 x < 1 =
1 for 1 x < 2
Finally, F 0 (x) =
:
:
;
x for 2 < x 3

R x+c
277. If f : R ! R is continuous and c > 0; dene g : R ! R by g(x) = x c f (t)dt. Show that g is
dierentiable on R and nd g 0 (x):
R R R

R x c+h
x+h+c
x+c
x+c+h
Proof. we have g(x + h) g(x) = x+h c f (t)dt
f
(t)dt
=
f
(t)dt
f (t)dt:
x c
x+c
x c
Let > 0 be given. Since f is continuous at t = x c and t = x + c; we have that
For all t 2 [x c; x c + h]; f (x c)
For all t 2 [x + c; x + c + h]; f (x + c)

f (t)
f (t)

2
2

f (x c) + 2 ; and
f (x + c) + 2 :

Thus, we ge that
Rx

c+h
c
Rxx+c+h
x+c

f (x

c)

dt

f (x + c)

dt

Rx

This can be rewritten as


f (x

c)

f (x + c)

Rx

c+h
f (t)dt
x c
R x+c+h
f (t)dt
x+c

Therefore,
f (x

c) +

f (x + c)

Rx

c+h
f (t)dt
c
Rxx+c+h
f (t)dt
x+c

Rx

c+h

f (x

c
Rxx+c+h
x+c

f (x

c) +

dt; and

f (x + c) +

dt:

c) +

h; and

f (x + c) +

h:

c+h

f (t)dt
f (x
x c
R x+c+h
f (t)dt
f (x + c) +
x+c

c)
2

h; and

h:

Adding the two inequalities above and dividing by h we get:


(f (x + c)
But

hR

Hence,

x+c+h
x+c

f (x

f (t)dt

c)

Rx

c+h
x c

g(x+h) g(x)
h

hR

x+c+h
x+c

i
f (t)dt =h =

(f (x + c)

f (x

f (t)dt

Rx

c+h
x c

i
f (t)dt =h

(f (x + c)

f (x

c) + )

g(x+h) g(x)
:
h

c))

and so

g(x+h) g(x)
h

(f (x + c)

f (x

c))

Since our choice of > 0 was arbitrary, we conclude that g 0 (x) = lim g(x+h)h g(x) = f (x + c) f (x c):
h!0
R R
x
1
278. If f : [0; 1] ! R is continuous and 0 f = x f for all x 2 [0; 1]; show that f (x) = 0 for all x 2 [0; 1]:
R R R R R
R x
R x
R x
1
1
1
x
1
Proof. we have 0 f = 1 f = 0: Also, 0 f = 0 f + x f = 0 f + 0 f = 2 0 f:
Rx
Hence, 0 f = 0 for all x 2 [0; 1]:
Rx
Let F (x) = 0 f (t)dt: Then, since f is continuous on [0; 1]; F is an antiderivative for f , i.e.,
F 0 (x) = f (x) for all x 2 [0; 1]:
Moreover, F (x) = 0 for all x 2 [0; 1] =) F 0 (x) = 0 for all x 2 (0; 1):
Thus, f (x) = 0 for all x 2 (0; 1) and by continuity of f; we get that f (x) = 0 for all x 2 [0; 1]:

279. (a) If Z1 and Z2 are null sets, show that Z1 [ Z2 is a null set.
103

1
S
(b) More generally, if Zn is a null set for each n 2 N; show that
Zn is a null set.
n=1

Proof. Since (a) is a special case of (b) where Zn = ? for all n 3; it su ces to prove (b).
1
Let > 0 be given. Since Zn is a null set, there exists a countable collection of open intervals fJkn gk=1
1
1
S
P
jJkn j < 2n : (Here, jJkn j denotes the length of the open interval Jkn ):
such that Zn
Jkn and

Thus,

1
S

n=1
1
S

Zn

k=1
1
S

n;k=1

k=1

Jkn and

1
P

n;k=1

jJkn j =

1
P

1
P

n=1

k=1

jJkn j

<

1
P

n=1

2n

1
P

n=1

1
2n

= :

Hence,
Zn is a null set.
n=1

280. Let f; g 2 R[a; b]:


Rb
2
(a) If t 2 R; show that a (tf g)
0:
Rb
Rb 2 1 Rb 2
(b) Use (a) to show that 2 a f g
t a f + t a g for t > 0:
Rb 2
Rb
Rb
Rb
(c) If a f = 0; show that a f g = 0; and if a g 2 = 0; show that a f g = 0:
(d) Prove the following Schwartz Inequality:
Rb
a

Rb

fg

jf gj

Rb
a

f2

Rb
a

g2

Proof. (a) Let t 2 R: Since f 2 R[a; b]; tf 2 R[a; b]; and since g 2 R[a; b]; tf g 2 R[a; b]:
2
By the product rule for integrals, we get that (tf g) 2 R[a; b]:
R
b
2
2
Since (tf g)
0; we have that a (tf g)
0:

Rb
Rb
Rb
Rb
Rb
2
(tf + g) = a t2 f 2 + 2tf g + g 2 = t2 a f 2 + 2t a f g + a g 2 :
(b) We have 0
a
Hence, since t > 0; we can divide by t to get:
Rb 2
Rb
R
Rb
R b 2 1 R b 2 (1)
1 b 2
0 t a f + 2 a f g + t a g =) 2 a f g
t af +t ag
:
Rb
Rb
Rb
2
Also, we have 0
(tf g) = a t2 f 2 2tf g + g 2 = t2 a f 2
a
Hence, since t > 0; we can divide by t to get:
(2)
Rb
Rb
Rb
Rb
Rb
Rb
0 t a f 2 2 a f g + 1t a g 2 =) 2 a f g t a f 2 + 1t a g 2 :
By (1) and (2), we get that 2

Rb
a

fg

Rb
a

f2 +

1
t

Rb
a

2t

Rb
a

fg +

Rb
a

g2 :

g2 :

Rb
Rb
Rb 2
1
(c) Suppose that a f 2 = 0: Then by (b), a f g
2t a g for all t > 0: Letting t ! 1; we get that
Rb
Rb
f g = 0 and so a f g = 0:
a
Rb
Rb
R
t b 2
Suppose that a g 2 = 0: Then by (b), a f g
2 a f for all t > 0: Letting t ! 0; we get that
Rb
Rb
f
g
=
0
and
so
f g = 0:
a
a

Rb
Rb
Rb
(d) If a f 2 = 0 or a g 2 = 0; then, by (c), a f g = 0 and the inequality follows trivially.
Rb
Rb
So, suppose that a f 2 > 0 and a g 2 > 0:
R
1=2
1=2 R b
Rb
Rb
Rb
Rb
( b g2 )
t a f 2 + 1t a g 2 = 2 a f 2
g2
Let t = Rab 2 1=2 : Then t > 0; and so by (b), 2 a f g
a
( at )
Rb
R b 2 1=2 R b 2 1=2
Thus, a f g
f
g
:
a
a
2
Rb
Rb 2 Rb 2
Squaring, we get a f g
f
g :
a
a

104

1=2

Now,

Rb
a

Rb

fg

jf gj =)

Rb

Rb

fg

( )

jf gj

Also, applying the same argument to jf j and jgj ; we get that


2
2
2
Rb
Rb
Rb
Rb 2 Rb 2
Rb
jf gj = a jf gj = a jf j jgj
jf j
jgj = a f 2
a
a
a

Hence,

Rb
a

Rb

jf gj

f2

Rb
a

g2

Rb
a

g2 :

2
2
Rb
Rb
Rb 2 Rb 2
Hence, by ( ) and ( ), we get that a f g
jf
gj
f
g :
a
a
a

x
281. Show that lim x+n
= 0 for all x 2 R; x 0:
n!1

x
x
Proof. If x = 0; then x+n
= 0 for all n 2 N and so lim x+n
= 0:
n!1

x
x
If x > 0; then 0 < x+n
lim nx = x lim n1 = 0; the squeeze theorem implies that
n and since n!1
n!1
x
x
lim x+n
= 0: Thus, lim x+n
= 0 for all x 2 R; x 0:
n!1
n!1

nx
282. Show that lim 1+n
2 x2 = 0 for all x 2 R:
n!1

nx
nx
Proof. If x = 0; then 1+n
2 x2 = 0 for all n 2 N and so lim 1+n2 x2 = 0:
nx
If x > 0; then 0 < 1+n
2 x2
nx
Thus, lim 1+n
=
0.
2 x2

1
nx

1
n!1 nx

and since lim

n!1
1
lim n1
x n!1

= 0; the squeeze theorem implies that

n!1

1
nx
1
If x < 0; then nx
lim nx
= x1 lim n1 = 0; the squeeze theorem implies that
1+n2 x2 < 0 and since n!1
n!1
nx
Thus, lim 1+n
2 x2 = 0.
n!1
nx
Therefore, lim 1+n
2 x2 = 0 for all x 2 R:
n!1

nx
283. Evaluate lim 1+nx
for x 2 R; x 0:
n!1

nx
nx
Proof. If x = 0; then 1+nx
= 0 for all n 2 N and so lim 1+nx
= 0:
n!1

If x > 0; then

nx
1+nx

1 =

1
1+nx

! 0 as n ! 1:

0 if x = 0
nx
Hence, lim 1+nx
=
:
1 if x > 0
n!1

xn
284. Evaluate lim 1+x
0:
n for x 2 R; x
n!1

xn
xn
Proof. If x = 0; then 1+x
n = 0 for all n 2 N and so lim 1+xn = 0:
n!1

x
n
If 0 < x < 1; then 1+x
! 0 as n ! 1):
n ! 0 as n ! 1 (since x
n
x
1
1
xn
1
If x = 1; then 1+xn = 1+1 = 2 for all n 2 N and so lim 1+x
n = 2:
n!1

xn
1+xn

1
1+xn

1 =
! 0 as n ! 1 (since xn ! 1 as n ! 1):
8
9
0 if 0 x < 1 =
<
1
xn
if x = 1
Hence, lim 1+x
:
n =
n!1
: 2
;
1
if x > 1

285. Evaluate lim sin(nx)


for x 2 R; x 0:
n!1 1+nx

sin(nx)
Proof. If x = 0; then sin(nx)
1+nx = 0 for all n 2 N and so lim 1+nx = 0:
If x > 1; then

n!1

If x > 0; then 0

sin(nx)
1+nx

1
1+nx

Hence, by the squeeze theorem,

1
nx ! 0 as
lim sin(nx) = 0:
n!1 1+nx

<

n ! 1.

105

= 0 for x 2 R; x 0:
Thus, lim sin(nx)
n!1 1+nx

286. Evaluate lim e nx for x 2 R; x 0:


n!1

Proof. If x = 0; then e nx = 1 for all n 2 N and so lim e nx = 1:


If x > 0; then 0 < e

n!1

x n

< 1 and so lim (e

) = 0:

n!1

1 if x = 0
Thus, lim e nx =
:
0 if x > 0
n!1

287. Show that lim xe nx = 0 for x 2 R; x 0:


n!1

Proof. If x = 0; then xe nx = 0 for all n 2 N and so lim xe nx = 0:


If x > 0; then 0 < e

n!1

x n

< 1 and so lim x (e

) = x lim (e

n!1

x n

) = 0:

n!1

Thus, lim xe nx = 0 for x 2 R; x 0:


n!1

288. Show that lim x2 e nx = 0 and lim n2 x2 e nx = 0 for x 2 R; x 0:


n!1
n!1

Proof. If x = 0; then x2 e nx = 0 for all n 2 N and so lim x2 e nx = 0:


If x > 0; then 0 < e

Thus, lim x2 e
n!1

nx

< 1 and so lim x2 (e


n!1

= 0 for x 2 R; x

If x = 0; then n2 x2 e

n!1

x n

x n

) = x2 lim (e

) = 0:

n!1

0:

nx

= 0 for all n 2 N and so lim n2 x2 e nx = 0:


n!1
1
P
n3 x3
nk xk
nx
nx
If x > 0; then recall that e =
k! and so, since x > 0; e
3! :
k=0
6
2 2
nx
n3 x3 and so n x e
2 2
nx
lim n x e
= 0 for x 2
n!1

6n x
6
Hence, e
n3 x3 = nx ! 0 as n ! 1:
Therefore,
R; x 0:

2n
289. Show that lim (cos x) exists for all x 2 R. What is its limit?
n!1

2
2n
Proof. If x 2
= Z; then 1 < cos x < 1 and so, 0 (cos x) < 1 =) lim (cos x) = 0:
nx

n!1

If x 2 Z; then cos x =

2n

1 and so, (cos x) = 1 =) (cos x)

2n

= 1 for all n 2 N =) lim (cos x)


n!1

= 1:

0 if x 2
=Z
2n
Hence, lim (cos x) =
:
1 if x 2 Z
n!1
n

o1
x
290. Show that if a > 0; then the convergence of the sequence x+n
is uniform on [0; a]; but not
n=1

uniform on [0; 1):

x
Proof. Let fn (x) = x+n
for n = 1; 2; 3; ::: and x 0:
Then jfn (x)j < 1 for all n and for all x 0:
We showed in an exercise above that lim fn (x) = 0 for all x 0:
Also, kfn (x)

0k[0;a] = sup

x2[0;a]

x
x+n

n!1
a
a+n

! 0 as n ! 1: (Note that if 0

x < y; then

x
x+n

y
y+n ):

Therefore, the convergence of ffn (x)gn=1 is uniform on [0; a]:


n
To show that the convergence is not uniform on [0; 1); let 0 = 41 and note that fn (n) = n+n
= 12 > 0 :
n o1

nx
291. Show that if a > 0; then the convergence of the sequence 1+n
is uniform on [a; 1); but not
2 x2
n=1

uniform on [0; 1):

nx
Proof. Let fn (x) = 1+n
0: Let > 0 be given.
2 x2 for n = 1; 2; 3; ::: and x
106

1
Then 9n 2 N : n > a1 and so an
< :
1
1
Hence, for all x a; we have that jfn (x)j nx
an < :
1
Therefore, the convergence of ffn (x)gn=1 is uniform on [a; 1):

To show that the convergence is not uniform on [0; 1); let 0 = 14 and note that fn ( n1 ) = 12 > 0 :
n o1

xn
292. Show that if 0 < b < 1; then the convergence of the sequence 1+x
is uniform on [0; b]; but not
n
n=1

uniform on [0; 1]:

xn
Proof. Let fn (x) = 1+x
n for n = 1; 2; 3; ::: and x 2 [0; b]:
Then jfn (x)j 1 for all n and for all x 2 [0; b]:
We showed in an exercise above that lim fn (x) = 0 for all x 2 [0; b]:
Also, kfn (x)

0k[0;b] = sup

x2[0;b]

xn
1+xn

n!1
bn
= 1+b
n

! 0 as n ! 1:

Therefore, the convergence of ffn (x)gn=1 is uniform on [0; b]:


1
To show that the convergence is not uniform on [0; 1]; let 0 = 41 and note that fn ( 21=n
) = 13 > 0 :
n

o1
sin(nx)
293. Show that if a > 0; then the convergence of the sequence 1+nx
is uniform on [a; 1); but not
n=1

uniform on [0; 1):

Proof. Let fn (x) = sin(nx)


1+nx for n = 1; 2; 3; ::: and x 2 [a; 1): Let > 0 be given. Then there is an N 2 N
1
such that N a < : Hence, for all n N; we have that
jfn (x)j =

sin(nx)
1+nx

1
1+nx

1
nx

Therefore, the convergence of

1
na

1
N a < for all x 2 [a; 1):
1
ffn (x)gn=1 is uniform on [a; 1):

To show that the convergence is not uniform on [0; 1]; let 0 = 41 and note that fn ( 2n ) = 2+2 > 0 :

294. Show that the sequence x2 e nx converges uniformly on [0; 1):

Proof. Let fn (x) = x2 e nx for n = 1; 2; 3; ::: and x 2 [0; 1):


We showed in an exercise above that lim fn (x) = 0 for all x 2 [0; 1):
n!1

To calculate the uniform norm of fn (x)

0 = fn (x) on [0; 1); we nd the derivative and solve

fn0 (x) = e

nx

(2x

nx2 ) = 0

to obtain the points yn = 0 and xn = n2 :


Using the rst derivative test, we get that fn (x) attains its maximum on [0; 1) at xn =
Therefore, we obtain
kfn k[0;1) = fn (xn ) = fn ( n2 ) =

4
2
n2 e

2
n:

! 0 as n ! 1:

Thus we see that convergence is uniform on [0; 1):

1
295. Show that if a > 0; then the sequence n2 x2 e nx n=1 converges uniformly on [a; 1); but it does not
converge uniformly on [0; 1):

Proof. Let fn (x) = n2 x2 e nx for n = 1; 2; 3; ::: and x 2 [a; 1):


We showed in an exercise above that lim fn (x) = 0 for all x 2 [0; 1):
n!1

Now, we can nd N 2 N large enough so that N2 < a and so n2 < a for all n N:
Let n N: To calculate the uniform norm of fn (x) 0 = fn (x) on [a; 1); we nd the derivative and solve
107

fn0 (x) = e

nx

(2xn2

n3 x2 ) = 0

to obtain the points yn = 0 and xn = n2 :


A simple calculation shows that fn0 (x) < 0 for all x > n2 :
Thus, for n N; the function fn (x) is decreasing on the interval [a; 1):
Therefore, we obtain
kfn k[a;1) = fn (a) = n2 a2 e

na

! 0 as n ! 1:

Thus we see that convergence is uniform on [a; 1):


To show that the convergence is not uniform on [0; 1); let 0 = 41 and note that fn ( n1 ) = 1e > 0 :

296. Show that if (fn ) and (gn ) converge uniformly on the set A to f and g; respectively, then (fn + gn )
converges uniformly on A to f + g:

Proof. Let > 0 be given. Since fn


f and gn
g; we can nd N 2 N :
A

jfn (x)

f (x)j <

and jgn (x)

g(x)j <

for all n

N and for all x 2 A:

Hence, for all n N and for all x 2 A; we have that


j(fn (x) + gn (x)) (f (x) + g(x))j = j(fn (x) f (x)) + (gn (x) g(x))j jfn (x) f (x)j + jgn (x) g(x)j
<2+2= :
Hence, fn + gn
f + g:
A

297. Show that if fn (x) = x + n1 and f (x) = x for x 2 R; then (fn ) converges uniformly on R to f; but the
sequence fn2 does not converge uniformly on R. (Thus, the product of uniformly convergent sequences
of functions may not converge uniformly).

Proof. Let > 0 be given. Then 9N 2 N : N1 < : Hence, for all n N; we have that
jfn (x) f (x)j = x + n1 x = n1 < for all x 2 R:
Thus, fn
f:
R

Now, for each x 2 R; fn2 (x) = x + n1 = x2 + 2 nx + n12 ! x2 as n ! 1:


But fn2 (n) n2 = 2 + n12
2 =) fn2 does not converge uniformly on R.

298. Let (fn ) and (gn ) be sequences of bounded functions on A that converge uniformly on A to f and g;
respectively. Show that (fn gn ) converges uniformly on A to f g:

Proof. We rst show that there is a uniform bound for all of the fn s.
Since fn
f; we can nd N 2 N : jfn (x) f (x)j < 1 for all x 2 A and all n N:
A

Thus, in particular, jf (x)j < 1 + jfN (x)j for all x 2 A:


Hence, jfn (x)j < 1 + jf (x)j < 2 + jfN (x)j for all x 2 A and all n N:
Let M = 2 + jfN (x)j : Then jf (x)j M and jfn (x)j M for all x 2 A and all n
Similarly, we can nd L > 0 such that jg(x)j

L and jgn (x)j

( )

L for all x 2 A and all n

:
N

Now, we go back to the exercise. We want to show that (fn gn ) converges uniformly on A to f g:
Let > 0 be given. Then there exists N 2 N : for all n N; we have that
( ) and ( ) are satised.
jfn (x) f (x)j < 2L and jgn (x)

g(x)j <

2M :

108

Thus, jfn (x)gn (x) f (x)g(x)j = jfn (x)gn (x) fn (x)g(x) + fn (x)g(x)
= jfn (x) [gn (x) g(x)] + g(x) [fn (x) f (x)]j
jfn (x)j jgn (x) g(x)j + jg(x)j jfn (x) f (x)j
< M 2M + L 2L = ; for all x 2 A and all n N:

f (x)g(x)j

Therefore, fn gn
f g:
A

299. Let (fn ) be a sequence of functions that converges uniformly to f on A and that satises jfn (x)j M
for all n 2 N and all x 2 A: If g is continuous on the interval [ M; M ]; show that the sequence (g fn )
converges uniformly to g f on A:

Proof. We rst show that jf (x)j M for all x 2 A:


Let > 0 be given. Since fn
f; we can nd N 2 N : jfn (x) f (x)j < for all x 2 A and all n N:
A

Hence, jf (x)j
+ jfn (x)j
+ M:
Since our choice of > 0 was arbitrary, we conclude that jf (x)j

M for all x 2 A:

Now, we go back to the exercise. Suppose that g is continuous on the interval [ M; M ]:


Then g is uniformly continuous on the interval [ M; M ]:
Let > 0 be given. There exists > 0 : whenever x; y 2 [ M; M ] with jx yj < ; we have that
jg(x) g(y)j < :
For this ; since fn
f; we can nd N 2 N : jfn (x) f (x)j < for all x 2 A and all n N:
A

Thus, for all x 2 A and all n N; we have that


j(g fn ) (x) (g f ) (x)j = jg (fn (x)) g(f (x))j < since fn (x) and f (x) are in [ M; M ] for all x 2 A
and jfn (x) f (x)j < for all x 2 A and all n N:
Therefore, (g fn )
(g f ) :
A

xn
does not converge uniformly on [0; 2] by showing that the limit
300. Show that the sequence 1+x
n
function is not continuous on [0; 2]:

xn
xn
Proof. If x = 0; then 1+x
n = 0 for all n 2 N and so lim 1+xn = 0:
If 0 < x < 1; then
If x = 1; then
If 1 < x

n
lim x n
n!1 1+x

xn
1+xn

1
2

1+ lim xn

n!1

n!1

0
1+0

= 0:

xn
n
n!1 1+x
n
x
1
1+xn

for all n 2 N and so lim

xn
n
1+x
n!1

2; then lim

lim xn

n!1

= 1. (note that

Hence, the sequence of continuous functions

x
1+xn

= 12 :
=

1
1+xn

! 0 as n ! 1):

converges to the function f dened by:

8
9
< 0 if 0 x < 1 =
1=2
if x = 1
f (x) =
:
:
;
1 if 1 < x 2

Since f is not continuous on [0; 2]; the convergence cannot be uniform on [0; 2]:

301. Construct a sequence of functions on [0; 1] each of which is discontinuous at each point of [0; 1]
and which converges uniformly to a function that is continuous at every point.

Solution. For n = 1; 2; :::; dene fn : [0; 1] ! R as follows:


fn (x) =

1
n

if x 2 Q \ [0; 1]
if x 2 [0; 1]nQ

109

Note that each fn is discontinuous at each point of [0; 1].


Also, jfn (x) 0j n1 for all n 2 N and all x 2 [0; 1]:
Thus, the sequence (fn ) converges uniformly to the zero function on [0; 1] which is continuous
everywhere on [0; 1]:

302. Suppose (fn ) is a sequence of continuous functions on an interval I that converges uniformly on I to
a function f: If (xn ) I converges to x0 2 I; show that lim (fn (xn )) = f (x0 ) :
n!1

Proof. Let > 0 be given. Since each fn is continuous and fn


f; we conclude that f is continuous on I:
I

Thus, we can nd N 2 N such that:


(i) jfn (x) f (x)j <
(This is because fn

for all x 2 I and all n


f ):

N: In particular, jfn (xn )

f (xn )j <

for all n

N:

(ii) jf (xn )

f (x0 )j <

for all n

N: (This is by continuity of f ):

Hence, for all n N; jfn (xn ) f (x0 )j = jfn (xn ) f (xn ) + f (xn ) f (x0 )j jfn (xn ) f (xn )j+jf (xn ) f (x0 )j
<2+2= :
This is exactly what it means for lim (fn (xn )) to be f (x0 ) :
n!1

303. Let f : R ! R be uniformly continuous on R and let fn (x) = f (x + n1 ) for x 2 R: Show that (fn )
converges uniformly on R to f:

Proof. Let > 0 be given. Then 9 > 0 : whenever x; y 2 R with jx yj < ; we have jf (x) f (y)j < :
1
Let N 2 N be large enough so that N1 < : Then n1
for all n N:
N <
1
1
Let x 2 R: Then x (x + n ) = n < for all n N:
Hence, f (x) f (x + n1 ) < for all n N:
Thus, since our choice of x 2 R was arbitrary, we have proved that:
8 > 0 9N 2 N : 8x 2 R; jf (x)

fn (x)j = f (x)

f (x + n1 ) <

for all n

N:

This is exactly what it means for fn (x) to converge to f (x) uniformly on R.

1
304. Let fn (x) = (1+x)
n for x 2 [0; 1]: Find the pointwise limit f of the sequence (fn ) on [0; 1]:
Does (fn ) converge uniformly to f on [0; 1]?

1
Proof. For x = 0; fn (x) = (1+0)
n = 1 for all n and so lim fn (0) = 1:
n!1

1
If 0 < x < 1; then 1 + x > 1 =) (1 + x) ! 1 as n ! 1 =) (1+x)
n ! 0 as n ! 1:
1
If x = 1; then fn (x) = 2n ! 0 as n ! 1:
1
if x = 0
Thus, the pointwise limit f (x) =
:
0 if 0 < x 1
Since each fn is continuous on [0; 1]; but f (x) is not, we conclude that the convergence cannot be
uniform.

305. Suppose that the sequence (fn ) converges uniformly to f on the set A; and suppose that each fn
is bounded on A: (That is, for each n there is a constant Mn such that jfn (x)j Mn for all x 2 A):
Show that the function f is bounded on A:

Proof. Since (fn ) converges uniformly to f on the set A; 9N 2 N :

jfn (x)

f (x)j < 1 for all x 2 A and all n

110

In particular, jfN (x) f (x)j < 1 for all x 2 A and all n N:


Hence, jf (x)j < 1 + jfN (x)j 1 + MN for all x 2 A:
Therefore, f is bounded on A:

nx
306. Let fn (x) = 1+nx
2 for x 2 A = [0; 1): Show that each fn is bounded on A; but the pointwise limit f
of the sequence is not bounded on A: Does (fn ) converge uniformly to f on A?

nx
Proof. Note that for all x 0; 0 fn (x) = 1+nx
n: Hence, each fn is bounded on A:
2
0
For x = 0; fn (x) = 1+0 = 0 for all n and so, lim fn (0) = 0:
n!1

nx
x
1
For x > 0; fn (x) = 1+nx
2 ! x2 = x as n ! 1:
Hence, the pointwise limit of (fn ) is the function f dened by:

f (x) =

0
1
x

if x = 0
if x > 0

This function f is clearly not bounded on [0; 1):


The convergence cannot be uniform because by the previous exercise, if each fn is bounded, then the
pointwise limit, if it is a uniform limit, must also be bounded.

n
307. Let fn (x) = xn for x 2 [0; 1]: Show that the sequence (fn ) of dierentiable functions converges uniformly
to a dierentiable function f on [0; 1]; and that the sequence (fn0 ) converges on [0; 1] to a function g; but
that g(1) 6= f 0 (1):

n
1
Proof. Let > 0 be given. Then 9N 2 N : N1 < and so jfn (x) 0j = xn
N and
n < for all n
all x 2 [0; 1]: Let f (x) = 0 for all x 2 [0; 1]: We have shown that fn
f:
[0;1]

Now, fn0 (x) = xn

for all x 2 [0; 1] and as we saw before, fn0 ! g on [0; 1]; where
g(x) =

0 if 0 x < 1
1
if x = 1

Note that f 0 (1) = 0 6= 1 = g(1):

308. Let gn (x) = e nx =n for x 0; n 2 N: Examine the relation between lim (gn ) and lim (gn0 ) :
n!1
n!1

Proof. We have gn (x) = e nx =n n1 for all x 0 and all n 2 N:


Hence, lim (gn ) = 0 and the convergence is uniform on [0; 1):
n!1

1 if x = 0
:
0 if x > 0
Since each gn0 (x) is continuous on [0; 1) and h(x) is not continuous on [0; 1), we conclude that
the convergence cannot be uniform.
R
2
2
309. Show that lim 1 e nx dx = 0:
n!1

2
Proof. Let fn (x) = e nx for x 2 [1; 2] and n = 1; 2; 3; ::::
2
Then for all x 2 [1; 2]; jfn (x) 0j = e nx
e n < n1 for all n 2 N:
Hence, fn
0: Thus, we can interchange the limit with the integral sign.

Now, gn0 (x) =

nx

! h(x) =

[1;2]

R 2 nx2
R2
R2
2
That is, lim
e
dx = 1 lim e nx dx = 1 (0) dx = 0:
1
n!1
n!1

nx
for x 2 [0; 1]: Show that (fn ) converges nonuniformly to an integrable function
310. Let fn (x) = 1+nx
R1
R1
f and that 0 f (x)dx = lim 0 fn (x)dx:
n!1

111

0
if x = 0
:
1 if 0 < x 1
Note that each fn is continuous but f is not continuous, and so the convergence
R 1 cannot be
R 1 uniform.
Since f is continuous except at x = 1; we conclude that f is integrable and 0 f (x)dx = 0 1dx = 1:
R1
R 1 nx
R1
R1
R1 1
1
1
1
Now, 0 fn (x)dx = 0 1+nx
dx = 0 nx+1
dx = 1
1+nx dx = 0 1
1+nx dx = 1
n ln j1 + nxj
0 1+nx

Proof. It is easy to check, as we did many times before, that f (x) =

1
0

= 1 ln(1+n)
! 1 as n ! 1:
R 1n
R1
Hence, 0 f (x)dx = 1 = lim 0 fn (x)dx:
n!1
R
1
311. Let gn (x) = nx(1 x)n for x 2 [0; 1]; n 2 N: Discuss the convergence of (gn ) and 0 gn (x)dx :

Proof. It is clear that gn (x) converges pointwise to the function g(x) = 0 on [0; 1]:
This convergence is not uniform because gn ( n1 ) = n n1 (1 n1 )n = (1 n1 )n ! e 1 as n ! 1:
1
By taking the rst derivative, it can be easily checked that gn (x) attains its maximum at the point xn = 1+n
:
1
n
1
n
Hence, kgn (x)k[0;1] = gn ( 1+n ) = 1+n (1 1+n )
1 1 = 1 for all n 2 N:
Note that each gn 2 R[0; 1] and g 2 R[0; 1]: Also, we showed that jgn (x)j 1 for all x 2 [0; 1]; n 2 N:
R1
R1
Thus, by the bounded convergence theorem, we conclude that lim
gn (x)dx = 0 g(x)dx = 0:
0
n!1

312. Let fr1 ; r2 ;


; rn ; g be an enumeration of the rational numbers in [0; 1]; and let
fn (x) =

1 if x 2 fr1 ; r2 ;
; rn g
0
otherwise

Show that fn 2 R[0; 1] for each n 2 N; that f1 (x)

f2 (x)

fn (x)

:
; and that f (x) = lim (fn (x))
n!1

is the Dirichlet function which is not Riemann integrable on [0; 1]:

Proof. Each fn is continuous on [0; 1] except on a nite set fr1 ; r2 ;


; rn g : Hence, each fn 2 R[0; 1]:
By denition, fn (x) = fn+1 (x) for all x 2 [0; 1]nfrn+1 g and fn (rn+1 ) = 0 but fn+1 (rn+1 ) = 1:
Hence, fn (x) fn+1 (x) for all x 2 [0; 1] and for all n 2 N: That is, f1 (x) f2 (x)
fn (x)
:
Note that if x 2 [0; 1]nQ then fn (x) = 0 for all n 2 N and so lim (fn (x)) = 0:
n!1

Also, if x 2 [0; 1] \ Q; then x = rN for some N 2 N and so fn (x) = fn (rN ) = 1 for all n
Hence, lim (fn (x)) = 1:

N:

n!1

Thus, the pointwise limit of (fn ) is the function f dened by:


f (x) =

1 if x 2 Q \ [0; 1]
0 if x 2 [0; 1]nQ

This is precisely the Dirichlet function.

313. Show that if a convergent series contains only a nite number of negative terms, then it is absolutely
convergent.

1
P
Proof. Let
xn be such a series. Let N 2 N be large enough so that xn 0 for all n N:
Since
Say,

1
P

n=1

n=1
1
P

n=N
1
P

xn < 1; we get that

1
P

xn < 1 and so

n=N
NP1

xn = a < 1 and let b =

n=0
1
P

jxn j :

1
P

n=N

jxn j =

1
P

xn is absolutely convergent.

n=N

Then
jxn j = b + a < 1 and so
xn is absolutely convergent.
n=1
n=1

314. Show that if a series is conditionally convergent, then the series obtained from its positive terms is
divergent and the series obtained from its negative terms is divergent.

112

Proof. Let

1
P

xn be such a series. For n = 1; 2; 3; :::; dene the sequences (an ) and (bn ) as follows:

n=1

xn
0

an =
Then

1
P

an is the series of positive terms of

n=1

1
P

Suppose to the contrary that


Then, since
Since

1
P

n=1

1
P

n=1

jan j =

n=1

1
P

an < 1:

1
P

n=1

an < 1 and
1
P

1
P

n=1

jxn j =

1
P

1
P

xn

n=1

jbn j =
1
P

n=1

1
P

xn and

if xn 0
if xn < 0

bn is the series of negative terms of

n=1

1
P

1
P

an =

n=1

1
P

0
xn

and bn =

n=1

xn < 1; we get that

n=1

Hence,

if xn > 0
if xn 0

bn =

(xn

1
P

n=1

an ) =

1
P

n=1

bn < 1:

bn < 1; we conclude that

n=1

(jan j + jbn j) =

xn :

n=1

n=1
1
P

n=1

1
P

jan j +

1
P

n=1

jbn j < 1:

xn is absolutely convergent. A contradiction.

n=1

1
1
P
P
Therefore,
an = 1: Similarly,
bn = 1:
n=1
n=1

1
P
315. If
xn is conditionally convergent, give an argument to show that there exists a rearrangement
n=1

whose partial sums diverge to 1:

1
1
1
P
P
P
Proof. By the previous problem, the series
an and
bn of positive and negative terms of
xn are
both divergent. That is,

1
P

an = +1 and

n=1

n=1
1
P

n=1

bn =

n=1

n=1

( )

Consider the rearrangement dened as follows: (we can do this because of ( ))


1
1
P
P
Take poisitve terms from
an until the partial sum exceeds 1, then take negative terms from
bn
n=1

n=1

until the partial sum is less than 1, then take positive terms until the partial sum exceeds 2, then negative
terms untill the partial sum is less than 2, ... and so on.

1
P
316. Find an explicit expression for the n-th partial sum of
ln 1 n12 and show that this series
n=2

converges to ln 2: Is this convergence absolute?

n
n
n
n
P
Q
Q
Q
(k 1)(k+1)
k2 1
= ln
=
ln
Solution. sn =
ln 1 k12 = ln
1 k12
2
k
k2
k=2
(n 1)!(n+1)!
=
2(n!)2

k=2

ln

n+1
2n

Thus, lim sn = lim ln

n+1
2n

= ln

= ln

n!1

n!1

k=2

k=2

lim n+1 = ln(1=2) = ln 2: (The second equality


n!1 2n
n+1
is a sequence that converges to 1=2).
2n
all n 2 and so ln 1 n12 < 0 for all n 2:

follows because

ln x is continuous at x = 1=2 and


Now, note that 0 < 1 n12 < 1 for
Since all the terms of the series are negative, the convergence is indeed absolute.

1
1
P
P
317. (a) If
an is absolutely convergent and (bn ) is a bounded sequence, show that
an bn is absolutely
convergent.

n=1

n=1

(b) Give an example to show that if the convergence of

1
P

n=1

113

an is conditional and (bn ) is a bounded

1
P
sequence, then
an bn may diverge.
n=1

Proof. (a) Suppose that jbn j M for all n 2 N: Then jan bn j M jan j for all n 2 N; and so
1
1
1
1
P
P
P
P
jan bn j
M jan j = M
jan j < 1 since
an is absolutely convergent.
n=1

Hence,

n=1

1
P

n=1

n=1

an bn is absolutely convergent.

n=1
n

(b) Let an = ( n1) and bn = ( 1) for n = 1; 2; 3; ::::


1
P
Then
an is conditionally convergent and (bn ) is a bounded sequence.
n=1
1
P

1
1
P
P
n
( 1)n
1
Also,
an bn =
(
1)
=
n
n = 1:
n=1
n=1
n=1

1
1
P
P
318. Give an example of a convergent series
an such that
a2n is not convergent.
n=1
n=1

1
1
1
n
P
P
P
1
Solution. Let an = ( p1)n for n 2 N: Then
an is convergent, but
a2n =
n = 1:
n=1
n=1
n=1

1
P
319. Give an example of a divergent series
an with (an ) decreasing and such that lim (nan ) = 0:
n!1
n=1

1
P
1
Solution. Let an = n ln
an = 1, and (an ) is decreasing.
n for n 2 N: Then
n=1

Also, lim (nan ) = lim ln1n = 0:


n!1
n!1

1
P
320. If (an ) is a sequence and if lim n2 an exists in R; show that
an is absolutely convergent.
n!1
n=1

Proof. Since lim n2 an exists in R, the sequence n2 an must be bounded, say by M .


n!1

That is, n2 an
1
P
Hence,
jan j
n=1

1
P

M for all n 2 N and so jan j


1
P
M
n2 < 1:

M
n2

for all n 2 N:

n=1

Therefore,
an is absolutely convergent.
n=1

1
1
P
P
321. If (ank ) is a subsequence of (an ), then the series
ank is called a subseries of
an : Show that
n=1

1
P

n=1

an is absolutely convergent if and only if every subseries of it is convergent.

1
P
Proof. (=)) Suppose that
an is absolutely convergent, and let (ank ) be a subsequence of (an ) :
n=1

We want to show that

1
P

n=1

ank is convergent.

n=1

For n 2 N; dene

bn =
Note that
Hence,

1
P

1
P

n=1

n=1

ank =

1
P

n=1

bn . Also, jbn j

jank j < 1 and so

1
P

n=1

ank
0

if n = nk
otherwise

jan j for all n 2 N and so

ank < 1:

((=) Suppose that every subseries of

1
P

:
1
P

n=1

jbn j

1
P

n=1

jan j < 1:

an is convergent. Then, in particular, the subseries of positive

n=1

114

terms and the subseries of negative terms are convergent and hence are absolutely convergent as we did
1
P
in a previous problem. Hence,
an is absolutely convergent.
n=1

1
P
1
322. Let a > 0: Show that the series
1 and is convergent if a > 1:
1+an is divergent if 0 < a
n=1

1
P
1
1
1
Proof. If 0 < a < 1; then lim an = 0 and so lim 1+a
n = 1+0 = 1: Thus,
1+an is divergent.
If a = 1; then

1
P

n=1

If a > 1; then

1
a

1
1+an

n!1
1
P
1
2
n=1
1
P

n!1

= 1:

< 1 and so

n=1

1
1+an

1
P

<

n=1

1
an

n=1

1
P

1 n
a

n=1

1=a
1 (1=a)

1
a 1

< 1:

1
P
1
Hence, the series
1 and is convergent if a > 1:
1+an is divergent if 0 < a
n=1

323. Establish the convergence or divergence of the series whose nth term is:
1=2
(a) (n (n + 1))
;
1=2
(b) n2 (n + 1)
;
(c) n!=nn ;
n
(d) ( 1) n= (n + 1) :

= 1:
: Let bn = n1 for n 2 N: Then lim abnn = lim p n
Solution. (a) an = p 1
n(n+1)

n!1

Thus, by the Limit Comparison Test and since

1
P

n!1

bn is divergent, we get that

n=1

(b) an = p

1
:
n2 (n+1)

Let bn =

1
n3=2

n=1

an
n!1 bn

for n 2 N: Then lim

Thus, by the Limit Comparison Test and since

1
P

n(n+1)
1
P

3=2

= lim p n2

n (n+1)

n!1

= lim

(c) We have
Hence, lim

n!1

an+1
an
an+1
an

(n+1)!nn
n!(n+1)n+1

= lim

1
n+1

n!1

n
n+1
n

= 1

=e

1
n(n+1)

n!1

is divergent.

n3
n2 (n+1)
1
P

bn is absolutely convergent, we get that

n=1

is absolutely convergent.

n=1

1
n+1

= 1:
p

1
n2 (n+1)

< 1:

an+1
n!1 an

Thus, by the Ratio Test and since lim

< 1; we get that

1
P

n=1

n!
nn

is absolutely convergent.

1
P
1)n n
( 1)n n
(d) Note that lim ( n+1
does not exist and so
n+1 is divergent.
n!1
n=1

1
P
p
324. If a; b > 0; show that
(an + b) converges if p > 1 and diverges if p 1:
n=1

1
P
p
p
Proof. If p < 0; then lim (an + b) = 1 and so
(an + b) diverges.
n!1

If p = 0; then

1
P

(an + b)

n=1

n=1

If 0 < p
1
P
Since

n=1

1; let cn =
1
np

1
np :

Then

= 1 for 0 < p

1
P

1 = 1 and so

n=1
(an+b)
cn

1
P

(an + b)

diverges.

n=1

np
(an+b)p

1
ap

as n ! 1:
1
P
1; we get by the comparison test that
(an + b)

diverges.

n=1

n
1
If p > 1; let cn = n1p : Then (an+b)
= (an+b)
p !
cn
ap as n ! 1:
1
1
P
P
p
1
Since
<
1
for
p
>
1;
we
get
by
the
comparison
test
that
(an + b) converges.
p
n
n=1
n=1

325. Discuss the series whose nth term is:

115

(a)
(b)
(c)

n!
3 5 7 (2n+1) ;
(n!)2
(2n)! ;
2 4 (2n)
3 5 (2n+1) ;
2 4 (2n)
5 7 (2n+3) :

(d)

5 7 (2n+1))
n+1
1
= (3(n+1)!(3
Solution. (a) aan+1
5 7 (2n+1)(2n+3))n! = 2n+3 ! 2 < 1 as n ! 1: Hence, by the Ratio Test, we
n
1
P
get that the series
an converges.
(b)

an+1
an

n=1
((n+1)!)2 (2n)!
(2n+2)!(n!)2
1
P

(n+1)2
(2n+1)(2n+2)

n2 +2n+1
4n2 +6n+2

1
4

< 1 as n ! 1: Hence, by the Ratio Test, we

get that the series


(c)

an+1
an

[2 4
[3 5

an converges.
n=1
(2n)(2n+2)][3 5 (2n+1)]
(2n+1)(2n+3)][2 4 (2n)]

2n+2
2n+3 !
2n+2
2n+3 = 1

1 as n ! 1: Hence, the Ratio Test is not applicable.

1
So, we try the Raabes Test. Note that
1 n1 for all n 2 N:
2n+3
1
1
P
P
Hence,
an does not converge absolutely and since the terms of
an are all positive, we conclude
n=1

that

1
P

n=1

an does not converge.

n=1

(d) It can be checked that in this case,

an+1
an

2n+2
2n+5
1
P

=1

3
2n+5

a
n

for any 1 < a < 3=2 and

for all n 2 N: Let a = 4=5: Then by Raabes test,


an is convergent.
n=1

is divergent.
326. Show that the series 1 + 12 13 + 14 + 15 16 + +

1
9n2 2
Proof. Note that the sum 3n1 2 + 3n1 1 3n
= (3n 2)(3n
1)(3n) :
Let (sn ) be the sequence of partial sums of our series.
n
n
1
P
P
P
1
1
1
9k2 2
9k2 2
Then s3n =
3k 2 + 3k 1
3k =
(3k 2)(3k 1)(3k) and so lim s3n =
(3k 2)(3k 1)(3k) :
k=1

But the series whose nth term is


Hence, lim s3n = 1:

k=1
9n2 2
(3n 2)(3n 1)(3n)

n!1

k=1

is divergent. (compare it with

1
n ):

n!1

Since (s3n ) is a divergent subsequence of (sn ) ; we conclude that (sn ) is divergent.


Hence, the series 1 + 21 13 + 14 + 15 16 + +
is divergent.

327. For n 2 N; let cn = 11 + 12 +


+ n1 ln n: Show that (cn ) is a decreasing sequence of positive
numbers. The limit C of this sequence is called Eulers Constant and is approximately equal to 0:577.
Show that if we put
bn =

1
1

1
2

1
3

1
2n

then the sequence (bn ) converges to ln 2: [Hint: bn = c2n cn + ln 2:]

R n 1
+ n 1 is the Riemann sum of the function f (x) = x1
Proof. Note that ln n = 1 x1 dx and that 11 + 12 +
with respect to the tagged partition whose intervals are of the form [k 1; k] for 2 k n and whose tags
are the left endpoints of the intervals.
Since f (x) = x1 is decreasing on [1; 1); we get that
Rn
ln n = 1 x1 dx 11 + 12 +
+ n 1 1 < 11 + 12 +
+ n 1 1 + n1 :
Hence, cn =

1
1

1
2

1
n

ln n > 0 for all n 2 N

(1)

To show that the sequence (cn ) is decreasing, note that


116

R n+1
n

1
x dx

>

1
n+1

for all n 2 N:

1
1
Hence, for all n 2 N; ln(n + 1) ln(n) > n+1
=) ln n > n+1
ln(n + 1)
1
1
1
1
1
1
1
=) cn = 1 + 2 +
+ n ln n > 1 + 2 +
+ n + n+1 ln(n + 1) = cn+1 :

Thus, the sequence (cn ) is strictly decreasing

(2)

By (1) and (2), we get that the sequence (cn ) is convergent indeed.
1
Let bn = 11 12 + 13
cn + ln 2 for all n 2 N; then we would
2n : If we show that bn = c2n
get that lim bn = lim (c2n cn + ln 2) = lim c2n
lim cn + ln 2 = C C + ln 2 = ln 2 and so we will
n!1
n!1
n!1
n!1
be done.
1
1
1
1
+
+ 2n
+ ln (n) ln (2n) + ln 2 = n+1
+
+ 2n
= dn :
Note that c2n cn + ln 2 = n+1
Thus, we need to check that bn = dn for all n 2 N:
We will prove this by induction.
For n = 1; we have b1 = 1 1=2 = 1=2 and d1 = 1=2:
Now, suppose the result is true for n: That is, bn = dn : We need to show that the result holds for n + 1;
1
1
+
+ 2n+2
:
i.e., bn+1 = dn+1 = n+2
1
n+2 +
1
2(n+1) = bn

Note that dn+1 =

1
2n+2
1
2n+1

1
n+1
1
2(n+1)

1
n+2

1
2n+2

1
n+1

= dn +

1
2n+1

1
2n+2

1
n+1

1
= dn + 2n+1
+
= bn+1 :
Hence, by the induction theorem, bn = dn = c2n cn + ln 2 for all n 2 N:

328. Test the following series for convergence and for absolute convergence:
1
1
1
1
P
P
P
P
n+1 ln n
( 1)n+1
( 1)n+1
( 1)n+1 n
(a)
;
(b)
;
(c)
;
(d)
( 1)
2
n +1
n+1
n+2
n :
n=1
n=1
n=1
n=1

1
1
n+1
P
P
( 1)n+1
1
1
Solution. (a) ( n1)
= n21+1
for
all
n
2
N.
Since
<
1;
the
series
2 +1
2
2
n
n
n2 +1 converges
n=1

n=1

absolutely and so it converges.


(b)

( 1)n+1
n+1

1
n+1

and since

1
n+1

= 1 (comparing it with

1
n );

the series does not converge

1
absolutely. Since xn = n+1
is a decreasing sequence the converges to 0, and since the sequence of partial
1
P
n+1
sums of the series
( 1)
, which is f1; 0; 1; 0; 1; 0; :::g; is bounded, the Dirichlets test implies that the

series

1
P

n=1

n=1

( 1)n+1
n+1

converges.

( 1)n+1 n
n+2
n!1

(c) Note that lim

(d) Note that xn =

ln n
n

does not exist and so the series

1
P

n=1

is decreasing for n

( 1)n+1 n
n+2

diverges.

4 and xn ! 0 as n ! 1 by LHospitals Rule.


1
P
n+1
Also, the sequence of partial sums of the series
( 1)
; which is f1; 0; 1; 0; 1; 0; :::g; is bounded.
n=1
1
P

Hence, the Dirichlets test implies that the series


Now,

R1
1

n+1 ln n
n

( 1)

n=1

ln x
x dx

1
2

converges.

(ln x) ]1
1 = 1 and so, by the integral test, we get that the series

1
P

n=1

n+1 ln n
n

( 1)

does

not converge absolutely.

329. Give an example to show that the Alternating Series Test may fail if (an ) is not a decreasing sequence.

Solution. For n 2 N; dene the sequence (an ) as follows:


an =

1=n2
1=n

if n is odd
if n is even

117

Then, clearly, (an ) is a sequence of strictly positive numbers and (an ) converges to 0.
1
P
n
Also,
( 1) an = 1 + 1=2 1=9 + 1=4 1=25 + 1=6 1=49 + 1=8
is divergent because if
n=1

it were conditionally convergent, then the series obtained from its negative terms must be divergent.
It is clear, though, that the series of negative terms is absolutely convergent.
1
P
n
Hence,
( 1) an is divergent.
n=1

Note that all of this happened because the sequence (an ) was not decreasing to 0.

330. Consider the series


1

1
2

1
3

1
4

1
5

1
6

1
7

++

where the signs come in pair. Does it converge?

Proof. Let (xn ) = n1 and (yn ) = f1; 1; 1; 1; 1; 1; 1; 1; 1; 1; 1; g:


1
P
Then the sequence (xn ) is decreasing to 0 and the sequence of partial sums of the series
yn is given
n=1

by f1; 0; 1; 0; 1; 0; 1; 0; 1; 0; 1; 0; g is bounded by 1. Hence, by the Dirichlets Test, we deduce that


the series 1 12 13 + 14 + 15 16 17 + +
converges.

1
1
P
P
an
an
331. Let an 2 R for n 2 N and let p < q: If the series
is
convergent,
show
that
the
series
p
n
nq is
n=1

n=1

also convergent.

1
1
1
1
P
P
P
P
an
an
1 an
Proof. Note that
=
=
=
xn yn ; where xn = nq1 p and yn = annp for n 2 N:
q
q
p+p
q
p
p
n
n
n
n
n=1

n=1

n=1

n=1

Note that, since q

p > 0; the sequence (xn ) is monotonically decreasing and xn ! 0 as n ! 1:


1
1
P
P
an
Moreover, the series
yn =
np is given to be convergent.
n=1

n=1

1
1
P
P
an
Hence, by Abels test, we get that the series
xn yn =
nq is convergent.
n=1
n=1

1
p
P
n
is a convergent series.
332. If p and q are positive numbers, show that
( 1) (lnnn)
q
n=2

p
Proof. If we dene f (x) = (lnxx)
;
then we get that f 0 (x) < 0 for x su ciently large. Hence, the function
q
f (x) is decreasing for x su ciently large, say for x M: Let N 2 N be such that N M:
p
Then we get that the sequence (lnnn)
is decreasing for all n N:
q
(ln n)p
Moreover, lim nq = 0 by applying LHospitals Rule.
n!1
1
p
P
n
Hence, by the Alternating Series Test, we deduce that the series
( 1) (lnnn)
is a convergent series.
q
n=2

333. Discuss the series whose nth term is:


n
n
n
n
nn
nn
(a) ( 1) (n+1)
(b) (n+1)
(c) ( 1) (n+1)
(d) (n+1)
n+1 ;
n+1 ;
nn ;
nn+1 :
n
n
1)n
nn
nn
1
Solution. (a) ( 1) (n+1)
1 n+1
and yn = (n+1
:
n+1 = xn yn ; where xn = (n+1)n =
1
1
n
P
P
( 1)
Note that the sequence (xn ) is monotone and converges to e 1 : Also,
yn =
n+1 is convergent.

Hence, by Abels test, we get that the series

1
P

xn yn =

n=1
nn
(n+1)n+1

1
P

n=1

n=1

( 1)

nn
(n+1)n+1

n=1

is convergent.

n+1

n
for n 2 N: Then abnn = n+1
! e 1 as n ! 1:
1
1
P
P
1
Hence, by the Comarison Test and since
=
1;
we
deduce
that
the
series
an diverges.
n

(b) Let an =

and bn =

1
n

n=1

n=1

118

n (n+1)n
nn

does not exist and so the series

(c) Note that lim ( 1)


n!1

(n+1)n
nn+1

1
P

n (n+1)n
nn

( 1)

n=1
n+1
n+1
!
n

diverges.

for n 2 N: Then abnn =


e as n ! 1:
1
1
P
P
1
Hence, by the Comarison Test and since
=
1;
we
deduce
that
the
series
an diverges.
n+1
n=1
n=1

1
1
P
P
334. If the partial sums of
an are bounded, show that the series
an e nt converges for t > 0:
n=1
n=1

n
for n 2 N: Then the sequence (xn ) is decreasing and
Proof. Fix t > 0; and let xn = e nt = e1t
t
converges to 0. (Note that for t > 0; e < 1 and so e nt ! 0 as n ! 1):
1
P
an e nt converges.
Hence, by the Dirichlets Test, we deduce that
n=1

1
1
1
P
P
P
an
sn
335. If the partial sums sn of
an are bounded, show that the series
n converges to
n(n+1) :
n=1
n=1
n=1

Proof. Recall the Abels Lemma which says that if (an ) and (bn ) are sequences in R and if the sequence
of partial sums of (an ) is denoted by sn ; with s0 = 0; and if m > n; then

(d) Let an =

1
n+1

and bn =

m
P

ak bk = (bm sm

bn+1 sn ) +

k=n+1

In our case, bn =
m
P

1
n:

Note that
Hence,

1
P

b1 s0 ) +

k=1

m
P1

(bk

bk+1 ) sk = bm sm +

ak
k

= lim

sm
m

m
P1
k=1

1
k

1
k+1

sk =

sm
m

0 since (sm ) is a bounded sequence.


m
P

m!1 k=1

ak
k

sm
m!1 m

lim

m
P1

(bk

bk+1 ) sk

k=1

k=1

k=1
lim smm =
m!1
ak
k

bk+1 ) sk

Also, note that

ak bk = (bm sm

m
P

(bk

k=n+1

k=1

In other words,

m
P1

lim

m
P1

m!1 k=1

m
P1
k=1

1
k(k+1) sk :

1
k(k+1) sk

=0+

1
P

k=1

1
k(k+1) sk

1
P

k=1

1
k(k+1) sk :

336. Show that the hypothesis that the sequence (xn ) is decreasing in the Dirichlets Test can be replaced
1
P
by the hypothesis that
jxn xn+1 j is convergent.
n=1

Proof. We recall the Dirichlets Test:


Dirichlets Test: If (xn ) is a decreasing sequence with lim xn = 0; and if the partial sums (sn ) of the series
n!1
1
1
P
P
yn are bounded, then the series
xn yn is convergent.
n=1

n=1

What we need to prove, thus, is the following modied version:


Modied Dirichlets Test: If (xn ) is a sequence such that
if the partial sums (sn ) of the series

1
P

n=1

jxn

xn+1 j is convergent, with lim xn = 0;and

yn are bounded, then the series

n=1

Proof. Let jsn j

1
P

n!1

1
P

xn yn is convergent.

n=1

B for all n 2 N: If m > n; then it follows by Abels Lemma (see the previous problem)
1
P
and the fact that
jxn xn+1 j is convergent, that: (Here, we use Cauchys Criterion for Convergence)
n=1
"
#
m
m
m
P
P1
P1
xk yk
jxm xn+1 j B +
jxk xk+1 j B = B jxm xn+1 j +
jxk xk+1 j < B B = ,
k=n+1

k=n+1

k=n+1

119

for all su ciently large m and n; with m > n: [ > 0 was chosen so that things would work!].
1
P
Hence, by Cauchys Criterion for Convergence of Series, we deduce that
xn yn is convergent.
n=1

1
P
337. Show that if the partial sums sn of the series
ak satisfy jsn j M nr for some r < 1; then the

series

1
P

k=1

k=1

ak
k

converges.

Proof. Recall the Abels Lemma which says that if (an ) and (bn ) are sequences in R and if the sequence
of partial sums of (an ) is denoted by sn ; with s0 = 0; and if m > n; then
m
P

ak bk = (bm sm

k=n+1

In our case, bn =

1
n:

k=n+1

=M

1
r
mm

ak
k

bk+1 ) sk

Thus,

k=n+1
m
P

(bk

k=n+1

m
P

Hence,

m
P1

bn+1 sn ) +

1
m

1
r
n+1 n

jsm j +

1
m sm

1
n+1 sn

m
P1 jsk j
jsn j +
k(k+1)
! k=n+1

1
n+1

m
P1

ak
k

kr
k(k+1)

k=n+1
1
r
mm !

m
P1

k=n+1

1
r
mMm

sk
k(k+1)

1
r
n+1 M n

m
P1

k=n+1

M kr
k(k+1)

1
Note that, since r < 1;
0 as m ! 1; and n+1
nr ! 0 as n ! 1.
1
1
P
P
kr
1
1
Also,
k(k+1) < 1 since we can compare it with k2 r and
k2 r < 1 since 2
k=1

r > 1:

k=1

Hence, as m; n ! 1; we get that

m
P1

k=n+1

kr
k(k+1)

! 0:

Gathering all the information that we got, we deduce that


Cauchys Criterion for Convergence, we get that the series

m
P

ak
k

! 0 as m; n ! 1 and so, by the

k=n+1
1
P
ak
k converges.
k=1

1
1
P
P
338. Suppose that
an is a convergent series of real numbers. Either prove that
bn converges or
n=1

n=1

give a counter-example, where


we dene bn by:
p
p
a
(a) ann ;
(b) n n (an 0)
(c) ann (an 0);
(d) n1=n an :

1
P
Solution. (a) Since the sequence n1 is monotone and converges to 0, and since
an is a convergent
series, Abels Test implies that

1
P

n=1

n=1

an
n

is convergent.

(b) Note that, by Cauchy-Schwartzinequality,


1
P

n=1

an
n

1 p
P

an

1=2

n=1

1
P

n=1

1 2
n

1=2

1
P

n=1

1=2

an

1
P

n=1

1=2
1
n2

This is a product of two convergent series and so it converges. Hence, by the Comparison Test, we deduce
1 p
P
an
that the series
n is convergent.
n=1

(c) Let an =

1
n(ln n)2

(Note that the series

1
P

n=2

1
n(ln n)2

is convergent by the Integral Test).

120

Then

1 p
P
an
n

n=2

1 q
P

n=2

1
n2 (ln n)2

1
P

n=2

1
n ln n

which is divergent by the Integral Test again.

(d) The sequence n1=n is monotonically decreasing for n

1
P

3 with lim n1=n = 1; and since


n!1

an

n=1

1
P
is a convergent series, Abels Test implies that
n1=n an is convergent.
n=1

1
P
339. Discuss the convergence and uniform convergence of the series
fn where fn is given by:
n=1

(a) x + n
(b) (nx)
(x 6= 0)
(c) sin(x=n )
(d) (xn + 1)
(x 0)

1
P
1
1
1
Solution. (a) We have x2 +n
for
all
x
2
R
and
all
n
2
N:
Also,
2
2
n
n2 < 1:

Hence, by the Weierstrass M-Test, we conclude that

1
P

n=1

x2 + n2

n=1

(b) For any x 6= 0;

1
P

n=1

1
n2 x2

1
x2

For a xed a > 0; we claim that

1
P

n=1
1
P

n=1

a =) x2

This is because jxj

1
n2

the Weierstrass M-Test implies that

< 1: Hence, the series converges for all x 6= 0:

1
n2 x2

a2 =)
1
P

converges uniformly on R.

converges uniformly on the set jxj

1
x2

1
a2

(nx)

n=1

=)

n2 x2

n2 a2

; and since

a:
1
P

n=1

1
n2 a2

< 1;

converges uniformly on Rn( a; a).

The series does not converge uniformly on Rnf0g because if we take the sequence (xn ) =
1
1
P
P
fn (xn ) =
1 = 1:
n=1

1
n

; then

n=1

(c) We have sin

x
n2

jxj
n2

for all x 2 R and so

1
P

sin

n=1

x
n2

converges for all x 2 R:

The convergence is not uniform on R because if (xn ) = n2 ; then

1
P

sin

n=1

xn
n2

1
P

n=1

sin (1) = 1:

Yet, on any interval of the form [ a; a] ; with a > 0; we claim that the convergence is uniform.
jxj
a
This is because sin nx2
n2
n2 for all x 2 [ a; a] and the Weierstrass M-Test implies
uniform convergence.
(d) If x = 0; then fn (x) = 1 for all n 2 N and so the series diverges.
1
If 0 < x < 1; then fn (x) = 1+x
n ! 1 as n ! 1 and so the series diverges.
1
If x = 1; then fn (x) = 2 for all n 2 N and so the series diverges.
1
P
1
1 n
1 n
If x > 1; then fn (x) = 1+x
and
< 1: Hence, the series converges.
n
x
x
n=1

The convergence is not uniform on (1; 1) since if we choose (xn ) = n1=n then xn ! 1+ but
1
P
1
1
fn (xn ) = 1+n
and
1+n = 1:
n=1

We claim that for any a > 1; the series converges uniformly on [a; 1):
1
P
1
1
1
1
This is because for all x a; jfn (x)j = 1+x
and
n
n
n
x
a
an < 1:
n=1

Hence, by the Weierstrass M-Test, we conclude that the convergence is uniform on [a; 1):

1
1
P
P
340. If
an is an absolutely convergent series, show that the series
an sin nx is absolutely and
n=1

n=1

uniformly convergent.

1
P
Proof. We have jan sin nxj jan j for all x 2 R and all n 2 N: Also,
jan j < 1:
n=1

Hence, by the Weierstrass M-Test, we conclude that the convergence is uniform on R:


121

Clearly, by the same inequality, we see that the convergence is absolute.

1
P
341. Determine the radius of convergence of the power series
an xn , where an is given by:
n=1

(a) 1=npn ;
(b) n =n!;
(c) nn =n!;
(d) (ln n)
(n 2);
(e) (n!) =(2n)!;
n
:
(f ) n

1=n
Solution. (a) We have lim sup jan j
= lim 1=n = 0; and so R = +1:
1=n
1=n
n =n
(b) We have lim sup jan j
= lim sup n =n = (n!)
= lim (n!)
1=n = 0; and so R = +1:
n

(c) We have lim

an
an+1

n (n+1)!
n
= lim n!(n+1)
1
n+1 = lim (n+1)n = lim

(d) We have lim

an
an+1

= lim ln(n+1)
ln n

(e) We have lim

an
an+1

LHospitals Rule

1
n+1

=e

; and so R = e

1; and so R = 1:

(2n+1)(2n+2)
(n!) (2n+2)!
= lim (2n)!((n+1)!)
= 4; and so R = 4:
2 = lim
(n+1)2
1=n

(f ) We have lim sup jan j


= lim sup n 1= n = lim n 1= n = 1; and so R = 1=1 = 1:

1 if n = k 2
1 if n = k!
342. If an =
and bn =
for k 2 N, nd the radius od convergence
0 otherwise
0 otherwise
1
1
P
P
of the power series
an xn and of
bn xn :
n=1
n=1

1=n
Solution. We have lim sup jan j
= lim supf1; 0; 0; 1; 0; 0; 0; 0; 1; 0; g = 1; and so R = 1=1 = 1:
1=n
Also, lim sup jbn j
= lim supf1; 1; 0; 0; 0; 1; 0; 0; g = 1; and so R = 1=1 = 1:

1
P
343. If 0 < p jan j q for all n 2 N; nd the radius of convergence of
an xn :
n=1

1=n
Solution. We have 0 < p1=n jan j
q 1=n and so, since lim p1=n = lim q 1=n = 1; the Squeeze Theorem
1=n
1=n
implies that lim jan j
= 1: Hence, lim sup jan j
= 1 and so, R = 1=1 = 1:

1
P
an xn for jxj < R: If f (x) = f ( x) for all jxj < R; show that an = 0 for all n odd.
344. Let f (x) =
n=1
(n)

n
Proof. We have an = f n!(0) for all n 2 N: Note that f (n) (x) = ( 1) f (n) ( x) for all n 2 N:
Hence, f (n) (0) = f (n) (0) for all n odd and so f (n) (0) = 0 for all n odd.
Consequently, an = 0 for all n odd.

345. Prove that if f is dened for jxj < r and if there exists a constant B such that f (n) (x)
B for all
jxj < r and n 2 N; then the Taylor series expansion
1
P

n=1

f (n) (0) n
n! x

converges to f (x) for jxj < r:

Proof. We have to show that the sequence (Rn (x)) of remainders converges to 0 for each x with jxj < r:
Recall that
Rn (x) =
for some c between 0(n+1)
and x:
f
Hence, jRn (x)j = (n+1)!(c) xn+1

f (n+1) (c) n+1


(n+1)! x

B
n+1
! 0 as n ! 1:
(n+1)! r
1
(n)
P
f
(0) n
n! x converges to f (x)
n=1

Hence, the Taylor series expansion


for jxj < r:

122

Rx
2
346. Find a series expansion for 0 e t dt for x 2 R:
n
1
1
P
P
2
( t2 )
( 1)n t2n
Solution. For each t 2 R; we have e t =
=
:
n!
n!
Rx

Rx

n=0
1
P

n=0

Rx

1
P
( 1)n x2n+1
Hence, 0 e dt = 0
dt =
t2n dt =
n!
2n+1 :
0
n=0
n=0
n=0
R
347. Let f : R ! R be a Lebesgue integrable function, i.e., R f dm < 1.
(a) Prove that m(fx 2 R : f (x) = 1g) = 0:
(b) Prove that 8 > 0; m(fx 2 R : jf (x)j
g) < 1:
(c) Prove that the Lebesgue integral is absolutely continuous
with respect to the Lebesgue measure, i.e.,
R
if f is Lebesgue integrable on A, then 8 > 0 9 >
R 0 : B jf j dm < whenever B A with m(B) < :
(d) Prove that 8 > 0; 9 a compact set K R : RnK jf j dm < :
R
(e) Prove that 8 > 0; 9 M > 0 and a measurable set A R : jf (x)j M on A; and RnA jf j dm < :
R R
Proof. (a) If m(fx 2 R : f (x) = 1g) = a > 0; then R f dm
f dm = 1:
R fx2R:f (x)=1g
R
(b) Let > 0 be given. If m(fx 2 R : jf (x)j
g) = 1; then R jf j dm
jf j dm
fx2R:jf (x)j g
m(fx 2 R : jf (x)j
g) = 1:
(c) Let > 0 be given. f is Lebesgue integrable on A , jf j is Lebesgue integrable on A.
jf j 0 =) 9 a sequence of simple functions 'n % f on A, 'nR (x) 0 on RA for each n = 1; 2; 3; ::::
Thus, by the Lebesgue monotone convergence theorem, lim A 'n dm = A jf j dm:
n!1
R
Hence, 9N : A (jf j 'n ) dm < 2 8n N:
k
k
X
[
Write 'N =
ai (Ai ); where A =
Ai : Let a = max ai :
1 i k
i=1
i=1
R
Then for
B A with m(B) < :
R = 2a ; weRhave that B j'N j dm < aR m(B) = a = 2 whenever
R
Hence, B jf j dm = B (jf j 'N + 'N ) dm = B (jf j 'N ) dm + B 'N dm < 2 + 2 = :
1
X
R
R
(d) Let > 0 be given and for n 2 Z; set An = [n; n + 1]. Then R jf j dm =
jf j dm < 1:
An

Thus, 9N :

t2

N
X
R

n= 1

An

1
P

( 1)n t2n
n!

jf j dm +

1
X
R

n=N

( 1)n
n!

n= 1

An

jf j dm < :

N
1
X
X
R
R
R
Let K = [ N; N ]: Then K is compact, and RnK jf j dm =
jf
j
dm
+
jf j dm < :
An
An
n=
1
n=N
R
(e) Let > 0 be given. By (c), 9 R > 0 : B jf j dm < 2 whenever B R with m(B) < :
By (d), 9 a compact set K R : RnK jf j dm < 2 .
n!1

Let Dn = fx 2 K : jf (x)j > ng: Then m(Dn ) ! 0: Thus, 9M : m(Dn ) < for all n M:
Let A = KnDM and note that RnA = (RnK) [
R (DM ) :
R
R
Then A is measurable, jf (x)j M on A, and RnA jf j dm = RnK jf j dm + DM jf j dm < 2 + 2 = :

348. Show that if f is a Lebesgue integrable function on A and An = fx 2 A : jf (x)j ng; then
lim n m(An ) = 0:
n!1

Proof. Since f is a LebesgueR integrable function on A,Rm(An ) ! 0 as n ! 1:


Thus, by problem 1(c), lim An jf j dm = 0: Note that An jf j dm n m(An ) 0:
n!1

Hence, lim n m(An ) = 0:


n!1

R
349. Show that if f 0 on a set A, m(A) > 0; and A f dm = 0; then f = 0 a.e. on A.

1
[
Proof. A = A0 [
Bn ; where A0 = fx 2 A : f (x) = 0g and Bn = fx 2 A : f (x) > n1 g:
n=1

123

Note that B =

1
[

n=1

Bn = fx 2 A : f (x) 6= 0g:

1
X
m(Bn ); there is N : m(BN ) = a > 0:
If m(B) > 0; then since m(B)
n=1
R
R
R
Then BN f dm > N1 m(BN ) > 0: But BN f dm
f dm = 0; a contradiction.
A
Thus, m(B) = 0 and so f = 0 a.e. on A.
R
350. Show that if B f dm = 0 for every measurable subset B of A; m(A) > 0; then f = 0 a.e. on A.

Proof. Let B1 = fx 2 A : f (x) 0g and B2 = fx 2 A


R : f (x) 0g:
R
Both B1 and B2 are measurable subsets of A and so B1 f dm = 0 and B2 f dm = 0:
R
R
R
R
0:
But B1 f dm = A f + dm and B2 f dm = A f dm: Also, f + 0 and f
+
Hence, by problem 3, f = 0 a.e. on A, and f = 0 a.e. on A.
But f = f + f : Thus, f = 0 a.e. on A.

R
R
351. Show that if f is Lebesgue integrable on A and A f dm = A jf j dm; then either f 0 a.e. on A,
or f 0 a.e. on A.

Proof. We rst
that if a; bR> 0 and ja bj = a + b; then either a = 0 or b = 0:
R note
+
Now, let
a
=
f
dm
andRb = A f dm:
A
R
Then A f dm = a b and A jf j dm =Ra + b:
R
Thus, either a = 0 or b = 0 =) either A f + dm = 0 or A f dm = 0
f + ;f

=) either f + = 0 a.e. on A or f = 0 a.e. on A =) either f 0 a.e. on A, or f 0 a.e. on A.


R
352. Let ffn g be a sequence of nonnegative and measurable functions on A such that lim A fn dm = 0:
m

n!1

Show that fn ! 0: ( fn ! 0 means fn converges to 0 in measure).


R R
f dm
f dm:
Proof. Let > 0 be given and set An = fx 2 A : fn (x)
g: Then 0
m(An )
A n
An n
R
n!1
n!1
m
Thus, since A fn dm ! 0; m(An ) ! 0 and so fn ! 0:

353. Show that in problem 6, convergence in measure cannot be replaced by a.e. convergence.

Proof. Let A = [0; 1]: Let f1 = [0;1] ; f2 = [0;1=2] ; f3 = [1=2;1] ; f4 = [0;1=4] ; f5 = [1=4;1=2] ;
R
f6 = [1=2;3=4] ; f7 = [3=4;1] ; f8 = [0;1=8] ; :::; and so on. Clearly, lim A fn dm = 0:
n!1
But fn 9 0 a.e. on A.

R
jfn j
354. Let ffn g be a sequence of measurable functions on a set A, m(A) < 1: Show that lim A 1+jf
dm = 0
nj
n!1

if and only if fn ! 0:
R
jfn j
Proof. (=)) Suppose that lim A 1+jf
dm = 0, let > 0; and set An = fx 2 A : jfn (x)j
g:
nj
n!1
R
R
n!1
jfn j
jfn j
jfn j
Note that jfn (x)j
() 1+jfn j
dm
dm ! 0:
1+ : Hence, 0
1+ m(An )
An 1+jfn j
A 1+jfn j
n!1

Thus, m(An ) ! 0 and consequently, fn ! 0:


m
((=) Suppose that fn ! 0, let > 0; and set An = fx 2 A : jfn (x)j
g:
jfn j
<
:
Note that jfn (x)j < () 1+jf
1+
nj
R
R
R
R
R
jfn j
jfn j
jfn j
Now, A 1+jfn j dm = An 1+jfn j dm + AnA 1+jf
dm
1dm
+
dm = m(An ) +
An
AnA 1+
nj
m(An ) +

1+

m(A)

n!1

jfn j
Hence, lim A 1+jf
dm
nj
n!1
R
jfn j
Thus, lim A 1+jf
dm =
nj
n!1

1+

1+

m(A):

m(A): But m(A) < 1 and our choice of

0:
124

> 0 was arbitrary.

1+

m(AnAn )


355. Show that the assumption m(A) < 1 is essential in problem 8.

m
Proof. For n = 1; 2; 3; :::; let fn (x) = n1 : Then fn ! 0 on A = R:
R
R 1=n
R 1
R
jfn j
1
But A 1+jf
dm = R 1+1=n
dm = R n+1
dm = n+1
dm = 1:
R
nj

356. Suppose that f is nonnegative and measurable on A, m(A) < 1: Let Ak = fx 2 A : k f (x) < k + 1g:
1
X
Prove that f is Lebesgue integrable on A if and only if
km(Ak ) < 1:
k=0

1
[
R
Proof. We rst note that km(Ak )
f dm < (k + 1)m(Ak ) for k = 0; 1; 2; :::: Also, A =
Ak and
Ak
the Ak s are mutually disjoint and so m(A) =
Hence,

1
X

1
X
R

km(Ak )

k=0

Thus,

1
X

f dm <
Ak

k=0

km(Ak )

k=0

1
X

1
X

m(Ak ) and

k=0

(k + 1)m(Ak ) = m(A) +

k=0

f dm < m(A) +

1
X

f dm =
1
X

1
X
R

k=0

Ak

f dm:

k=0

km(Ak ):

k=0

km(Ak ):

k=0

Therefore, f is Lebesgue integrable on A if and only if

1
X

k=0

km(Ak ) < 1:

357. Suppose that f is nonnegative and measurable on A, m(A) < 1: Let Bk = fx 2 A : f (x) kg:
1
X
Prove that f is Lebesgue integrable on A if and only if
m(Bk ) < 1:
k=0

1
1
X
X
Proof. We rst note that
m(Bk ) =
(k + 1)m(Ak ), where Ak = fx 2 A : k f (x) < k + 1g: Thus,
1
X

k=0

m(Bk ) < 1 ,

1
X

k=0

1
X

k=0

km(Ak ) < 1

k=0

k=0

(k + 1)m(Ak ) < 1 ,

Problem 10

1
X

m(Ak ) +

1
X

k=0

k=0

km(Ak ) < 1 , m(A) +

1
X

k=0

km(Ak ) < 1

f is Lebesgue integrable on A.

1
X
k m(Ak );
358. Suppose that f is nonnegative and integrable on A, m(A) < 1: For > 0; dene S( ) =
k=0
R
where Ak = fx 2 A : k
f (x) < (k + 1) g: Prove that lim S( ) = A f dm:
!0

1
[
R
Proof. We rst note that k m(Ak )
f
dm
<
(k
+
1)
m(A
)
for
k
=
0;
1;
2;
::::
Also,
A
=
Ak and
k
Ak
the Ak s are mutually disjoint and so m(A) =
1
X

1
X
R

1
X

m(Ak ) and

k=0

1
X

f dm =

1
X
R

k=0

Ak

k=0
1
X

f dm:

k m(Ak ) = m(A) + S( ):
k m(Ak )
f dm <
(k + 1) m(Ak ) = m(A) +
Hence, S( ) =
Ak
k=0
k=0
k=0
k=0
R
R
Thus, S( )
f dm < m(A) + S( ): Letting ! 0; we get that lim S( )
f dm lim S( ):
A
A
!0
!0
R
Thus, lim S( ) = A f dm:
!0

359. RLet ffn g be


R suppose that
R a sequence of nonnegative functions converging to a functionRf on R; and
lim R fn dm = R f dm < 1: Show that for each measurable set A R; lim A fn dm = A f dm:
n!1
n!1

125

R
R
R
R
Proof. By Fatous Theorem, A f dm = A limfn dm lim A fn dm lim A fn dm
R
R
R
R
R
R
lim( R fn dm
f dm) = lim R fn dm lim RnA fn dm = R f dm lim RnA fn dm
RnA n
Fatous Theorem R
R
R
R
R
f dm
limfn dm = R f dm
f dm = A f dm:
R
RnA
RnA
R
R
R
R
R
R
R
Hence, A fRdm lim AR fn dm lim A fn dm
f
dm
and
so
lim
f
dm
=
lim
f
dm
=
f dm:
n
n
A
A
A
A
Thus, lim A fn dm = A f dm:
n!1

360. Suppose that


R ffn g is a sequence
R of measurable
R functions Ron A such that jfn j g; where g is integrable
on A. Show that A limfn dm lim A fn dm lim A fn dm
limfn dm:
A

Proof. Let gn = g fn ; n =R1; 2; 3; :::: Then each


R gn is nonnegative and measurable on A.
Hence, by Fatous Theorem, A limgn dm lim A gn dm:
R
R
R
R
But limgn = lim(g fn ) = g limfn ; and so A limgn dm = A (g limfn )dm = A gdm
limfn dm:
A
R
R
R
R
Also, lim A gn dm = lim A (g fn )dm = A gdm lim A fn dm:
R
R
R
R
R
R
Thus, A gdm
limfn dm
gdm lim A fn dm and so lim A fn dm
limfn dm :
A
A
A
Now, let hn = g + fn ; n = 1; R2; 3; :::: Then each Rhn is nonnegative and measurable on A.
Hence, by Fatous Theorem, A limhn dm lim
R A hn dm:
R
R
R
But limhnR = lim(g + fn ) R= g + limfn ; andRso A limhn dm
R = A (g + limfn )dm = A gdm + A limfn dm:
Also, lim A hn dm = lim A (g + fn )dm = A gdm + lim A fn dm:
R
R
R
R
R
R
Thus, A gdm + A limfn dm
gdm + lim A fn dm and so A limfn dm lim A fn dm :
A
R
R
It is always true that lim A fn dm lim A fn dm :
R
R
R
R
Thus, A limfn dm lim A fn dm lim A fn dm
limfn dm:
A

361. Suppose that ffRn g is a sequence


of real-valued functions integrable on A, m(A) < 1: Show that
R
if fn
f , then lim A fn dm = A f dm: ( fn
f means fn converges uniformly to f on A).
n!1
A
A

Proof. Let > 0 be given. Since fn


f; there exists N : jfn f j < m(A) for all x 2 A and for all n N:
AR
R
R
R
R
Hence, for n N; we have A fn dm
f dm = A (fn f )dm
jfn f j dm < A m(A) dm = :
A
A
R
R
Thus, lim A fn dm = A f dm:
n!1

362. Let fn (x) = nxn 1 (n + 1)xn ; x 2 (0; 1):


1
1
X
X
R
R
(a) Show that (0;1)
fn dm 6=
f dm:
(0;1) n
(b) Show that

1
X
R

n=1

(0;1)

n=1

n=1

jfn j dm = 1:

1
1
1
X
X
X
d
d
n 1
n
n
n+1
Proof. We rst note that for x 2 (0; 1);
nx
(n + 1)x =
x
= dx
xn xn+1
dx x
n=1

d x x2
dx ( 1 x )

d
dx (x)

(a) We have that

= 1:
1
X
R
(0;1)

fn dm =

n=1

On the other hand,


Hence,

(0;1)

1
X

n=1

1
X
R

(0;1)

(0;1)

fn dm =

n=1

1
X

nxn

(n + 1)xn dm =

n=1

1
X
R

n 1

(0;1)

nx

(n + 1)x

n=1

1
X
R

(0;1)
n=1
n 1

(0;1)

dm =

n=1

fn dm = 1 6= 0 =

(b) We rst note that jfn j =

n=1

1 dm = m ((0; 1)) = 1:

1
X

n=1

fn dm:

nx
(n + 1)xn
n
(n + 1)x
nxn 1

126

n
for x 2 (0; n+1
)
n
for x 2 ( n+1 ; 1)

[x (1

1
x)]0

1
X

n=1

0 = 0:

Thus,
=
=

1
X

n=1
1
X
n=1

1
X
R

n=1

jf j dm =
(0;1) n

xn

xn+1

n
n+1

>

n=1

2
e

1
X

nxn

n
(0; n+1
)

+ xn+1

2
1
1 n n+1
(1+ n
)

1
X
R

xn

1
n+1

1
n
n+1

(n + 1)xn dm +
1
X

n n+1
( n+1
)

n n
( n+1
)

(n + 1)xn

n
( n+1
;1)

nxn

dm

n n+1
n n
( n+1
)
+ ( n+1
)

n=1

= 1:

n=1

1
X
R
363. Let ffn g be a sequence of measurable functions on A such that
jf j dm < 1:
A n
(a) Show that the function f =
(b) Show that

1
R X
A

1
X

n=1

fn is Lebesgue integrable on A.

n=1

fn dm =

n=1

1
X
R

fn dm:

n=1

n
1
X
X
fk : Let g =
jfk j :
Proof. (a) For n = 1; 2; 3; ::: let gn =
k=1

Then each gn is measurable on A, and jgn j =


Note that

gdm =
A

1
R X
A

k=1

jfk j dm

n
X

fk

k=1

k=1
n
X

k=1

jfk j

M onotone Convergence Theorem

g:

1
X
R

k=1

jfk j dm < 1:

R
R
Thus, by the Lebesgue dominated convergence theorem, lim A gn dm = A lim gn dm:
n!1
!
!
! n!1
n
n
1
X
X
R X
R
R
R
This means that lim
fk dm = A lim
fk dm = A
fk dm = A f dm:
A
n!1
n!1
k=1
k=1
k=1
!
!
1
1
n
n
X
X
X
R
R
R
R
R X
jf j dm < 1:
f
dm
f
dm
=
f
dm
=
lim
Thus, A f dm = lim
k
k
k
A k
A
A
A
n!1

n!1

k=1

Hence, f is Lebesgue integrable on A.


R
(b) As we did in part (a), A f dm = lim

n!1

n
R X
A

fk dm

k=1

k=1

k=1

k=1

= lim

n!1

n
X
R

k=1

f dm
A k

1
X
R

fk dm:

k=1

x
R1
sin( )
364. Compute lim 0 (1+ xn)n dx and justify your answer.
n
n!1
x x
sin( )
sin( )
1
2
Solution. For n = 1; 2; 3; :::; let fn (x) = (1+ xn)n : Then jfn (x)j = (1+ xn)n
x n
(1+ n
)
ex for n large.
n
n
R1 2
Note that 0 ex dx = 2 < 1 and so by the Lebesgue dominated convergence theorem, we get that
R1
R1
R1
R1
sin( x )
lim 0 fn (x)dx = 0 lim fn (x)dx = 0 lim (1+ xn)n dx = 0 sin(0)
ex dx = 0:
n
n!1
n!1
n!1

R n
365. Compute lim 0 (1 nx )n eix dx and justify your answer.
n!1

(1 nx )n eix
for 0 x n
Solution. For n = 1; 2; 3; :::; let fn (x) =
:
0
for x > n
Rn
R
1
Then lim 0 (1 nx )n eix dx = lim 0 fn (x)dx:
n!1

n!1

Note that for n large, jfn (x)j = (1 nx )n eix


e x for all x 2 [0; 1) and for all n 2 N:
R1 x
Also, 0 e dx = 1 < 1 and so by the Lebesgue dominated convergence theorem, we get that
R1
R1
R1
R1
R1
lim 0 fn (x)dx = 0 lim fn (x)dx = 0 lim (1 nx )n eix dx = 0 e x eix dx = 0 e( 1+i)x dx = 12 + 12 i:
n!1
n!1
n!1

1
n
for 0 < x < n
366. Let fn (x) =
:
0 for n1
x<1
(a) Find lim fn (x):
n!1

127

(b) Find lim

R1

n!1 0

fn (x)dx:

(c) Is there a function g(x) 2 L1 (0; 1) : g(x) fn (x) for all n?

Solution. (a) Let x 2 (0; 1): Then there is N large so that N1


x; and so fn (x) = 0 for all n N:
Thus, lim fn (x) = 0:
n!1
1
R1
R1
(b) lim 0 fn (x)dx = lim 0n ndx = lim [nx]0n = lim 1 = 1:
n!1
n!1
n!1
R1
R n!1
1
(c) Since lim 0 fn (x)dx = 1 6= 0 = 0 lim fn (x)dx; the Lebesgue dominated convergence theorem implies
n!1
n!1
that no such g exists.

R b
367. (a) Let (a; b) (0; 2 ) be an open interval. Show that lim a cos(nx)dx = 0:
R n!1
(b) Let A (0; 2 ) be a measurable set. Show that lim A cos(nx)dx = 0:
n!1
h i
b
Rb
Rb
2
Proof. (a) a cos(nx)dx = sin(nx)
= sin(nb) nsin(na) ; so a cos(nx)dx
n
n ; and so
a
Rb
Rb
lim a cos(nx)dx = 0: Hence, lim a cos(nx)dx = 0:
n!1

n!1

(b) Let A
A

> 0 be given. Then there exists and open set G such that
1
[
(ak ; bk ); a disjoint union of open intervals.
G and m(GnA) < : Thus, G = A [ (GnA): Write G =

Then lim

n!1

(0; 2 ) be a measurable set and

cos(nx)dx = lim
G

n!1

1
X
R bk
ak

cos(nx)dx =

1
X

k=1

lim

R bk

n!1 ak

(a)

cos(nx)dx =

1
X

0 = 0:

k=1
k=1
Rk=1
Let N be large enough so that G cos(nx)dx < 2 for all n N:
R
R
R
Since m(GnA) < ; GnA cos(nx)dx
jcos(nx)j dx
dx = m(GnA) < 2 :
GnA
GnA
R
R
R
R
R
Thus, for n N; A cos(nx)dx = G cos(nx)dx
cos(nx)dx
cos(nx)dx + GnA cos(nx)dx
GnA
G
R
< 2 + 2 = : Therefore, lim A cos(nx)dx = 0:
n!1

368. Let fn (x) = 1+nn2 x2 :


(a) Does fn ! 0 a.e. on [0; 1]?
(b) Does fn ! 0 in L1 [0; 1]?
R1
(c) Find lim 0 sinx x fn (x)dx and justify your answer.
n!1

1
1
Solution. (a) For x 2 (0; 1]; lim fn (x) = lim 1+nn2 x2 = lim 1 +nx
2 = 0+1 = 0:
n!1

Thus, fn ! 0 a.e. on [0; 1]:


R1
R1
(b) lim 0 fn (x)dx = lim 0
n!1

n!1

n!1

n!1

Rn 1
u=nx
n
dx
=
lim
2
2
2 du
1+n x
n!1 0 1+u

= lim [arctan u]0 = lim (arctan n) =


n!1

n!1

2:

Thus, fn 9 0 in L [0; 1]:


u
R1
R 1 n sin x
sin( n
u=nx R n
)
(c) 0 sinx x fn (x)dx = 0 x(1+n
= 0 u (1+u
du:
2 x2 ) dx
2)
(n)
)
( sin u
(n)
for
0
u
n
u
2
( n )(1+u )
: (Remember that lim sinx x = 1):
For n = 1; 2; 3; :::; let gn (u) =
x!0+
0
for u > n
u
R1
R n sin( n )
R1
du = lim 0 gn (u)du:
Then lim 0 sinx x fn (x)dx = lim 0 u (1+u
2)
(
)
n!1
n!1
n!1
n
u
sin( n
)
1
Note that jgn (u)j = u (1+u2 )
(1+u2 ) for all u 2 [0; 1):
(n)
R1 1
Also, 0 (1+u2 ) du = 2 < 1 and so by the Lebesgue dominated convergence theorem, we get that
R1
R1
R1
R1 1
lim 0 sinx x fn (x)dx = lim 0 gn (u)du = 0
lim gn (u) du = 0 (1+u
2 ) du = 2 :
n!1
n!1
n!1

R1
sin x
369. Compute lim
dx and justify your answer.
1 1+nx2
n!1

128

sin x
1
1
Solution. Note that 1+nx
2
1+nx2
1+x2 for all n 2 N and for all x 2 R:
R1 1
< 1: Thus, by the Lebesgue dominated convergence theorem, we get that
Also, 1 1+x
2 dx =
R 1 sin x
R1
R1
sin x
lim
dx
=
lim
dx
=
0dx = 0:
2
2
1 1+nx
1 n!1 1+nx
1
n!1
R
n
370. Let f be a nonnegative Lebesgue integrable function on (0; 1): Prove that lim n1 0 xf (x)dx = 0:
n!1

x
for 0 x n
n f (x)
:
Proof. For n = 1; 2; 3; :::; let fn (x) =
0
for x > n
R
R
R
n
n
1
Then lim n1 0 xf (x)dx = lim 0 nx f (x)dx = lim 0 fn (x)dx:
n!1
n!1
n!1
R1
Note that jfn (x)j = nx f (x) f (x) for all x 2 [0; 1) and for all n 2 N: Also, 0 f (x)dx < 1:
Thus, by the Lebesgue dominated convergence theorem, we get that
Rn
R1
R1
R1
lim n1 0 xf (x)dx = lim 0 fn (x)dx = 0
lim fn (x) dx = 0 0dx = 0:
n!1
n!1
n!1

371. Let f be nonnegative and measurable on [0; 1]: Let A = fx 2 [0; 1] : f (x) > 1g:
R1
n
Show that lim 0 (f (x)) dx exists if and only if m(A) = 0:
n!1

Proof. (=)) Suppose that m(A) > 0:R Then 9r > 1 : m(fx 2 [0; 1] : f (x) rg) > 0:
n
Let B = fx 2 [0; 1] : f (x) rg: Then B (f (x)) dx rn m(B):
R1
R
n
n
Thus, lim 0 (f (x)) dx
lim
(f (x)) dx = lim rn m(B) = 1:
n!1
n!1 B
n!1
R1
R
R
R
n
n
n
n
((=) Suppose that m(A) = 0: Then 0 (f (x)) dx = A (f (x)) dx + [0;1]nA (f (x)) dx = [0;1]nA (f (x)) dx:
R
m(A)=0
n
Note that on [0; 1]nA; (f (x))
1 for all n 2 N and [0;1]nA 1dx = m([0; 1]nA) = 1 < 1:
R1
n
Thus, by the Lebesgue dominated convergence theorem, we get that lim 0 (f (x)) dx =
n!1
R
R
n
n
lim
(f
(x))
dx
=
lim (f (x)) dx = m(C) 1; where C = fx 2 [0; 1] : f (x) = 1g:
[0;1]nA
[0;1]nA n!1
n!1
R
b
372. Show that if f is Riemann integrable on [a; b] and f (x) = 0 for x 2 [a; b] \ Q; then a f (x)dx = 0:

Proof. We have that f is Riemann integrable on [a; b] if and only if f is continuous a.e. on [a; b]:
Let D = fx 2 [a; b] : f is discontinuous at xg: Then m(D) = 0 and f is continuous on [a; b]nD:
Since [a; b] \ Q is dense in [a; b]; we get that f (x) = 0 for all x 2 [a; b]nD:
R
R
R
Rb
m(D)=0
Thus, a f (x)dx = D f (x)dx + [a;b]nD f (x)dx = 0 + [a;b]nD 0dx = 0:

373. Prove the following Schwartz inequality:


2
Rb
Rb
Rb
2
2
If f and g are Lebesgue integrable on (a; b); then a f (x)g(x)dx
(f
(x))
dx
(g(x)) dx :
a
a
R R R R
2
b b
b b
2
Proof. 0
(f (x)g(y) f (y)g(x)) dxdy = a a f (x)g(y))2 2f (x)f (y)g(x)g(y) + (f (y)g(x) dxdy
a a
RbRb
R
R
R
R
b b
b b
= a a (f (x))2 (g(y))2 dxdy
2 a a f (x)f (y)g(x)g(y)dxdy + a a (f (y))2 (g(x))2 dxdy
Rb
Rb
Rb
Rb
Rb
Rb
= a (f (x))2 dx
(g(y))2 dy
2 a f (x)g(x)dx
f (y)g(y)dy + a (f (y))2 dy
(g(x))2 dx
a
a
a
Rb
Rb
Rb
Rb
Rb
Rb
= a (f (x))2 dx
(g(x))2 dx
2 a f (x)g(x)dx
f (x)g(x)dx + a (f (x))2 dx
(g(x))2 dx
a
a
a
2
Rb
Rb
Rb
2
= 2 a (f (x))2 dx
(g(x))
dx
2
f
(x)g(x)dx
:
a
a
2
Rb
Rb
R
b
2
2
Hence, a f (x)g(x)dx
(f (x)) dx
(g(x)) dx :
a
a

374. Let f : [a; b] ! R be nonnegative and Lebesgue integrable.


2
2
2
Rb
Rb
Rb
Prove that a f (x) cos xdx + a f (x) sin xdx
f
(x)dx
:
a

2
2
2
2
p
p
Rbp
Rb
Rb
Rbp
f
(x)
f
(x)
cos
xdx
+
f
(x)
f
(x)
sin
xdx
Proof.
f
(x)
cos
xdx
+
f
(x)
sin
xdx
=
a
a
a
a

129

Rb
Rb
Rb
f (x)dx
f (x) cos2 xdx + a f (x)dx
f (x) sin2 xdx
a
a
h
i
Rb
Rb
Rb
= a f (x)dx
f (x) cos2 xdx + a f (x) sin2 xdx
a
hR
i
2
Rb
Rb
b
2
2
= a f (x)dx
f
(x)
cos
x
+
sin
x
dx
= a f (x)dx :
a

R
x2
for x 2 [0; 1]nQ
: Is f Riemann integrable on [0; 1]?
375. Find [0;1] f dm; where f (x) =
1
for x 2 [0; 1] \ Q

Solution. Since [0; 1] \ Q is dense in [0; 1] and f (x) = x2 for x 2 [0; 1]nQ; we have that f is not
continuous at any point in (0; 1): Thus, f is not Riemann integrable.
Now, m([0; 1] \ Q) = 0 and so m([0; 1]nQ) = 1:
R1
R
R
R
R
Thus, [0;1] f dm = [0;1]nQ x2 dm + [0;1]\Q 1dm = [0;1]nQ x2 dm = 0 x2 dx = 31 :

376. Let C be the Cantor set.


8
9
sin(
x)
for
x
2
[0;
1=2]nC
<
=
R
cos( x)
for x 2 [1=2; 1]nC
(a) Find [0;1] f dm; where f (x) =
:
:
;
x2
for x 2 C
8
9
0
for x 2 C
>
>
<
=
n 1
2[
(b) Let f be dend on [0; 1] as follows: f (x) =
;where Cn;k is a removed
>
for x 2
Cn;k >
: n
;
k=1
R
interval of length 31n : Find [0;1] f dm:

Solution.
(a) WeRrst note that m(C) =R 0 and so m([0; 1=2]nC)R = 1=2 and m([1=2; 1]nC) = 1=2:
R
Now, [0;1] f dm = [0;1=2]nC sin( x)dm + [1=2;1]nC cos( x)dm + C x2 dm
R
R 1=2
R1
Thus, [0;1] f dm = 0 sin( x)dx + 1=2 cos( x)dx + 0 = 0:
0n 1 1
n 1
n 1
n 1
1 2X
1 2X
1 2X
1
2X
X
X
X
X
R
R
R
1 A
(b)
f dm =
f dm =
ndm =
n m(Cn;k ) =
n@
n
Schwartz

Rb
a

[0;1]

1
X

n=1

2n 1
3n

Cn;k
n=1 k=1
1
X
= 12
n( 23 )n =
n=1

Cn;k

n=1 k=1

n=1 k=1

n=1

k=1

3:

377. Dene the Cantor-Lebesgue function ' : [0; 1] ! [0; 1] as follows:


1
X
an
If x is an element of the Cantor set C and x =
3n with an = 0 or 2; then we put
n=1

'(x) = '(

1
X

n=1

an
3n )

1
X

an 1
2 2n ;

n=1

that is, if an is the n-th ternary digit for x; then the n-th binary digit for '(x) is
We extend ' to [0; 1] by setting:

an
2 :

'(x) = supf'(y) : y 2 C; y < xg


(a) Show that the Cantor function maps the Cantor set C onto [0; 1].
(b) Show that ' is increasing and continuous on [0; 1]:
(c) Show thatR'0 (x) = 0 a.e. on [0; 1]:
(d) Compute [0;1] 'dm and justify your answer.

Proof. See my notes on Cantor Lebesgue Function: http://www.adwan.net/RealAnalysis/Cantor.pdf.

1 for x 2 A
378. Let A [0; 1] be a perfect nowhere dense set with m(A) > 0: Show that A (x) =
0 for x 2
=A
130

is not Riemann integrable on [0; 1] but it is Lebesgue integrable on [0; 1].

Solution.
Since A R [0; 1]; weRhave that 0 < m(A) 1:
R
Now, [0;1] A dm = A 1dm + [0;1]nA 0dm = m(A): Thus, A (x) is Lebesgue integrable on [0; 1].
Now, B = [0; 1]nA is dense in [0; 1] and A (x) 0 on B:
Let a 2 A: Then 9 a sequence fbn g in B such that lim bn = a:
n!1

Note that A (bn ) = 0 for all n 2 N; and A (a) = 1: This implies that A is not continuous at a:
Thus, A D = fx 2 [0; 1] : A is discontinuous at xg and so m(D) m(A) > 0:
Hence, A (x) is not Riemann integrable on [0; 1]:
X

1
379. Let frn g be the sequence of rational numbers and dene f (x) =
2n :
fn:rn <xg

(a) Show that f is continuous on the irrationals.


(b) Show thatR f is discontinuous on the rationals.
(c) Compute [0;1] f dm and justify your answer.

Proof. (a) Let x0 be an irrational number, and let > 0 be given.


1
1
X
X
1
1
Since
<
1;
we
can
nd
N
:
2n
2n < :
n=1

n=N

Now, let

> 0 be small enough so that (x0


; x0 + ) \ fr1 ; r2 ; :::; rN 1 g = ?:
X
X
X
1
1
Then 8x 2 (x0
; x0 ); f (x0 ) f (x) =
=
n
n
2
2
fn:rn <x0 g

Also, 8x 2 (x0 ; x0 + ); f (x)

f (x0 ) =

fn:rn <xg

1
2n

fn:rn <xg

1
2n

fn:rn <x0 g

1
X

fn:rn <xg

1
2n

n=1

(m ((rn ; 1) \ [0; 1])) :

fn:x0 rn <xg

1 A
dm
2n

[0;1]

<

1
2n

<

1
(rn ;1) (x) 2n

n=1

dm =

1 A
2n
1
X

n=N
1
X

1
2n

< :

1
2n

< :

n=N

fn:x0 <rn <xg

fn:x0 <rn <xg

1
X

1
X

fn:x rn <x0 g

Thus, 8x 2 (x0
; x0 + ); jf (x) f (x0 )j < :
Hence, f is continuous at x0 :
(b) Let x0 be a rational number. Then x0 = rm for some
0 m 2 N:
X
X
1
@ 1m +
Thus, if x > x0 ; then f (x) f (x0 ) =
2n =
2
Thus, f is discontinuous
0 at x0 :
X
R
R
(c) [0;1] f dm = [0;1] @

1
2n

>

1
2n

n=1

1
2m :

[0;1]

(rn ;1) (x)

dm

1
X
1 p 1
380. Let frn g be the sequence of rational numbers in [a; b] and dene f (x) =
:
2n
jx rn j

n=1
Rb
(a) Show that a f (x)dx < 1:
Rb
2
(b) Show that a (f (x)) dx = 1:

1
1
Rb
RbX
M onotone Convergence Theorem X R b 1
1 p 1
p 1
Proof. (a) a f (x)dx = a
dx
=
dx
2n
a 2n

=
=

1
X

n=1
1
X
n=1

1
2n
1
2n

hR

n=1

rn
p 1
dx
a
rn x

p
2 rn

Rb

p
a+2 b

rn

jx rn j

p 1
dx
x rn

rn

1
X

1
X

1
2n

n=1
p
b a+ b a
n
2 1

n=1

p
[ 2 rn
p
=2 b

n=1

p
r
x]an + [2 x

1
X

n=1

131

1
2n

rn ]rn

p
=4 b

jx rn j

a < 1:

(b)

Rb
a

(f (x)) dx =

1
X

Rb
a

1
2n

1
jx rn j

!2

dx

1
X

Rb
a

1
1
4n jx rn j

dx

n=1
n=1
1
1
1
hR
i X
X
Rb 1
M onotone Convergence Theorem X R b
rn
1
1
1
1
1
dx
=
dx
+
dx
=
n
n
4
4n
a rn x
rn x rn
a 4 jx rn j
n=1
n=1
n=1
hR
i
hR
i
b
b
b
1
1
1
1
1
Now, 4n rn x rn dx = 4n lim
dx = 4n lim [log(x rn )]R
R x rn
R!rn +
R!rn +
1
hR
i
X
b
1
1
dx
= 1:
= 41n lim [log(b rn ) log(R rn )] = 1 for all n 2 N: Thus,
n
4
rn x rn
R!rn +
n=1
Rb
2
Consequently, a (f (x)) dx = 1:

hR

b
1
dx
rn x rn

381. Let fxn g be a sequence in [0; 1] and fan g be a sequence of nonnegative real numbers such that
1
1
X
X
p an
converges for almost all x 2 [0; 1]:
an < 1: Show that
n=1

n=1

jx xn j

1
1
[
X
p an
and set f (xn ) = 0 for n = 1; 2; 3; ::::
Proof. For x 2 [0; 1]n
fxn g; dene f (x) =
n=1

We have that
=
=

1
X

n=1
1
X

an

hR

R1
0

f (x)dx

xn
p dx
0
xn x

p
p
an 2 xn + 2 1

R1

fxn g is countable

R1

xn

p dx
x xn

1
X

1
X

n=1

an

n=1

p
p
xn = 2 xn + 2 1

n=1

n=1

jx xn j

M onotone Convergence Theorem


an
dx
=
jx xn j

p
[ 2 xn
1
X

xn

x
x]0 n

an

p
+ [2 x
[2 + 2]

n=1

1
X

1
xn ]xn

an = 4

n=1

Thus, f is Lebesgue integrable on [0; 1] and so m(fx 2 [0; 1] : f (x) = 1g) = 0:


1
X
p an
Therefore, f (x) =
is nite for almost all x 2 [0; 1]:

1
X

n=1

1
X
Rb
a

n=1

an
dx
jx xn j

an < 1:

jx xn j

n=1

382. Suppose f : [0; 1] ! [0; 1) is measurable and satises m(fx 2 [0; 1] : f (x) yg) y(log1 y)2 8 y 2:
Show that f is Lebesgue integrable on [0; 1]:

Proof. Let An = fx 2 [0; 1] : f (x) ng for n = 0; 1; 2; ::::


1
1
1
1
X
X
X
X
1
1
Then
m(An ) = m(A0 ) + m(A1 ) +
m(An ) 1 + 1 +
=
2
+
2
n(log n)
n(log n)2 < 1:
n=0

(By the integral test,

n=2

R1
2

dx
x(log x)2

1
ln 2

< 1 and so

1
X

n=2

n=2

1
n(log n)2

n=2

< 1):

Thus, by problem 11, we see that f is Lebesgue integrable on [0; 1]:


R
p
383. For p > 0; let B = ff : R ! R such that f is measurable and R jf j dm 1g:
m
Suppose that fn 2 B; and fn ! f on R: Prove that f 2 B:
R
p
Proof. fn 2 B =) R jfn j dm 1 for all n 2 N:
m
a.e.
p a.e.
p
Since fn ! f on R; 9 a subsequence
fnkR ! f on R and so jfnk jR ! jf j on R:
R
p
p
p
1:
Now, Fatous Theorem =) R jf j dm = R (lim jfnk j ) dm lim R jfnk j dm
Therefore, f 2 B:

384. TrueRor False: If f is Lebesgue integrable on R; then there exists a continuous function g : R ! R
such that R jf gj dm = 0:

1 for x 2 [0; 1]
Solution. Let f (x) = [0;1] (x) =
.
0 for x 2
= [0; 1]
132

Then f is Lebesgue integrable on R and

f dm = 1:
R
Problem 3
Then for such a continuous function g; we have R jf gj dm = 0 =) jf gj = 0 a.e. on R
=) f = g a.e. on R =) m(fx 2 R : 0 < g(x) < 1g) = 0 =) m(g 1 (0; 1)) = 0:
But g is continuous =) g 1 (0; 1) is open =) m(g 1 (0; 1)) > 0: A contradiction.
Hence, there is no such g and the statement is False.

385. We say that functions fn ; n = 1;


R 2; 3; :::; integrable on A are equi-integrable if 8 > 0 9 > 0 such that
for every measurable subset B of A, B jfn j dm < for n = 1; 2; 3; :::; whenever m(B) < :
Show that ifR ffn g is a convergent
sequence, say fn ! f; of equi-integrable functions on a set A, m(A) < 1;
R
then lim A fn dm = A f dm:
n!1

Proof. Let > R0 be given. By equi-integrability of ffn g; 9 > 0 such that for every measurable
subset B of A, B jfn jRdm < 3 for Rn = 1; 2; 3; :::; whenever
R m(B) < :
By Fatous Theorem, B jf j dm = B lim jfn j dm lim B jfn j dm < 3 :
By Egorovs Theorem, 9 a measurable subset C A; m(AnC) < such that fn
f:
R

Thus, 9N : jfn (x) f (x)j < 3m(C) for all n N and for all x 2 C:
R
R
R
R
R
R
Therefore, for n N; A fn dm
f dm = A (fn f )dm
jf
f j dm = C jfn f j dm+ AnC jfn f j dm
A
A n
m(AnC)< R
R
R
R
jf
f
j
dm
+
jf
j
dm
+
jf
j
dm
<
dm + 3 + 3 = :
n
n
AnC
C 3m(C)
C
AnC R
R
Thus, lim A fn dm = A f dm:
n!1
R R
m
386. (Fatou in measure) Show that if fn 0 and fn ! f on A, then A f dm lim A fn dm:
R
Proof. Let an = A fn dm and a = liman : Then there is a subsequence ank ! a as k ! 1:
m
m
a.e.
Since fn ! f; we get that fnk ! f and so 9 a subsequence fnkm ! f on A as m ! 1:
R
R
R
Thus, Fatous Theorem =) A f dm = A lim fnkm dm
lim
f
dm = lim ankm = a:
A n km
m!1
m!1
m!1
R
R
Hence, A f dm lim A fn dm:

m
387. (Monotone
Convergence
in measure) Show that if fn 0; fn fn+1 ; and fn ! f on A,
R
R
then lim A fn dm = A f dm:
n!1

m
a.e.
Proof. Since fn ! f , 9 a subsequence fnk ! f on A as k !
R 1:
R
Thus, by the Lebesgue Monotone Convergence Theorem, lim A fnk dm = A f dm:
R
R k!1
Since fn fn+1 ; we conclude that lim A fn dm = A f dm:
n!1

m
388. (Lebesgue DCT in measure) Show that if fn ! f on A, and there
g integrable
R exists a function
R
on A such that jfn (x)j g(x) for all n 2 N and for all x 2 A; then lim A fn dm = A f dm:
n!1
R R
Proof. Let an = A fn dm and a = A f dm: If lim an 6= a; then 9 a subsequence fank g : ank ! b 6= a:
m

n!1

a.e.

Since fn ! f; we get that fnk ! f and so 9 a subsequence fnkm ! f on A as m ! 1:


Now, fnkm (x)
g(x) for all m 2 N and for all x 2 A:
R
R
Thus, by the Lebesgue Dominated Convergence Theorem, we get that lim
fnkm dm = A f dm:
A
m!1
In other words, lim ankm = a: But lim ank = b =) lim ankm = b 6= a: A contradiction.
m!1
m!1
R
R m!1
Hence, lim A fn dm = A f dm:
n!1

389. (Equi-integrability in measure) Show that if ffn g is a sequence of equi-integrable functions on


R
R
m
a set A, m(A) < 1; and fn ! f on A, then lim A fn dm = A f dm:
n!1
R R
Proof. Let an = A fn dm and a = A f dm: If lim an 6= a; then 9 a subsequence fank g : ank ! b 6= a:
n!1

133

a.e.

Since fn ! f; we get that fnk ! f and so 9 a subsequence fnkm ! f on A as m ! 1:


R
R
Since ffnkm g are equi-integrable, we can use problem 39 to get that lim
f
dm = A f dm:
A nk m
m!1
In other words, lim ankm = a: But lim ank = b =) lim ankm = b 6= a: A contradiction.
m!1
m!1
R m!1
R
Hence, lim A fn dm = A f dm:
n!1

a.e.
390. Let f; fn : RR ! R be integrable functions such thatR fn ! f onRR:
Show that lim R jfn f j dm = 0 if and only if lim R jfn j dm = R jf j dm:
n!1
n!1
R
R R
R
jf j dm
jf j dm = R (jfn j jf j) dm
jf
f j dm ! 0 as n ! 1:
Proof. (=))
R nR
R
R n
R 0
Thus, lim R jfn j dm = R jf j dm:
n!1

((=) We prove it rst for sets of nite measure. So let A R be a measurable set, m(A) < 1 and let
> 0 be Rgiven. Since jf j is integrable
R on A; 9 >
R 0 : whenever C A is measurableRand m(C) < ;
we have C jf j dm < 4 : Since lim A jfn j dm = A jf j dm; we can nd N1 such that C jfn j dm < 2 8n N1 :
n!1

a.e.

Since fn ! f on A, Egorovs Theorem =) 9 a measurable set B

A, m(AnB) <

Now, Let Am = [m; m + 1]: Then R =


Hence, lim

n!1

1
[

Am and so

m= 1

jf
R n

f j dm = lim

n!1

1
X
R

m= 1

jf
Am n

jf
R n

f j dm =

1
X

f j dm =

m= 1

f:

Thus, 9N2 : jfn (x) f (x)j < 4m(B) for all n N2 and for all x 2 B:
R
R
R
Let N = maxfN1 ; N2 g: Then, for n N; A jfn f j dm = B jfn f j dm + AnB jfn
m(AnB)<
R
R
R
R
dm + AnB jfn j dm + AnB jf j dm
dm + 2 + 4 = :
4m(B)
B 4m(B)
B
R
Therefore, lim A jfn f j dm = 0:
n!1

such that fn

lim

n!1

1
X
R

m= 1

jf
Am n

f j dm

jfn

f j dm:

f j dm =

1
X

Am

0 = 0:

m= 1

R jxj
2
391. Use Fubinis theorem to prove that Rn e
dx = n=2 :
R
R
R R R
R R x2 x2 x2
2
x21 x22
x2n
Proof. Rn e jxj dx = R R
e
dx
dx
dx
=
e 1e 2
e n dx1 dx2
dxn
1
2
n
R
R R
R
R
R
2
2
2
Fubini R
=
e x1 dx1
e x2 dx2
e xn dxn :
R
R
R
2
R
R
R
2
2
2
2
2
2
Fubini R R
Polar Co ordinates R 2 R 1
=
e r (rdrd )
Now, R e x dx = R e x dx
e x dx
= R R e x y dxdy
0
0
R
h
i1
R1
R
2
Fubini R 2
1
r2
r2
dr
=
(2
)
(
=
d
re
e
) = : Thus, R e x dx = 1=2 :
2
0
0
0
R
R
R
R
2
2
2
x22
1=2
1=2
Therefore, Rn e jxj dx = R e x1 dx1
e
dx2
e xn dxn = 1=2
= n=2 :
R
R

R 1
x sin x
392. Let a > 1: Prove that 0 1+(nx)a dx = o( n1 ):
R
1 x sin x
nx sin x
Proof. We need to show that lim n 0 1+(nx)
a dx = 0: Let fn (x) = 1+(nx)a ; n = 1; 2; 3; ::::
n!1

Then lim fn (x) = 0 since a > 1:


n!1

Now, if x 2 [0; n1 ]; then nx 1 and so nx 1 + (nx)a which imples that jfn (x)j 1:
nx
1
If x 2 [ n1 ; 1]; then (nx)
1 since (nx)a 1 1: Hence, nx 1 + (nx)a and so jfn (x)j 1:
a = (nx)a 1
Therefore, for all n and for all x 2 [0; 1]; jfn (x)j 1:
R1
Moreover, 0 1dx = 1 < 1 and so, by the Lebesgue DCT, we get that
R 1 x sin x
R1
R1
nx sin x
lim n 0 1+(nx)
lim 1+(nx)
a dx = 0
a dx = 0 0dx = 0:
n!1
n!1

R1
393. Compute lim 0 (1+ x dx
n x1=n and justify your answer.
n!1
n)

Solution. For x 2 (0; 1); let fn (x) = (1+ x )1n x1=n ; n = 1; 2; 3; :::: Then lim fn (x) = e x :
n!1

For x 2 (0; 1); we have fn (x)

x1=n

for n = 1; 2; 3; ::::

134

For x 2 [1; 1); we have fn (x)


1

1
x n
(1+ n
)

1
2
(1+ x
2)

for n

2:

for x 2 (0; 1)
for n 2: Then fn (x) gn (x) on (0; 1) for n
for x 2 [1; 1)
i
i
h
hR
R1
R1
1 1
dx + 0 (1+1x )2 dx = lim nn 1 + 23 = 53 :
Then lim 0 gn (x)dx = lim 0 x1=n
Let gn (x) =

x1=n
1
2
(1+ x
2)

n!1

n!1

Also, lim gn (x) = g(x); where g(x) =


n!1

n!1

1
2
(1+ x
2)

2:

for x 2 (0; 1)
for x 2 [1; 1)

; and

R1
0

g(x)dx = 53 :

Thus, gn (x) ! g(x) in L1 ((0; 1)):


Therefore,
DCT,R (see problem 56), we
R1
R 1get xthat
R 1 by the variant of Lebesgue
1
=
lim
f
(x)dx
=
lim
f
(x)dx
=
e dx = 1:
lim 0 (1+ x dx
n
n x1=n
0 n!1 n
0
n!1 0
n!1
n)

R 1 log(x+n) x
e cos x dx and justify your answer.
394. Compute lim 0
n
n!1

Solution. Let fn (x) = log(x+n)


e x cos x; n = 1; 2; 3; ::::: Then lim fn (x) = 0 on (0; 1):
n
n!1
R1
log(x+n)
x
x
Now, jfn (x)j =
e cos x
e for n = 1; 2; 3; :::: Since 0 e x dx = 1 e 1 < 1;
n
R1
R1
R1
log(x+n)
x
x
e
cos
xdx
=
e
cos
x
dx
=
0dx = 0:
the Lebesgue DCT implies that lim 0 log(x+n)
lim
n
n
0 n!1
0
n!1
R
1
nx
395. (a) Compute lim 0 1+n
2 x2 dx and justify your answer.
n!1
R 1 n3=2 x
(b) Compute lim 0 1+n2 x2 dx and justify your answer.
n!1

R 1 nx
R 1 2n2 x
log(1+n2 )
1
1
2 2 1
dx
=
lim
dx
=
lim
= 0:
Solution. (a) lim 0 1+n
log
1
+
n
x
=
lim
2 x2
2 2
2n
0
n!1
n!1 2n 0 1+n x
n!1 2n
n!1
R 1 n3=2 x
R 1 2n2 x
log(1+n2 )
1
1
2 2 1
p
p
p
(b) lim 0 1+n
= lim
= 0:
2 x2 dx = lim 2 n
2 x2 dx = lim 2 n log 1 + n x
1+n
0
2 n
0
n!1
n!1
n!1
n!1

1
X
n
p
R =2
396. Compute
1
sin x cos xdx and justify your answer.
0
n=0

p n
Solution. For x 2 [0; 2 ]; let fn (x) = 1
sin x cos x: Then fn (x) 0 for all n and for all x 2 [0; 2 ]:
n
1
X
X
Now, if one sets gn (x) =
fk (x); then one gets that gn (x) % g(x) =
fk (x) as n ! 1:
k=0

k=0

Thus, By the Lebesgue monotone convergence theorem, we get !


that
!
1
1
k
p
R =2
R =2
R =2 X
R =2 X
1
lim
gn (x)dx = 0
lim gn (x) dx = 0
fk (x) dx = 0
sin x cos x dx
0
n!1
n!1
k=0
k=0
!
p
1
X
k
p
R =2
R =2 cos x
(1 sin x) 1 R =2
cospx
p
= 0
cos x
1
sin x
dx
=
dx = 0
dx = 2:
0
sin x
1 (1
sin x)
k=0

R 1
397. Compute lim 0 (1 + nx ) n sin( nx )dx and justify your answer.
n!1

Solution. Let fn (x) = (1 + nx ) n sin( nx ); n = 1; 2; 3; ::::: Then lim fn (x) = 0 on (0; 1):
n!1

Now, jfn (x)j = (1 + nx ) n sin( nx )


(1 + nx ) n (1 + x2 ) 2 for all x 2 [0; 1] and for all n 2:
R1
R1
Since 0 (1 + x2 ) 2 dx = 23 < 1; the Lebesgue DCT implies that lim 0 (1 + nx ) n sin( nx )dx =
n!1
R1
R1
x
x
n
lim (1 + n ) sin( n ) dx = 0 0dx = 0:
0 n!1

R 1 1+nx
398. Compute lim 0 (1+x)
n dx and justify your answer.
n!1

1+nx
Solution. Let fn (x) = (1+x)
n ; n = 1; 2; 3; ::::: Then lim fn (x) = 0 on (0; 1):
n!1
R1
1+nx
Now, jfn (x)j = (1+x)n
1 for all n:Since 0 1dx = 1 < 1; the Lebesgue DCT implies
135

R1
R1
R 1 1+nx
1+nx
dx = 0 0dx = 0:
that lim 0 (1+x)
lim (1+x)
n dx = 0
n
n!1
n!1

R1
1 for x = n1
is Riemann integrable on [0; 1] and compute 0 f (x)dx:
399. Show that f (x) =
0 otherwise

Proof. Note that f (x) is continuous on A = (0; 1]nf n1 : n 2 Ng and m(f n1 : n 2 Ng [ f0g) = 0:
Thus, f is continuous a.e. on [0; 1] and so f is Riemann integrable on [0; 1]:
R1
R
R
R
R
Now, 0 f (x)dx = A f (x)dx + [0;1]nA f (x)dx = A 0dx + [0;1]nA f (x)dx = 0 + 0 = 0:

400. (Variant of Fatous Theorem) Let fgn g be a sequence of integrable functions on A such that
a.e.
gn ! g in L1 (A): Suppose that ffn g is a sequence
R of measurable
R functions on A
R such that fn ! f on A,
and fn gn a.e. for each n 2 N: Prove that lim A fn dm
limfn dm = A f dm:
A

Proof. Let hn (x) = gn (x) R fn (x): Then hn (x) R 0 a.e. for each n 2 N:
Now, Fatous theorem =) A (limhn ) dm lim A hn dm
R
R
=) A (lim (gn fn )) dm lim A (gn fn ) dm
R
R
R
=) A (limgn )
limfn dm lim A gn dm
f dm
A n
R
R
R
R
=) A (limgn ) dm
limfn dm lim A gn dm
lim A fn dm
A
R
R
Now, since gn ! g in L1 (A); we get that lim A gn dm = A gdm and so
n!1 R
R
R
R
=) A gdm
limf
dm
gdm
lim A fn dm
n
A
A
R
R
R
=) lim A fn dm
limf
dm
=
f
dm:
n
A
A

401. (Variant of Fatous Theorem in measure) Let fgn g be a sequence of integrable functions on A
such that gn ! g in L1 (A): Suppose that ffn g is a sequence of measurable functions on A such that
R
R
m
fn ! f on A, and fn gn a.e. for each n 2 N: Prove that lim A fn dm
limfn dm:
A
R
Proof. Let an = A fn dm and a = liman : Then there is a subsequence ank ! a as k ! 1:
m
m
a.e.
Since fn ! f; we get that fnk ! f and so 9 a subsequence fnkm ! f on A as m ! 1:
R
R
R
Thus, Variant of Fatous Theorem =) a = lim A fnkm dm
limfnkm dm = A f dm:
A
R
R
R
Hence, liman = lim A fn dm
limfn dm = A f dm:
A

402. (Variant of Lebesgues DCT) Let fgn g be a sequence of integrable functions on A such that
a.e.
gn ! g in L1 (A): Suppose that ffn g is a sequence ofR measurableRfunctions on A such that fn ! f on A,
and jfn j gn a.e. for each n 2 N: Prove that lim A fn dm = A f dm:
n!1

Proof. Applying Fatous theorem and the variant of Fatous theorem, we get that:
Fatou
Variant of Fatou R
R
R
R
R
jf j dm = A (lim jfn j) dm
lim A jfn j dm
lim A jfn j dm
lim jfn j dm
AR
R
R
R A
R
R
= A jf j dm: Hence, A jf j dm = lim A jfn j dm = lim A jfn j dm and so lim A jfn j dm = A jf j dm:
a.e.

n!1
a.e.

a.e.

Now, to get the result, we note that fn ! f on A =) fn+ ! f + and fn ! f on A.

403. (Variant of Lebesgues DCT in measure) Let fgn g be a sequence of integrable functions on A
such that gn ! g in L1 (A): Suppose that ffn g is a sequence of measurable functions on A such that
R
R
m
fn ! f on A, and jfn j gn a.e. for each n 2 N: Prove that lim A fn dm = A f dm:
n!1
R R
Proof. Let an = A fn dm and a = A f dm: If lim an 6= a; then 9 a subsequence fank g : ank ! b 6= a:
m

n!1

a.e.

Since fn ! f; we get that fnk ! f and so 9 a subsequence fnkm ! f on A as m ! 1:


Now, fnkm (x)
gn (x) for all m 2 N and for a.e. x 2 A:
R
R
Thus, by the variant of Lebesgue DCT, we get that lim
f
dm
=
f dm:
n
k
A
A
m
m!1
In other words, lim ankm = a: But lim ank = b =) lim ankm = b 6= a: A contradiction.
m!1
m!1
R m!1
R
Hence, lim A fn dm = A f dm:
n!1

136


a.e.
404. Let 1 p < 1; fn 2 Lp (R) such thatR fn ! f on R: Suppose that
p
(i) 9n1 > 0 and A R with m(A) < 1 : RnA jfn j dm 1 8n n1 ; and
R
p
(ii) 9n0 > 0 and 0 < < 1 : whenever m(F) < ; F jfn j dm 1 8n n0 :
Show that f 2 Lp (R):

a.e.
p a.e.
p
Proof. fn ! f on R
R =)pjfn j !
R jf j onp R:
R
p
By Fatou
jfRj dm = R lim jfn j dm lim R jfn j dm: R
R
R s Theorem,
R
p
p
p
p
Thus, RnA jf j dm lim RnA jfn j dm 1; and whenever m(F) < ; F jf j dm lim F jfn j dm 1.
p
p
Now, since m(A) < 1; Egorovs Theorem =) 9B A : m(AnB) < and jfn j
jf j :
p

Thus, 9N
B, jf j
nj
R 1 : jjf
Rjf j j 1p 8x 2 B
R andp8n NR1 =) on
R1 + jfN1 j . p
p
p
Hence, R jf j dm = RnA jf j dm + B jf j dm + AnB jf j dm 1 + B (1 + jfN1 j )dm + 1
R
p
= 2 + m(B) + B jfN1 j dm < 1:
p
Therefore, f 2 L (R):

405. Prove the following Tchebyshevs Inequality:


Let f be nonnegative and measurable on A:
R
1
If > 0; then m(fx 2 A : f (x) > g)
f
dm:
A

R R R
Proof. Let B = fx
f dm
dm = m(B):
B
B
R 2 A : f (x) > g: Then A f dm
1
Thus, m(B)
f
dm:
A
p

406. Let fn (x) = ne nx on [0; 1]: Prove that:


(a) fn (x) ! 0 pointwise in (0; 1]:
R1
(b) 0 (fn (x))2 dx C for all n 2 N:
(c) fn does not converge in L2 (0; 1):
R1
(d) 0 fn (x)g(x)dx ! 0 for each g 2 L2 (0; 1):
p
p
n LHopital
Proof. (a) Let x 2 (0; 1]: Then lim fn (x) = lim ne nx = lim enx
=
lim p 1 nx = 0:
n!1
n!1
n!1
n!1 2 nxe
R
R
R1
1 p
1
(b) 0 (fn (x))2 dx = 0 ( ne nx )2 dx = 0 ne 2nx dx = 12 1 e 2n
1 for all n 2 N:
R1
R1
1
1
2
2n
(c) lim 0 (fn (x)) dx = lim 2 1 e
= 2 ; whereas 0 lim (fn (x))2 dx = 0:
n!1

n!1

n!1

(d) Let ' 2 L (0; 1) be a simple function. Write '(x) =

Then lim

n!1

= lim

n!1

R1

k
X
i=1

ai

fn (x)'(x)dx = lim
R

Ai

fn (x)dx

n!1

R1
0

fn (x)

k
X

ai

i=1

k
X
i=1

ai

Ai (x);

Ai (x)

dx

where [0; 1] =

k
[

Ai :

i=1

= lim

n!1

k
X
i=1

ai

R1
0

fn (x)

Ai (x)dx

R1p
R1
n!1
fn (x)dx = 0 ne nx dx = pn1 0 ne nx dx = p1n (1 e n ) ! 0: Hence, fn converges in L1 (0; 1):
R
R
R
R1
Thus, lim Ai fn (x)dx = Ai lim fn (x)dx = Ai 0dx = 0; and so lim
f (x)'(x)dx = 0:
0 n

Now,

R1
0

n!1

n!1

n!1

Now, let g 2 L2 (0; 1) and let > 0 be given: Then, by density of simple functions in L2 (0; 1);
R1
2
2
9 a simple function ' 2 L2 (0; 1) : 0 jg(x) '(x)j dx < 4 :
R1
By what we did above, 9N : 0 jfn (x)'(x)j dx < 2 for all n N:
R1
R1
Thus, 0 jfn (x)g(x)j dx = 0 jfn (x)g(x) fn (x)'(x) + fn (x)'(x)j dx
R1
R1
jfn (x)g(x) fn (x)'(x)j dx + 0 jfn (x)'(x)j dx
0
R1
R1
= 0 jfn (x)j jg(x) '(x)j dx + 0 jfn (x)'(x)j dx

137

Schwartz

jfn (x)j dx
{z
1 by (b)

1=2

R1

1=2

R1
0

for all n

jg(x)

1=2

'(x)j dx

R1
0

jfn (x)'(x)j dx

N:

Thus, 0 fn (x)g(x)dx ! 0 for each g 2 L2 (0; 1):

1
407. Let fn (x) =
1=2 on (0; 1): Prove that:
jx n1 j
(a) fn (x) converges pointwise on (0; 1):
(b) fn (x) converges in measure on (0; 1):
(c) fn converges in L1 (0; 1):
(d) @ g 2 L1 (0; 1) : fn (x) g(x) for a.e. x 2 (0; 1) and for all n 2 N:

1
1
Proof. (a) Let x 2 (0; 1)nf m
: m 2 Ng: Then lim fn (x) = lim
= p1x :
1 1=2
n!1
n!1 jx n
j
1
1
If x = m
for some m 2 N; then for n m + 1; we also get that lim fn ( m
) = lim 1 11 1=2 = p1 1 :
n!1
n!1 j m n j
m
m
1
(b) Since m ((0; 1)) = 1 < 1; and fn (x) ! px pointwise in (0; 1); fn (x) ! p1x :
R1
R1
(c) We have 0 lim fn (x) dx = 0 p1x dx = 2:
n!1
h q
i1=n h q
i1
R1
R1
R 1=n 1
1
1
1
p
p
dx
+
dx
=
lim
2
x
+
2
x
Now, lim
f
(x)dx
=
lim
1
1
n
n
1=n
0 n
0
x n
n!1
n!1
n!1
0
1=n
n x
q
q
1
1
= lim 2 n + 2 1 n = 2:
n!1
R1
R1
Thus, 0 lim fn (x) dx = lim
f (x)dx and so, fn converges in L1 (0; 1):
0 n
n!1
n!1

a.e.
408. Let p 1: Suppose fn 2 Lp (R), supn jfn j 2 Lp (R); and fn ! f in R. Prove that f 2 Lp (R) and
that fn ! f in Lp (R):

p
Proof. Let g(x) = supn jfn (x)j : Then jfn (x)j
g(x)p for all x 2 R and for all n 2 N:
a.e.
p a.e.
p
Now, fn ! f in R =)
R jfn j p ! jf jR in R:
R
R
p
p
Fatous theorem =) R jf (x)j dxR = R lim jfn (x)jR dx lim R jfn (x)j dx
g(x)p dx < 1:
R
p
p
Now, Lebesgues DCT =) lim R jfn (x)j dx = R jf (x)j dx:
n!1

409. Let A R be measurable with m(A) < 1: Let ffn g be a sequence of functions in L1 (A) such that
R
m
1=2
fn ! 0 on A, and kfn kL1 (A) M for all n 2 N. Prove that lim A jfn gj dm = 0 for all g 2 L1 (A):
n!1

R 1=2
Proof. Let g 2 L1 (A) and suppose that A jfn gj dm 9 0:
R
1=2
Then 9 a subsequence A jfnk gj dm r > 0 for all k = 1; 2; 3; ::::
Schwartz R
R
R
1=2
1=2
1=2 R
1=2
1=2
dm = A fnkm
jgj dm
f
dm
jgj dm
:
Now, A fnkm g
A nkm
A
m
a.e.
Since fnk ! 0; 9 a subsequence fnkm ! 0 as m ! 1:
Let > 0 be given. Since m(A) < 1; Egorovs theorem =) 9B A : m(AnB) < 2M and fnkm
0:
B

Thus, 9N : fnkm (x) < 2m(B) 8x 2 B and 8m N:


R
Now, AnB fnkm dm
fnkm L1 (A) m(AnB) M 2M = 2 :
R
R
R
R
R
Thus, 8m N; A fnkm dm = B fnkm dm + AnB fnkm dm < B 2m(B) dm + AnB fnkm dm < 2 + 2 = :
R
R
1=2
1=2 R
1=2
1
Hence, lim
f
dm
=
0
and
since
g
2
L
(A);
lim
f
dm
jgj dm
= 0:
n
n
k
k
A
A
A
m
m
m!1
m!1
R
R
1=2
1=2
Thus, lim A fnkm g
dm = 0; a contradiction to A jfnk gj dm r > 0 for all k = 1; 2; 3; ::::
m!1
R
1=2
Therefore, lim A jfn gj dm = 0 for all g 2 L1 (A):
n!1

138

410. Let f be a measurable function on [0; 1] and let A = fx 2 [0; 1] : f (x) 2 Zg:
R1
2n
Prove that A is measurable and that lim 0 [cos( f (x))] dx = m(A):
n!1
[

1
1
Proof. A =
f fng ; and f fng = f 1 [n; 1) n f 1 (n; 1) is measurable since f is measurable.
n2Z

Thus, A is a countable union of measurable sets and so is measurable.


2n
2n
Now, let fn (x) = [cos( f (x))] : Then for x 2 A; f (x) = m 2 Z; and so fn (x) = [cos( f (x))]
2n
= [cos( m)] = 1; and for x 2
= A; f (x) 2
= Z and so 0 fn (x) < 1:
1 x2A
Hence, lim fn (x) =
= A (x):
0 x2
=A
n!1
R1
Now, jfn (x)j 1 for all n 2 N and for all x 2 [0; 1]: Also, 0 1dx = 1 < 1; and so by the Lebesgue DCT
R1
R1
R1
2n
2n
we get that lim 0 [cos( f (x))] dx = 0 lim [cos( f (x))] dx = 0 A (x)dx = m(A):
n!1
n!1

411. Show that if m(A) < 1; and 0 < p1 < p2 < 1; then Lp2 (A) Lp1 (A); and the inequality
kf kp1

kf kp2 (m(A)) p1

1
p2

holds for f 2 Lp2 (A):

Proof. Let p = pp21 > 1 and let p0 be such that p1 + p10 = 1: Then p10 = 1 pp12 :
1
1
R p0
R
R
p0
p
p1 p
p
1
dm
Then by Hlders inequality: A jf j 1 dm
jf
j
dm
:
A
A
1
R
R
1
p
p
Thus, A jf j 1 dm
jf j 2 dm p (m(A)) p0
A
1
1
1
R
R
R
1
1
1=p2
p
p
p
=) A jf j 1 dm p1
(m(A)) p1 p2 :
jf j 2 dm pp1 (m(A)) p0 p1 = A jf j 2 dm
A
1

Thus, if f 2 Lp2 (A); then kf kp1 kf kp2 (m(A)) p1 p2 and since m(A) < 1; f 2 Lp1 (A):
R
x
1
f (t)dt:
412. Given f 2 L2 ([0; 1]) dene K(x) = x4=3
0
Show that kKk1 C kf k2 where C is a constant independent of f:
R R R
R
1
x
1
1
1
1 x
Proof. kKk1 = 0 x4=3
f
(t)dt
dx
=
dx
1=3
x 0 f (t)dt
0
0 x
R1
R
1
1 x
dx
x 0 jf (t)j dt
0
x1=3
1
1
Schwartz R 1
R
2
2
2
2
1 1 Rx
1
dx
jf
(t)j
dt
dx
0 x1=3
0 x 0
1
1
R
R1
2
2
M ean Value Theorem
1 1
2
=
dx
jf
(c
)j
dx
; for some 0 < cx < x
x
0 x2=3
0
1
1
R1
R1
2
2
2
2
Now, 0 jf (cx )j dx
jf (x)j dx
= kf k2 :
0
1
p
R1 1
2
Also, 0 x2=3 dx
= 3:
1
1
1
p R1
p
R1 1
R1
2
2
2
2
2
Hence, kKk1
dx
jf
(c
)j
dx
3
jf
(x)j
dx
= 3 kf k2 :
x
2=3
0 x
0
0

413. Recall that the convolution of two integrable functions f and g is dened by:
(f
Let f

0 such that

g) (x) =

f (x

y)g(y)dy:

f (x)dx = A < 1: Dene the sequence fn = f


|

{z

n tim es

f:
}

Prove that fn 2 L1 (R) for each n 2 N, and fn ! 0 in L1 (R):

R R R Fubini
R R
Proof. Note that R (f f ) (x)dx = R R f (x y)f (y)dydx = R f (y) R f (x y)dx dy
R
R
R
u=x y R
=
f
(y)
f
(u)du
dy
=
f
(u)du
f (y)dy = A2 :
R
R
R
R
139

R
We claim thatR R fn (x)dx = An : To prove it, we proceed by induction.
Suppose that R fn 1 (x)dx = An 1 :
R
R
R
as ab ove R
Then R fn (x)dx = R (fn 1 f )(x)dx =
f
(x)dx
f (x)dx = An 1 A = An < 1 < 1:
R n 1
R
1
Thus, fn 2 L (R) for each n 2 N:
R
R
f 0
0 A<1
Now, lim R jfn (x)j dx = lim R fn (x)dx = lim An = 0: Hence, fn ! 0 in L1 (R):
n!1
n!1
n!1

m
414. Let ffn g be a sequence of measurable functions on A, m(A) < 1; such that fn ! f on A,
and jfnR (x)j C for xR2 A, and n = 1; 2; 3; :::: Show that if g is continuous on [ C; C]; then
lim A g(fn )dm = A g(f )dm:
n!1

Proof. Let > 0 be given. Since g is continuous on a compact set [ C; C]; g is uniformly continuous
on [ C; C]: Thus, 9 > 0 : whenever x; y 2 [ C; C]; and jx yj < =) jg(x) g(y)j < :
m
We now show that g(fn ) ! g(f ) on A.
Note that fx 2 A : jg(fn ) g(f )j
g fx 2 A : jfn f j
g; and consequently,
0 m(fx 2 A : jg(fn ) g(f )j
g) m(fx 2 A : jfn f j
g) ! 0 as n ! 1:
m
Thus, lim m(fx 2 A : jg(fn ) g(f )j
g) = 0 and so g(fn ) ! g(f ) on A.
n!1

Now, jg(fn )j

M=

max g(x) for all n = 1; 2; 3; ::: and for all x 2 A:


R
Also, since m(A) < 1; we see that A M dm = M m(A) < 1:
R
R
Thus, by the Lebesgue DCT in measure, problem 42, we get that lim A g(fn )dm = A g(f )dm:
n!1

m
415. Let ffnRg be a sequence Rof measurable functions on A, m(A) < 1; such that fn ! f on A.
Then lim A sin(fn )dm = A sin(f )dm:
n!1

m
Proof. As in the previous problem, we get that sin(fn ) R! sin(f ) on A.
Now, jsin(fn )j 1 for all n and for all x 2 A: Note that A 1dm = m(A)R < 1:
R
Thus, by the Lebesgue DCT in measure, problem 42, we get that lim A sin(fn )dm = A sin(f )dm:
n!1

416. Suppose that g satises a Lipschitz condition on R; i.e., 9C > 0 : 8x; y 2 R; jg(x) g(y)j C jx yj :
Show that if f is Lebesgue integrable on [a; b]; then g(f ) is Lebesgue integrable on [a; b]:
R
Proof. Let > 0 be given. Since f 2 L1 ([a; b]); 9 a continuous function ' such that [a;b] jf 'j dm < :
R
R
R
R
Thus, [a;b] jg(f )j dm = [a;b] jg(f ) g(') + g(')j dm
jg(f ) g(')j dm + [a;b] jg(')j dm
[a;b]
R
R
g ' is continuous
C [a;b] jf 'j dm + [a;b] jg(')j dm
<
C (b a) + M (b a) < 1; where M = max jg(')j :
[a;b]
R
Hence, [a;b] jg(f )j dm < 1 and g(f ) is Lebesgue integrable on [a; b]:

417. Suppose that g satises a Lipschitz condition on R; i.e., 9C > 0 : 8x; y 2 R; jg(x) g(y)j C jx yj :
m
Show that if ffn g is a sequence of measurable functions on [a; b] such that fn ! f and there is a Lebesgue
R
R
integrable function G such that jfn (x)j G(x); then lim
g(fn )dm = [a;b] g(f )dm:
[a;b]
n!1

Proof. Let > 0 be given. Then fx 2 [a; b] : jg(fn ) g(f )j


g fx 2 [a; b] : jfn f j C g; and
consequently, 0 m(fx 2 [a; b] : jg(fn ) g(f )j
g) m(fx 2 [a; b] : jfn f j C g) ! 0 as n ! 1:
m
Thus, g(fn ) ! g(f ) on [a; b].
Now, jg(fn ) g(G)j C jfn Gj 2C jGj and so jg(fn )j 2C jGj + jg(G)j for all n and for all x 2 [a; b]:
We have shown, in problem 70, that g(G) is Lebesgue integrable on [a; b]:
Hence, 2C jGj + jg(G)j is Lebesgue integrable on [a; b]:
R
R
Thus, by the Lebesgue DCT in measure, problem 42, we get that lim
g(f
)dm
=
g(f )dm:
n
[a;b]
[a;b]
n!1

R
R
p
p
418. Let ffn g be a sequence ofR functions converging to f on A such that RA jfn j dm < 1R and A jf j dm < 1;
p
p
p
1 p < 1: Show that lim A jfn f j dm = 0 if and only if lim A jfn j dm = A jf j dm:
n!1
n!1

x2[ C;C]

140

Proof. (=)) Suppose that lim

n!1

kfn kp = kfn
Thus, 0

f + f kp

kfn kp

Hence, lim kfn kp


n!1

kfn

kf kp

jfn

f j dm = 0: Then by Minkowskis inequality, we get that

f kp + kf kp and kf kp = kf

fn + fn kp

kfn

f kp + kfn kp :

kfn

kf kp

f kp ! 0 as n ! 1:
R
R
p
p
= 0 and so lim A jfn j dm = A jf j dm:
n!1

R
R
p
p
((=) We prove it rst for m(A) < 1: Suppose that lim A jfn j dm = A jf j dm and let > 0 be given.
n!1
R
R
p
p
We can nd N1 large so that A jfn j dm < A jf j dm + 2p for all n N1 :
p
Since jf j is Lebesgue integrable on A, we
R can puse problem 1c to nd > 0 such that whenever B A is
measurable and m(B) < ; we have that B jf j dm < 2p+1 :
Now, Egorovs theorem implies that we can nd a measurable set C A with m(AnC) < such that fn
f:
C

1=p

Thus, we can nd N2 large so that for all x 2 C; we have that jfn (x) f (x)j < m(C)1=p for all n N2 :
Hence, for n N = maxfN1 ; N2 g; we have
R
R
R
R
R
R
p
p
p
p
p
p
p
jf
f
j
dm
=
jf
f
j
dm+
jf
f
j
dm
jf
f
j
dm+2
jf
j
dm
+
jf j dm
n
n
n
n
AnC
C
AnC
AnC n
A
C
R
R
R
R
R
p
p
p
p
p
jf
f j dm + 2p AnC jf j dm + AnC jf j dm + 2p
= C jfn f j dm + 2p 2 AnC jf j dm + 2p
C n
R
R
R
R
p
p
p
p
jf
f j dm + 2p+1 AnC jf j dm +
jf
f j dm + 2p+1 2p+1 +
jf
f j dm + 2
C n
C n
C n
p
R
R
1=p
p
dm + 2
+ 2 = 3 : Thus, lim A jfn f j dm = 0:
C m(C)1=p
n!1
R
R
p
p
If m(A) = 1; then, by problem 1, 9 a measurable
R set B p A; m(B) < 1 so that A jf j dm < B jf j dm + :
Then we proceed as above to show that lim A jfn f j dm = 0:
n!1

a.e.
419. Let ffn g be a sequence of functions in Lp [a; b]; 1 < p < 1; such that fn ! f on [a; b]:
Suppose that 9C > 0 : kfn kp C for n = 1; 2; 3; :::: Prove that 8g 2 Lq [a; b]; with p1 + 1q = 1;
R
R
lim
fn gdm = [a;b] f gdm:
[a;b]
n!1

kfn kp C
R
R
p
p
Proof. By Fatous theorem, [a;b] jf j dm lim [a;b] jfn j dm
C p : Thus, f 2 Lp [a; b]
R
R
p
p
Let us show that lim
jfn j dm = [a;b] jf j dm:
[a;b]
n!1
R
p
This is equivalent, by problem 72, to proving that lim
jf
f j dm = 0:
[a;b] n
So let

> 0 be given. Then for

n!1

> 0; we have the following:


Hlder R
R
p
=) 8 n 2 N, B jfn j dm
jf j dm
B n
Cq

1=p R
1=q
whenever B [a; b]; and m(B) < 1
1q dm
B
q
= kfn kp (m(B))1=q C( C q )1=q = :
R
Also, we can nd 2 > 0 :whenever B [a; b]; and m(B) < 2 ; we have that B jfRj dm < : R
Let = minf 1 ; 2 g: Then whenever B [a; b]; and m(B) < ; we have that 8n; B jfn j dm; B jf j dm < :
Now, Egorovs theorem =) 9A [a; b]; m([a; b]nA) < : fnp
f p:
A

1=p

Thus, 9N : jfn (x) f (x)j < (m(A))1=p for all x 2 A; and for all n N:
Hence,
for n N , weR have
R
R
p
p
p
jf
f j dm = A jfn f j dm + [a;b]nA jfn f j dm
n
[a;b]
p
R
R
R
1=p
p
p
dm + 2p [a;b]nA jfn j dm + [a;b]nA jf j dm
1=p
A (m(A))
m([a;b]nA)< R
dm + 2p ( + )
A m(A)
(2p+1 + 1) :
R
R
R
p
p
p
Thus, lim [a;b] jfn f j dm = 0 and so, by problem 72, lim
jfn j dm = [a;b] jf j dm:
[a;b]
n!1
n!1
R
R
Similarly, to show that lim
fn gdm = [a;b] f gdm 8g 2 Lq [a; b]; we show that
[a;b]
n!1
R
lim [a;b] jfn g f gj dm = 0 and then use the result in problem 72.
n!1

141

R
R
(Note that, by Hlders inequality, [a;b] fn gdm < 1 and [a;b] f gdm < 1):
1=p R
1=q n!1
Hlder R
R
R
p
q
Now, 0
jf
g
f
gj
dm
=
jf
f
j
jgj
dm
jf
f
j
dm
jgj
dm
! 0:
n
n
n
[a;b]
[a;b]
[a;b]
[a;b]
R
Thus, lim [a;b] jfn g f gj dm = 0 and we are done by problem 72.
n!1

420. Let ffn g be a sequence of functions in L2 (R): Suppose that kfn kL2 (R) M for all n 2 N; and
R
R
a.e.
fn ! f on R: Prove that lim R fn gdm = R f gdm for all g 2 L2 (R):
n!1

Proof. Let us do it rst when


be exactly problem 73
R A R is Rsuch that m(A) < 1: 2Note that this would
2
with p = q = 2: Thus, lim A fn gdm = A f gdm for all g 2 L (R): (Here, g 2 L (R) is not a typo, why?).
[ n!1
Now, write R =
[m; m + 1]; and let g 2 L2 (R):
m2Z
!
XR
R
R
m and n are indep endent X
Then lim R fn gdm = lim
f gdm
=
lim [m;m+1] fn gdm
[m;m+1] n
n!1

X R

[m;m+1]

m2Z

n!1

f gdm =

m2Z

m2Z

n!1

f gdm:

421. Let ffn g be a sequence of functions in Lp [a; b]; 1 p < 1; which converges in norm to f 2 Lp [a; b];
a.e.
and let fgn g be a sequence of measurable functions such that jgn j C for n = 1; 2; 3; :::; and gn ! g:
Show that fn gn ! f g in Lp [a; b]:
R
R p
p
p
p
Proof. Note that for all n; jf gn j
C p jf j and [a;b] C p jf j dm = C p [a;b] jf j dm < 1:
R
R
a.e.
Moreover, f p gnp ! f p g p on [a; b] and so, by the Lebesgue DCT, lim
f p gnp dm = [a;b] f p g p dm:
[a;b]
n!1
R
p
Hence, by problem 72, we have that lim [a;b] jf gn f gj dm = 0.
n!1
1=p
1=p
R
R
p
p
= [a;b] jfn gn f gn + f gn f gj dm
Thus, we have [a;b] jfn gn f gj dm
1=p
1=p
Hlder R
R
p
p
jf
g
f
g
j
dm
+
jf
g
f
gj
dm
n
n
n
n
[a;b]
[a;b]
1=p
1=p
R
R
p
p
p
p
= [a;b] jgn j jfn f j dm
+ [a;b] jf j jgn gj dm
1=p
1=p
R
R
p
p
p
C p [a;b] jfn f j dm
+ [a;b] jf j jgn gj dm
! 0 as n ! 1:
R
R
R
p
p p
Thus, lim [a;b] jfn gn f gj dm = 0; and so, by problem 72, lim
f
g
dm
=
f p g p dm:
n
n
[a;b]
[a;b]
n!1

n!1

Therefore, fn gn ! f g in Lp [a; b]:

a.e.
422. Let f; fn : R ! R be measurable functions such that fn ! f on R and there exists a function g
integrable on R such that jfn (x)j g(x) a.e. for all n 2 N. Show that fn ! f almost uniformly on R:
(That is, 8 > 0; 9 a measurable set A R : m(RnA) < and fn
f ):
A
R R
Proof. Since jfn (x)j g(x) a.e. for all n 2 N; R jfn j dmR
gdm < 1R a.e. for all Rn 2 N. Thus, each fn
R
is Lebesgue integrable on R: Now, Fatous theorem =) R jf j dm lim R jfn j dm
gdmR< 1.
R
R
Thus, f is Lebesgue integrable on R: By the Lebesgue DCT, we get that lim R jfn j dm = R jf j dm:
n!1
R
Thus, by problem 72, we get that lim R jfn f j dm = 0:
R n!1
Let > 0 be given. Then 9N : R jfn f j dm < 2 for all n N:
Now, by Tchebyshevs inequality, problem 59, we get that if B = fx 2 R : jfn (x) f (x)j > g; then
R
2
m(B) < 1 R jfn f j dm <
= ; and so if A = RnB; then m(RnA) = m(B) < ; and on A, we have that
jfn (x) f (x)j
for all n N: This means that fn
f and so we are done.
A

423. Suppose that f is nonnegative and measurable


on a set A, where
R
R 20 < m(A) < 1: Suppose also that
1
1
there are positive and such that m(A)
f
dm
and
:
m(A) A f dm
A
142

Show that if 0 < < 1 and A = fx 2 A : f (x)


g; then m(A ) m(A)(1
)2 :

R
R 2
R 2
2 Schwartz R
2
2
Proof. A f dm
f
dm
1
dm
=
f dm m(A) ( m(A)) m(A) = (m(A)) < 1:
A
A
A
Thus, f is Lebesgue integrable on A:
2 Schwartz R
R
R
R 2
2
2
Now, A f dm
f
dm
1
dm
f dm (m(A ))
m(A)m(A ):
A
A
A
R
On the other hand, AnA f dm
m(AnA )
m(A):
R
R
R
R
Thus, m(A)
f dm = AnA f dm + A f dm
m(A) + A f dm:
A
2
R
R
2
2
Hence, A f dm
m(A)
m(A) = (1
)m(A) and so A f dm
(1
)2 (m(A)) :
2
R
2
Therefore, 2 (1
)2 (m(A))
f
dm
m(A)m(A ):
A
2

This immediately implies that m(A ) m(A)(1


)2 :

424. We say that a measurable function f on a set A is essentially bounded if m(fx 2 A : jf (x)j > rg) = 0
for some real number r: In this case, we dene the essential supremum of f by:
kf k1 = inffr : m(fx 2 A : jf (x)j > rg) = 0g:
1=p
R
p
Show that if f is essentially bounded on [a; b]; then lim
jf j dm
= kf k1 :
[a;b]
p!1

1=p
R
p
Proof. If kf k1 = 0; then f = 0 a.e. in [a; b] and so lim
jf j dm
= 0 = kf k1 :
[a;b]
p!1

If kf k1 = r > 0; then let B = fx 2 [a; b] : jf (x)j > rg: Then m(B) = 0 and on [a; b]nB; jf j r:
1=p
1=p m(B)=0 R
1=p
R
R
R
R
p
p
p
p
Thus, [a;b] jf j dm
= [a;b]nB jf j dm + B jf j dm
=
jf j dm
rp dm
[a;b]nB
[a;b]nB
1=p
R
1=p m(B)=0
1=p
1=p
= r [a;b]nB dm
= kf k1 (m([a; b]nB))
= kf k1 (m([a; b]))
= kf k1 (b a) :
1=p
R
p
1=p
jf
j
dm
lim kf k1 (b a)
= kf k1 :
Thus, lim
[a;b]

1=p

p!1

p!1

Now, given any 0 < < r; we have, by denition of r; that m(fx 2 A : jf (x)j > r
1=p
1=p
R
R
R
p
p
Thus, [a;b] jf j dm
jf j dm
(r
fx2A:jf (x)j>r g
fx2A:jf (x)j>r g
1=p
R
p
jf j dm
r
= kf k1
:
and so lim
[a;b]

g) =
p

) dm

> 0:

1=p

= (r

1=p

p!1

Therefore, Letting

! 0; we get that lim

p!1

Thus, kf k1

lim
p!1

[a;b]

jf j dm

1=p

lim

p!1

1=p

[a;b]

jf j dm
p

[a;b]

jf j dm

kf k1 :

1=p

kf k1 :

1=p
R
p
Hence, lim
jf j dm
= kf k1 :
[a;b]
p!1

425. Prove the following Hlders Inequality: If 1 p 1 and p1 + p10 = 1; then kf gk1 kf kp kgkp0 ;
1=p0
R
R
R
R
1=p R
p
p0
that is, A jf gj dm
jf
j
dm
jgj
dm
; and A jf gj dm kf k1 A jgj dm:
A
A
0
p
p
Proof. Let f1 (x) = jf j (x) and f2 (x) = jgj (x): Then Hlders inequality, problem 92, implies that
0
R 1=p
R
R
1=p
1=p0
1=p
1=p
1=p0
f
(x)f
(x)dx
f
(x)dx
f
(x)dx
:
But,
f
=
jf
j
;
and
f
= jgj :
1
2
1
2
1
2
A
A
A
1=p0
R
R
R
0
1=p
p
p
Thus, A jf gj dm
jf j dm
jgj dm
:
A
A
R
R
Now, a.e. on A, we have jf gj kf k1 jgj ; and so A jf gj dm kf k1 A jgj dm:

426. (a) Prove the following Minkowskis Inequality: If 1 p 1; then kf + gkp kf kp + kgkp ;
R
R
R
1=p
1=p
1=p
p
p
p
that is, A jf + gj dm
jf j dm
+ A jgj dm
; and kf + gk1 kf k1 + kgk1 :
A

143

(b) Show that Minkowskis Inequality fails for 0 < p < 1:

Proof. (a) We consider 3 cases:


(CaseR i) p = 1:
R
R
R
Then A jf + gj dm
(jf j + jgj) dm = A jf j dm + A jgj dm:
A
(Case ii) p = 1:
Then we have jf j kf k1 and jgj kgk1 a.e. on A. Thus, jf + gj jf j + jgj kf k1 + kgk1 a.e. on A,
and consequently, kf + gk1 kf k1 + kgk1 :
(Case iii) 1 < p < 1: R
R
R
R
p
p
p 1
p 1
p 1
Then we have kf + gkp = A jf + gj dm = A jf + gj
jf + gj dm
jf + gj
jf j dm+ A jf + gj
jgj dm:
A
p 1
p
0
In the last integral, apply Hlders inequality to jf + gj
and jgj with exponents p = p 1 and p; respectively.
R
R
p 1
p 1
p 1
p 1
This gives A jf + gj
jgj dm kf + gkp kgkp : Similarly, A jf + gj
jf j dm kf + gkp kf kp :
p
p 1
p 1
p 1
Thus, kf + gkp kf + gkp kf kp + kf + gkp kgkp = kf + gkp (kf kp + kgkp ):
p 1
Dividing both sides by kf + gkp ; which we can assume to be 6= 0 and 6= 1; we get that
kf + gkp kf kp + kgkp :
(b) Let A = (0; 1); f (x) = (0;1=2) (x); and g(x) = (1=2;1) (x):
Then kf + gkp = 1; but kf kp + kgkp = 2 1=p + 2 1=p = 21 (1=p) < 1:

1=p
R1
427. Let f : [0; 1] ! R be positive and continuous. For p 6= 0; let N (p) = 0 f p (x)dx
:
(a) Compute lim N (p):
p!1

(b) Compute lim N (p):


p! 1

(c) Compute lim N (p):


p!0

f is continuous
Solution. (a) By problem 78, lim N (p) = kf k1
=
max jf (x)j :
p!1

(b) For p < 0; p =


= lim

jpj!1

R1
0

jpj : Thus, lim N (p) = lim

( f1 )jpj dx

p! 1

1=jpj

(c) lim ln(N (p)) = lim ln


p!0

= lim

p!0

R
( 1 f p (x)dx)
R 10
f p (x)dx
0

because f p (x) ln(f (x))

R1

why?

= lim

p!0

p!0

1
1=jpj
( 1 )jpj dx)
0 f

f p (x)dx

1=p

R1

p! 1

R1

(
jpj!1
lim

p!0

d
dp

x2[0;1]

R1

ln(f (x)); and f p (x)


lim (ln N (p))

k f1 k1

1=p

f is continuous

1
max
x2[0;1] f (x)

R
ln( 01 f p (x)dx) LHopital
=
lim
p
p!0
p!0
R
( 01 f Rp (x) ln(f (x))dx)
= lim
1 p
f (x)dx
p!0
0

= lim

d
f p (x)dx
0 dp
R1
p
f (x)dx
0

[0;1]

f p (x)dx

p!0

1:

[0;1]
R1
ln(f (x))dx
0

d
dp

= min jf (x)j :
x2[0;1]

ln

R1
0

R1
0

f p (x)dx

ln(f (x))dx;

Hence, lim N (p) = lim eln N (p) = ep!0


=e
:
p!0
p!0

Rb 2
1
428. Suppose f 2 C ([a; b]); f (a) = f (b) = 0; and a f (x)dx = 1:
Rb
(a) Show that a xf (x)f 0 (x)dx = 12 :
Rb 0
Rb 2 2
2
(b) Show that 41
(f (x)) dx
x f (x)dx :
a
a
R R R
integration by parts
b
b
b
b
Proof. (a) a xf (x)f 0 (x)dx = a xf (x)d(f (x))
=
xf 2 (x) a
f (x)d(xf (x))
a
Rb
R
R
Rb
f (a)=f (b)=0
b
b
=
0
f (x) (f (x) + xf 0 (x)) dx =
f 2 (x)dx
xf (x)f 0 (x)dx = 1
xf (x)f 0 (x)dx:
a
a
a
a
Rb
R
b
Thus, 2 a xf (x)f 0 (x)dx = 1 and so a xf (x)f 0 (x)dx = 12 :
2 Schwartz R b
Rb
Rb 0
2
2
(b) 41 = ( 12 )2 = a xf (x)f 0 (x)dx
(xf (x)) dx
(f (x)) dx :
a
a

144

429. (a) Find min

R1

f 2A 0

(1 + x2 )f 2 (x)dx; where A = ff 2 C([0; 1]) :

R1
0

f (x)dx = 1g; and nd a function for

which the minimum is attained.


R1
R1
R1
(b) Let A = ff 2 C([0; 1]) : 0 f (x)dx = 3; and 0 xf (x)dx = 2g: Find min 0 f 2 (x)dx; and a function for
f 2A

which the minimum is attained.

Hlder R 1
R1 p
R1
R1 1
1
Proof. (a) We have 1 = 0 f (x)dx = 0
1 + x2 f (x) p1+x
dx
(1 + x2 )f 2 (x)dx
dx
2
0
0 1+x2
R1
R
1
= 0 (1 + x2 )f 2 (x)dx 4 = 4 0 (1 + x2 )f 2 (x)dx:
R1
4
Thus, 0 (1 + x2 )f 2 (x)dx
:
R
R1 1
R1
2R1
1
4 1
1
Let f (x) = 1+x2 : Then 0 f (x)dx = 4 0 1+x
(1 + x2 )f 2 (x)dx = 4
dx = 4 :
2 dx = 1; and
0
0 1+x2

2 Schwartz R 1
R1
R1 2
R1 2
1
2
(b) We have 4 = 0 xf (x)dx
f
(x)dx
x
dx
=
f (x)dx :
3
0
0
0
R1 2
Thus, 0 f (x)dx 12: Equality would hold if f (x) = cx:
R1
R1
Now, we solve for c: 2 = 0 xf (x)dx = c 0 x2 dx = 3c =) c = 6 =) f (x) = 6x:
R1
R1
Note that f 2 A since 0 f (x)dx = 0 6xdx = 3:
R1
We have to admit that this was a lucky solution since we did not use the fact that 0 f (x)dx = 3.
For the real solution, we go as follows:
2
2
R1
R1
R1
2
(x + a)f (x)dx = 0 xf (x)dx + a 0 f (x)dx = (2 + 3a) :
0
2 Schwartz R 1
R1 2
R1
R1 2
2
Thus, (2 + 3a) = 0 (x + a)f (x)dx
(x + a)2 dx
f (x)dx = a2 + a + 13
f (x)dx :
0
0
0
R1 2
2
2
(2+3a)
attains its maximum value at g(0) = 12:
and g(a) = a(2+3a)
Hence, 0 f (x)dx
( 2 +a+ 13 )
(a2 +a+ 13 )
R1
Thus, 0 f 2 (x)dx
12: The rest is as above.

430. Show that if f is continuous on [0; 1]; dierentiable on (0; 1); f (0) = 0; and 0 < f 0 (x) 1 on (0; 1);
2
R1
R1
3
then 0 f (x)dx
(f (x)) dx: Show also that equality occurs if and only if f (x) = x:
0
R R
2
x
x
3
Proof. For 0 < x < 1; let g(x) = 0 f (t)dt
(f (t)) dt: h
0
i
Rx
Rx
3
2
Then g(0) = 0; and g 0 (x) = 2f (x) 0 f (t)dt
(f (x)) = f (x) 2 0 f (t)dt
(f (x)) :

Since f (0) = 0 and R0 < f 0 (x) 1 on (0; 1); f is strictly increasing on (0; 1) and so f (x) > 0 for x 2 (0; 1):
x
2
Now, let h(x) = 2 0 f (t)dt
(f (x)) : Then h(0) = 0; and h0 (x) = 2f (x) [1 f 0 (x)] 0:
Thus, h(x) is increasing and so h(x) 0 on [0; 1]:
Hence, g 0 (x) 0 and so g is increasing. In particular, g(1) 0:
2
2
R1
R1
R1
R1
3
3
f
(t)dt
(f (t)) dt :
Thus, g(1) = 0 f (t)dt
(f
(t))
dt
0;
and
so,
0
0
0
f (x)>0

Equality will occur when h(x) = 0 =) h0 (x) = 0 =) 2f (x) [1 f 0 (x)] = 0 =) f 0 (x) = 1 =) f (x) = x:

R b f(a)+f
(b)
431. Show that if f is a convex function on [a; b]; then f a+b
(b
a)
f
(x)dx
(b a):
2
2
a

Proof. Using the substitution x = (1 t)a + tb for t 2 [0; 1]; we get that t = xb aa and so,
Rb
R1
f (x)dx = 0 f ((1 t)a + tb)(b a)dt
a
convexity
R1
(b a) 0 [(1 t)f (a) + tf (b)] dt
R1
= (b a) 0 [f (a) + t(f (b) f (a))] dt
= (b a) f (a) + 12 (f (b) f (a))
(b)
= f (a)+f
(b a):
2
For the other inequality, we use the substitution x = a+b
2 + t and we get
Rb
R b 2 a a+b
f (x)dx = a b f ( 2 + t)dt
a
2

145

=
=
=

R0
a

2
b a
2

f ( a+b
2 + t)dt +
f ( a+b
2

a
2

=2

a
2

0
convexity

1
a+b
2f( 2

t)

a
2

f ( a+b
2 + t)dt

2
f ( a+b
2 + t)dt
0
a+b
+ f ( 2 + t) dt
t) + 12 f ( a+b
2 + t) dt

t)dt +

f ( a+b
2

f ( a+b
2 ) dt
a+b
= f 2 (b a):
R
b
432. Show that if f 2 C([a; b]) is positive and strictly concave on [a; b]; then a f (x)dx > 21 (b a) max f (x):
x2[a;b]

Proof. Let c 2 [a; b] be such that f (c) = max f (x):


x2[a;b]
Rc
Rb
R1
R1
Rb
Then a f (x)dx = a f (x)dx + c f (x)dx = 0 f ((1 t)a + tc)(c a)dt + 0 f ((1 t)c + tb)(b c)dt
R1
R1
= 0 f ((1 t)a + tc)(c a)dt + 0 f ((1 t)c + tb)(b c)dt
R1
R1
concavity
>
(c a) 0 ((1 t)f (a) + tf (c)) dt + (b c) 0 ((1 t)f (c) + tf (b)) dt
2

f (a)+f (c)
+ (b
2
b a
c a
= 2 f (c) + 2 f (a)
f is p ositive
b a
>
f (c):
2

= (c

a)

f (c)+f (b)
2
b c
f
(b)
2

c)

433. Prove the following Opials inequality: If f is continuously dierentiable on [0; a] and f (0) = 0; then
Ra
Ra
2
jf (x)f 0 (x)j dx a2 0 (f 0 (x)) dx: Show also that the constant a2 is the best possible.
0
R
x
Proof. For x 2 [0; a]; dene h(x) = 0 jf 0 (t)j dt: Then h0 (x) = jf 0 (x)j :
Rx 0
Rx 0
Also, since f (0) = 0, jf (x)j = 0 f (t)dt
jf (t)j dt = h(x):
0
Ra
Ra
Thus, jf (x)f 0 (x)j h(x)h0 (x); and so 0 jf (x)f 0 (x)j dx
h(x)h0 (x)dx:
0
Ra
R
R
Ra 0
Ra
by parts
2
a
a
a
0
Now, 0 h(x)h0 (x)dx = 0 h(x)dh(x) =
h2 (x) 0
h(x)h
(x)dx
=
jf
(t)j
dt
h(x)h0 (x)dx:
0
0
0
Ra
Ra
Ra 2
Ra
2 Schwartz 1 R a
2
2
Hence, 0 h(x)h0 (x)dx = 12 0 jf 0 (t)j dt
jf 0 (t)j dt
1 dt = a2 0 (f 0 (x)) dx:
2
0
0
Ra
R
R
a
a
2
Therefore, 0 jf (x)f 0 (x)j dx
h(x)h0 (x)dx a2 0 (f 0 (x)) dx:
0
To show
constantR a2 is the best possible,
we pick f (x) = cx:
R a that the
Ra
a
0
Then 0 jf (x)f (x)j dx = 0 c2 x dx = c2 0 xdx = 12 a2 c2 :
Ra
Ra 2
Ra
2
Also, a2 0 (f 0 (x)) dx = a2 0 (c) dx = 12 a2 c2 = 0 jf (x)f 0 (x)j dx:

434. Let f 0 be measurable on [0; 1] and f n g be a sequence of positive numbers such that n # 0 as n ! 1:
R1
R1
Prove that 0 f (x)dx < 1 if and only if lim n f (x)dx < 1:
n!1

0
if 0 x < n
Proof. For x 2 [0; 1] and for n 2 N; dene fn (x) = [ n ;1] (x)f (x) =
:
f (x)
if n x 1
R1
R1
Note that since f 0; fn (x) % f (x) as n ! 1: Also, n f (x)dx = 0 fn (x)dx:
R1
(=)) Suppose that 0 f (x)dx < 1: Then by the Lebesgue monotone convergence theorem, we get that
R1
R1
R1
R1
lim n f (x)dx = lim
fn (x)dx = 0 lim fn (x) dx = 0 f (x)dx < 1:
0
n!1
n!1
n!1
R1
((=) Suppose that lim n f (x)dx < 1: Then By Fatous theorem, we get that
n!1
R1
R1
R1
R1
f
(x)dx
=
(limf
lim 0 fn (x)dx = lim n f (x)dx < 1:
n (x)) dx
0
0
n!1

435. Assume that f 2 C 1 ([0; 1)); f 0 is monotonic on [0; 1); and lim f (x) = l < 1:
x!1
R 1 f (bx) f (ax)
b
Show that for 0 < a < b; 0
dx = (l f (0)) ln a :
x

146

Proof.

R1

0
lim f (x)=l R

x!1

f (bx) f (ax)
dx
x
b

R1 Rb
0

f 0 (yx)dy dx

f 0 is m onotonic

Rb R1
a

f 0 (yx)dx dy =

Rb

1
a y

([f (yx)]0 ) dy

1
(l f (0)) dy = (l f (0)) [ln jyj]a = (l f (0)) ln ab :
=
a y
R
bx
ax
1
436. (a) Find 0 e x e dx; 0 < a < b:
R1
(b) Find 0 cos(bx)x2cos(ax) dx; 0 < a < b:
R
R R R
R1
R b1 b
b
b
1
1 e bx e ax
xy
xy
Solution. (a) 0
dx = 0
e
dy dx =
e
dx dy =
dy = ln a :
x
a
a
0
a y
R 1 cos(bx) cos(ax)
R1 1 Rb
R b R 1 sin(yx)
Rb
(b) 0
dx = 0 x a sin(yx)dy dx =
dx dy =
dy = 2 (b a):
x2
x
a
0
a 2
r

2
R1p
R1
437. (a) Show that if f 2 C[0; 1] and jf (x)j 1 8x 2 [0; 1]; then 0 1 f 2 (x)dx
1
f
(x)dx
:
0

1=2
R1
R1
1=2 R 1
2
(b) Show that if f 2 C 1 [0; 1) and lim f (x) = 0; then 0 f 2 (x)dx 2 0 x2 f 2 (x)dx
(f 0 (x)) dx
:
0
x!1

2 Schwartz R 1
R1
R1 2
R1p
Proof. (a) We have 0 1 f 2 (x)dx
1 f 2 (x)dx ( 0 12 dx) = 1
f (x)dx :
0
0
r
2
R1
R1p
1
f (x)dx :
Thus, 0 1 f 2 (x)dx
0
R1
R1
R1 2 2
R1 2
(b) If 0 f 2 (x)dx = 1; then 0 x2 f 2 (x)dx
x f (x)dx
f (x)dx = 1; and f 0 (x) 6= 0 a.e. in R:
1
1
1=2
R1
R1
R1
1=2 R 1
2
2
Thus, 0 x2 f 2 (x)dx = 1; and 0 (f 0 (x)) dx > 0: Hence, 2 0 x2 f 2 (x)dx
(f 0 (x)) dx
= 1:
0
R1 2
2
So, suppose that 0 f (x)dx = A < 1: Then, since lim f (x) = 0; we get that lim xf (x) = 0:
x!1
x!1
R1
R1
R1
by parts
1 R1
2
0
f (x)d (xf (x)) = 0 0 f (x) (f (x) + xf 0 (x)) dx
Now, 0 xf (x)f (x)dx = 0 xf (x)df (x) =
xf (x) 0
0
R1 2
R1
=
f (x)dx
xf (x)f 0 (x)dx:
0R
0
R1
R1
R1
R1
1 2
Thus, 0 f (x)dx = 2 0 xf (x)f 0 (x)dx; and so 0 f 2 (x)dx = 0 f 2 (x)dx = 2 0 xf (x)f 0 (x)dx
1=2
Hlder
R1
R1
1=2 R 1
2
2 0 jxf (x)j jf 0 (x)j dx
2 0 x2 f 2 (x)dx
(f 0 (x)) dx
:
0

438. Prove the following Hlder inequality: If f1 ; f2 ; :::; fn are nonegative


! andnRiemann integrable on [a; b];
n
n
X
Y Rb
Rb Y
i
f (x)dx
:
and if 1 ; 2 ; :::; n > 0 are such that
fi i (x) dx
i = 1; then a
a i
i=1

i=1

i=1

n
n
X
Y
Proof. Let us rst show that if x1 ; x2 ; :::; xn > 0; and
=
1,
then
xi i
i
1 x1 +
n xn :
i=1
i=1
!
n
Y
log is concave
Note that log
xi i = 1 (log x1 ) +
log ( 1 x1 +
n (log xn )
n xn ) :
i=1

Thus, since log is increasing,


Now, assume WLOG that

Rb
a

n
Y

xi i

1 x1

n xn :

i=1

fi (x)dx > 0 for 1

i (x)

n; and let xi = R b f i(x)dx i = R b ffi (x)


(a i
)
!
! a i (x)dx
n
Rb X
i
fi (x)
dx
dx:
i R b f (x)dx
a

n
Rb Y
f (x)
Rb i
Then xi > 0 for each i; and so a
f (x)dx
a i
a i
i=1
i=1
!
n
n
n
X
X
Rb X
R b fi (x)
fi (x)
Rb
But, a
dx =
dx
=
i R b f (x)dx
i
i = 1:
a
f (x)dx
a i
a i
i=1
i=1
i=1
!
!
n
n
n
Y
Rb Y
Rb Y
Rb
i
fi (x)
i
Rb
Hence, a
dx
1
and
hence,
f
(x)
dx
f (x)dx
i
a
a i
f (x)dx
i=1

i=1

i=1

439. Suppose that f and g are nonnegaive and Riemann integrable on [a; b]: For p 6= 0; Let q be its conjugate,
i.e., p1 + 1q = 1: Prove that:

147

(a) If p > 1; then

Rb
a

Rb

f (x)g(x)dx

f p (x)dx

1=p

Rb
a

1=q

g q (x)dx

1=p R b
1=q
Rb p
Rb
(b) If 0 < p < 1; and f and g are positive, then a f (x)g(x)dx
f (x)dx
g q (x)dx
:
a
a

Proof. (a) Let f1 (x) = f p (x) and f2 (x) = g q (x): Then Hlders inequality, problem 92, implies that
1=p R b
1=q
R b 1=p
Rb
1=q
1=p
1=q
f
f
(x)dx
f
(x)dx
: But, f1 (x) = f (x); and f2 (x) = g(x):
(x)f
(x)dx
1
2
1
2
a
a
a
1=p R b
1=q
Rb p
Rb
q
f
(x)dx
g
(x)dx
:
Thus, a f (x)g(x)dx
a
a
p

(b) Let r = p1 > 1; and s be such that 1r + 1s = 1: Also, let f1 (x) = (f g) (x) and f2 (x) = gp1(x) :
Then f (x)g(x) = f1r ; f p (x) = f1 (x)f2 (x); and g q (x) = f2s (x); and so, by part (a),
1=r R b
1=r R b
1=s
Rb p
Rb
Rb r
Rb
s
f
(x)dx
=
f
(x)f
(x)dx
f
(x)dx
f
(x)dx
=
f
(x)g(x)dx
g q (x)dx
1
2
1
2
a
a
a
a
a
a
p Rb
1 p
Rb
= a f (x)g(x)dx
g q (x)dx
:
a
p
p 1
Rb
Rb p
Rb q
Thus, a f (x)g(x)dx
f (x)dx
g (x)dx
a
a

1=s

p 1
1=p R b
1=p R b
1=q
Rb
Rb p
Rb
p
q
=) a f (x)g(x)dx
f
(x)dx
g
(x)dx
= a f p (x)dx
g q (x)dx
:
a
a
a

440. Suppose that f is continuous on [0; 1] and there is a > 0 such that 0 f (x) a2=3 8x 2 [0; 1]:
R1
R1p
Show that if 0 f (x)dx = a; then 0 f (x)dx a2=3 :

1=p 2p 1
R1p
R1
R1
1
1
+1 2p
dx
f (x)dx
a 3p :
Proof. By Hlders inequality, with p > 1; a = 0 f (x)dx = 0 (f (x)) 2p
0
1=p
1=p
R1p
R1p
R1p
2p 1
p+1
p+1
3p
Thus, a1 ( 3p )
f
(x)dx
=)
a
f
(x)dx
=) a 3
f (x)dx:
0
0
0
p
R1p
R
p+1
1
f (x)dx =) a2=3
f (x)dx:
Since this holds for any p > 1; lim a 3
0
0
p!1+

441. Prove the following Minkowski inequality for integrals: If f1 ; f2 ; :::; fn are nonnegative and Riemann
integrable on [a; b]; then
1=p
1=p
1=p
Rb
Rb p
Rb
p
(a) If p > 1; then a (f1 (x) +
+ fn (x)) dx
f (x)dx
+
+ a fnp (x)dx
:
a 1
1=p
1=p
1=p
Rb
Rb p
Rb
p
(b) If 0 < p < 1; and fi 0, then a (f1 (x) +
+ fn (x)) dx
f (x)dx
+ + a fnp (x)dx
:
a 1

Proof. (a) If one sets Sn (x) = f1 (x) + f2 (x) +


+ fn (x); then one gets that
Rb p
Rb
p 1
S
(x)dx
=
(f
(x)
+
f
(x)
+
+
f
(x))
S
(x)dx
1
2
n
n
aR n
a
Rb
Rb
b
p 1
p 1
+ a fn (x)Snp 1 (x)dx
= a f1 (x)Sn (x)dx + a f2 (x)Sn (x)dx +
1=p R b
1=q
1=p R b
1=q
Hlders inequality R b
Rb p
p
p 1 q
p 1 q
f
(x)dx
S
(x)dx
+
+
f
(x)dx
S
(x)dx
n
n
n
a 1
a
a
a
1
1
p + q =1

Rb

(p 1)q=p

=
Rb
a

Now,

f1p (x)dx

Snp (x)dx
Rb

1=q

p
Sn
(x)dx
1=q
p
S
(x)dx
n
a

Rb

Rb

1=p

Rb

Snp (x)dx

f p (x)dx
a 1

Rb
a

1=q

1=p

Snp (x)dx

+
1

1
q

+
=

+
Rb
a

Rb
a

Rb
a

fnp (x)dx

fnp (x)dx

Snp (x)dx

1=p

Rb
a

1=p

Snp (x)dx

1=q

1=p

1=p
1=p
1=p
Rb
Rb p
Rb
Thus, a Snp (x)dx
f (x)dx
+
+ a fnp (x)dx
:
a 1
(b) It su ces to apply the inequality in problem 93(b).

442. Assume that f1 ; f2 ; :::; fn are nonnegative and Riemann integrable on [a; b]:
Rb p
Rb
Rb p
p
+ fn (x)) dx
f
(x)dx
+
+
f (x)dx:
(a) If p > 1; then a (f1 (x) +
1
a R
a nR
Rb
b p
b
p
(b) If 0 < p < 1; then a (f1 (x) +
+ fn (x)) dx
f (x)dx +
+ a fnp (x)dx:
a 1

148

Proof. (a) If we show that for x1 ; :::; xn > 0, and for p > 1; xp1 +
+ xpn (x1 +
+ xn ) ; then the result
p
follows immediately. Let us show that for x1 ; x2 > 0; and p > 1; we have xp1 + xp2 (x1 + x2 ) :
Dividing both sides by xp2 ; we get that

x1
x2

x1
x2

+1

; and so it su ces to prove that tp + 1


p 1

1: Then f (0) = 0; and f 0 (t) = p (t + 1)

tp

for all t > 0 and p > 1: Let f (t) = (t + 1)

+1

tp

(t + 1)
1

> 0:

1 p 1
t
p

(This is because for p > 1; and t > 0; we have that 1 +


> 1).
p
Thus, f is increasing and so f (t) 0 for all t > 0: Hence, t + 1 (t + 1) for all t > 0 and p > 1:
Rb
R
Rb
b
p
Therefore, if p > 1; then a (f1 (x) +
+ fn (x)) dx
f p (x)dx +
+ a fnp (x)dx:
a 1
p
(b) If we show that for x1 ; :::; xn 0, and for 0 < p < 1; (x1 +
+ xn )
xp1 +
+ xpn ; then the result
p
p
p
follows immediately. Let us show that for x1 ; x2 > 0; and 0 < p < 1; we have x1 + x2 (x1 + x2 ) :
Dividing both sides by xp2 ; we get that

x1
x2

x1
x2

+1
p

for all t > 0 and 0 < p < 1: Let f (t) = (t + 1)

tp

tp + 1

+ 1; and so it su ces to prove that (t + 1)


p 1

1: Then f (0) = 0; and f 0 (t) = p (t + 1)


1 p 1
t
p

tp

< 0:

(This is because for 0 < p < 1; and t > 0; we have that 1 +


< 1).
Thus, f is decreasing and so f (t) 0 for all t > 0: Hence, (t + 1)
tp + 1 for all t > 0 and 0 < p < 1:
Rb
R
Rb
b
p
Therefore, if 0 < p < 1; then a (f1 (x) +
+ fn (x)) dx
f p (x)dx +
+ a fnp (x)dx:
a 1

443. Suppose that f 2 C([0; c]); c > 0; is strictly increasing on [0; c] and f (0) = 0: Let f 1 (x) denote
Rx
R f (x) 1
the inverse of f: Show that for x 2 [0; c]; 0 f (t)dt + 0
f (t)dt = xf (x):
R R
x
f (x)
Proof. For x 2 [0; c]; let g(x) = 0 f (t)dt + 0
f 1 (t)dt:
0
R
R
0
x
f (x)
Then g 0 (x) = 0 f (t)dt + 0
f 1 (t)dt = f (x) + f 0 (x)f 1 (f (x)) = f (x) + xf 0 (x):
Rx
Rx
Rx 0
Rx
Now, g(x) = 0 g (t)dt = 0 (f (t) + tf 0 (t)) dt = 0 f (t)dt + 0 tf 0 (t)dt
Rx
Rx
Rx
by parts R x
x
=
f
(t)dt
+
[tf
(t)]
f
(t)dt
=
f
(t)dt
+
xf
(x)
f (t)dt = xf (x):
0
0
0
0
0

444. Prove the following Young inequality: Suppose that f 2 C([0; c]); c > 0; is strictly increasing on [0; c]
and f (0) = 0: Let f 1 (x) denote the inverse of f:
Rb
Ra
(a) Show that for any a 2 [0; c] and for any b 2 [0; f (c)]; we have 0 f (t)dt + 0 f 1 (t)dt ab:
(b) Show that equality holds in part (a) if and only if b = f (a):

Proof. (a) WLOG, say f (a) b: (otherwise, b < f (a) and so f 1 (b) < a and we apply the same method).
R f (a) 1
Rb
Rb
Rb
Ra
Ra
problem 97
=
af (a) + f (a) f 1 (t)dt:
f (t)dt + f (a) f 1 (t)dt
Now, 0 f (t)dt + 0 f 1 (t)dt = 0 f (t)dt + 0
Rb
Rb
adt = a(b f (a) = ab af (a):
Also, since f is strictly increasing on [0; c]; we get that f (a) f 1 (t)dt
f (a)
Ra
Rb 1
Rb
1
Hence, 0 f (t)dt + 0 f (t)dt = af (a) + f (a) f (t)dt af (a) + ab af (a) = ab:
(b) ((=)If b = f (a); then this is exactly problem 97.
Ra
Rb
Rb
(=)) If 0 f (t)dt + 0 f 1 (t)dt = ab; and WLOG f (a) b; then ab = af (a) + f (a) f 1 (t)dt:
Rb
If f (a) < b; then f (a) f 1 (t)dt > ab af (a); and we would get that ab > ab a contradiction.

445. Show that for a; b 0; (1 + a) ln(1 + a) (1 + a) + (eb b) ab:

Proof. Let f (x) = ln(1 + x) for x 2 [0; c]; c > 0: Then f is strictly increasing on [0; c] and f (0) = ln 1 = 0:
Ra
Rb
Also, f 1 (x) = ex 1: Thus, by problem 98, we have that for any a; b 0; 0 f (t)dt + 0 f 1 (t)dt ab:
(Of course, we will have a and b rst and then we choose c > maxfa; bg):
Ra
Rb
Ra
Rb
But 0 f (t)dt + 0 f 1 (t)dt = 0 ln(1 + t)dt + 0 (et 1) dt = eb b 1 + ((a + 1) ln (a + 1) a) :

446. Suppose that f is positive and Riemann integrable on [a; b]:


Rb
Rb 1
dx :
(a) Show that (b a)2
f (x)dx
a
a f (x)
Rb
Rb 1
(m+M )2
(b) Show that if 0 < m f (x) M; then a f (x)dx
dx
a)2 :
4mM (b
a f (x)
149


2
2
Schwartz R b
Rb
Rb 1
Rbp
dx :
f (x)dx
Proof. (a) (b a)2 = a 1dx = a f (x) p 1 dx
a
a f (x)

(b)

=)

f (x)
(f (x) m)(f (x) M )
Since
< 0; we get that f (x) M m + mM
f (x)
f (x) < 0
Rb
Rb 1
mM
f (x) + f (x) < M + m =) a f (x)dx + mM a f (x) dx
(M +

m) (b

a)

2
Rb
Rb
Rb 1
Rb
(m+M )2 (b a)2
=) mM a f (x)dx
dx
f (x)dx (M + m) (b a)
f (x)dx
:
4
a f (x)
a
a
Rb
Rb 1
2
2
(m+M ) (b a)
Thus, a f (x)dx
dx
:
4mM
a f (x)

1
X
447. Given an innite sequence of complex numbers (cn ) ; let f (x) =
cn einx : Show:
1
X

(a) If
(b) If

n=1
1
X

n=1

n=1

jcn j < 1; then f is continuous on R:


n jcn j < 1; then f is continuously dierentiable on R:

k
1
1
X
X
X
Proof. (a) Let fk (x) =
cn einx and > 0 be given. Then
jcn j < 1 =) 9N :
jcn j < :
n=1

Thus, for all x and all m

n=1

N; jf (x)

1
X

fm (x)j =

n=N

1
X

cn einx

n=m+1

n=m+1

Thus, the sequence ffk g converges to f uniformly.


Since each fk is continuous, so is f .

(b) We have f (x) =


W hy?

1
X

lim f (x+h)h f (x)


h!0

cn einx lim e

inh

n=1

h!0

1
X

= lim

h!0

1
X

cn e

n=1

cn einx [in] =

n=1

in(x+h) A

1
X

1
X

cn e

n=1

jcn j < :

inx A

1
X

= lim

h!0

cn einx

n=1

einh 1
h

incn einx :

n=1

Thus, we can dierentiate term by term and f is dierentiable.


1
1
1
X
X
X
0
0
inx
To show that f is continuous, let dn = incn : Then f (x) =
dn e ; and
jdn j =
n jcn j < 1:
n=1

n=1

n=1

Hence, by (a), f 0 is continuous.

Z1
2
448. Show that the function f (t) =
e tx dx is nite and continuous for t 1; dierentiable for t > 1;
1

and satises the dierential equation 2tf 0 (t) + f (t) = 0:


0
0
1
0
1
1
2
Z1
Z1
Z1
2
2
tx2
tx2
A
@
A
@
@
e ty dy A
dx
=
e
dx
Proof. For any t > 0; we have (f (t)) =
e
Fubini

1
2t

Z1 Z1
1

(2 ) e

e t(x
tr

+y

1
0

) dxdy

Polar Co ordinates

2 Z
1
Z
0

02
1 01
Z
Z
2
e tr rdrd = 2t1 @ d A @
0

2tre

tr 2

drA

1=2

Hence, f (t) =
t
for all t > 0:
This is nite, continuous
and dierentiable for all t > 0:p
1p
1
0
3=2
Now, f (t) = 2
t
and so 2tf 0 (t) + f (t) = 2t
t
2
150

3=2

1=2

1=2

1=2

= 0:


449. Let f be continuous on [0; 1]; and suppose that
continuous
function on R with
0 g1 is a nonnegative
10 1
1
Z1
Z
Z
g(x + 1) = g(x): Show that lim
f (x)g(nx)dx = @ f (x)dxA @ g(x)dxA :
n!1

Z1
Proof. Let a = g(x)dx and M = sup jf (x)j :
x2[0;1]

Then, since g(x) 0; we0


have that g(x)
1 0 1 a for all1x 2 [0;1 1]:
Z
Z
Z1
Z1
f (x)g(nx)dx
Now,
f (x)g(nx)dx @ f (x)dxA @ g(x)dxA =
=

f (x) (g(nx)

a) dx

Z1

Z1

jf (x)j jg(nx)

aj dx

a g(x) 8x2[0;1]

Z1

(a

Z1

Z1

af (x)dx

jf (x)j (a

g(nx)) dx

g(nx)) dx:

We will be done if we show that lim

n!1

Z1

(a

g(nx)) dx = 0:

Note that

Z1

Now, lim

Z1

(a

g(nx)) dx = a

n!1

Z1

g(nx)dx and so we have to show that lim

n!1

g(nx)dx

y=nx

=
lim 1
dy=ndx n!1 n

Zn

g(y)dy

g(y)=g(y+1)

lim 1
n!1 n

@n

Z1
0

Z1

g(nx)dx = a:

g(y)dy A =

Z1

g(y)dy = a:

450. Let f 2 L1 (R). Show that there exists a sequence xn ! 1 such that xn f (xn ) ! 0:

Proof. For x 2 R; let g(x) = xf (x): Suppose that @ a sequence xn ! 1 such that g(xn ) ! 0:
Then we can nd ; M > 0 : for all x M; we have that jg(x)j
:
g(x)
g(x)
Now, f (x) = x and so jf (x)j = x
M:
jxj for all x
R
1
= L (R); a contradiction.
But jxj M jxj dx = 1 and so f 2
Hence, there is a sequence xn ! 1 such that xn f (xn ) ! 0:
R
451. Let g : Rd ! R be
= 1; let g (x) = d g(x= ) for > 0; and dene
R nonnegative with Rd g(y)dy
f (x) = (g f ) (x) = Rd g (y)f (x y)dy: If f : Rd ! R is continuous and bounded, then show that
for any compact subset K Rd , f
f as ! 0:
K

R
R
R
y=x=
Proof. We rst note that for any > 0; Rd g (x)dx = d Rd g(x= )dx
=
g(y)dy = 1:
Rd
dy=

d dx

Let K R be a Rcompact subset. ThenRfor all x 2 K; we have:


R
jf (x) f (x)j = Rd g (y)f (x y)dy
g (y)f (x)dy = Rd g (y) [f (x y)
Rd
R
R
z=y=
= Rd d g(y= ) [f (x y) f (x)] dy
=
g(z) [f (x
z) f (x)] dz
Rd
dz=

d dy

Since f is bounded, 9M > 0 : jf (y) f (x)j M for all x; y 2 Rd :


Since f is continuous on K, f is uniformly continuous on K:
We are now ready to show that f
f as ! 0:
K
R
Let > 0 be given. Since g 2 L1 (R); 9N 2 N : Rd nBN (0) g(y)dy < 2M :
R
Now, (1) can be written as: (Remember that f is bounded and so Rd g(z) jf (x
151

f (x)] dy
R
g(z) jf (x
Rd

z)

f (x)j dz

(1)

z)

f (x)j dz

M)

Rd

g(z) jf (x

z)

f (x)j dz =

For I1 ; we have that I1

Z
|

Rd nBN (0)

Rd nBN (0)

g(z) jf (x
{z

z)

I1

g(z)M dz = M

f (x)j dz +

Rd nBN (0)

BN (0)

g(z) jf (x

{z

z)

I2

g(z)dz < M 2M = 2 ; 8x 2 Rd :

f (x)j dz :
}

For I2 ; we go as follows: (our aim is to make I2 < 2 )


Since f is uniformly continuous on K; we can nd 0 > 0 : whenever jx yj < 0 ; we have jf (x) f (y)j < 2 :
0
0
Choose small
R enough so that N < : Then j Rzj < for all z 2 BRN (0):
R
Hence, I2 = BN (0) g(z) jf (x
z) f (x)j dz
g(z) 2 dz = 2 BN (0) g(z)dz 2 Rd g(z)dz = 2 ; 8x 2 K:
BN (0)
Hence, we have shown that for small enough, and for all x 2 K; we have that jf (x) f (x)j < :
Thus, for any compact subset K Rd , f
f as ! 0:
K

152

Você também pode gostar